You are on page 1of 544

1

MAT023
A PUNTES DE CLASES

Departamento Matematica UTFSM Santiago


Parte I

Ecuaciones diferenciales ordinarias

5
Indice general

Indice general 1

I Ecuaciones diferenciales ordinarias 5


1. Ecuaciones diferenciales ordinarias de primer orden 6
1.1. Definiciones . . . . . . . . . . . . . . . . . . . . . . . . . . . . . . . . . . . 6
1.2. Modelos simples . . . . . . . . . . . . . . . . . . . . . . . . . . . . . . . . . 11
1.3. Metodos Elementales de Resolucion . . . . . . . . . . . . . . . . . . . . . . 17
1.3.1. Integracion directa . . . . . . . . . . . . . . . . . . . . . . . . . . . 18
1.3.2. Ecuaciones de variable separable . . . . . . . . . . . . . . . . . . . . 19
1.3.3. Ecuacion lineal de primer orden . . . . . . . . . . . . . . . . . . . . 23
1.3.4. Ecuacion de Bernoulli . . . . . . . . . . . . . . . . . . . . . . . . . 29
1.3.5. Ecuacion de Ricatti . . . . . . . . . . . . . . . . . . . . . . . . . . . 32
1.3.6. Ecuaciones homogeneas . . . . . . . . . . . . . . . . . . . . . . . . 36
1.3.7. Otros cambios de variables . . . . . . . . . . . . . . . . . . . . . . . 38
1.4. Modelos simples: Segunda parte . . . . . . . . . . . . . . . . . . . . . . . . 44
1.5. Analisis cualitativo . . . . . . . . . . . . . . . . . . . . . . . . . . . . . . . 56
1.5.1. Metodos cualitativos . . . . . . . . . . . . . . . . . . . . . . . . . . 57
1.5.2. Ecuaciones diferenciales autonomas . . . . . . . . . . . . . . . . . . 59
1.5.3. Equilibrio y estabilidad . . . . . . . . . . . . . . . . . . . . . . . . . 63
1.6. Ejercicios del captulo . . . . . . . . . . . . . . . . . . . . . . . . . . . . . . 72

2. Ecuaciones diferenciales lineales de orden superior 80


2.1. Elementos de transformaciones lineales . . . . . . . . . . . . . . . . . . . . 80
2.1.1. Definiciones . . . . . . . . . . . . . . . . . . . . . . . . . . . . . . . 80
2.1.2. Nucleo e imagen . . . . . . . . . . . . . . . . . . . . . . . . . . . . 88
2.1.3. Isomorfismo . . . . . . . . . . . . . . . . . . . . . . . . . . . . . . . 94
2.1.4. Matriz asociada a una transformacion lineal . . . . . . . . . . . . . 94
2.1.5. Calculo con coordenadas . . . . . . . . . . . . . . . . . . . . . . . . 101
2.1.6. Ejercicios resueltos de Transformaciones lineales . . . . . . . . . . . 104
2.2. Ecuaciones diferenciales lineales . . . . . . . . . . . . . . . . . . . . . . . . 140
2.2.1. Definiciones . . . . . . . . . . . . . . . . . . . . . . . . . . . . . . . 140
2.3. Teorema de Existencia y Unicidad . . . . . . . . . . . . . . . . . . . . . . . 145
2.4. El wronskiano . . . . . . . . . . . . . . . . . . . . . . . . . . . . . . . . . . 148
2.5. Ecuaciones diferenciales a coeficientes constantes . . . . . . . . . . . . . . . 153
2.5.1. Definiciones . . . . . . . . . . . . . . . . . . . . . . . . . . . . . . . 153
2.5.2. La ecuacion de orden 2 . . . . . . . . . . . . . . . . . . . . . . . . . 154

1
Apuntes Mat023 (Segundo semestre 2014)

2.5.3. La ecuacion de orden superior . . . . . . . . . . . . . . . . . . . . . 158


2.6. Metodo de variacion de parametros . . . . . . . . . . . . . . . . . . . . . . 162
2.7. Metodo del anulador . . . . . . . . . . . . . . . . . . . . . . . . . . . . . . 168
2.8. Movimiento vibratorio . . . . . . . . . . . . . . . . . . . . . . . . . . . . . 181
2.9. Ejercicios del captulo . . . . . . . . . . . . . . . . . . . . . . . . . . . . . . 187

3. Sistemas de ecuaciones diferenciales 192


3.1. Definiciones . . . . . . . . . . . . . . . . . . . . . . . . . . . . . . . . . . . 192
3.2. Ecuacion con coeficientes constantes . . . . . . . . . . . . . . . . . . . . . . 196
3.2.1. Matriz A diagonalizable . . . . . . . . . . . . . . . . . . . . . . . . 196
3.3. Variacion de parametros en sistemas . . . . . . . . . . . . . . . . . . . . . 201
3.4. Analisis cualitativo de sistemas . . . . . . . . . . . . . . . . . . . . . . . . 207
3.4.1. Valores propios reales y distintos (no nulos) . . . . . . . . . . . . . 207
3.4.2. Valores propios complejos . . . . . . . . . . . . . . . . . . . . . . . 211
3.4.3. Valores propios repetidos (no nulos) . . . . . . . . . . . . . . . . . . 214
3.5. Ejercicios del captulo . . . . . . . . . . . . . . . . . . . . . . . . . . . . . . 225

4. Transformacion integral de Laplace 229


4.1. Definiciones y teoremas fundamentales . . . . . . . . . . . . . . . . . . . . 229
4.2. Calculo de transformadas . . . . . . . . . . . . . . . . . . . . . . . . . . . . 233
4.3. Primer Teorema de la Traslacion . . . . . . . . . . . . . . . . . . . . . . . 236
4.4. Transformada de la derivada . . . . . . . . . . . . . . . . . . . . . . . . . . 237
4.5. Funciones escalonadas y Segundo Teorema de la Traslacion . . . . . . . . . 243
4.6. La Transformada de integrales de convolucion . . . . . . . . . . . . . . . . 247
4.7. Ejercicios del captulo . . . . . . . . . . . . . . . . . . . . . . . . . . . . . . 255

5. Series de Fourier 262


5.1. Definiciones . . . . . . . . . . . . . . . . . . . . . . . . . . . . . . . . . . . 262
5.1.1. El espacio SC [a, b] . . . . . . . . . . . . . . . . . . . . . . . . . . . 262
5.1.2. Teorema de la mejor aproximacion . . . . . . . . . . . . . . . . . . 267
5.2. Convergencia Puntual de series de Fourier . . . . . . . . . . . . . . . . . . 278
5.3. Series de Fourier de senos y cosenos . . . . . . . . . . . . . . . . . . . . . . 282
5.4. Derivacion e integracion de Series de Fourier . . . . . . . . . . . . . . . . . 292
5.5. Ejercicios del captulo . . . . . . . . . . . . . . . . . . . . . . . . . . . . . . 293

II Calculo diferencial en varias variables 298


6. Elementos de topologa de Rn 299
n
6.1. El espacio euclidiano R . . . . . . . . . . . . . . . . . . . . . . . . . . . . 299
6.2. Producto interno y norma . . . . . . . . . . . . . . . . . . . . . . . . . . . 299

2
Apuntes Mat023 (Segundo semestre 2014)

6.3. Elementos de topologa de Rn . . . . . . . . . . . . . . . . . . . . . . . . . 302


6.4. Ejercicios del captulo . . . . . . . . . . . . . . . . . . . . . . . . . . . . . . 316

7. Funciones de varias variables 318


7.1. Definiciones basicas . . . . . . . . . . . . . . . . . . . . . . . . . . . . . . . 318
7.2. Graficos, conjuntos de nivel y trazas . . . . . . . . . . . . . . . . . . . . . . 321
7.3. Ejercicios del captulo . . . . . . . . . . . . . . . . . . . . . . . . . . . . . . 327

8. Lmites y continuidad 329


8.1. Definiciones . . . . . . . . . . . . . . . . . . . . . . . . . . . . . . . . . . . 329
8.2. Calculo de lmites . . . . . . . . . . . . . . . . . . . . . . . . . . . . . . . . 336
8.2.1. Algebra de lmites . . . . . . . . . . . . . . . . . . . . . . . . . . . . 336
8.2.2. Desigualdades y Teorema del Sandwich o teorema de acotamiento . 337
8.3. Continuidad . . . . . . . . . . . . . . . . . . . . . . . . . . . . . . . . . . . 339
8.4. Algebra de funciones continuas . . . . . . . . . . . . . . . . . . . . . . . . 341
8.5. Continuidad de funciones vectoriales . . . . . . . . . . . . . . . . . . . . . 342
8.6. Ejercicios del captulo . . . . . . . . . . . . . . . . . . . . . . . . . . . . . . 347

9. Diferenciacion en varias variables 349


9.1. Derivadas parciales . . . . . . . . . . . . . . . . . . . . . . . . . . . . . . . 349
9.2. Interpretacion de la derivada parcial . . . . . . . . . . . . . . . . . . . . . 359
9.3. Diferenciabilidad . . . . . . . . . . . . . . . . . . . . . . . . . . . . . . . . 365
9.4. Derivadas de orden superior y funciones de clase C n . . . . . . . . . . . . . 383
9.5. Gradiente y matriz jacobiana . . . . . . . . . . . . . . . . . . . . . . . . . 387
9.6. La regla de la cadena . . . . . . . . . . . . . . . . . . . . . . . . . . . . . . 390
9.7. Gradiente y planos tangentes . . . . . . . . . . . . . . . . . . . . . . . . . 406
9.8. Derivada direccional y direcciones de crecimiento maximo . . . . . . . . . . 410
9.9. Ejercicios del captulo . . . . . . . . . . . . . . . . . . . . . . . . . . . . . . 415

10.Maximos y mnimos 427


10.1. Extremos locales . . . . . . . . . . . . . . . . . . . . . . . . . . . . . . . . 427
10.2. Maximos y mnimos en compactos y/o con restricciones . . . . . . . . . . . 443
10.3. Extremos restringidos Multiplicadores de Lagrange . . . . . . . . . . . . . 445
10.3.1. Criterio de la segunda derivada para extremos condicionados . . . . 458
10.4. Ejercicios del captulo . . . . . . . . . . . . . . . . . . . . . . . . . . . . . . 463

11.Funciones implcitas e inversas 468


11.1. El teorema de la funcion implcita . . . . . . . . . . . . . . . . . . . . . . . 468
11.2. El teorema de la funcion inversa . . . . . . . . . . . . . . . . . . . . . . . . 483
11.3. Ejercicios del captulo . . . . . . . . . . . . . . . . . . . . . . . . . . . . . . 492

3
Apuntes Mat023 (Segundo semestre 2014)

III Evaluaciones de anos anteriores 495


12.Controles 496

13.Certamenes 509

Bibliografa 541

4
Parte I

Ecuaciones diferenciales ordinarias

5
Captulo 1 : Ecuaciones diferenciales ordinarias de primer orden

Definiciones

En general, entenderemos el modelamiento matematico como el proceso de establecer


un modelo matematico (es decir, un sistema expresado en terminos de variables, funciones,
ecuaciones, etc.) que represente una situacion principalmente de naturaleza fsica, su
resolucion matematica, y finalmente la interpretacion de los resultados en los terminos
fsicos originales.
Como muchos conceptos de la naturaleza, tales como velocidad, aceleracion, las reaccio-
nes qumicas, los cambios de temperatura observados en un cuerpo, etc. se expresan como
razones de cambio tiene pleno sentido el uso de derivadas de funciones adecuadas. En este
tipo de situaciones, un modelo matematico es frecuentemente una ecuacion que contiene
una o mas derivadas de una funcion desconocida. Tal modelo matematico es llamado una
ecuacion diferencial [1].

Ejemplo 1.1.1. Son ejemplos de ecuaciones diferenciales las siguientes expresiones ma-
tematicas:

d dy

1. dx
(1 x2 ) dx + n (n + 1) y = 0 (Ecuacion de Legendre)
d y 2 dy
2. x2 dx 2 2
2 + x dx + (x ) y = 0 (Ecuacion de Bessel)

d y 3 2
2d y dy
3. x3 dx 3 + 4x dx2 + x dx + 5y = 0 (Ecuacion de Euler)

4. y 00 + ky = A sin(o x), A, o R (Problemas de resortes)

5. y 00 xy = 0 (Ecuacion de Airy)

Definicion 1.1.1. Una ecuacion diferencial se dice de orden n si n corresponde al mayor


orden de derivada de la variable dependiente y presente en la ecuacion.
d y 2
3
Ejemplo 1.1.2. La ecuacion dx 2 + (5x) y = y 6 es una ecuacion diferencial de segundo
 3 4
d y dy
grado. La ecuacion dx 3 = dx + 5y es una ecuacion diferencial de grado tres.

Comenzaremos el estudio de las ecuaciones diferenciales con las ecuaciones diferenciales


de primer orden.

6
Apuntes Mat023 (Segundo semestre 2014)

Definicion 1.1.2. Sea f : U R2 R una funcion de dos variables. Una ecuacion


diferencial de primer orden es una ecuacion de la forma:

y 0 = f (x, y) (1.1)

La variable x en este caso se conoce como variable independiente. Si la variable inde-


pendiente es el tiempo t, frecuentemente una ecuacion diferencial se anota como:

y 0 = f (t, y) o bien y = f (t, y)

Observacion 1.1.1. Una ecuacion diferencial (ordinaria) de orden n es una ecuacion de


la forma:
f x, y, y 0 , y 00 , . . . , y (n) = 0


para una cierta funcion f .


Ejemplo 1.1.3. Son ecuaciones diferenciales de primer orden:

1. y 0 = f (x) , f funcion integrable.

2. y 0 + y = cos x

3. x3 y 0 y 2 = 0, x>0

Definicion 1.1.3. Sea I R un intervalo abierto del tipo ]a, b[, o posiblemente intervalos
abiertos infinitos del tipo ], b[ , ]a, +[, o bien ], +[. Una funcion : I R R
se dice solucion de la ecuacion diferencial (1.1) en el intervalo I si:

1. t, (t) U, t I

2. 0 (t) = f t, (t) , t I


1
Ejemplo 1.1.4. La funcion : ]1, +[ R, t (t) = t1
es solucion de la EDO
dy
= y 2
dt
pues, para t ]1, +[
 
d 1 1
=
dt t1 (t 1)2
 2
1
=
t1
Ejemplo 1.1.5. Verifique que la funcion (x) = sin x definida en R es solucion de la
ecuacion diferencial de segundo orden:

y 00 + y = 0

7
Apuntes Mat023 (Segundo semestre 2014)

d d2
Solucion. Derivando la funcion obtenemos: dx
= cos x y dx2
= sin x se sigue

d2 (x)
+ (x) = 0
dx2
para todo x R.

Observacion 1.1.2. Note que al escribir:

C (x) = sin x + C

con C una constante cualquiera, C tambien es solucion de la ecuacion diferencial.

Ejemplo 1.1.6. Consideremos la ecuacion:

xy 0 x2 y = 0, x>0

Se sabe que toda funcion de la forma:

y = x2 + Cx, CR (1.2)

es solucion de la ecuacion diferencial. En efecto, derivando la ecuacion (1.2), tenemos:

y 0 = 2x + C

Reemplazando en la ecuacion diferencial, se tiene que:

x (2x + C) x2 x2 + Cx = 0


para todo x > 0.

Ejemplo 1.1.7. Hallar una ecuacion diferencial para la familia de parabolas:

y = C1 (x C2 )2

donde C1 y C2 constantes arbitrarias.

Solucion. Despejamos C1
y
= C1
(x C2 )2
luego  
d y
=0
dx (x C2 )2
esto es
y 0 (x C2 )2 y2 (x C2 )
=0
(x C2 )4

8
Apuntes Mat023 (Segundo semestre 2014)

se sigue
y 0 (x C2 )2 y2 (x C2 ) = 0
luego
y 0 (x C2 ) = 2y
de donde obtenemos
2y
x C2 =
y0
derivando  0
2y
1=
y0
se sigue
1 (y 0 )2 yy 00
=
2 (y 0 )2
luego
(y 0 )2
= yy 00
2

Ejercicios de la seccion

1. Establezca el orden de la ecuacion diferencial dada:


 7
00 0 (4) dy
(6) 00
(a) (1 x) y 4xy + 3y = tan x (b) y 2xy y =
s dx
4 2
d3 y d2 y
  
dy dy
(c) x 3 + ex y = 0 (d) = 1+
dx dx dx2 dx
2. Comprobar que las siguientes funciones satisfacen las ecuaciones dadas y dar un
intervalo en el cual esto se cumpla:
(a) y = A sin (x + B) ; y 00 + y = 0 (b) y = ex ex ; y 00 y = 0
(c) y = tan (x) ; y 0 = 1 + y 2 (d) y 0 = 25 + y 2 ; y = 5 tan 5x

3. Demostrar que la ecuacion dada define implcitamente una solucion de la ecuacion


dada
y3
(a) x2 y =1 dy
dx
= x2xy
2 y 2 para x 6= y
3
3 2 0
(b) x + 3xy = 1 2xyy + x2 + y 2 = 0 para x ]0, 1[

4. Muestre que la familia de funciones


Z x
x2 2 2
y=e et dt + Cex
0

9
Apuntes Mat023 (Segundo semestre 2014)

son soluciones de la ecuacion


dy
+ 2xy = 1
dx
5. Muestre que la funcion definida por tramos
x2 si x < 0

(x) =
x2 si x 0
es una solucion de la ecuacion diferencial xy 0 2y = 0 en R.

6. Determine R para que la funcion y = x sea solucion de 2x2 y 00 y = 0. Si


encuentra mas de un valor, muestre que cualquier combinacion lineal de esas dos
funciones resulta ser una solucion del problema.

7. Encontrar valores de m para los cuales la funcion es solucion de la ecuacion dada:

a) y (x) = emx donde y 000 3y 00 4y 0 + 12y = 0


b) y (x) = xm donde x2 y 00 + 2xy 0 6y = 0

8. Encontrar una E.D.O. de la forma y 00 + A (x) y 0 + B (x) y = 0 que tenga entre sus
soluciones las funciones y1 (x) = ex y y2 (x) = xex .

9. Juan, Leo y Roberto estan tomando cafe y un joven del paralelo 19 les pregunta por
la solucion de la ecuacion diferencial
dy y+1
=
dt t+1
despues de un rato, Juan dice y (t) = t, Leo y (t) = 2t + 1 y Roberto y (t) = t2 2
Quien esta en lo correcto?

10. Construir una ecuacion diferencial de la forma


dy
= 2y t + g (y)
dt
que tenga la funcion y (t) = e2t como solucion.
dy
11. Construir una ecuacion diferencial de la forma = f (t, y) que tenga por solucion
2
dt
y (t) = et donde f (t, y) dependa explcitamente de t y y.

12. Hallar una ecuacion diferencial para la familia de curvas:

y = C1 + (x C2 )2

donde C1 y C2 constantes arbitrarias.

13. Construya una ecuacion diferencial que no tenga ninguna solucion real.

10
Apuntes Mat023 (Segundo semestre 2014)

Modelos simples

Estudiaremos algunos ejemplos elementales de modelamiento matematico:

Problema 1.2.1 ([2]). Desde una cierta altura se ha arrojado un cuerpo de masa m.
Determinar la ley segun la cual vara la velocidad de cada v, si sobre el cuerpo, ademas
de la fuerza de gravedad, actua la fuerza de resistencia del aire que es proporcional a la
velocidad v.

Solucion. Sea m la masa del cuerpo en cada libre. En virtud de la Segunda Ley de
Newton: X
Fi = ma
i
P
donde i Fi representa la suma (vectorial) de fuerzas aplicadas al cuerpo y a representa la
aceleracion del cuerpo, se obtiene:
dv
m = mg kv (1.3)
dt
donde g es la aceleracion de gravedad y k es la constante (positiva) de proporcionalidad.
Resolver esta ecuacion diferencial significa encontrar una funcion v = f (t) que satisfaga
identicamente la ecuacion diferencial dada. Existen una infinidad de funciones de este tipo
(esto sera probado mas adelante). Es facil comprobar que toda funcion del tipo:
k mg
v (t) = Ce m t + (1.4)
k

11
Apuntes Mat023 (Segundo semestre 2014)

satisface la ecuacion (1.3), cualquiera que sea la constante C. Pero, cual de estas funciones
dara la dependencia buscada entre v y t? Para encontrar dicha relacion, se debe utilizar un
condicion adicional. Esta condicion adicional se llama condicion inicial. Supongamos que
en el momento inicial del experimento arrojamos el cuerpo con una velocidad inicial v0
conocida. As, la funcion v = f (t) que deseamos encontrar debe satisfacer la condicion:

f (0) = v0

Es decir, v = v0 en t = 0. As, reemplazando en (1.4), se obtiene:


mg
C = v0
k
Por lo tanto, la ley segun la cual vara la velocidad de cada esta dada por la ecuacion:
 mg  k t mg
v (t) = v0 e m +
k k
Observacion 1.2.1. Una cuestion de interes es el comportamiento asintotico de la solucion.
Es decir, el comportamiento de la solucion para t suficientemente grande. Mas precisamente,
nos interesa:
lm v (t)
t+

En particular, para el ejemplo:


n mg  k t mg o
lm v (t) = lm v0 e m +
t+ t+ k k
mg
=
k
La interpretacion del resultado anterior, es que cuando el tiempo t es suficientemente
grande, la velocidad final del cuerpo no depende de la velocidad inicial.

Observacion 1.2.2. Observar que desde la ecuacion (1.3), si suponemos k = 0, es decir,


suponemos que la resistencia del aire es tan pequena que puede ser despreciada, se obtiene:

v (t) = v0 + gt

Definicion 1.2.1. Se llamara solucion general de la ecuacion diferencial de primer


orden:
y 0 = f (x, y)
a la funcion:
y = (x, C)
que depende de una constante arbitraria C y satisface las condiciones siguientes:

1. satisface la ecuacion diferencial para cualquier valor de la constante C.

12
Apuntes Mat023 (Segundo semestre 2014)

2. cualquiera que sea la condicion inicial y = y0 para x = x0 , es decir, y (x0 ) = y0


se puede encontrar un valor C = C0 tal que la funcion y = (x, C0 ) satisfaga la
condicion inicial dada.

Problema 1.2.2 (Espejo parabolico). Hallar la forma que debe tener un espejo convexo,
simetrico respecto de un eje, llamado eje focal, de tal modo que si un haz de luz es apuntado
hacia el espejo, paralelo al eje focal, se refleje directamente en un punto F fijo (llamado
foco) del eje focal.

Solucion. Obtendremos tal espejo mediante la rotacion de una curva en el plano. Sea
C : y = f (x) tal curva. Consideremos un sistema de coordenadas de tal modo que el
origen del sistema coincida con el foco F de la curva. En particular, el eje focal de la curva
coincide con el eje de las abscisas de tal sistema de referencia. Considere un punto P (x, y)
en la curva C y sea T la recta tangente a C en el punto P (x, y). Denotemos por Q el punto
de interseccion de T con el eje focal (o de las abscisas del sistema de referencia). Si L es
una recta que representa el haz de luz paralelo al eje focal y que incide en P , por la ley de
Snell, el angulo de incidencia ]T P L y angulo de reflexion ]QP F coinciden. Es decir, se
tiene que:
]T P L = ]QP F =
Entonces, del triangulo 4QP F se obtiene:
y
tan 2 = (1.5)
x
por ser F el origen del sistema de coordenadas. Como T es tangente a la curva C, se tiene
que ]P QF es tambien . Luego:
tan = y 0 (1.6)

13
Apuntes Mat023 (Segundo semestre 2014)

Reemplazando las ecuaciones (1.5) y (1.6), en la identidad trigonometrica siguiente:


2 tan
tan 2 =
1 tan2
se obtiene la ecuacion diferencial de primer orden:

y 2y 0
= (1.7)
x 1 (y 0 )2

Problema 1.2.3 (Braquistocrona). [3] Hallar la forma que debe tener un alambre de
modo que una argolla que se desliza por el, sin roce, bajo la accion de la gravedad de un
punto A a un punto B de menor altura en el mismo plano y no exactamente bajo el punto
A, lo haga en el menor tiempo posible.

Solucion. Este problema fue planteado en 1696 por Jean Bernoulli a la comunidad cientfi-
ca de su epoca. La solucion que el mismo encontro (independientemente tambien lo hicieron
Leibnitz, LHopital, Newton) usa una version generalizada de la Ley de Snell de la optica.
Comenzaremos modelando primeramente esta situacion: la version generalizada de la Ley
de Snell. A modo de ejercicio lo haremos utilizando las herramientas del calculo diferencial,
en particular minimizacion. Considere, entonces, el siguiente problema:

Supongamos que un atleta, situado en la orilla oriental de un ro de ancho a debe atravesarlo


nadando a velocidad constante v1 hasta un cierto punto C en la orilla occidental. Luego de
esto, debe correr a velocidad constante v2 por sobre la arena de la ribera de ancho b del ro
hasta la meta en el punto B. Se supone que v1 < v2 . Se desea encontrar el punto C en la orilla
occidental de tal modo que el tiempo empleado por el atleta desde el punto A hasta la meta
en B sea el menor posible.

14
Apuntes Mat023 (Segundo semestre 2014)

Consideremos las rectas paralelas (y verticales) l1 , l2 y l3 . Supongamos que la recta l1


representa la orilla oriental del ro, l2 representa la ribera del ro y l3 la lnea de meta. Por
la condiciones del problema tenemos:

1. A l1

2. C l2

3. B l3

4. a es la distancia entre l1 y l2

5. b es la distancia entre l2 y l3

Supongamos, ademas, que c y x son las distancias verticales de A a B, y de A a C,


respectivamente. Entonces, el tiempo total de la carrera esta dado por:


q
2
x +a 2 (c x)2 + b2
T (x) = +
v1 v2
Derivando e igualando a 0, se obtiene la coordenada x0 de tiempo mnimo. Es decir, x0
debe cumplir con:
x c x0
p 0 = q
v1 x20 + a2 v (c x )2 + b2
2 0

Si introducimos los angulos de incidencia y respecto de la recta normal a l2 para este


valor de x0 se obtiene:
sin sin
=
v1 v2
que es la conocida Ley de Snell.
Ahora bien, si consideramos otro segmento (es decir, otra recta vertical l4 ) por el cual
el atleta se deba desplazar a velocidad constante v3 , se obtendra:
sin 1 sin 2 sin 3
= = = constante
v1 v2 v3
De manera analoga, la argolla de masa m que cae bajo la accion de la gravedad g tiene
una velocidad v que va en aumento de acuerdo a la distancia vertical recorrida y. As,
igualando energas potencial y cineticas se obtiene la ecuacion:
1 2
mv = mgy
2
de donde: p
v= 2gy (1.8)

15
Apuntes Mat023 (Segundo semestre 2014)

Por esta razon, Bernoulli conjeturo la Ley de Snell generalizada siguiente:


sin
= , constante (1.9)
v
donde es el angulo que instantaneamente forma la tangente a la curva en la posicion de
la argolla y la vertical. Finalmente, notando que:
 
sin = cos
2
1
= q
1 + tan2 2


1
= q
1 + (y 0 )2

Utilizando, entonces, las formulas (1.8) y (1.9) se obtiene la ecuacion:


p 1
2gy = q
1 + (y 0 )2

O bien:  
2
y 1 + (y 0 ) = k 2
1
donde k = 2g2
.

Ejercicios de la seccion

1. Determine la ecuacion que debe cumplir la familia de curvas que forman un angulo
de 45 grados en la interseccion con la familia de curvas y (x + c) = 1.

2. Determine la ecuacion de la familia de curvas ortogonales a la familia y 2 = cx3 .

3. En este problema se analiza la cada de una gota de agua. Supongamos que al


caer esta se evapora y mantiene su forma esferica, la rapidez con que se evapora es
proporcional al area con una constante de proporcionalidad < 0 y no se considera
la resistencia del aire. Designemos por la densidad del agua, r0 el radio de la gota
cuando t = 0 y la direccion positiva es hacia abajo.

a) Muestre que el radio de la gota r (t) disminuye de acuerdo a la ley


 

r (t) = t + r0

b) Obtener la ecuacion diferencial que satisface la velocidad v (t) de la gota en su


cada libre. Determine la velocidad si la gota cae del reposo.

16
Apuntes Mat023 (Segundo semestre 2014)

4. En la figura suponga que el eje y y la recta x = 1 corresponden a las orillas oeste


y este de un ro de 1 [km] de ancho. El ro fluye hacia el norte con una velocidad
vr donde kvr k = vr [km/h]. Un hombre entra al ro en el punto (1, 0) en la costa
este y nada en direccion a la ribera contraria a una velocidad constante de kvs k = vs
[km/h]. El hombre quiere llegar al punto (0, 0) de modo que nada de forma tal que
su vector velocidad vs siempre apunta a (0, 0). Muestre que la trayectoria que sigue
el nadador satisface la ecuacion
p
dy vs y vr x2 + y 2
=
dx vs x

Metodos Elementales de Resolucion

Recordaremos primeramente un teorema esencial para resolver ecuaciones diferenciales


elementales: el Teorema Fundamental del Calculo.

Teorema 1.3.1. (Teorema Fundamental del Calculo)


Sea f : [a, b] R funcion integrable en [a, b]. Dado x0 [a, b] e y0 R se tiene que la
funcion F : [a, b] R dada por
Zx
F (x) = f (t)dt + y0 con x [a, b],
x0

es continua en [a, b] y F (x0 ) = y0 . Ademas, si f es continua en [a, b], entonces la funcion


F es derivable en [a, b] y satisface la ecuacion:

F 0 (x) = f (x)

Observacion 1.3.1. Recordemos que en este contexto la funcion F es conocida como una
primitiva de f .

17
Apuntes Mat023 (Segundo semestre 2014)

Integracion directa

Definicion 1.3.1. Una ecuacion diferencial ordinaria de primer orden, digamos y 0 = f (x, y),
se dice que es resoluble por integracion directa si existe una funcion g integrable sobre
un intervalo abierto I R tal que f (x, y) = g(x)

Observacion 1.3.2. En particular, una ecuacion diferencial de integracion directa es de


la forma:
y 0 = g(x) (1.10)
para todo x I.
As, si g : D R R es una funcion integrable, entonces integrando en ambos lados de
la igualdad 1.3.2 y usando el Teorema Fundamental del Calculo, obtenemos que la solucion
general de la ecuacion diferencial es de la forma:

(x) = F (x) + C,

donde F es una primitiva de g y C R es una constante.

Ejemplo 1.3.1. Una partcula se mueve a lo largo de una lnea recta de manera que su
velocidad en el instante t es 2 sin t. Si f (t) indica su posicion en el tiempo t medio a partir
del punto de partida, se tiene que f 0 (t) = 2 sin t. Por la observacion anterior, se concluye
que:
f (t) = 2 cos t + C
Note que para fijar la funcion posicion se necesita algun otro dato. En particular, si se
conoce el valor de f en algun instante en particular, entonces se puede determinar C. Por
ejemplo, si f (0) = 0, entonces C = 2 y la funcion posicion es f (t) = 2 cos t + 2.

Ejemplo 1.3.2. Determine la solucion del problema


dy 2
= ex
dx
y (0) = 5
2
ex dx no es una funcion elemental, sin embargo,
R
Solucion. Se puede demostrar que
podemos expresar la solucion por
Z x
2
y (x) = et dt + 5
0

18
Apuntes Mat023 (Segundo semestre 2014)

Ecuaciones de variable separable

Definicion 1.3.2. Una ecuacion diferencial de primer orden y 0 = f (x, y) se dice una
ecuacion de variable separable si:
f (x, y) = Q (x) R (y) (1.11)
para Q y R funciones continuas sobre algun intervalo abierto I R.
Observacion 1.3.3. [5] Si R (y) 6= 0 se puede dividir por R (y) y escribir la ecuacion
(1.11) en la forma:
A (y) y 0 = Q (x) (1.12)
donde A (y) = 1/R (y). Recordemos, ademas, que y representa una cierta funcion descono-
cida y = Y (x), luego la ecuacion (1.12), se expresa como:
A (Y (x)) Y 0 (x) = Q (x)
Integrando la ecuacion anterior, se sigue que:
Z Z
0
A (Y (x)) Y (x) dx = Q (x) dx + C

Haciendo la sustitucion y = Y (x) en la integral de la izquierda, se tiene que dy = Y 0 (x) dx.


Por consiguiente, se obtiene:
Z Z
A (y) dy = Q (x) dx + C

Ahora bien, si G es una primitiva de A y H es una primitiva de Q, la ecuacion anterior se


puede escribir como:
G (y) = H (x) + C (1.13)
Recprocamente, si y es una funcion que satisface la ecuacion (1.13), entonces:
G0 (y) y 0 = H 0 (x)
es decir, tenemos A (y) y 0 = Q (x). Por lo tanto, la ecuacion (1.13) da una representacion
implcita de una familia de soluciones a un parametro.
Observacion 1.3.4. En particular, el formalismo anterior se reduce considerablemente si
se considera la notacion de Leibnitz para las derivadas. Es decir, escribimos y 0 = dy/dx y
separamos las variables, con lo cual la ecuacion (1.12) se escribe directamente como:
A (y) dy = Q (x) dx
y la ecuacion anterior tiene sentido al ser dy la diferencial de la funcion desconocida y.
Formalmente, podemos integrar la ecuacion anterior, obteniendo:
Z Z
A (y) dy = Q (x) dx + C

pero esto solo es formal, pues las variables de integracion son distintas.

19
Apuntes Mat023 (Segundo semestre 2014)

Ejemplo 1.3.3. Resolver la ecuacion


dy
= eyx
dx
Solucion. Se trata de una E.D.O. de variables separadas pues
dy
= eyx = ex ey
dx
se sigue
ey(x) y 0 (x) = ex
integrando respecto a x Z Z
y(x) 0
e y (x) dx = ex dx

se sigue
ey(x) = ex + C
despejando
ey(x) = ex + K
luego
y (x) = ln ex + K


es una familia de soluciones.

Ejemplo 1.3.4. Resolver la ecuacion


dy
= x2 y + x2
dx
Solucion. Note que x2 y+x2 = x2 (y + 1) luego se trata de una EDO de variables separables
dy
= x2 (y + 1)
dx
se sigue Z Z
dy
= x2 dx
y+1
integrando

x3
ln |y + 1| = +C
3

x3
y + 1 = Ke 3

as
x3
y (x) = Ke 3 1

20
Apuntes Mat023 (Segundo semestre 2014)

Ejemplo 1.3.5. Hallar la solucion general de la ecuacion diferencial:



x cos y + x + 1 sen y y 0 = 0

indicando claramente el dominio en el cual esta definida.

Solucion. Notamos primeramente que la ecuacion esta definida para x > 1 y es una
ecuacion de variable separable. Luego, anotamos:
dy
x cos y + x + 1 sen y =0
dx
Es decir:
dy
x cos y = x + 1 sen y
dx

Ahora bien, si y 6= 2
+ k, con k Z, escribimos la ecuacion anterior como:

x dy
= tan y
x+1 dx
Separando variables e integrando, obtenemos:
Z Z
x
dx = tan y dy + C
x+1
con C una constante de integracion. Entonces, la ecuacion anterior queda:

Z Z
1
x + 1 dx dx = ln | cos y| + C
x+1
Es decir, obtenemos:
2
q 3
x+1 2 x + 1 = ln | cos y| + C
3
Por tanto, obtenemos:
2
x + 1 (x 2) + ln | cos y| = C
3
Finalmente, notamos que las funciones constantes:

y= + 2k, kZ
2
son soluciones de la ecuacion diferencial. Por lo tanto, la solucion general de la ecuacion
esta dada por:

2 x + 1 (x 2) + ln | cos y| = C , si y = 3
6 , ,...
3 2 2
y = + 2k, k Z
2

21
Apuntes Mat023 (Segundo semestre 2014)

Ejemplo 1.3.6. Diremos que una ecuacion diferencial es una ecuacion diferencial lineal
de primer orden homogenea si es de la forma:

y 0 + P (x) y = 0 (1.14)

si P es una funcion continua sobre su dominio abierto.

Solucion. Separando las variables e integrando:


Z Z
dy
= P (x) dx + K
y
con K una constante de integracion. Luego, la ecuacion anterior implica que:
Z
ln |y| = P (x) dx + K

As: R
y = eK e P (x)dx

Por consiguiente: R
y = C e P (x)dx

Observacion 1.3.5. Note que en el procedimiento anterior, se ha dividido por y, por


tanto, debe
R
explicarse que toda solucion de (1.14) se puede expresar mediante la formula
P (x)dx
y =Ce . Sea y una solucion de (1.14) y considere la funcion g definida por:
R
P (x)dx
g (x) = y e

Luego:
R R
g 0 (x) = y 0 e P (x)dx
+ P (x) ye P (x)dx
R
= e P (x)dx
(y 0 + P (x) y)
= 0

como x pertenece a un intervalo abierto, se obtiene que g (x) = C, por el Teorema del Valor
Medio. Por tanto: R
y = C e P (x)dx

Observacion 1.3.6. El razonamiento anterior da origen a un metodo de resolucion de


ecuaciones diferenciales del tipo:

y 0 + P (x) y = Q (x)

donde P y Q son funciones continuas en un intervalo abierto I R. La ecuacion diferencial


anterior se llama ecuacion diferencial lineal de primer orden.

22
Apuntes Mat023 (Segundo semestre 2014)

Ecuacion lineal de primer orden

Definicion 1.3.3. Una ecuacion diferencial de primer orden se dice lineal si es de la forma:

y 0 + P (x) y = Q (x) (1.15)

para dos funciones P y Q continuas en un intervalo abierto I R.

Definicion 1.3.4. Un factor integrante o un factor de integracion para la ecuacion


(1.15) es una expresion de la forma:
R
P (x)dx
(x) = e (1.16)

Observacion 1.3.7. Multipliquemos la ecuacion diferencial (1.15) por el factor integrante


en (1.16), luego: R R
e P (x)dx {y 0 + P (x) y} = e P (x)dx Q (x)
Note que la ecuacion anterior, puede escribirse como:
d n R P (x)dx o R
ye = e P (x)dx Q (x)
dx
Integrando respecto de x, obtenemos:
R
Z R
P (x)dx P (x)dx
ye = e Q (x) dx + C

Finalmente: Z 
R R
P (x)dx P (x)dx
y=e e Q (x) dx + C

Teorema 1.3.2 (Formula de Leibnitz). Sean P y Q dos funciones continuas sobre un


intervalo abierto I R. La solucion general de la ecuacion diferencial lineal:

y 0 + P (x) y = Q (x)

esta dada por la formula:


R
Z R

P (x)dx P (x)dx
y (x) = e e Q (x) dx + C
R
P (x)dx
Observacion 1.3.8. Si (x) = e es el factor integrante, la formula anterior queda:
Z 
1
y (x) = (x) Q (x) dx + C
(x)
Ejemplo 1.3.7. Resolver la ecuacion lineal de primer orden:

xy 0 + (1 x) y = e2x

23
Apuntes Mat023 (Segundo semestre 2014)

Solucion. Supongamos que x 6= 0. La ecuacion anterior queda como:

e2x
 
0 1
y + 1 y =
x x

Utilizando las notaciones del teorema anterior, tenemos que P (x) = 1/x1 y Q (x) = e2x /x.
Calculamos, primeramente, el factor integrante:
1
(x) = e ( x 1)dx
R

= |x| ex

Luego:
Z 
1
y (x) = (x) Q (x) dx + C
(x)
ex 2x
Z 
x e
= |x| e dx + C
|x| x

Si x > 0, se tiene que |x| = x y la solucion es:

ex
Z 
x
y = e dx + C
x
e2x ex
= +C
x x
Por otro lado, si x < 0, se tiene que |x| = x y la solucion es:

ex
 Z 
x
y = e dx + C
x
e2x ex
= C
x x
Ahora bien, como C es una constante arbitraria, si x 6= 0 se puede escribir:

e2x ex
y= +C
x x
Ejemplo 1.3.8. Deternine una funcion f : D R R tal que
Z t
f (t) = t + et
eu f (u) du + tf (t)
0

Solucion. Notemos que f (0) = 0 y


Z t
t
(f (t) t tf (t)) e = eu f (u) du
0

24
Apuntes Mat023 (Segundo semestre 2014)

usando el teorema fundamental del calculo

(f 0 (t) 1 f (t) tf 0 (t)) et (f (t) t tf (t)) et = et f (t)

se sigue
f 0 (t) 1 f (t) tf 0 (t) f (t) + t + tf (t) = f (t)
as
(1 t) f 0 (t) (3 t) f (t) = 1 t
es una EDO de primer orden lineal
df 3t
f =1
dt 1t
el factor de integracion es
3t
R
dt
(t) = e 1t = e2 ln(t1)t
= (t 1)2 et

se sigue
d
(t 1)2 et f = (t 1)2 et

dt
integrando Z
2 t
(t 1) e f = (t 1)2 et dt

es decir
(t 1)2 et f = et t2 + 1 + C


de donde
et (t2 + 1) + C
f (t) =
(t 1)2 et
como f (0) = 0 se sigue
0 = 1 + C C = 1
finalmente
et (t2 + 1) + 1
f (t) =
(t 1)2 et
Ejemplo 1.3.9. Resolver la ecuacion
dy
2xy = x
dx

25
Apuntes Mat023 (Segundo semestre 2014)

2
R
2xdx
Solucion. Es una ecuacion lineal, el factor integrante es (x) = e = ex , multipli-
cando la ecuacion por (x) se tiene
2 dy 2 2
ex 2xex y = xex
dx
luego
d  x2  2
e y = xex
dx
integrando
Z
x2 2
e y = xex dx
1 2
= ex + C
2
as
1 2
y (x) = + Cex
2
Ejemplo 1.3.10. Considere el problema de valor inicial:

(1 + x2 )y 0 + 2xy = f (x), y(0) = 0.

en donde: 
x, si x [0, 1),
f (x) =
x, si x 1.
Hallar, si acaso existe, una solucion continua para este problema.

Solucion. Por definicion de f , notamos primeramente que el dominio de la ecuacion


diferencial es el intervalo [0, +). Entonces, si x [0, 1) tenemos el problema de valor
inicial siguiente:
(1 + x2 )y 0 + 2xy = x, y(0) = 0 (1.17)
Entonces:
2x x
y0 + 2
y=
1+x 1 + x2
As:
Z
1  2 x 
y = (1 + x ) dx + C
1 + x2 1 + x2
1  x2 
= +C
1 + x2 2
Ahora bien, como y(0) = 0, obtenemos que C = 0. Por tanto, la unica solucion del problema
de valor inicial (1.17) esta dada por:
x2
y=
2(1 + x2 )

26
Apuntes Mat023 (Segundo semestre 2014)

Por otro lado, si x [1, +) tenemos la ecuacion diferencial:

(1 + x2 )y 0 + 2xy = x

O bien:
2x x
y0 + 2
y= (1.18)
1+x 1 + x2
cuya solucion general esta dada por:

1  x2 
y= +Q (1.19)
1 + x2 2
As, para que exista una solucion continua, se debe cumplir que:

lm y[0,1) (x) = y[1,+) (1)


x1

donde y[0,1) denota la solucion para 0 x < 1 e y[1,+) denota la solucion para x 1.
Entonces, como:
x2 1
lm y[0,1) (x) = lm 2
=
x1 x1 2(1 + x ) 4
tenemos que elegir el valor de Q en la familia de funciones (1.19) tal que y[1,+) (1) = 14 .
Entonces, Q = 1. Por tanto, el problema:

(1 + x2 )y 0 + 2xy = f (x), y(0) = 0.

en donde: 
x, si x [0, 1),
f (x) =
x, si x 1.
tiene una solucion continua y = y(x) y esta dada por:

x2

, 0x<1


2(1 + x2 )

y(x) =  x2
1

+ 1 , x 1.
1 + x2 2
Ejemplo 1.3.11. Una curva continua y = f (x) en el primer cuadrante parte desde el
origen y tiene la propiedad de que el area debajo del cuadrado de dicha curva, desde el
origen hasta un punto (x, y) en ella, es igual al cubo de la abscisa de (x, y) mas el cuadrado
del area del rectangulo que tiene a los puntos (0, 0) y (x, y) como vertices opuestos. Hallar
una ecuacion, como solucion de una ecuacion diferencial de primer orden, que defina
implcitamente la curva y = f (x). Z
1
Indicacion: No intente calcular la integral x2 e x dx.

27
Apuntes Mat023 (Segundo semestre 2014)

Solucion. Sean C : y = f (x) una curva continua en el primer cuadrante y (x, y) C.


Sabemos que el area bajo el cuadrado de la curva C : y = f (x) desde el origen (0, 0) hasta
Rx
el punto (x, y), esta dada por la integral: 0 y(t)2 dt. As, la ecuacion que define la curva C
es la ecuacion: Z x
y(t)2 dt = x3 + (xy)2 (1.20)
0

con x > 0, en donde la expresion (xy)2 representa el cuadrado del area del rectangulo
que tiene a los puntos (0, 0) y (x, y) como vertices opuestos. Como y = f (x) es continua,
entonces, por el Teorema Fundamental del Calculo, la integral indefinida del lado izquierdo
es derivable. Luego, derivando la ecuacion (1.20) respecto de x, obtenemos:

y 2 = 3x2 + 2(xy) y + xy 0 ,

x>0

Esto es:
y 2 = 3x2 + 2xy 2 + 2x2 yy 0 , x>0 (1.21)
Ahora, sea u = y 2 , entonces u0 = 2yy 0 . Reemplazando en (1.21), tenemos:

u = 3x2 + 2xu + x2 u0 , x>0

Reordenando la ecuacion, obtenemos la ecuacion diferencial lineal de primer orden siguiente:


 
0 2 1
u + u = 3, x>0 (1.22)
x x2
Ahora bien, note que: Z  
2 1 1
2 dx = 2 ln x + +Q
x x x
Luego, el factor integrante (x) de (1.22), esta dado por:
2 1
e ( x x2 ) dx = x2 e x
1
R

Entonces, por la formula de Leibniz:


 Z 
1
u(x) = 3 (x) dx + C
(x)
en donde C R, se obtiene que:
1
e x
 Z 
2 x1
u(x) = 2 C 3 x e dx
x

Sin embargo, u = y 2 , por lo tanto, la ecuacion que define implcitamente la curva C : y =


f (x) esta dada por:
1 1 Z
2 e x e x 1
y =C 2 3 2 x2 e x dx
x x

28
Apuntes Mat023 (Segundo semestre 2014)

Ecuacion de Bernoulli
Observacion 1.3.9. Numerosas aplicaciones pueden ser modeladas con ecuaciones di-
ferenciales ordinarias que no son lineales, sin embargo, mediante cambios de variables
adecuados y algo de manipulacion algebraica, estas ecuaciones pueden ser transformadas
en ecuaciones diferenciales lineales. Un caso importante es la llamada ecuacion de Bernoulli.

Definicion 1.3.5. Sean P (x) y Q (x) funciones continuas sobre un intervalo abierto I R.
Una ecuacion diferencial de la forma:

y 0 + P (x) y = Q (x) y , 6= 1 (1.23)

se llama ecuacion de Bernoulli.

Observacion 1.3.10. Multipliquemos la ecuacion de Bernoulli en (1.23) por y , de donde


obtenemos:
y y 0 + P (x) y 1 = Q (x) (1.24)
Sea z = y 1 . Luego, por la regla de la cadena, obtenemos:

z 0 = (1 ) y y 0

Reemplazando en la ecuacion (1.24), se obtiene:


1
z 0 + P (x) z = Q (x)
1
O bien:
z 0 + (1 a) P (x) z = (1 ) Q (x) (1.25)
que es una ecuacion diferencial lineal de primer orden. Es importante notar que una vez
resuelta la ecuacion (1.25) se debe volver a la variable original y = y (x).

Ejemplo 1.3.12. Hallar la solucion de la ecuacion diferencial:


dy
+ xy = x3 y 3
dx
Solucion. Dividiendo todos los terminos por y 3 , tenemos:

y 3 y 0 + xy 2 = x3 (1.26)

Consideremos, tal como en la observacion anterior, el cambio de variables z = y 2 . Luego:


dz dy
= 2y 3
dx dx

29
Apuntes Mat023 (Segundo semestre 2014)

Reemplazando la ecuacion anterior en (1.26), se obtiene la ecuacion diferencial lineal:


dz
2xz = 2x3
dx
cuya solucion es:
2
z = 1 + x2 + Cex
Por consiguiente, la solucion general de la ecuacion diferencial dada es:
1
y=
1+ x2 + Cex2
con C una constante cualquiera.

Ejemplo 1.3.13 (Ecuacion logstica). Resuelva la siguiente ecuacion de Bernoulli, conocida


como ecuacion logstica:
dN
= N ( N ) (1.27)
dt
con , > 0.

Solucion. Note que podemos escribir la ecuacion (1.27) como:


dN
= N N 2
dt
Sean A = y B = , entonces:
dN
AN = BN 2
dt
Dividiendo la ecuacion anterior por N 2 , obtenemos:
dN
N 2 AN 1 = B (1.28)
dt
Sea z = N 1 , luego:
dz dN
= N 2
dt dt
Reemplazando en la ecuacion (1.28), obtenemos:
dz
Az = B
dt
Es decir:
dz
+ Az = B
dt
cuya solucion exacta es:
B
z = CeAt +
A
30
Apuntes Mat023 (Segundo semestre 2014)

1
Pero z = N
, luego:
1 1
N=
=
z CeAt + B/A
Finalmente, como A = y B = , obtenemos:

N (t) =
1 + Cet
Ejemplo 1.3.14. Se ha determinado experimentalmente que la variacion de peso de un
tipo de pez sigue la ley
dp
= e 3 t p2/3 p
dt
donde p = p (t) representa el peso del pez, y son constantes positivas. Si p (0) = p0 > 0
determine el peso maximo del pez.
Solucion. Se trata de una ecuacion de Bernoulli
dp
+ p = e 3 t p2/3
dt
dp 2/3
multiplicando por p2/3 se sigue dt
p + p1/3 = e 3 t hacemos el cambio u = p1/3
entonces du
dt
= 13 p2/3 dp
dt
as
du
3 + u = e 3 t
dt
du t
+ u = e 3
dt 3 3

resolvemos esta ecuacion lineal al multiplicar por el factor (t) = e 3 t se sigue
d  t
ue 3 =
dt 3 
u = t + C e 3 t
3
volvemos a la variable original
 3
p (t) = t + C et
3
como p (0) = p0 se tiene p0 = C 3 as
 3
p (t) = t + p0 et
3
3
para buscar el maximo derivamos
0
 2
t
 3
p (t) = t + p0 e
3
t + p0 et
3
3 3
 2
t
  
= t + p0 e
3
t + p0
3
3  3 
 2
t
= t + p0 e
3
p0
3
t
3 3

31
Apuntes Mat023 (Segundo semestre 2014)

as
3 p0
t=
3
note que antes de ese numero la derivada es positiva y despues negativa, el punto es punto
de maximo global, si este numero es negativo, el maximo se alcanza en t = 0 y corresponde
a p0 , en otro caso, el peso maximo es
!
3 p0
pmax = p
3
! !3 
3 p0

3 p0
=
+ 3
p0 e 3
3 3
 3

e3(1 p0 )
3
=

Ecuacion de Ricatti
Definicion 1.3.6. Una ecuacion de Ricatti es una ecuacion diferencial de la forma:

y 0 + P (x) y + Q (x) y 2 = R (x) (1.29)

donde P, Q y R son funciones continuas sobre un intervalo I R.

Observacion 1.3.11. Existe un metodo para obtener una familia de soluciones de una
ecuacion de Ricatti si se conoce una solucion particular u = u (x). Supongamos esto y
consideremos el cambio de variables:
1
y =u+ (1.30)
v
con v = v (x) la nueva variable. Derivando la ecuacion anterior, obtenemos:
1 0
y 0 = u0 v (1.31)
v2
Reemplazando las ecuaciones (1.30) y (1.31) en la ecuacion de Ricatti (1.29), se sigue que:
     2
0 1 0 1 1
u 2 v + P (x) u + + Q (x) u + = R (x)
v v v

Reordenando los terminos de la ecuacion anterior, tenemos que:


v0
 
 0 2 1 u 1
u + P (x) u + Q (x) u 2 + P (x) + Q (x) 2 + 2 = R (x)
v v v v

32
Apuntes Mat023 (Segundo semestre 2014)

Ahora bien, como u es una solucion particular de la ecuacion (1.29), obtenemos:


v0
 
1 u 1
2 + P (x) + Q (x) 2 + 2 = 0
v v v v
Amplificando la ecuacion anterior por v 2 , finalmente se obtiene la ecuacion:

v 0 P (x) v Q (x) u (x) v Q (x) = 0

Es decir:
v 0 {P (x) + u (x) Q (x)} v = Q (x)
que es una ecuacion lineal de primer orden.

Ejemplo 1.3.15. Resuelva la ecuacion de Ricatti:


2
y0 = y2 (1.32)
x2
Solucion. Notamos primeramente que u = x1 es una solucion particular de la ecuacion.
En efecto:
2 1 2
u2 2 = 2 2
x x x
1
= 2
x
0
= u
1
Ahora bien, sea y = x
+ v1 , luego:
dy 1 1 dv
= 2 2
dx x v dx
1
Reemplazando la ecuacion anterior y el cambio de variables y = x
+ v1 en la ecuacion (1.32),
obtenemos:  2
1 1 dv 1 1 2
2 2 = + 2
x v dx x v x
Simplificando y agrupando terminos semejantes, se tiene:
1 dv 2 1
2
= + 2
v dx vx v
Amplificando por v 2 , finalmente se obtiene:
2
v 0 + v = 1
x
Como se puede observar, la ecuacion anterior es una ecuacion lineal de primer orden cuya
solucion exacta es:
C x
v= 2
x 3
33
Apuntes Mat023 (Segundo semestre 2014)

1
Sin embargo, recordemos que y = x
+ v1 . Luego, la solucion obtenida es:

1 1
y= + C x
x x2
3

O bien:
1 3x2
y= +
x C x3
Ejemplo 1.3.16. Obtenga una solucion del problema
dy 1 1
= 2 y2 y + 1
dx x x
y (1) = 3

Indicacion:Primero buscar una solucion dela ecuacion de la forma y = ax + b.

Solucion. Usamos la indicacion para buscar una solucion (la ecuacion es de Ricatti)
entonces
x2 a = (ax + b)2 x (ax + b) + x2
se sigue
ax2 = a2 a + 1 x2 + (2ab b) x + b2


as
b=0
y
a = a2 + 1 a
que tiene solucion a = 1, se sigue que una solucion es

y=x

(la cual no cumple y (1) = 3) hacemos el cambio de variables


1
y= +x
u
entonces  2  
1 du 1 1 1 1
2 +1= 2 +x +x +1
u dx x u x u
as
1 du 1 1
2
+1= + 2 2 +1
u dx ux u x
eliminando
1 du 1 1
2
= + 2 2
u dx ux u x

34
Apuntes Mat023 (Segundo semestre 2014)

luego
du u 1
= 2
dx x x
es lineal y tiene solucion
C 1
u= ln x
x x
1
as, como y = u
+ x se sigue
1
y= C
+x
x
x1 ln x
pero y (1) = 3 as
1
3= +1
C
1
luego C = 2
reemplazando
1
y= 1 +x
2x
x1 ln x
es la solucion del problema.

Ejemplo 1.3.17. Resolver la ecuacion

y 0 xy 2 + (2x 1) y = x 1

si se sabe que tiene una solucion constante.

Solucion. La solucion constante es y = 1 (verificar) entonces podemos hacer el cambio


1
y= +1
z
dy dz
= z 2
dx dx
reemplazamos
 2 
2 dz 1 1
z x +1 + (2x 1) +1 x+1 = 0
dx z z
dz 1
z 2 2 (x + z) = 0
dx z
as
dz
+ (x + z) = 0
dx
la cual es lineal, la solucion es z = Aex x + 1 as
1
y= +1
Aex x+1

35
Apuntes Mat023 (Segundo semestre 2014)

Ecuaciones homogeneas

Definicion 1.3.7. Una funcion f : U R2 R se dice homogenea de grado n si:

f (tx, ty) = tn f (x, y)

para todo t R tal que (tx, ty) U .

Observacion 1.3.12. Si M y N son funciones homogeneas de grado n, entonces la


ecuacion:
M (x, y) dx + N (x, y) dy = 0 (1.33)
se escribe como:
M 1, xy

dy M (x, y) y
= = = F
N 1, xy

dx N (x, y) x
donde F : R R. Luego, la ecuacion (1.33) puede escribirse como:
y
y0 = F
x
Definicion 1.3.8. La ecuacion de primer orden:

y 0 = f (x, y)

se llama homogenea, si la funcion f (x, y) es homogenea de grado 0.

Observacion 1.3.13. En vista de la observacion anterior, toda ecuacion homogenea puede


escribirse en la forma: y
0
y =F (1.34)
x
haciendo t = x1 . Para resolver esta ecuacion se considera el cambio de variables z = xy ; o
bien, y = zx. Luego, al derivar respecto de x tenemos que:
dy dz
=z+x
dx dx
Reemplazando la ecuacion anterior en la ecuacion (1.34), se obtiene:

dz
z+x = F (z)
dx
la cual es una ecuacion de variable separable. Finalmente, podemos escribir:
Z Z
dz dx
= +C
F (z) z x

con C una constante arbitraria.

36
Apuntes Mat023 (Segundo semestre 2014)

Ejemplo 1.3.18. Resuelva la ecuacion diferencial:


x+y
y0 =
xy
Solucion. Se trata de una ecuacion con funcion homogenea de grado cero pues
y
0 x+y 1+ x
y
y = = y =F
xy 1 x
x
y
ponemos u = x
luego u + xu0 = y 0 reemplazando

1+u
u + xu0 =
1u
luego
 
du 1 1+u
= u
dx x 1u
1 u2 + 1
 
=
x 1u

esta ecuacion es de variables separadas, se sigue


1u
Z Z
dx
2
du =
1+u x
luego
1
ln 1 + u2 = ln |x| + C

arctan u
2
volvemos a la variable
y   y 2 
1
arctan ln 1 + = ln |x| + C
x 2 x

Ejemplo 1.3.19. Resuelva el P.V.I.


dy 1/2
x = y + x2 y 2
dx
y (1) = 0

Solucion. Note que


dy y
  y 2 1/2
= + 1
dx x x
es homogenea, hacemos el cambio
y du dy
u= u+x =
x dx dx
37
Apuntes Mat023 (Segundo semestre 2014)

luego
du
u+x = u + 1 u2
dx
se sigue
du 1 u2
=
dx x
resolvemos esta ecuacion de variables separables
Z Z
du dx
=
1u 2 x
arcsin u = ln |x| + C

as
u = sin (ln |x| + C)
luego
y = x sin (ln |x| + C)
evaluando
0 = sin (ln |1| + C) = sin C
se sigue C = k con k Z, as

y = x sin (ln |x| + k) con k Z

Otros cambios de variables


Observacion 1.3.14. La regla de la cadena nos permite cambiar variables en las ecuaciones
diferenciales para llevarlas a ecuaciones diferenciales que se resuelven mediante los metodos
elementales. Consideremos los siguientes ejemplos:

Ejemplo 1.3.20. Recordemos la ecuacion (1.7) obtenida en el problema del espejo pa-
rabolico. Es decir, la ecuacion:
y 2y 0
=
x 1 (y 0 )2
Utilizando un cambio de variables adecuado, resolveremos esta ecuacion diferencial. En
primer lugar, despejando y 0 de la ecuacion, se obtiene:
2
y (y 0 ) + 2xy 0 y = 0

Por la formula de la ecuacion de segundo grado, encontramos que:


p
x x2 + y 2
y0 =
y

38
Apuntes Mat023 (Segundo semestre 2014)

p
Por la simetra de la curva y = f (x) y el hecho que |x| < x2 + y 2 , se obtiene que y 0 > 0.
Luego: p
x + x2 + y 2
y0 = (1.35)
y
Consideremos, ahora, el cambio de variables z = x2 + y 2 . Entonces:
dz dy
= 2x + 2y
dx dx
Reemplazando la ecuacion anterior en la ecuacion (1.35), se obtiene:
p !
dz x + x2 + y 2
= 2x + 2y
dx y
p
= 2x 2x + 2 x2 + y 2

Es decir, obtenemos la ecuacion de variable separable:


dz
=2 z
dx
As: Z Z
dz
= dx + C
2 z
implica que:

z =x+C
Por tanto, elevando al cuadrado y como z = x2 + y 2 , se obtiene finalmente:

x2 + y 2 = x2 + 2Cx + C 2

O bien:
y 2 = 2Cx + C 2
con C R.

Ejemplo 1.3.21. Resuelva la ecuacion diferencial:


dy
= (x + y + 1)2 2
dx
utilizando para ello un cambio de variables adecuado.

Solucion. Haciendo z = x + y + 1, tenemos z 0 = 1 + y 0 , de modo que:

z0 1 = z2 2

o bien:
z0 = z2 1

39
Apuntes Mat023 (Segundo semestre 2014)

Separando variables, obtenemos:


Z Z
dz
2
= dx + C
z 1
Integrando se tiene que:
1
{ln (z 1) ln (z + 1)} = x + C
2
o:
z1
ln = 2x + C
z+1
Luego:
z1
= e2x+C
z+1
Es decir, obtenemos:
z 1 = Ce2x (z + 1)
Despejando z y recordando que z = x + y + 1, se obtiene finalmente:

1 + Ce2x
y= x1
1 Ce2x
Ejemplo 1.3.22. Para un valor de n N adecuado, el cambio de variables u = xn y
transforma la ecuacion diferencial
dy p
x2 + 2xy = 1 y 2 x4 x2
dx
en una ecuacion de variables separables. Determine tal valor de n y resolver la ecuacion.

Solucion. Note que


y = xn u
luego
dy du
= nxn1 u + xn
dx dx
reemplazando
 
n du
 q
2
x nxn1
u+x + 2x x u = 1 (xn u)2 x4 x2
n
dx

ordenando
du  p
nxn+1 u + x2n + 2 xn+1 u = 1 (x42n u2 )x2
dx
se sigue
du  p
x2n + (2 n) xn+1 u = 1 (x42n u2 )x2
dx

40
Apuntes Mat023 (Segundo semestre 2014)

tomando n = 2 obtenemos
du
= 1 u 2 x2
dx
que es de variables separables, se sigue
Z Z
du
= x2 dx
1u 2

as
x3
arcsin u = +C
  3
x3
luego u = sin 3
+ C y finalmente
 
x3
sin 3
+C
y=
x2
Ejemplo 1.3.23. Resolver la ecuacion
2
yy 00 = y 0 y 2 + (y 0 )
haciendo el cambio u = y 0 .
Solucion. Si u = y 0 entonces
du dy
y 00 =
dy dx
du
= u
dy
reemplazando
du
yu = uy 2 + (u)2
dy
du
y = y2 + u
dy
esta ecuacion es lineal
du 1
u=y
dy y
que tiene solucion
u = y 2 + cy
luego
dy
= y 2 + cy
dx
es de variables separadas, tiene solucion
c
y (x) = cx
Ae 1
Obs.: Esta tecnica reduce el orden de la E.D.O. si es de la forma F (y, y 0 , y 00 ) es decir,
no depende de la variable x.

41
Apuntes Mat023 (Segundo semestre 2014)

Ejemplo 1.3.24. Resuelva la ecuacion diferencial:


2
3xy 2 y 0 + xex + y 3 = 0

Solucion. Considere el cambio de variables u = y 3 . Luego, u0 = 3y 2 y 0 , entonces la ecuacion:


2
3xy 2 y 0 + xex + y 3 = 0

queda como:
2
xu0 + xex + u = 0
la cual es lineal. As, si x 6= 0, la ecuacion queda como:
1 2
u0 + u = ex
x
Denotando por = (x) el factor de integracion de la ecuacion anterior, tenemos que:
1
R
(x) = e x
dx
= eln |x| = |x|

Por tanto, si x > 0 la solucion u esta dada por:


Z
1  x2

u = (x)e dx + C
(x)
Z
1  2

= |x|ex dx + C
|x|
Z
1 2

= xex dx + C
x
2
C ex
=
x 2x
Por otro lado, si x < 0, entonces:
Z
1  2

u = |x|ex dx + C
|x|
Z
1 2

= xex dx + C
x
2
1  ex 
= +C
x 2
2
C ex
=
x 2x
Por tanto, si x 6= 0 tenemos que:
2
C ex
u=
x 2x

42
Apuntes Mat023 (Segundo semestre 2014)

con C R. Finalmente, como u = y 3 , obtenemos que la solucion de la ecuacion diferencial


esta dada por:
2
3 C ex
y =
x 2x
r
3 C e x2
y =
x 2x
con C una constante arbitraria.

Observacion 1.3.15. Como muestran los ejemplos anteriores, en general, se puede efectuar
cualquier cambio de variables o sustitucion que se desee al intentar resolver una ecuacion
diferencial. Sin embargo, cualquier no asegura el exito en facilitar la resolucion de la
ecuacion diferencial. Por tanto, debe buscarse el cambio de variables adecuado, [4].

Ejercicios de la seccion

1. Resuelva las siguientes ecuaciones mediante integracion directa


 dy dy
(a) x2 + 4 =4 (b) = x ln x
dx dx
1 1 dy
(c) y0 = 2
(d) ex = sin x
arctan x (1 + x ) dx
2. Resuelva las siguientes ecuaciones de variables separables:
(a) yy 0 x = xy 2 (b) y 0 = ex+y
(c) y ln y + xy 0 = 0 (d) (1 + ex ) y 0 = ey
(e) y 0 = 1 + x + y + xy (f) y 0 = x2 y 2 + x2

3. Verifique que las siguientes ecuaciones son homogeneas y resuelvalas:


dy dy p
(a) 3x y + (2y x) = 0 (b) x = y + y 2 x2
dx dx
2 2 dy 2 2
 dy
(d) 3x2 y 2

(c) 4x + xy 3y + 5x + 2xy + y = 0 = 2xy
dx dx
dy x+y dy
(e) = (f) x2 y = x3 + y 3
dx xy dx
4. Resuelva las siguientes ecuaciones lineales de primer orden:
(a) xy 0 + y = xex (b) y 0 + y cos (x) = sin (x) cos (x)
(c) y 0 + 2y = x2 + 2x (d) y 0 + 2y = sin x
1 1
1 + x2 y 0 + xy = 1 (f) y 0 + y = 2

(e)
x x

43
Apuntes Mat023 (Segundo semestre 2014)

5. Resolver las siguientes ecuaciones de Bernoulli:


1.- y 0 4y = 2ex y 1/2 3.- xy 0 2y = 4x3 y 1/2
2.- y 0 y + y 2 (x2 + x + 1) = 0 4.- xy 0 + y = y 2 x2 ln x

6. Las ecuaciones en este ejercicios pueden ser transformadas en lineales mediante un


cambio de variables adecuado, descubra tal cambio y resuelva la ecuacion:

a) y 0 + x tan y = x2 sec y
b) 2xyy 0 + (1 + x) y 2 = ex

7. Sean A, B, C, D, E, F constantes. Muestre que utilizando un cambio de coordenadas


conveniente podemos transformar una ecuacion del tipo
 
dx Ax + Bt + C
=f
dt Dx + Et + F

en una ecuacion de variables separables o en una homogenea. Ind.: x = X+h, y = Y +k


donde h, k son constantes por determinar.

8. Resolver el P.V.I.
dx 2x + t 1
=
dt x + 2t + 1
x (0) = 1

9. Resuelva la ecuacion de Ricatti:

y 0 2x3 + 1 y = x2 y 2 x4 x + 1


sabiendo que u (x) = x es una solucion particular de la ecuacion.

10. Si u (x) = x2 es una solucion particular de la ecuacion:

y 0 + 3 2x2 sin x y = y 2 sin x + 2x + 3x2 x4 sin x




Hallar una familia infinita de soluciones.

Modelos simples: Segunda parte

Observacion 1.4.1. Ahora consideraremos algunos ejemplos mas: el problema de mezclas,


o en terminos mas generales, el problema de analisis de compartimientos, [4], el problema
de las curvas de persecucion y un problema geometrico.

44
Apuntes Mat023 (Segundo semestre 2014)

Observacion 1.4.2 (Analisis de compartimientos, [4]). Un proceso fsico o biologico com-


plejo puede ser dividido algunas veces en varios estados distintos. El proceso total puede
describirse por la interaccion entre los estados individuales. Cada estado se llama comparti-
miento (lo podemos considerar como un tanque) y se supone ademas que el contenido de
cada compartimiento esta mezclado homogeneamente. En cada compartimiento se transfiere
material que es inmediatamente incorporado al siguiente en el sistema.
Considere un sistema formado por un solo compartimiento, suponga que un material es
introducido en tal compartimiento a una razon e (t), el cual se incorpora a una cantidad
x (t) de material existente al interior del compartimiento, y luego se extrae material (que
puede pasar a otro compartimiento) a una razon de s (t). Por consiguiente, la variacion de
material al interior del compartimiento esta dada entonces por la ecuacion diferencial:
dx
= e (t) s (t)
dt
Consideremos algunos ejemplos:

Ejemplo 1.4.1. Considere un tanque que contiene 100 litros de agua, en el cual se han
disuelto 50 kilogramos de sal. Suponga que 2 litros de salmuera cada uno con 1 kilogramo
de sal disuelta, entran por minuto al tanque, y la mezcla que se mantiene homogenea
revolviendola a gran velocidad, sale del tanque a razon de 2 litros por minuto. Hallar la
cantidad de sal al interior del tanque en el tiempo t.

Solucion. Sea x (t) el numero de kilogramos de sal disueltos en el tanque en t minutos.


Notamos que las unidades ayudan bastante en la extraccion de informacion. En efecto,
dx/dt esta en [kg/ mn] y entonces, e (t) y s (t) deben esta en las mismas unidades. As:
    
kg lt kg
e (t) = 2 =2
lt mn mn

45
Apuntes Mat023 (Segundo semestre 2014)

y      
lt x (t) kg x (t) kg
s (t) = 2 =
mn 100 lt 50 mn
Por tanto, la ecuacion diferencial queda:
dx x (t)
=2
dt 50
la cual es una ecuacion lineal de primer orden. As, por la formula de Leibnitz, tenemos
que:
 Z 
t/50 t/50
x (t) = e 2 e dt + C

= 100 + Cet/50

pero sabemos que en t = 0, x (0) = 50. As, 50 = 100 + C. Por tanto:

x (t) = 100 50et/50

Observacion 1.4.3 (Curvas de persecucion, [4]). Utilizando la propiedad geometrica de


que la pendiente de la recta tangente a una curva y en un punto dado de la curva es y 0 , se
pueden construir ecuaciones diferenciales que permiten estudiar la trayectoria que describe
un depredador tras su presa.

Consideremos algunos ejemplos:

Ejemplo 1.4.2. Un esquiador acuatico P localizado en el punto (a, 0), con a > 0, es
halado por un bote de motor Q localizado en el origen y que viaja hacia arriba a lo largo
del eje Y . Hallar la trayectoria del esquiador si este se dirige en todo momento haca el
bote. La trayectoria se denomina tractriz.

Solucion. Observemos que la recta que une P y Q, digamos P Q es tangente al camino
recorrido por P . Por tanto, su pendiente esta dada por:

dy a2 x 2
= (1.36)
dx x
46
Apuntes Mat023 (Segundo semestre 2014)

puesto que la longitud del segmento P Q es a. La ecuacion diferencial (1.36) es de integracion


directa, luego: Z 2
a x2
y= dx + C
x
As1 :

 
a+ a2 x 2
y = a ln a2 x 2 + C
x
Como y = 0 cuando x = a, vemos que C = 0, de modo que la ecuacion de la trayectoria es:


 
a + a2 x 2
y = a ln a2 x 2
x

Ejemplo 1.4.3. Suponga que un halcon P situado en el punto (a, 0) descubre una paloma
Q en el origen, la cual vuela a lo largo del eje Y a una velocidad v. El halcon emprende el
vuelo inmediatamente hacia la paloma a una velocidad w. Cual sera el camino seguido
por el halcon en su vuelo?

Solucion. Sea t = 0 el instante en que el halcon comienza a volar hacia la paloma. Despues
de t segundos la paloma estara en el punto Q = (0, vt) y el halcon en P (x, y). Como la

recta T = P Q es otra vez tangente a la trayectoria, encontramos que su pendiente mT = y 0 .
Luego, se obtiene la ecuacion diferencial:
y vt
y0 = (1.37)
x
Debemos ahora eliminar t de la ecuacion anterior, pues y 0 = dy/dx. Para ello debemos
calcular la longitud del camino recorrido por el halcon. Si ds representa un elemento
diferencial de longitud del arco formado por la trayectoria, tenemos que:
Z a
wt = ds
x
q
pero ds = 1 + (y 0 )2 dx. As, la formula anterior queda como:
Z a
1
q
t= 1 + (y 0 )2 dx
w x

Despejando t de la ecuacion (1.37) e igualando con la ecuacion anterior, obtenemos:

y xy 0 1 a
Z q
= 1 + (y 0 )2 dx
v w x
Z a + a2 x2

1 a2 x2 p
dx = a2 x2 a ln +C

x x

47
Apuntes Mat023 (Segundo semestre 2014)

Derivando:
y 0 (y 0 + xy 00 ) 1
q
= 1 + (y 0 )2
v w
Ordenando la ecuacion anterior, se tiene que:
v
q
xy =00
1 + (y 0 )2 (1.38)
w
Note que la ecuacion anterior es una ecuacion diferencial de segundo orden. Sin embargo,
mediante el cambio de variables:
u = y0
la ecuacion (1.38) queda como:
v
xu0 = 1 + u2
w
que es una ecuacion de primer orden de variable separable. Entonces, separando variables
e integrando, obtenemos: Z Z
du v dx
= +C
1+u 2 w x
Luego:   v
2
ln u + 1 + u = ln x + K
w
0
pero u = y = 0 cuando x = a, se sigue que K = (v/w) ln a. Tomando exponenciales a
ambos lados de la ecuacion anterior se tiene:
 x v/w
u + 1 + u2 =
a
que, despues de algunas operaciones algebraicas, nos da:
 
dy 1  x v/w  x v/w
=
dx 2 a a

Suponiendo que w > v, se obtiene finalmente que:


( )
a (x/a)1+v/w (x/a)1v/w
y= +C
2 1 + v/w 1 v/w

Ejemplo 1.4.4. Determine la curva que pasa por 21 , 32 y corta a cada miembro de la


familia x2 + y 2 = c2 con c R+ formando un angulo de 45o .

Solucion. El angulo entre dos curvas es dado por el angulo entre sus rectas tangentes
luego
m1 m2
tan =
1 + m1 m2

48
Apuntes Mat023 (Segundo semestre 2014)

note que la pendiente de las rectas tangentes a la familia de curvas es


x
2x + 2yy 0 = 0 m1 =
y
as x
 
y
y0
tan =
4 1 xy y 0
esto es x
y
y0
1=
1 xy y 0
se sigue
x x
1 y0 = y0
y y

x x 0
1+ = y y0
y y
luego
dy 1 + xy y
+1
= x = x y
dx y
1 1 x
y
que es una ecuacion homogenea , hacemos el cambio u = x
de donde

du u+1
u+x =
dx 1u
luego
du u+1
x = u
dx 1u

u2 + 1 1

du
=
dx 1u x

resolvemos
1u
Z Z
1
du = dx
1 + u2 x
se sigue
1
ln 1 + u2 = ln |x| + C

arctan u
2
volvemos a la variable y,
y 1
ln x2 + y 2 = C

arctan
x 2

49
Apuntes Mat023 (Segundo semestre 2014)

1 3

y determinamos la constante con el punto ,
2 2
 
1 5
arctan (3) ln =C
2 2

as la curva es y  
1 1 5
ln x2 + y 2 = arctan 3 ln

arctan
x 2 2 2
Ejemplo 1.4.5. Hallar una curva C : y = f (x), con x > 0, que pase por el punto
(x0 , y0 ) = (1, 1) y que tenga la propiedad de que el segmento sobre la recta tangente a dicha
curva, trazado entre un punto de tangencia P (x, y) y un punto en el eje y, es bisectado
por el eje x.

Solucion. Considere el diagrama siguiente, asociado al problema geometrico:

y
y = f (x)
y P (x, y)

PM B x

x x
2

De la figura e hipotesis del problema, notamos que el punto PM es el punto medio del
segmento P Q, donde P = P (x, y) es el punto de tangencia a la curva incognita y = f (x).
Por tanto, del triangulo 4PM BP rectangulo en B, se obtiene que:
y
tan = x
2

Por otro lado, si denotamos por T la recta tangente a la curva y = f (x), entonces P Q T .
As:
y 0 = mT = tan
Ahora bien, la curva C : y = f (x) debe pasar por (x0 , y0 ) = (1, 1). Por tanto, obtenemos
el problema de valor inicial:
2y
y0 = , y(1) = 1
x
Separando variables e integrando obtenemos:
Z Z
dy dx
=2 +C
y x

50
Apuntes Mat023 (Segundo semestre 2014)

Esto es:
ln |y| = 2 ln x + C
O bien:
y = Cx2
pero y(1) = 1, luego C = 1. Por tanto, la curva con la propiedad buscada en el problema
es:
C : y = x2

Ejemplo 1.4.6. Un paracaidista se deja caer desde un avion y despues de 5,5 sg. abre
2
el paracadas provocando una fuerza de resistencia del aire igual wv
256
[lbs.] , donde w es
el peso total del hombre y del paracadas y v la velocidad con que va cayendo. Hallar la
velocidad en cualquier momento despues de abierto el paracadas. Asumir que antes de
abierto el paracadas no hay resistencia del aire.

Solucion. Primero modelamos la situacion antes de abrir el paracadas:


dv
m = mg v = gt v (5,5) = (32) (5,5) = 176
dt
ahora cuando se abre el paracadas ponemos t = 0 nuevamente pero con una velocidad
inicial
dv mgv 2
m = mg
dt 256
v (0) = 176

as
dv 32v 2
= 32
dt 256
la cual tiene solucion
16C + 16e4t
v (t) =
C e4t
reemplazando la C.I.
16C + 16
176 =
C 1
5
se obtiene C = 6
as
5

16 6
+ 16e4t
v (t) = 5
6
e4t
es decir
6e4t + 5
 
v (t) = 16
6e4t 5

51
Apuntes Mat023 (Segundo semestre 2014)

Ejemplo 1.4.7. Suponga que un tanque cilndrico recto con radio de la base 12 metro
y altura 4 metros tiene inicialmente 2 litros de agua pura. Una solucion de salmuera se
1

bombea hacia el tanque a una rapidez de 1 + 1+t litros por minuto, la concentracion de
1
sal en el flujo de entrada es de 2 kilogramo por litro. La solucion en el tanque es homogenea
1
y se extrae a 1+t litros por minuto. Determinar la cantidad de sal en el tanque cuando este
se llena.

Solucion. Sea x (t) la cantidad de sal en el tanque en minuto t entonces


dx
= (Sal que entra por minuto) - (Sal que sale por minuto)
dt
notemos que
    
1 lt 1 kg
Sal que entra por minuto = 1+
1+t min 2 lt
   
1 1 kg
= 1+
2 1+t min
y
 
  
1 kg lt x (t)
Sal que sale por minuto =
1+t lt min 2+t
   
1 kg
= x (t)
(1 + t) (2 + t) min
se sigue
   
dx 1 1 1
= 1+ x
dt 2 1+t (1 + t) (2 + t)
x (0) = 0

se trata de un problema de valores iniciales con E.D.O lineal de primer orden.


Resolvemos aplicando la tecnica del factor integrante
R dt t+1
(t) = e (1+t)(2+t) =
t+2
as     
d t+1 1 1 t+1 1
x = 1+ =
dt t+2 2 1+t t+2 2
se sigue
t+1 t
x= +K
t+2 2
as    
t t+2 t+2
x (t) = +K
2 t+1 t+1

52
Apuntes Mat023 (Segundo semestre 2014)

como x (0) = 0 se sigue


K=0
y as   
t t+2
x (t) =
2 t+1
es la cantidad de sal en el tanque en el tiempo t. La cantidad de litros de agua en el tanque
es
l (t) = 2 + t
la cantidad de litros de capacidad del tanque es
 2
1
V = 4 metros3
2
= m3
= 103 litros

los cuales se logran a


t = 103 2 minutos
y en la cantidad de sal es
103 2 103
  
3

x 10 2 =
2 103 1
1570. 3 kg

Ejercicios de la seccion

1. Determinar la curva que pasa por 21 , 32 y corta a cada miembro de la familia de




curvas x2 + y 2 = c2 formando un angulo de 30o .

2. Encuentre la curva que pertenece a la familia de trayectorias ortogonales de la familia


de curvas x + y = cey que pasa por (0, 5).

3. Suponga que un halcon situado en (a, 0) descubre una paloma en el origen, la


cual vuela a lo largo del eje Y a una velocidad v; El halcon emprende el vuelo
inmediatamente hacia la paloma con una velocidad de w. Cual es el camino seguido
por el halcon en su vuelo persecutorio?.

4. Un destructor esta en medio de una niebla muy densa que se levanta por un momento
y deja ver un submarino enemigo en la superficie a cuatro kilometros de distancia.
Suponga:

a) que el submarino se sumerge inmediatamente y avanza a toda maquina en una


direccion desconocida.

53
Apuntes Mat023 (Segundo semestre 2014)

b) que el destructor viaja tres kilometros en lnea recta hacia el submarino.

Que trayectoria debera seguir el destructor para estar seguro que pasara directamente
sobre el submarino, si su velocidad v es tres veces la del submarino?

5. Suponga que el eje Y y la recta x = b forman las orillas de un ro cuya corriente tiene
una velocidad v en la direccion negativa del eje Y . Un hombre esta en el origen y
su perro esta en el punto (b, 0). Cuando el hombre llama al perro, este se lanza al
ro y nada hacia el hombre a una velocidad constante w (con w > v). Cual es la
trayectoria seguida por el perro?.

6. Cuatro caracoles situados en las esquinas de un cuadrado [0, a] [0, a] comienzan a


moverse con la misma velocidad, dirigiendose cada uno hacia el caracol situado a su
derecha. Que distancia recorreran los caracoles al encontrarse?

7. Hallar la ecuacion de todas las curvas que tienen la propiedad de que el punto de
tangencia es punto medio del segmento tangente entre los ejes coordenados.

8. Un cuerpo se calienta a 110o C y se expone al aire libre a una temperatura de 100 C.


Si al cabo de una hora su temperatura es de 60o C. Cuanto tiempo adicional debe
transcurrir para que se enfre a 30o C?

9. Una persona de un pueblo de 1000 habitantes regreso con gripe. Si se supone que la
gripe se propaga con una rapidez directamente proporcional al numero de agripados
como tambien al numero de no agripados. Determinar el numero de agripados cinco
das despues, si se observa que el numero de agripados el primer da es 100.

10. Un colorante solido disuelto en un lquido no volatil, entra a un tanque a una velocidad
v1 galones de solucion/minuto y con una concentracion de c1 libras de colorante/galon
de solucion. La solucion bien homogeneizada sale del tanque a una velocidad de v2
galones de solucion/min. y entra a un segundo tanque del cual sale posteriormente a
una velocidad de v3 galones de solucion/min.

54
Apuntes Mat023 (Segundo semestre 2014)

Inicialmente el primer tanque tena P1 libras de colorante disueltas en Q1 galones


de solucion y el segundo tanque P2 libras de colorante disueltas en Q2 galones de
solucion. Encontrar dos ecuaciones que determinen las libras de colorante presentes
en cada tanque en cualquier tiempo t.

11. Un teatro de dimensiones 10 30 50m3 , contiene al salir el publico 0,1 % por


volumen de CO2 . Se sopla aire fresco a razon de 500 m3 por minuto y el sistema
de aire acondicionado lo extrae a la misma velocidad. Si el aire atmosferico tiene
un contenido de CO2 del 0,04 % por volumen y el lmite saludable es de 0,05 % por
volumen. En que tiempo podra entrar el publico?.

12. Un tanque contiene inicialmente agua pura. Salmuera que contiene 2 libras de sal/gal.
entra al tanque a una velocidad de 4 gal./min. Asumiendo la mezcla uniforme, la
salmuera sale a una velocidad de 3 gal./min. Si la concentracion alcanza el 90 % de
su valor maximo en 30 minutos, calcular los galones de agua que haban inicialmente
en el tanque.

13. Un tanque de una cierta forma geometrica esta inicialmente lleno de agua hasta una
altura H. El tanque tiene un orificio en el fondo cuya area es A pie2 . Se abre el orificio
y el lquido cae libremente. La razon volumetrica de salida dQdt
es proporcional a la
velocidad de salida y al area del orificio, es decir,
dQ
= kAv
dt

aplicando la ecuacion de energa 12 mv 2 = mgh se obtiene v = 2gh donde g = 32
pie/seg2 .
La constante k depende de la forma del orificio:

a) Si el orificio es de forma rectangular, la constante k = 0, 8.


b) Si el orificio es de forma triangular, la constante 0, 65 k 0, 75.
c) Si el orificio es de forma circular, la constante k = 0, 6.

Con estos datos:

a) Un tanque semiesferico tiene un radio de 1 pie; el tanque esta inicialmente lleno


de agua y en el fondo tiene un orificio de 1 pulg. de diametro. Calcular el tiempo
de vaciado.
b) Modelar el caso: Cilindro circular de altura H0 pies y radio r pies, dispuesto en
forma vertical y con un orificio circular de diametro (pulgadas), suponga que
esta lleno de agua y calcule el tiempo de vaciado.

55
Apuntes Mat023 (Segundo semestre 2014)

14. Un torpedo se desplaza a una velocidad de 60 millas/hora en el momento de agotarse


el combustible; si el agua se opone al movimiento con una fuerza proporcional a su
velocidad y si en una milla de recorrido reduce su velocidad a 30 millas/hora. A que
distancia se detendra?

15. Una bala se introduce en una tabla de h = 10 cm. de espesor con una velocidad v0 =
200 m/seg, traspasandola con v1 = 80 m/seg. Suponiendo que la resistencia de la
tabla al movimiento de la bala es proporcional al cuadrado de la velocidad. Hallar el
tiempo que demora la bala en atravesar la tabla.

16. Una cadena de 4 pies de longitud tiene 1 pie de longitud colgando del borde de una
mesa. Despreciando el rozamiento, hallar el tiempo que tarda la cadena en deslizarse
fuera de la mesa.

Analisis cualitativo

Es equivocado pensar que el objetivo principal del estudio de las ecuaciones diferenciales
consiste en encontrar artificios de calculo que permitan resolverlas. Anteriormente presenta-
mos una seleccion de tecnicas que permiten resolver algunas ecuaciones diferenciales. Como
en toda seleccion la lista no es completa. Existen tratados en donde se elaboran tablas de
soluciones de manera analoga a las tablas de antiderivadas.

La pericia para resolver ecuaciones diferenciales va perdiendo poco a poco importancia


con la llegada de los computadores y el diseno de software especializado para computacion
simbolica. La tendencia actual es dejar al computador este tipo de tareas de calculo. Un
programa como Mathematica puede resolver mediante instrucciones sencillas casi todas las
ecuaciones diferenciales tratadas en este curso.

Sin quitarle importancia a este tipo de programas debe quedar claro que ni el mas
refinado de los software ni el mas ingenioso de los matematicos puede resolver en terminos
de funciones elementales todas las ecuaciones diferenciales, ni siquiera las mas importantes
de ellas. El problema mas que de habilidad es de principio. En casos tan simples como
dx 1
= +t
dt x
se desconocen soluciones clasicas. La busqueda de recetas para resolver todas las ecuaciones
diferenciales en terminos de funciones elementales es una busqueda sin esperanzas. Ante este
hecho se presentan algunas alternativas: Los metodos cualitativos, los metodos numericos,
y los metodos de aproximacion. No es parte de los objetivos de estas notas un estudio
detallado al respecto. Se quiere sin embargo ilustrar los metodos cualitativos.

56
Apuntes Mat023 (Segundo semestre 2014)

Metodos cualitativos
En muchos problemas, mas que calculos cuantitativos puntuales, lo que interesa es el
comportamiento cualitativo de las soluciones en terminos de las condiciones iniciales o de
valores de los parametros. Saber que una solucion es creciente, que es concava o que tiene
un lmite en el infinito puede ser de ayuda en el entendimiento de un modelo. Ocurre, que
bajo ciertas circunstancias, podemos obtener tal informacion sin resolver explcitamente la
ecuacion diferencial. Analizaremos primero el siguiente modelo:

El modelo de Verhulst
Resumiremos los principales resultados concernientes al modelo de Verhulst para la
dinamica de poblaciones
dx
= x (a bx) (1.39)
dt
donde a, b > 0, esta ecuacion tiene dos soluciones constantes x1 (t) = 0 y x2 (t) = ab . Estas
soluciones dividen al plano xt en 3 regiones de poblaciones
n a o n ao
R1 = (t, x) : < x , R2 = (t, x) : 0 < x < , R3 = {(t, x) : x < 0}
b b
tales que el grafico de cualquier solucion no constante x = x(t) de (1.39) permanece
confinado en una y solo una de estas regiones. Mas aun, podemos determinar cuando es
creciente, y cuando es decreciente la solucion x = x(t) de (1.39) a partir de la condicion
inicial x(t0 ) = x0 .

Z Z
dx
= dt
x (a bx)
Z  
1 1 b
+ = t+C
ax a a bx
 
1 1
ln |x| ln |a bx| = t+C
a a

x
ln
= at + C
a bx
x
se sigue abx
= Keat ,
Kaeat a
x (t) = =
at
Kbe + 1 b + Ceat
 
a 1 a
1. Si x0 < 0 entonces x0 = b+Ceat0
, entonces C = eat 0
b x0
, se sigue
ax0
x (t) =
bx0 + (a bx0 ) ea(tt0 )

57
Apuntes Mat023 (Segundo semestre 2014)

note que ax0 < 0 y

bx0 + (a bx0 ) ea(tt0 ) = 0



a(tt0 )
(a bx0 ) e = bx0

bx0
ea(tt0 ) = >0
(a bx0 )
as
1 bx0
T = ln + t0 > t0
a (a bx0 )
i h
bx0
el intervalo de definicion de esta solucion es , a1 ln (abx0)
+ t0 ], T [ en
este intervalo
d ax0 a2 x0 ea(tt0 ) (a bx0 )
= <0
dt bx0 + (a bx0 ) ea(tt0 ) (bx0 + aeat0 at bx0 eat0 at )2
la funcion es estrictamente decreciente y
ax0
lm =
tT bx0 + (a bx0 ) ea(tt0 )
Diremos que esta solucion explota en un tiempo finito. Note que esta solucion no
representara una solucion asociada al problema de poblaciones pues siempre es
negativa.
ax0
2. Si 0 < x0 < ab entonces x (t) = bx0 +(abx0 )ea(tt0 )
esta bien definida en todo R pues el
denominador no se anula

0 < bx0 + (a bx0 ) ea(tt0 )

ademas
ax0 a
0 < x (t) = a(tt )
<
bx0 + (a bx0 ) e 0 b
y
d ax0 a2 x0 ea(tt0 ) (a bx0 )
= >0
dt bx0 + (a bx0 ) ea(tt0 ) (bx0 + aeat0 at bx0 eat0 at )2
la funcion es estrictamente creciente y
ax0 a
lm a(tt )
=
t+ bx0 + (a bx0 ) e 0 b
ademas
ax0
lm =0
t bx0 + (a bx0 ) ea(tt0 )

58
Apuntes Mat023 (Segundo semestre 2014)

i h
a ax0 1 bx0
3. Si x0 > entonces x (t) =
b bx0 +(abx0 )ea(tt0 )
esta bien definida en a ln (abx0 ) + t0 , +
]T, +[ donde T < t0 ademas
ax0 a
lm =
t+ bx0 + (a bx0 ) ea(tt0 ) b
y
d ax0 a2 x0 ea(tt0 ) (a bx0 )
= <0
dt bx0 + (a bx0 ) ea(tt0 ) (bx0 + aeat0 at bx0 eat0 at )2
la funcion es estrictamente decreciente, x (t) > ab .

En el grafico se muestra el comportamiento de las soluciones

Veremos ahora que es posible analizar los comportamientos de las soluciones sin la
necesidad de resolver la ecuacion.

Ecuaciones diferenciales autonomas

Definicion 1.5.1. Diremos que una ecuacion diferencial de primer orden es autonoma si
se puede expresar en la forma
dx
= f (x) (1.40)
dt
donde f : R es una funcion definida en el intervalo abierto .

2
Las ecuaciones dx
dt
= x (1 x2 ); dx
dt
= x (1 x); dxdt
= ex sin x son ejemplos de
dx
ecuaciones autonomas, mientras la ecuacion dt = et x + t no lo es.

59
Apuntes Mat023 (Segundo semestre 2014)

Teorema 1.5.1. Sean f : R una funcion de clase C 1 (), x0 y t0 R entonces


existe una unica solucion del P.V.I.
dx
= f (x)
dt
x (t0 ) = x0

x : I R de clase C 1 (I) definida en un intervalo abierto que contiene a t0 .

Definicion 1.5.2. Llamaremos intervalo maximal de definicion al mayor intervalo abierto


donde esta definida la solucion del P.V.I. anterior.

Ejemplo 1.5.1. Determine el intervalo maximal de la solucion del P.V.I.


dx
= x2
dt
x (0) = 2

Solucion. Aplicando la tecnica de separacion de variables


Z 0 Z
x (t)
dt = 1dt
x (t)2

1
= t+C
x (t)

1
x (t) =
t+C
1
usando la condicion inicial 2 = C
entonces c = 1/2 se sigue

1
x (t) =
t 12

el mayor intervalo que contiene a t = 0 en el cual esta funcion esta definida es I = , 12


 

el cual corresponde al intervalo maximal.

Observacion 1.5.1. Note que las ecuaciones autonomas son de variables separadas.

Definicion 1.5.3. Las soluciones constantes x (t) = c, t R de la ecuacion (1.40) son


llamadas soluciones de equilibrio.

60
Apuntes Mat023 (Segundo semestre 2014)

Note que si x (t) = c es una solucion de equilibrio


dx (t)
0= = f (x (t)) = f (c)
dt
en otras palabras las soluciones de equilibrio corresponden a las races de la ecuacion
f (x) = 0.

Ejemplo 1.5.2. Determine las soluciones de equilibrio de la ecuacion


dx
= sin x + sin2 x
dt
Solucion. Las soluciones de equilibrio corresponden a las races de la ecuacion

sin x + sin2 x = 0

esto es
(sin x) (1 + sin x) = 0
3
luego x = k con k Z o bien x = 2
+ 2l con l Z.

Teorema 1.5.2. Las soluciones de la ecuacion (1.40) son soluciones de equilibrio o


funciones monotonas estrictas.

Si x (t) es una solucion definida en su intervalo maximal I, mostraremos que la derivada


no se puede anular a menos que la solucion sea de equilibrio. Supongamos que x0 (tc ) = 0
para algun tc I y definamos (t) = x (tc ) = una funcion constante, como

0 = 0 (t)

y
x0 (tc ) = f (x (tc ))
se sigue
0 = 0 (t) = f ( (t))
luego (t) es solucion de equilibrio pero
dx
= f (x)
dt
x (tc ) =

tiene 2 soluciones, por el teorema de existencia y unicidad la solucion debe ser la misma,
es decir x (t) = para todo t.

Esta demostracion tiene otra consecuencia

61
Apuntes Mat023 (Segundo semestre 2014)

Teorema 1.5.3. Las graficas de dos soluciones distintas de (1.40) no se intersectan.

Teorema 1.5.4. Sean = ], [, x0 , t0 R. Si x = x (t) es solucion de (1.40) con


x (t0 ) = x0 y su intervalo maximal de definicion es ]a, b[ entonces los lmites
lm x (t) = A y lm x (t) = B
ta+ tb

existen o son , mas aun, A debe tomar el valor o si a =


6 , y B debe tomar uno
de los valores o si b 6= .

Ejemplo 1.5.3. Consideremos la ecuacion


dx
= x (1 x)
dt
en este caso = R as = y = +. Para 0 < x0 < 1 sean x (t) definida en ]a, b[ la
solucion que cumple x (t0 ) = x0 , note que x1 (t) = 0 y x2 (t) = 1 son soluciones de equilibrio
luego
0 < x (t) < 1
se sigue que 0 A, B 1 entonces a = y b = +. La solucion estara definida en
todo R.

Teorema 1.5.5. Si c y x (t) es una solucion de (1.40) tal que lmt+ x (t) = c o
lmt x (t) = c entonces (t) = c, t R es una solucion de equilibrio.

Demostracion. Tenemos que probar que f (c) = 0. Supongamos que f (c) > 0 entonces
por la continuidad de f existira un intervalo ]c , c + [ tal que x ]c , c + [ implica
f (x) > f (c)
2
> 0, como
lm x (t) = c
t+
se sigue que existe un t0 tal que t t0 implica |x (t) c| < se sigue
Z t
x (t) = x (t0 ) + x0 (u) du
t
Z 0t
= x (t0 ) + f (x (u)) du
t0
Z t
f (c)
> x (t0 ) + du
t0 2
f (c)
= x (t0 ) + (t t0 )
2
para t t0 pero esto contradice lmt+ x (t) = c (se puede llegar a una contradiccion
similar si f (c) < 0).

62
Apuntes Mat023 (Segundo semestre 2014)

Equilibrio y estabilidad

Notemos que en la ecuacion autonoma


dx
= f (x)
dt
se nos indica la pendiente de la recta tangente
a la grafica de la funcion solucion x = x (t) es-
ta viene dada por f (x), esto es, dado un valor
de x las pendientes siempre son las mismas
(independiente de t), las curvas isoclinas co-
rresponde x = c, ademas en los intervalos en
los cuales f (x) > 0 la solucion es estrictamen-
te creciente y en los intervalos en los cuales
f (x) < 0 la funcion solucion es estrictamente
decreciente.
Si x (t) es una solucion de
dx
= f (x)
dt
entonces la funcion (t) = x (t + c) tambien es solucion, en efecto

0 (t) = x0 (t + c) = f (x (t + c)) = f ( (t))

esto significa que las traslaciones de una solucion de la ecuacion autonoma tambien es
solucion, note que si se conoce la solucion x (t) de

dx
= f (x)
dt
x (t0 ) = x0

entonces la solucion de
dx
= f (x)
dt
x (0 ) = x0

es (t) = x (t + (t0 0 )), esto nos dice que en cada banda limitada por las las soluciones
de equilibrio las soluciones son traslaciones de una solucion dada.

63
Apuntes Mat023 (Segundo semestre 2014)

Ejemplo 1.5.4. Considere la ecuacion


dx
=x
dt
en este caso la solucion de equilibrio es
x (t) = 0. Por teorema las demas soluciones
son monotonas estrictas. Para x > 0 las so-
luciones son estrictamente crecientes y para
x < 0 estrictamente decrecientes. En este caso
es facil resolver explcitamente

x1 (t) = et

es una solucion en la region x > 0 las demas


soluciones son xc (t) = et+k = ek et = Cet
donde C > 0 son traslaciones de la solucion
anterior. Note tambien que x (t) = et es
una solucion en la region x < 0 y las demas
soluciones en esa region son xK (t) = et+k =
Ket
En el ejemplo anterior el comportamiento de las soluciones en el entorno de la solucion
de equilibrio es como indica el diagrama

diremos en este caso que el punto de equilibrio es un repulsor, las soluciones se alejan de
esta solucion de equilibrio.

Llamaremos diagrama de fases o lneas de fases al grafico del comportamiento de las


soluciones en el plano xt.
Definicion 1.5.4. Dependiendo del comportamiento local de las soluciones de la ecuacion
alrededor de un punto de equilibrio aislado x0 en las lneas de fases, se distinguen los
siguientes tipos:

64
Apuntes Mat023 (Segundo semestre 2014)

1. x0 es llamado Repulsor si existe > 0 tal que

x0 < x < x0 x = x0 x0 < x < x 0 +


signo f (x) 0 +++

2. x0 es llamado Atractor si existe > 0 tal que

x0 < x < x0 x = x0 x0 < x < x 0 +


signo f (x) +++ 0

3. x0 es llamado Atractor-repulsor si existe > 0 tal que

x0 < x < x0 x = x0 x0 < x < x 0 +


signo f (x) +++ 0 +++

4. x0 es llamado Repulsor-atractor si existe > 0 tal que

x0 < x < x0 x = x0 x0 < x < x 0 +


signo f (x) 0

Para analizar entonces el tipo de solucion de equilibrio tenemos que analizar el signo
de la funcion en el entorno de la solucion de equilibrio, adicionalmente se cuenta con el
siguiente teorema:

Teorema 1.5.6. Si x (t) = c es una solucion de equilibrio de

dx
= f (x)
dt
entonces:

1. Si f 0 (c) < 0, c es un atractor

2. Si f 0 (c) > 0, c es un repulsor.

65
Apuntes Mat023 (Segundo semestre 2014)

Ejemplo 1.5.5. Bosquejar el diagrama de


fases de la ecuacion
dx
= x2 (2 x) (x 3)
dt
Desarrollo: La funcion x2 (2 x) (x 3) esta
bien definida y es de clase C (R) existe so-
lucion unica en cada punto (t0 , x0 ) del plano
xt. Las soluciones de equilibrio corresponden
a x1 (t) = 0, x2 (t) = 2 y x2 (t) = 3, en el
siguiente diagrama se analiza el signo de f

0 2 3
2
x +++ 0 +++ + +++ + +++
x2 0 +++ + +++
x3 0 +++
f (x) = x2 (x 2) (x 3) 0 0 +++ 0

se sigue que x1 (t) = 0 es un repulsor-atractor, x2 (t) = 2 un repulsor y x2 (t) = 3 un


atractor.

Ejemplo 1.5.6. Sean a, b R, a 6= 0. Considere la ecuacion diferencial autonoma


dx
= a ((x 1) (x 4) b)
dt

66
Apuntes Mat023 (Segundo semestre 2014)

1. Determine los valores y/o condiciones sobre a y b de modo que la funcion x (t) 5
sea una solucion de equilibrio y ademas un atractor.
Si f (x) = a ((x 1) (x 4) b) entonces x (t) 5 sera una solucion de equilibrio si
y solo si
0 = f (5) = a (4 b) b = 4
(pues a 6= 0), luego
dx
= a ((x 1) (x 4) 4)
dt
= a x2 5x


notemos que

f 0 (x) = a (2x 5)
f 0 (5) = 5a 6= 0

si a < 0 entonces x (t) 5 es un atractor.

2. Para los valores y/o condiciones obtenidos en la parte anterior, bosquejar el diagrama
de fases.
De la parte anterior
dx
= 5x x2

dt
donde > 0 entonces las soluciones de equilibrio corresponden a x (t) = 0 y x (t) = 5,
el analisis de signo es

factor \ numero 0 5
x 0 +++ + +++
5x +++ + +++ 0
(5x x2 ) 0 +++ 0

el diagrama de fases es como en la figura:

67
Apuntes Mat023 (Segundo semestre 2014)

3. Para los valores y/o condiciones obtenidos en la parte I), analizar el comportamiento
de la solucion definida por el P.V.I.
dx
= a ((x 1) (x 4) b)
dt
x (0) = 1

esto es: Intervalos de crecimiento, concavidad, limites a si estos tienen sentido


(examinar el intervalo maximal de definicion).
De las partes anteriores, el problema es:
dx
= 5x x2

dt
x (0) = 1

Note que la condicion inicial implica que la solucion del P.V.I queda entre las soluciones
de equilibrio. Si el intervalo maximal de definicion es ]a, b[ entonces x : ]a, b[ R
debe cumplir

0 < x (t) < 5 para t ]a, b[ , 0 ]a, b[ y x (0) = 1

De teoremas sabemos que las soluciones de una ecuacion autonoma o son de equilibrio
o son monotonas estrictas, en este caso, por el intervalo al cual pertenece la solucion,
esta debe ser estrictamente creciente (ver parte anterior). Por el teorema del intervalo
maximal sabemos
lm+ x (t) = A y lm x (t) = B
ta tb

entonces A y B debe existir o ser , si a 6= entonces A debe ser esto no


se puede cumplir pues 0 < x (t) < 5 implica

0 lm+ x (t) , lm x (t) 5


ta tb

se sigue a = de manera similar b = + y la solucion esta definida en todo R.


Como x es estrictamente creciente entonces

lm x (t)
t+

existe y debe ser una solucion de equilibrio, se sigue

lm x (t) = 5
t+

de manera similar
lm x (t) = 0
t

68
Apuntes Mat023 (Segundo semestre 2014)

finalmente para la concavidad (recuerde que 0 < x < 5)

d2 x d 2

= 5x x
dt2 dt
= (5x0 2xx0 )
dx
= (5 2x)
dt
= (5 2x) 5x x2


= 2 (5 2x) 5x x2


luego, si x 25 , 5 es concava y si x 0, 52 es convexa.


   

Obs.: No se entregan puntos por resolver explcitamente y luego realizar el analisis.

Ejercicios de la seccion

1. En una red de computadores sea x (t) la proporcion de maquinas infectadas con el


virus MTA320 en un instante t dado (note que 0 x (t) 1). x0 (t) mide la rapidez
de propagacion sobre la red. Los estudios de expertos de internet determinan que el
virus satisface el problema con valores iniciales
  
0 1 2
x = x x x (1 x)
3 3
x (0) = x0

donde x0 es la proporcion de computadores en que el virus se implanta inicialmente


y > 0 es una constante.

a) Encuentre las soluciones de equilibrio, clasificarlas y dibujar el diagrama de fase


del sistema.
b) Demostrar que si 0 < x0 < 2/3 entonces el virus alcanzara finalmente a un
tercio del total.
c) Cual debe ser la cantidad de maquinas infectada inicialmente para que todas
se infecten?

2. Sea x : I R la solucion maximal del P.V.I.


dx 2
= x2 (1 x) (2 x) ex
dt
3
x (0) =
2
muestre que I = R y determine lmt+ x (t), lmt x (t).

69
Apuntes Mat023 (Segundo semestre 2014)

3. Considere la ecuacion diferencial


dy
= y 2/3
dt
a) Demuestre que y1 (t) = 0 para todo t R es solucion.
b) Comprobar que y2 (t) = t3 /27 es tambien solucion.
c) Notar que y1 (0) = y2 (0) pero ambas funciones no son iguales para toda t Por
que este ejemplo no contradice el teorema de unicidad?

4. Esboce las lneas de fase para la ecuacion diferencial dada. Identifique los puntos de
equilibrio:
dy dy dy
(a) dt
= 3y (1 y) (b) dt
= y 2 6y 16 (c) dt
= cos y

dw dw dy 1
(d) dt
= w cos w (e) dt
= (w 2) sin w (f) dt
= y2

dw dy
(g) dt
= w2 + 2w + 10 (h) dt
= tan y
5. Considere la ecuacion diferencial
dy
= y 2 4y + 2
dt
con las siguientes condiciones iniciales:

a) y (0) = 0
b) y (0) = 1
c) y (0) = 1
d ) y (0) = 10
e) y (0) = 10
f ) y (3) = 1

describir el comportamiento a largo plazo de la solucion.

6. Describir el diagrama de fases de la ecuacion


dx
= (x + ) x2 + (cos x + 2)

dt
para los distintos valores del parametro .
1
7. Determine los valores de R para los cuales la solucion de equilibrio x (t) = 3
de
 
dx 1
x x2

= (x )
dt 3
es:

70
Apuntes Mat023 (Segundo semestre 2014)

a) Un atractor.
b) Un repulsor.

o justifique la no existencia de tales valores.

8. Sea x (t) la solucion del P.V.I.

dx 2
= exx sin (x) arctan x
dt
x (3) = 4

a) Determine su intervalo maximal de definicion.


b) Es x (t) estrictamente creciente?
c) Si existen, calcular el valor de lmt+ x (t) y lmt x (t)

9. Sean , > 0. La ecuacion


dP
= P 2/3 P
dt
modela el peso de un pez en el tiempo t. Sin resolver la ecuacion, determine el peso
maximo del pez si P (0) = p0 > 0 (justificar sus calculos).

10. Muestre que la solucion de equilibrio y (t) 0 de la ecuacion

dy
= y cos y 5 + 2y 27y 4
 
dt
corresponde a un repulsor.

71
Apuntes Mat023 (Segundo semestre 2014)

Ejercicios del captulo

1. Resuelva las siguientes ecuaciones diferenciales:


dy 1x2 dy dy sec2 x
a) dx
= y2
b) dx
= y (2 + sin x) c) dx
= 1+x2

dy 1 dy dv 14v 2
d) dx
= xy 3
e) dx
= 3xy 2 f) x dx = 3v

dx dy dy2
g) dt
+ x2 = x h) dx
= 3x2 (1 + y 2 ) i) (x + xy 2 ) + ex y dx =0

2. Resuelva los siguientes problemas de valor inicial:


dy

a) y 0 = x3 (1 y) , y (0) = 3 b) dx
= (1 + y 2 ) tan x, y (0) = 3

dy 3x2 +4x+2 dy
c) dx
= 2y+1
, y (0) = 1 d) dx
= 2 y + 1 cos x, y (0) = 2

dy dy
e) dx
= y sin x, y () = 3 f) x2 + 2y dx = 0, y (1) = 2

3. Resuelva las siguientes ecuaciones lineales:


dy dy y dr
a) dx
y = e3x b) dx
= x
+ 2x + 1 c) d
+ r tan = sec

dy dy dy
d) x dx + 3y + 2x2 = x3 + 4x e) (x2 + 1) dx + xy = x f) dx
= x2 e4x 4y

4. Considere el problema con valor inicial:


dy p
+ y 1 + sin2 x = x, y (0) = 2
dx
Utilice la integral indefinida para mostrar que el factor integrante para la ecuacion
diferencial se puede escribir como:
Z x p 
2
(x) = exp 1 + sin t dt
0

5. Resuelva las siguientes ecuaciones diferenciales:


a) dy
(x y) + x dx =0 b) dy
dx
= (3x + 2y 1)2 + 2 c) dy
dx
= sin (x y)


x2 +t t2 +x2 dy y(ln yln x+1)
d) y 0 + 1 = x+y e) dx
dt
= tx
f) dx
= x

6. Hallar la solucion general de la ecuacion de Bernoulli:

2y 0 + y tan x = (2x sec x) y 3

72
Apuntes Mat023 (Segundo semestre 2014)

7. Resuelva la ecuacion:
3y 0 = (1 2t) y 4 y
sabiendo que y (0) = 1.

8. Resolver:
dy 1
=yx1+
dx xy2
9. Considere la ecuacion diferencial:

y xy 0 = a 1 + x2 y 0 ,

a>1

a) Hallar la solucion general de la ecuacion.


b) Encuentre una solucion particular de la ecuacion, tal que y (0) = 1.
c) Hallar el intervalo mas grande donde la solucion particular anterior este definida.

10. Demuestre que la ecuacion diferencial:

2x4 y y 0 + y 4 = 4x6

se reduce a una ecuacion homogenea mediante el cambio de variables:

y = zn

para cierto n Z. Determine el valor de n y resuelva la ecuacion.

11. Muestre que la ecuacion:

2 x3 y y 3 dy = 3 x5 + x4 y 4 dx
 

se reduce a una ecuacion homogenea, realizando el cambio de coordenadas:

x = up y = vq

para ciertas constantes adecuadas p, q R. Hallar tales constantes y resolver la


ecuacion diferencial.

12. Hallar la solucion general de la ecuacion:


dy
(x 2y 4) + (2x y + 2) =0
dx

13. Considere la ecuacion de Ricatti:

y 0 + 2 (1 x) y y 2 = x (x 2)

73
Apuntes Mat023 (Segundo semestre 2014)

a) Hallar constantes A, B R de modo que:

y = Ax + B

sea una solucion particular de la ecuacion.


b) Determine la solucion general de la ecuacion diferencial.

14. Resuelva la ecuacion:


dy y
= x3 (y x)2 +
dx x
15. Sea x > 0. Considere la ecuacion:
1 e2x
y 0 + e2x y 2 1 + 4x + 2x2 y = 1 + x + 2x2 + x3
 
x x
a) Hallar una solucion particular de la forma:

y = e2x (Ax + B)

b) Resuelva la ecuacion diferencial.

16. Un esquiador acuatico ubicado en el punto (a, 0) es tirado por un bote que se
encuentra en el origen O y viaja hacia el norte en direccion OY . Hallar la trayectoria
que sigue el bote si este se dirige en todo momento hacia el bote.

17. Considere un tanque que contiene inicialmente 1000 litros de agua pura, dentro del
cual empieza a fluir una solucion salina a razon de 6 litros por minuto. La solucion
dentro del estanque se mantiene bien agitada y fluye hacia el exterior del tanque
a una velocidad de 5 litros por minuto. Si la concentracion de sal en la salmuera
que entra en el estanque es de 1 kilogramo por litro, determine el instante en que la
concentracion de sal dentro del tanque sea de 6364
kilogramo por litro.

18. El modelo de Malthus para el crecimiento de poblaciones esta dado por la ecuacion:
dN
= rN, N (0) = N0
dt
donde r es una constante positiva intrnseca a la poblacion y N0 es la poblacion
inicial.

a) Resuelva la ecuacion diferencial.


b) En 1790, la poblacion de EE.UU. era de 3, 93 millones de habitantes, y en 1890
era de 62, 98 millones de habitantes. Estime la poblacion para EE.UU para el
ano 2000.

74
Apuntes Mat023 (Segundo semestre 2014)

c) El censo realizado en el ano 2000 en EE.UU. estimo en 281, 42 millones de


habitantes
d ) El modelo de Malthus solo considera muertes por causas naturales, sin embargo,
un analisis mas detallado indica que hay muertes debida a enfermedades, a
desnutricion, a crmenes. En resumen, por la competencia entre los miembros
de la poblacion. El modelo logstico implementa dichas consideraciones:
dN
= rN (N K) , N (0) = N0
dt
para constantes positivas adecuadas r y K. Resuelva la ecuacion diferencial.
e) Para la poblacion de EE.UU. se sabe, ademas, que esta alcanzo los 17, 07
millones de habitantes en 1840. Calcule, usando el modelo logstico, la poblacion
de EE.UU. en el ano 2000. Compare respecto al censo del ano 2000.
f ) Considerando el modelo logstico, calcule la poblacion lmite de EE.UU.
h 3i
19. La sangre conduce un medicamento a un organo a razon de 3 cm seg
y sale de el a la
misma razon. El organo tiene un volumen lquido de 125 [cm3 ]. Si la concentracion
 gr 
del medicamento en la sangre que entra al organo es de 0, 2 cm 3 :

a) Cual es la concentracion del medicamento en el organo en el instante t, si


inicialmente no haba rastros de dicho medicamento?
b) En que momento llegara la concentracion del medicamento en el organo a
 gr 
0, 1 cm 3

20. Una taza de cafe caliente, inicialmente a 95 C, se enfra hasta 80 C en 5 minutos,


al estar en un cuarto con temperatura de 21 C. Use solo la ley de enfriamiento de
Newton (ver MAT022) y determine el momento en que la temperatura del cafe estara a
unos agradables 50 C.

21. Era el medioda en un fro da de julio en Vina del Mar: 16 C. Un inspector de la


Polica de Investigaciones (PDI) llega a la escena de un crimen, encontrando un
detective sobre un cadaver. El detective dijo que haba varios sospechosos detenidos.
Dadas las coartadas se podra hallar al culpable sabiendo el momento exacto de
la muerte. El inspector saco un termometro y midio la temperatura del cadaver:
34 C. Luego de realizar peritos en la escena que duraron exactamente una hora,
midio nuevamente la temperatura del cuerpo: 33, 7 C. Bajo que la hipotesis de que
la temperatura normal de un cuerpo humano es de 37 C. Determine el momento en
que ocurrio el crimen.

75
Apuntes Mat023 (Segundo semestre 2014)

22. El economista ingles Thomas Malthus fue unos de los primeros en intentar modelar el
crecimiento poblacional humano por medio de matematicas en 1978. Basicamente, el
fundamento del modelo maltusiano es la suposicion de que la rapidez a la que crece
la poblacion de un pais en cierto tiempo es proporcional a la poblacion del pas en
ese momento.

a) Proponer una ecuacion para este modelo.


b) Determine la ecuacion diferencial si se permite inmigracion a tasa constante
r > 0.

23. La ley de enfriamiento (o calentamiento) de Newton establece que la rapidez a la


que cambia la temperatura de un cuerpo es proporcional a la diferencia entre la
temperatura del cuerpo y la temperatura del medio circundante. Obtener una E.D.O.
para esta ley. Si la temperatura del medio cambia en el tiempo, escribir la ecuacion
e identificarla.

24. Una enfermedad contagiosa se disemina en una comunidad por medio de la gente que
entra en contacto con otras personas. Suponer que la rapidez con que se disemina la
enfermedad es proporcional al numero de interacciones entre las personas contagiadas
y no contagiadas (puede ser considerado como el producto de los contagiados y no
contagiados). Suponga que en una poblacion pequena de n personas se introduce un
enfermo obtener una EDO cuya solucion permita obtener el numero de contagiados
en un tiempo t.

25. Resolver las siguientes ecuaciones diferenciales ordinarias:

76
Apuntes Mat023 (Segundo semestre 2014)

dy 1+y dy
1) = 2) 2xy = x
dx 1+x dx

dy dy yx
3) y tan x = cos x 4) = 2
dx dx x y2

dy 2y + x 2 dy 1 cos x
5) = 6) + y= 2
dx yx+1 dx x y

dy 1 dy
7) + x2 y = 2 y 4 8) cos y + 2x sin y = 2x
dx x dx

e2x
 
dy dy 1
9) = y 2 + 2x x4 10) + 1 y =
dx dx x x
 
dy dy y+1 y+1
11) 4y = 2ex y 1/2 12) = + exp
dx dx x+2 x+2
2
dy dy ey
13) = 3 |y|2/3 14) =
dx dx y (2x + x2 )

dy dy p
15) = (x y + 3)2 16) x = y + x2 + y 2
dx dx

dy dy y
17) + y sin x = sin3 x 18) + = (1 + x) y 4
dx dx 1 + x
 2
dy dy
19) y = x +
dx dx

26. Suponga que un gran tanque de mezclado tiene inicialmente 600 galones de salmuera.
Otra solucion de salmuera se bombea hacia en tanque a una rapidez de 4 galones
por minuto, la concentracion de sal en el flujo de entrada es de 2 libras por galon.
Cuando la solucion en el tanque esta bien agitada se bombea a 2 galones por minuto.
Determinar una EDO cuya solucion corresponda a la cantidad de sal en el tanque en
el momento t.

27. Identificar el campo de direcciones con la ecuacion correspondiente;

(A) y 0 = x + y
(B) y 0 = xy
(C) y 0 = y + 1

77
Apuntes Mat023 (Segundo semestre 2014)

(D) y 0 = y 2 x2

Obs: el campo de direcciones nos indica la direccion de las rectas tangentes a las
soluciones en un punto (x, y) dado.

28. Considere la ecuacion diferencial autonoma


dS
= S 3 2S 2 + S
dt
a) Hacer el diagrama de las lneas de fase..
b) Usando el dibujo anterior delinear las graficas de las soluciones S (t) con las
condiciones iniciales siguientes: S (0) = 0; S (0) = 12 ; S (0) = 32 y S (0) = 12 .
c) A que tienden sus soluciones cuando t +?

29. Considere la ecuacion


dy 1
=
dt 1y

a) Determine los valores de y R en los cuales es aplicable el teorema de existencia


y unicidad.
b) Obtener las soluciones explcitamente y mostrar que llegan la y = 1 en un
tiempo finito, determine los intervalos maximales de solucion

78
Apuntes Mat023 (Segundo semestre 2014)

30. La ardilla negra es un pequeno mamfero nativo de las montanas rocallosas. Esas
ardillas son muy territoriales, por lo que si su poblacion es grande, su razon de
crecimiento decrece y puede llegar a ser negativa. Por otra parte, si la poblacion es
demasiado pequena, los adultos fertiles corren el riesgo de no ser capaces de encontrar
parejas adecuadas y su tasa de crecimiento nuevamente es negativa. Muestre que el
modelo   
dP P P
= kP 1 1
dt N M
donde k, M, N > 0 y M < N puede ser utilizado para describir la poblacion de
Ardillas Que interpretacion tienen N y M ?

79
Captulo 2 : Ecuaciones diferenciales lineales de orden superior

Elementos de transformaciones lineales

Definiciones

En lo que sigue K representa R o bien C.

Definicion 2.1.1. Sean U y V espacios vectoriales sobre K. Una transformacion lineal


es una funcion T : U V tal que para todo u, v U y K cumple:

1. T (u + v) = T (u) + T (v)

2. T ( u) = T (u)

Observacion 2.1.1. Usualmente, los puntos (1) y (2) de la definicion anterior, pueden
escribirse como

1. u, v U, K, T ( u + v) = T (u) + T (v)
o bien

2. u, v U, , K, T ( u + v) = T (u) + T (v)
las tres definiciones obtenidas son equivalentes.

Observacion 2.1.2. Note que si U, V son K espacios vectoriales, T : U V es una


transformacion lineal, para i = 1, , n, i son escalares en K y ui U entonces
n
! n
X X
T i ui = i T (ui )
i=1 i=1

luego las transformaciones lineales envan combinaciones lineales del espacio de partida a
combinaciones lineales del espacio de llegada.

Ejemplo 2.1.1. La funcion nula 0 y la funcion identidad 1 de un espacio vectorial en


s mismo son transformaciones lineales. En efecto, Si V es un K espacio vectorial y
0 : V V , v 0 (v) = V (el neutro de la adicion) entonces

0 (u + v) = V
= v + v
= 0 (u) + 0 (v)

80
Apuntes Mat023 (Segundo semestre 2014)

de manera similar, si 1 : V V , v 1 (v) = v entonces

1 (u + v) = u + v
= 1 (u) + 1 (v)

Ejemplo 2.1.2. T : R2 R4 , (x, y) T (x, y) = (x y, 2x, x + y, 2y) es una transfor-


macion lineal. En efecto:

1. Para cada (a, b), (u, v) R2 se tiene

T ((a, b) + (u, v))


= T (a + u, b + v)
= (a + u b v, 2a + 2u, a + u + v + b, 2b + 2v)
= ((a b) + (u v) , 2a + 2u, (a + b) + (u + v) , 2b + 2v)
= (a b, 2a, a + b, 2b) + (u v, 2u, u + v, 2v)
= T (a, b) + T (u, v)

2. Para cada (u, v) R2 y R se cumple

T ( (u, v)) = T (u, v)


= (u v, 2u, u + v, 2v)
= (u v, 2u, u + v, 2v)
= T (u, v)

de esta forma T verifica las condiciones de transformacion lineal.

Ejemplo 2.1.3. T : R R2 , x T (x) = (x, 2x + 1) no es una transformacion lineal. En


efecto,

T (x + y) = (x + y, 2x + 2y + 1)
= (x, 2x + 1) + (y, 2y)
6= T (x) + T (y)

basta usar un contraejemplo:

T (1 + 2) = T (3) = (3, 7)

pero
T (1) + T (2) = (1, 3) + (2, 5) = (3, 8)
de esta forma
T (1 + 2) 6= T (1) + T (2)
luego no es una transformacion lineal.

81
Apuntes Mat023 (Segundo semestre 2014)

Ejemplo 2.1.4. Si R3 [x] es el espacio vectorial de los polinomios de grado menor igual a
3 entonces
T : R3 [x] M (R)
 22
p (0) p0 (0)

p T (p) =
p00 (0) p000 (0)
Es una transformacion lineal. En efecto:

1. Si p, q R3 [x] entonces

(p + q) (0) (p + q)0 (0)


 
T (p + q) =
(p + q)00 (0) (p + q)000 (0)

pero recordar que (p + q)0 (x) = p0 (x) + q 0 (x), (p + q)00 (x) = p00 (x) + q 00 (x) y
(p + q)000 (x) = p000 (x) + q 000 (x) se sigue

(p + q) (0) (p + q)0 (0)


 
T (p + q) =
(p + q)00 (0) (p + q)000 (0)
p0 (0) + q 0 (0)
 
p (0) + q (0)
=
p00 (0) + q 00 (0) p000 (0) + q 000 (0)
p (0) p0 (0) q (0) q 0 (0)
   
= +
p00 (0) p000 (0) q 00 (0) q 000 (0)
= T (p) + T (q)

2. Si R y p R3 [x] entonces

(p) (0) (p)0 (0) p (0) p0 (0)


   
T (p) = =
(p)00 (0) (p)000 (0) p00 (0) p000 (0)
p (0) p0 (0)
 
= = T (p)
p00 (0) p000 (0)

Note que no es necesario considerar la expresion del polinomio p (x) = a3 x3 + a2 x2 +


a1 x + a0 en esta demostracion de que es transformacion lineal, lo que utilizamos son
las propiedades de la derivada y matrices.

Ejemplo 2.1.5. Sea T : R3 R2 la funcion definida por:

T (x, y, z) = (x + y + z, x y)

muestre que T es una transformacion lineal.

82
Apuntes Mat023 (Segundo semestre 2014)

Solucion. Sean R, (u, v, w) , (x, y, z) R3 entonces


T ( (u, v, w) + (x, y, z))
= T ((u, v, w) + (x, y, z))
= T (u + x, v + y, w + z)
= ((u + x) + (v + y) + (w + z) , (u + x) (v + y))
= (u + v + w, u v) + (x + y + z, x y)
= (u + v + w, u v) + (x + y + z, x y)
= T (u, v, w) + T (x, y, z)
Ejemplo 2.1.6. Sean I R un intervalo abierto y C 1 (I) el espacio vectorial real de las
funciones de clase C 1 sobre I. Defina la transformacion D : C 1 (I) C (I) mediante:
df (x)
D (f ) (x) = , x I
dx
Entonces, D es una trasformacion lineal.
Solucion. Por propiedades de la derivada, si R, f, g C 1 (I) entonces
d
D (f + g) (x) = {(f + g) (x)}
dx
df (x) dg (x)
= +
dx dx
= D (f ) (x) + D (g) (x)
= (D (f ) + D (g)) (x)
as
D (f + g) = D (f ) + D (g)
Ejemplo 2.1.7. Sea C el espacio vectorial de todas las funciones continuas de R en R y
sea T : C C, f T (f ) donde T (f ) es la funcion definida por:
Z x
T (f ) (x) = f (t) dt
1
Entonces, T es una transformacion lineal.
Solucion. Sean R, f, g C entonces
Z x
T (f + g) (x) = (f (t) + g (t)) dt
1
Z x Z x
= f (t) dt + g (t) dt
1 1
Z x Z x
= f (t) dt + g (t) dt
1 1
= T (f ) (x) + T (g) (x)
= (T (f ) + T (g)) (x)

83
Apuntes Mat023 (Segundo semestre 2014)

as
T (f + g) = T (f ) + T (g)

Ejemplo 2.1.8. Sean x, y Rn . Considere la funcion P : Rn R definida por:


n
X
x 7 P (x) = xi y i
i=1

Entonces, P es una transformacion lineal. Esto es directo por las propiedades del producto
punto en Rn .

Observacion 2.1.3. El conjunto de todas las transformaciones lineales de U en V se


denotara por LK (U, V ), o mas sencillamente por L (U, V ) si el cuerpo de escalares no
presenta confusion. Es decir:
n o
L (U, V ) = T : U V T es transformacion lineal

Teorema 2.1.1. Sea T : U V una transformacion lineal, entonces T (U ) = V .

Demostracion. Note que

T (U ) = T (U + U )
= T (U ) + T (U )

sumando el inverso aditivo T (U ) V se tiene

T (U ) + T (U ) = (T (U ) + T (U )) + T (U )

luego

V = T (U ) + (T (U ) + T (U ))
= T (U ) + V

y as
T (U ) = V

Ejemplo 2.1.9. Considere la funcion T : R2 R3 definida por la ecuacion:

T (x, y) = (x + y, 1 y, x)

En vista del teorema anterior, la funcion T no puede ser transformacion lineal, pues:

T (0, 0) = (0, 1, 0)

no es el vector nulo de R3 .

84
Apuntes Mat023 (Segundo semestre 2014)

Teorema 2.1.2. Sean U, V, W espacios vectoriales sobre K:

1. L (U, V ) es un espacio vectorial sobre K.

2. Si T L (U, V ) y S L (V, W ) entonces S T L (U, W ).

3. Si T L (U, V ) es biyectiva entonces T 1 L (V, U )

Demostracion. En efecto:

1. Mostraremos que es un subespacio del espacio de las funciones de U en V . L (U, V ) no


es vaco pues existe la transformacion lineal : U V definida por u (u) = V .
Sean , K y T, L L (U, V ) entonces T + L es una transformacion lineal pues
si K, u1 , u2 U entonces

(T + L) (u1 + u2 ) = T (u1 + u2 ) + L (u1 + u2 )


= (T (u1 ) + T (u2 )) + (L (u1 ) + L (u2 ))
= (T (u1 ) + L (u1 )) + (T (u2 ) + L (u2 ))
= (T + L) (u1 ) + (T + L) (u2 )

luego L (U, V ) es no vaco, cerrado para la suma y producto escalar.

2. Sean u, v U y K, entonces:

(S T ) ( u + v) = S (T ( u + v))
= S ( T (u) + T (v))
= S (T (u)) + S (T (v))
= (S T ) (u) + (S T ) (v)

Luego, S T es una transformacion lineal.

3. Si T 1 : V U existe entonces es una transformacion lineal, en efecto, sean v1 , v2 V


y K, por la biyectividad existen unicos u1 , u2 U tales que

T (u1 ) = v1
T (u2 ) = v2

luego

T 1 (v1 + v2 ) = T 1 (T (u1 ) + T (u2 ))


= T 1 (T (u1 + u2 ))
= u1 + u2
= T 1 (v1 ) + T 1 (v2 )

85
Apuntes Mat023 (Segundo semestre 2014)

Observacion 2.1.4. Con respecto al teorema anterior, denotaremos la composicion de


transformaciones lineales S T mediante la notacion producto ST .

Teorema 2.1.3. Sean U un espacio vectorial sobre K tal que u1 , u2 , . . . , un es una base
para U y v1 , v2 , . . . , vn un subconjunto cualquiera de un espacio vectorial V arbitrario (se
pueden repetir elementos), entonces existe una unica transformacion lineal T : U V tal
que:
T (ui ) = vi , i = 1, 2, . . . , n (2.1)

Observacion 2.1.5. Note que si u1 , u2 , . . . , un es una base para un espacio vectorial U y


u U , entonces existen unicos escalares x1 , x2 , . . . , xn K tales que:
n
X
u= xi u i
i=1

Si, ademas, T esta definida por las ecuaciones (2.1), entonces se tiene que:
n
!
X
Tu = T xi ui
i=1
n
X
= xi T (ui )
i=1
n
X
= xi vi
i=1

Ejemplo 2.1.10. Determinaremos la formula T (x, y, z) de la transformacion lineal T :


R3 R3 [x] tal que:

T (1, 0, 2) = x3 + 2x2 + 1
T (1, 1, 1) = 2x2 2 (2.2)
3
T (1, 2, 1) = x 1

Notamos primeramente que u1 = (1, 0, 2) , u2 = (1, 1, 1) y u3 = (1, 2, 1) forman una


base para R3 , pues:

1 0 2
det 1 1 1 = 5
1 2 1
Por otro lado, si:

(a, b, c) = x1 (1, 0, 2) + x2 (1, 1, 1) + x3 (1, 2, 1)

86
Apuntes Mat023 (Segundo semestre 2014)

entonces x1 = 35 a + 25 b + 15 c, x2 = 25 c 15 b 45 a y x3 = 15 c 35 b 25 a, as:
     
3 2 1 2 1 4 1 3 2
(x, y, z) = a + b + c (1, 0, 2)+ c b a (1, 1, 1)+ c b a (1, 2, 1)
5 5 5 5 5 5 5 5 5
por la linealidad de T y su definicion en la base, tenemos que:

 
3 2 1
T (a, b, c) = a + b + c T (1, 0, 2) +
5 5 5
   
2 1 4 1 3 2
+ c b a T (1, 1, 1) + c b a T (1, 2, 1)
5 5 5 5 5 5
   
3 2 1 3 2 2 1 4
c b a 2x2 2 +
 
= a + b + c x + 2x + 1 +
5 5 5 5 5 5
 
1 3 2
c b a x3 1

+
5 5 5
     
1 1 2 3 14 6 2 2 13 7 4
= a b+ c x + a+ b c x + a+ b c
5 5 5 5 5 5 5 5 5

Es decir, T : R3 R3 [x] esta definida por:


     
1 1 2 3 14 6 2 2 13 7 4
T (a, b, c) = a b+ c x + a+ b c x + a+ b c
5 5 5 5 5 5 5 5 5

Ejemplo 2.1.11. Hallar la formula de una transformacion lineal T de R2 en:

U = (x, y, z) R3 : x + y + z = 0


tal que T (1, 1) = (1, 2, 3) y T (2, 1) = (5, 3, 2).

Solucion. Note que (1, 2, 3), (5, 3, 2) pertenecen a U , como (1, 1) y (2, 1) son
linealmente independiente, por el teorema existe una unica transformacion lineal T : R2 U
tal que

T (1, 1) = (1, 2, 3)
T (2, 1) = (5, 3, 2)

para determinar una formula, notemos que

(a, b) = (1, 1) + (2, 1)

implica    
1 2 a 1 0 a + 2b

1 1 b 0 1 a+b

87
Apuntes Mat023 (Segundo semestre 2014)

as
(a, b) = (a + 2b) (1, 1) + (a + b) (2, 1)
as

T (a, b) = (a + 2b) T (1, 1) + (a + b) T (2, 1)


= (a + 2b) (1, 2, 3) + (a + b) (5, 3, 2)
= (6a + 7b, b a, 5a 8b)

Nucleo e imagen

Definicion 2.1.2. Sean U y V espacios vectoriales sobre K y T : U V una transforma-


cion lineal. Llamaremos:

1. Nucleo de T o Kernel de T al subconjunto de U definido como:

ker T = {u U : T (u) = 0}

2. Imagen de T al subconjunto de V definido como:

ImT = {v V : u U, T (u) = v}

Observacion 2.1.6. El siguiente teorema indica que el nucleo y la imagen de T poseen


estructura de espacio vectorial.

Teorema 2.1.4. Sea T L (U, V ), entonces:

1. ker T U .

2. ImT V

Ejemplo 2.1.12. Hallar el nucleo y la imagen de T : R3 R3 definida por:

T (x, y, z) = (2x y + z, x y + z, x)

Solucion. Primero determinaremos el nucleo

ker (T ) = (x, y, z) R3 : T (x, y, z) = (0, 0, 0)




= (x, y, z) R3 : (2x y + z, x y + z, x) = (0, 0, 0)




2x y + z = 0
= (x, y, z) R3 : x y + z = 0

x = 0

88
Apuntes Mat023 (Segundo semestre 2014)

note que
2 1 1 0

1 0 0 0
1 1 1 0 0 1 1 0
1 0 0 0 0 0 0 0
luego
 
x
3 = 0
ker (T ) = (x, y, z) R :
yz = 0
3

= (0, y, y) R : y R
= h{(0, 1, 1)}i

un espacio de dimension 1. Determinemos la imagen

(a, b, c) R3 : (x, y, z) R3 , T (x, y, z) = (a, b, c)



ImT =
= (a, b, c) R3 : (x, y, z) R3 , (2x y + z, x y + z, x) = (a, b, c)


2x y + z = a
= (a, b, c) R3 : (x, y, z) R3 , x y + z = b

x = c

en otras palabras, para que valores de a, b, c R el sistema

2x y + z = a
xy+z = b
x = c

tiene solucion, notemos que

2 1 1 a 2 1 1

a
1 1 1 b 0 1 1 2b a
1 0 0 c 0 0 0 c b + a

luego el sistema tiene solucion si y solo si

c b + a = 0

se sigue

(a, b, c) R3 : c b + a = 0

ImT =
= (b + c, b, c) R3 : b, c R


= h{(1, 1, 0) , (1, 0, 1)}i

es un espacio de dimension 2.

89
Apuntes Mat023 (Segundo semestre 2014)

Definicion 2.1.3. Sea T L (U, V ) con U y V espacios de dimension finita, entonces:


1. Llamaremos nulidad de T , denotado (T ), al numero definido como:
(T ) = dim ker T

2. Llamaremos rango de T , denotado (T ) al numero definido como:


(T ) = dim ImT

Teorema 2.1.5. Sean U, V un espacios vectoriales sobre K, u1 , u2 , . . . , un una base para


U y T L (U, V ), entonces:
ImT = h{T (u1 ) , T (u2 ) , . . . , T (un )}i
Demostracion. Suponga que v ImT entonces, por definicion, existe un u U tal que
T (u) = v, como {u1 , u2 , . . . , un } es una base de U existiran escalares 1 , 2 , . . . , n tales
que
Xn
u= i ui
i=1
se sigue que !
n
X n
X
v = T (u) = T i ui = i T (ui )
i=1 i=1
es decir, v es una combinacion lineal de los elementos T (u1 ) , T (u2 ) , . . . , T (un ), luego
{T (u1 ) , T (u2 ) , . . . , T (un )} genera ImT .

Ejemplo 2.1.13. Si T : M22 (R) M22 (R)


      
x y x y 1 1 x y
T =
z w z w 1 1 z w
sabemos que        
1 0 0 1 0 0 0 0
, , y
0 0 0 0 1 0 0 1
forman una base de M22 (R) luego
      
1 0 1 1 1 0 1 0
T = =
0 0 1 1 0 0 1 0
      
0 1 1 1 0 1 0 1
T = =
0 0 1 1 0 0 0 1
      
0 0 1 1 0 0 1 0
T = =
1 0 1 1 1 0 1 0
      
0 0 1 1 0 0 0 1
T = =
0 1 1 1 0 1 0 1

90
Apuntes Mat023 (Segundo semestre 2014)

generan la imagen, se sigue


       
1 0 0 1 1 0 0 1
ImT = , , ,
1 0 0 1 1 0 0 1
   
1 0 0 1
= ,
1 0 0 1
de donde obtenemos (T ) = 2.
Ejemplo 2.1.14. Demuestre que existe una transformacion lineal T : M2 (R) R3 tal
que:
ImT = {(x, y, z) : x 2y + z = 0}
y la nulidad de T sea 2.
Solucion. Notemos que
ImT = {(x, y, z) : x 2y + z = 0}
= {(x, y, z) : x = 2y z}
= {(2y z, y, z) : y, z R}
= h{(2, 1, 0) , (1, 0, 1)}i
este espacio tiene dimension 2. Si necesitamos que la nulidad sea 2 entonces el Nucleo debe
estar generado por dos elementos L. I. Definamos
 
1 0
T = (2, 1, 0)
0 0
 
0 1
T = (1, 0, 1)
0 0
 
0 0
T = (0, 0, 0)
1 0
 
0 0
T = (0, 0, 0)
0 1
existe una unica T : M2 (R) R3 que cumple lo anterior, para esta transformacion se
tiene:
        
1 0 0 1 0 0 0 0
ImT = T ,T ,T ,T
0 0 0 0 1 0 0 1
= h{(2, 1, 0) , (1, 0, 1)}i
Note que  
a b
T = (2a b, a, b)
c d
demuestre que el Nucleo tiene dimension 2..

91
Apuntes Mat023 (Segundo semestre 2014)

Observacion 2.1.7. El siguiente resultado relaciona la nulidad y el rango de una trans-


formacion lineal:
Teorema 2.1.6. Sean U y V espacios vectoriales sobre K con dim U < . Considere
T L (U, V ), entonces:
dim U = (T ) + (T )
Demostracion. Supongamos que dim U = n. Sea u1 , u2 , . . . , ur (r n) una base del nucleo
de T . Considere ur+1 , ur+2 , . . . , un una completacion de la base u1 , u2 , . . . , ur del nucleo de
T al espacio vectorial U . Luego, si v U , entonces:
n
X
v= i ui
i=1

As:
n
!
X
T (v) = T i ui
i=1
n
X
= i T (ui )
i=1
Xn
= T (ui )
i=r+1

y por tanto, T (ur+1 ) , T (ur+2 ) , . . . , T (un ) generan la imagen de T . Basta verificar, ahora,
que son linealmente independientes. En efecto, supongamos que:
nr
X
i T (ur+i ) = 0
i=1

Luego: !
nr
X
T i ur+i =0
i=1

es decir, si w = nr
P
i=1 i ur+i , se tiene entonces que w ker T . Por tanto, existen escalares
1 , 2 , . . . , r K tales que:
Xr
w= i ui
i=1
As:
r
X nr
X
i ui = i ur+i
i=1 i=1
O bien:
r
X nr
X
i ui i ur+i = 0
i=1 i=1

92
Apuntes Mat023 (Segundo semestre 2014)

Ahora bien, como u1 , u2 , . . . , ur , ur+1 , . . . , un es una base de U se tiene que:

1 = 2 = = r = 1 = 2 = = nr = 0

As, T (ur+1 ) , T (ur+2 ) , . . . , T (un ) es una base para la imagen de T . Por tanto, (T ) = r
y (T ) = n r.

Definicion 2.1.4. Sea T L (U, V ). Diremos que:

1. T es inyectiva si:
T (u) = T (v) = u = v
para todos u, v U .

2. T es epiyectiva si:
ImT = V

3. T es biyectiva si es inyectiva y epiyectiva, simultaneamente.

Ejemplo 2.1.15. No existe transformacion lineal T : R2 R3 que sea sobreyectiva. En


efecto, si T es sobreyectiva ImT = R3 luego (T ) = 3 pero del teorema de las dimensiones

(T ) + (T ) = Dim R2 = 2


como (T ) = 3 se obtiene
(T ) = 1
esto es una contradiccion pues (T ) 0.

Teorema 2.1.7. Sea T L (U, V ). Entonces, T es inyectiva, si y solo si, ker T = {U }.

Demostracion. Supongamos que ker T = {U }. Sean u, v U tales que T (u) = T (v) .


Como T es una transformacion lineal, se tiene T (u v) = V , es decir, u v ker T . As,
u v = U , y por tanto, T es inyectiva.
Por otro lado, supongamos que T es inyectiva, como {U } ker T , bastara verificar
que ker T {0}. En efecto, sea u ker T , esto es T (u) = V y como T es transformacion
lineal, se tiene que T (U ) = V . Es decir, T (u) = T (U ), luego u = U , pues T es inyectiva.
As, ker T = {U }.

93
Apuntes Mat023 (Segundo semestre 2014)

Isomorfismo
Definicion 2.1.5. Dos espacios vectoriales U y V sobre K se dicen isomorfos si existe una
transformacion lineal T : U V biyectiva. Tal transformacion T biyectiva se llamara iso-
morfismo entre U y V . El concepto de isomorfismo entre dos espacios se representara por:
U 'V.

Ejemplo 2.1.16. Sea U un espacio vectorial sobre R tal que u1 , u2 , . . . , un es una base de
U , entonces L : Rn U definida por:
n
X
(x1 , x2 , . . . , xn ) 7 L (x1 , x2 , . . . , xn ) = xi ui
i=1

es un isomorfismo entre U y Rn , esto se debe a que


n
X
(x1 , x2 , . . . , xn ) ker T xi ui = U
i=1
i, xi = 0

(pues u1 , u2 , . . . , un es una base de U ) as (x1 , x2 , . . . , xn ) ker T (x1 , x2 , . . . , xn ) =


(0, 0, . . . , 0) luego L es inyectiva y por el teorema de las dimensiones

n = dim Rn = 0 + (L)

as
(L) = n
esto implica ImT = U .

Teorema 2.1.8. Sean U y V espacios vectoriales de dimension finita sobre un cuerpo K.


Entonces, U ' V si y solo si dim U = dim V .

Ejemplo 2.1.17. M2 (R) ' R3 [x] ' R4

Ejemplo 2.1.18. Kmn ' Mmn (K)

Matriz asociada a una transformacion lineal


Definicion 2.1.6. Sean U un espacio vectorial sobre K y B = {u1 , u2 , . . . , un } una base
de U . Diremos que B es una base ordenada de U , si se considera B como la sucesion
finita de vectores en U :
B = (u1 , u2 , . . . , un )

94
Apuntes Mat023 (Segundo semestre 2014)

Definicion 2.1.7. Sea B = {u1 , u2 , . . . , un } una base ordenada de un espacio vectorial U


y sea u U . Los coeficientes i en la combinacion

u = 1 u1 + 2 u2 + + n un

son llamadas coordenadas de u con respecto a B y desde ahora usaremos la notacion



1
2
[u]B = .

..
n

Observacion 2.1.8. Estos escalares existen y son unicos por la definicion de base, el orden
de los elementos de la base es importante.

Ejemplo 2.1.19. Calcular las coordenadas del vector 1 + 2x + 3x2 en la base de R2 [x]
dada por B = {1 + x, x2 , 1} .

Solucion. Tenemos que encontrar escalares , , tales que

1 + 2x + 2x2 = (1 + x) + x2 + 1

desarrollando
1 + 2x + 3x2 = ( + ) + x + x2
as

1 = +
2 =
3 =

se sigue = 3 entonces

1 + 2x + 3x2 = 2 (1 + x) + 3 x2 3 (1)


de donde obtenemos
2
1 + 2x + 3x2 B = 3
 

Observacion 2.1.9. Dado en vector de coordenadas [v]B y conocida la base B se puede


recuperar el vector v.

95
Apuntes Mat023 (Segundo semestre 2014)

Ejemplo 2.1.20. Si el determinado vector v en M22 (R) tiene vector de coordenadas



1
1

[v]B =
2

2
       
1 1 1 1 1 1 1 0
en la base B = , , , entonces
1 1 1 0 0 0 0 0
      
1 1 1 1 1 1 1 0
v = 1 + (1) +2 + (2)
1 1 1 0 0 0 0 0
 
0 2
=
0 1

Observacion 2.1.10. Sean U y V espacios vectoriales sobre K tales que B = {u1 , u2 , . . . , un }


y D = {v1 , v2 , . . . , vm } son bases ordenadas de U y V , respectivamente. Note que para
cada i = 1, 2, . . . , n se tiene que T (ui ) V , por tanto, existen escalares Aji K tales que:
m
X
T (ui ) = A1 i v1 + A2 i v2 + + Am i vm = Aji vj , i = 1, 2, . . . , n
j=1

Es decir, los escalares A1 i , A2 i , . . . , Am i son las coordenadas de T (ui ) en la base D. As:



A1 i
A2 i
[T (ui )]D = . , i = 1, 2, . . . , n

. .
Am i

Definicion 2.1.8. Con respecto a las notaciones de la observacion anterior, se define la


matriz de T respecto de las bases B y D como la matriz A tal que A = (Ai j ) Mmn (R).
Esto es:

[T ]D

B = [T (u1 )D ] [T (u2 )D ] [T (un )D ]

A11 A12 A1n
A21 A22 A2n
= .

. .. . . ..
. . . .
Am1 Am2 Amn

96
Apuntes Mat023 (Segundo semestre 2014)

Ejemplo 2.1.21. Dada la transformacion lineal T : R3 R2 definida por:

T (x, y, z) = (3x 2y + z, x y)

Determine la matriz asociada a T respecto de las bases:

1. canonicas de R3 y R2 , respectivamente.
Solucion.

T (1, 0, 0) = (3, 1) = 3 (1, 0) + 1 (0, 1)


T (0, 1, 0) = (2, 1) = 2 (1, 0) 1 (0, 1)
T (0, 0, 1) = (1, 0) = 1 (1, 0) + 0 (0, 1)

as  
C 3 2 1
[T ]CR23 =
R 1 1 0

2. las bases B = {(1, 1, 0) ; (1, 0, 1) ; (1, 2, 3)} y D = {(2, 1) ; (1, 1)}


Solucion.

T (1, 1, 0) = (3 2, 1 1) = (1, 0) = 1 (2, 1) + 1 (1, 1)


T (1, 0, 1) = (3 + 1, 1) = (4, 1) = 2 (2, 1) + 2 (1, 1)
T (1, 2, 3) = (3 4 + 3, 1 2) = (2, 1) = 3 (2, 1) + 3 (1, 1)

determinemos los escalares, los determinaremos en un solo sistema


1 0 31 5 1
   
2 1 1 4 2 3 3

1 1 0 1 1 0 1 13 32 34
se sigue
1 5 1
 
[T ]D
B = 3 3 3
31 23 43

Ejemplo 2.1.22. Sea D : R3 [x] R3 [x] el operador lineal derivada, es decir, p D (p) =
p0 y considere las bases de R3 [x] dadas por

B1 = 2x3 , 3x x2 , 1 x, 1


B2 = 1, x, x2 , x3


luego

D 2x3 = 0 1 + 0x + 6x2 + 0x3



2
= 3 (1) 2 (x) + 0x2 + 0x3

D 3x x
D (1 x) = 1 (1) + 0x + 0x2 + 0x3
D (1) = 0 (1) + 0x + 0x2 + 0x3

97
Apuntes Mat023 (Segundo semestre 2014)

entonces
0 3 1 0
0 2 0 0

[D]BB21 =

6 0 0 0


0 0 0 0

usted puede calcular [D]B1


B2 y vera que es una matriz diferente.

Ejemplo 2.1.23. Si B = {v1 , v2 , . . . , vn } es una base del espacio vectorial V y I : V V


la transformacion identidad entonces

[I]BB = Inn (la matriz identidad)

En efecto, Iv1 = v1 = 1v1 + 0v2 + 0v3 + + 0vn se sigue



1
0
[Iv1 ]B = .

..
0

similarmente Iv2 = v1 = 0v1 + 1v2 + 0v3 + + 0vn se sigue



0

1

[Iv2 ]B =
0

..
.
0

as [I]BB es
[I]BB = Inn

Definicion 2.1.9. Sean U un espacio vectorial sobre K tales que B = {u1 , u2 , . . . , un } y


D = {v1 , v2 , . . . , vn } son bases ordenadas distintas de U . Sea 1 : U U la transformacion
lineal identidad. Se define la matriz cambio de base como la matriz asociada a 1 respecto
de las bases B y D.

Teorema 2.1.9. Sean U y V espacios vectoriales finito dimensionales sobre K con bases B
y D, respectivamente. Sean, ademas, S, T : U V dos transformaciones lineales, entonces:

[T + S]D D D
B = [T ]B + [S]B

98
Apuntes Mat023 (Segundo semestre 2014)

Teorema 2.1.10. Sean U, V y W espacios vectoriales de dimension finita sobre K tales que
B, D y E son bases de U, V y W , respectivamente. Suponga que T : U V y S : V W
son transformaciones lineales, entonces:

[ST ]EB = [S]ED [T ]D


B

Observacion 2.1.11. Sean U y V espacios vectoriales finito dimensionales sobre K con


bases B y E para U , y D y F para V . Considere, ademas, T : U V una transformacion
lineal. Note que en vista del teorema anterior el cambio de representacion matricial puede
expresarse mediante el siguiente diagrama conmutativo:

E[T ]F
UE VF
[1]EB [1]D
F
B[T ]D
UB VD

donde la notacion del tipo UE , por ejemplo, representa el espacio U considerado con la
base E. En efecto, utilizando la convencion usual para la composicion de funciones, tenemos
que:
[T ]D D F E
B = [1]F [T ]E [1]B

Teorema 2.1.11. Sean U y V espacios vectoriales finito dimensionales sobre K con bases
B y D, respectivamente. Sea, ademas, T : U V una transformacion lineal entonces T es
invertible si y solo si [T ]D
B es invertible, ademas

 1 B  D 1
T D = [T ]B

Teorema 2.1.12. Sean U y V espacios vectoriales sobre el mismo cuerpo K tales que
B = {u1 , u2 , . . . , un } es una base ordenada para U y D = {v1 , v2 , . . . , vm } es una base
ordenada para V , entonces:
[T u]D = [T ]D
B [u]B

para todo u U .

Ejemplo 2.1.24. Considere las bases de R3

B1 = {(1, 1, 1) , (1, 1, 0) , (1, 0, 0)}

y
B2 = {(0, 0, 1) , (1, 0, 0) , (0, 1, 0)}

99
Apuntes Mat023 (Segundo semestre 2014)

1. Calcular [I]BB21 y [I]BB12 .

2. Calcular [v]Bi para i = 1, 2 donde v = (3, 1, 3)

1. Notemos que

I (1, 1, 1) = (1, 1, 1) = 1 (0, 0, 1) + 1 (1, 0, 0) + 1 (0, 1, 0)


I (1, 1, 0) = (1, 1, 0) = 0 (0, 0, 1) + 1 (1, 0, 0) + 1 (0, 1, 0)
I (1, 0, 0) = (1, 0, 0) = 0 (0, 0, 1) + 1 (1, 0, 0) + 0 (0, 1, 0)

entonces
1 0 0
[I]BB21 = 1 1 1
1 1 0
 1  1
como [I]BB12 = [I]BB21 se sigue que [I]BB12 = [I]BB21

1 0 0 1 0 0 1 0 0 1 0 0
1 1 1 0 1 0 0 1 0 1 0 1
1 1 0 0 0 1 0 0 1 0 1 1

as
1 0 0
[I]BB12 = 1 0 1
0 1 1

2. Notemos que (3, 1, 3) = 3 (0, 0, 1) + 3 (1, 0, 0) + 1 (0, 1, 0)



3
[(3, 1, 3)]B2 = 3
1

luego

[(3, 1, 3)]B1 = [I]BB12 [(3, 1, 3)]B2



1 0 0 3
= 1 0 1 3
0 1 1 1

3
= 2
2

100
Apuntes Mat023 (Segundo semestre 2014)

Ejemplo 2.1.25. Sean B = {(1, 1, 1) ; (0, 0, 1) ; (1, 0, 1)} y D = {(1, 1) ; (0, 1)} bases de
R3 y R2 , respectivamente. Sea T una transformacion lineal tal que:
 
D 4 2 2
[T ]B =
3 1 1

Hallar explcitamente T (x, y, z).

Solucion. Primero buscamos [(x, y, z)]B esto es

(x, y, z) = (1, 1, 1) + (0, 0, 1) + (1, 0, 1)

luego

1 0 1 x 1 0 0 y
1 0 0 y 0 1 0 zx
1 1 1 z 0 0 1 xy
as
y
[(x, y, z)]B = z x
xy
se sigue que

[T (x, y, z)]D = [T ]D
B [(x, y, z)]B

  y
4 2 2
= zx
3 1 1
xy
 
6y 4x + 2z
=
2x 4y z

de donde obtenemos

T (x, y, z) = (6y 4x + 2z) (1, 1) + (2x 4y z) (0, 1)


= (6y 4x + 2z, 2y 2x + z)

Calculo con coordenadas


Observacion 2.1.12. Como se puede observar de la seccion anterior, las matrices asociadas
a las transformaciones lineales dependen de las coordenadas. Es decir, dependen de las
bases consideradas en los espacios vectoriales involucrados. Por consiguiente, el calculo
de los subespacios notables (nucleo e imagen) asociados a una transformacion lineal no
es directo por definicion. Sin embargo, con los procedimientos algebraicos adecuados se
pueden obtener los espacios notables asociados a las transformaciones a traves de calculos
con coordenadas. Veamos algunos ejemplos:

101
Apuntes Mat023 (Segundo semestre 2014)

Ejemplo 2.1.26. Sean:

B = {(1, 1, 1) ; (0, 1, 1) ; (1, 0, 1)}

y:
D = {(1, 1, 1) ; (1, 0, 1) ; (1, 2, 3)}
bases de R3 y T L (R3 , R3 ) tales que:

2 2

4
[T ]D
B =
3 1 1
1 2 2

Calcule el nucleo y la imagen de T , sin explicitar T (x, y, z).

Solucion. Sea u un vector cualquiera en el nucleo de T , es decir T (u) = 0. Por tanto,


[T u]D = [0]D = 0. Sin embargo, sabemos que [T u]D = [T ]D
B [u]B , luego:

2 2

4 a 0
3 1 1 b = 0
1 2 2 c 0
T
en donde [u]B = a b c . Escalonando la matriz de coeficientes:

2 2

4 1 0 0
3 1 1 0 1 1
1 2 2 0 0 0

se obtiene que a = 0 y b = c. Por lo tanto:



0
[u]B = c , cR
c

Luego:
u = 0 (1, 1, 1) + c (0, 1, 1) + c (1, 0, 1) = (c, c, 2c) , cR
Por lo tanto, obtenemos de lo anterior que:

ker T = h (1, 1, 2) i

Para calcular la imagen de T , debemos notar primeramente que de:

2 2

4
[T ]D
B =
3 1 1
1 2 2

102
Apuntes Mat023 (Segundo semestre 2014)

se obtiene que:

2

4 2
[T (1, 1, 1)]D = 3 , [T (0, 1, 1)]D = 1 , [T (1, 0, 1)]D = 1
1 2 2

Por consiguiente:

T (1, 1, 1) = 4 (1, 1, 1) + (3) (1, 0, 1) + 1 (1, 2, 3) = (2, 6, 4)


T (0, 1, 1) = 2 (1, 1, 1) + (1) (1, 0, 1) + 2 (1, 2, 3) = (3, 6, 7)
T (1, 0, 1) = (2) (1, 1, 1) + 1 (1, 0, 1) + (2) (1, 2, 3) = (3, 6, 7)

Ahora bien:

ImT = hT (1, 1, 1) , T (0, 1, 1) , T (1, 0, 1)i


= h{(2, 6, 4) , (3, 6, 7) , (3, 6, 7)}i
= h{(2, 6, 4) , (3, 6, 7)} i

note que el rango de T es 2.

Ejemplo 2.1.27. Sean:

B = {(1, 1, 1) ; (0, 2, 1) ; (1, 0, 1)}

y:
D = 1 + 2x + x2 ; 1 + 3x + 2x2 ; 2 + x + 3x2


bases de R3 y R2 [x], respectivamente. Sea T L (R3 , R2 [x]) tales que:

2 2

5
[T ]D
B =
3 1 1
1 4 3

Verifique que T es un isomorfismo entre R3 y R2 [x]. Calcule, ademas, T 1 .

Solucion. Para verificar que T es un isomorfismo, basta notar que det [T ]D 6 0. En efecto:
B =

5 2 2
det [T ]D

B = 3 1 1 = 1

1 4 3

Por otro lado, para calcular T 1 , procedemos por coordenadas. En efecto:


 1 B
T (a, b, c) B = T 1 D [(a, b, c)]D
 
 1
= [T ]DB [(a, b, c)]D

103
Apuntes Mat023 (Segundo semestre 2014)

Luego:
1
2 2

5
1 B
 
T D
= 3 1 1
1 4 3
1 2 0

= 8 13 1
11 18 1
Necesitamos, ahora, las coordenadas de a + bx + cx2 respecto de la base D. Esto es:
1 + 2x + x2 + 1 + 3x + 2x2 + 2 + x + 3x2 = a + bx + cx2
  

Resolviendo el sistema anterior, obtenemos:




a + bx + cx2 D
 
=

7
a + 14 b 54 c

4
= 41 b 54 a + 43 c
1
4
c + 14 a 14 b
Luego:
7
1 2 0 a + 14 b 54 c

4
T 1 a + bx + cx2
 
B
= 8 13 1 41 b 54 a + 43 c
1
11 18 1 4
c + 14 a 41 b
3
a 34 b 14 c

4
= 52 a 11
2
b + 21 c
7
2
a 15
2
b + 21 c
As:
   
1 2
 3 3 1 5 11 1
T a + bx + cx = a b c (1, 1, 1) + a b + c (0, 2, 1)
4 4 4 2 2 2
 
7 15 1
+ a b + c (1, 0, 1)
2 2 2
 
17 33 1 23 47 3 1 5 1
= a b + c, a b + c, a b + c
4 4 4 4 4 4 4 4 4

Ejercicios resueltos de Transformaciones lineales

1. Considere la transformacion
T : M22 (R) R4
   
a b a b
T = (a + b, a + c b, c, d)
c d c d

104
Apuntes Mat023 (Segundo semestre 2014)

a) Determine una base de Ker(T ) e Im(T ).


Solucion. Vamos a buscar el Nucleo de la transformacion,
    
a b a b
ker (T ) = M22 (R) : T = (0, 0, 0, 0)
c d c d
  
a b
= M22 (R) : (a + b, a + c b, c, d) = (0, 0, 0, 0)
c d


a + b = 0


 
a b ab+c = 0

= M22 (R) :

c d c = 0

d = 0
  
a b
= M22 (R) : a = b = c = d = 0
c d
 
0 0
=
0 0

del teorema de las dimensiones se sigue que

Dim (ker (T )) + Dim (Im (T )) = Dim (M22 (R))


0 + Dim (Im (T )) = 4

as Dim (Im (T )) = 4 y luego Im (T ) = R4 (T es biyectiva)


b) Si
       
1 1 0 1 0 1 1 1
B1 = , , ,
2 0 2 1 1 1 0 3
y
B2 = {(1, 0, 2, 0) , (1, 0, 2, 1) , (0, 0, 1, 1) , (1, 1, 0, 3)}
son bases de M22 (R) y R4 respectivamente determinar [T ]BB21 .
Solucion. Como
 
1 1
T = (0, 4, 2, 0)
2 0
 
0 1
T = (1, 3, 2, 1)
2 1
 
0 1
T = (1, 0, 1, 1)
1 1
 
1 1
T = (2, 0, 0, 3)
0 3

105
Apuntes Mat023 (Segundo semestre 2014)

se sigue que necesitamos resolver los sistemas


(0, 4, 2, 0) = 1 (1, 0, 2, 0) + 1 (1, 0, 2, 1) + 1 (0, 0, 1, 1) + 1 (1, 1, 0, 3)
(1, 3, 2, 1) = 2 (1, 0, 2, 0) + 2 (1, 0, 2, 1) + 2 (0, 0, 1, 1) + 2 (1, 1, 0, 3)
(1, 0, 1, 1) = 3 (1, 0, 2, 0) + 3 (1, 0, 2, 1) + 3 (0, 0, 1, 1) + 3 (1, 1, 0, 3)
(2, 0, 0, 3) = 4 (1, 0, 2, 0) + 4 (1, 0, 2, 1) + 4 (0, 0, 1, 1) + 4 (1, 1, 0, 3)
y as la matriz asociada sera

1 2 3 4
1 2 3 4

[T ]BB21 =

1 2 3 4


1 2 3 4
vamos a resolver los 4 sistema de una sola vez

1 1 0 1 0 1 1 2 1 0 0 0 6 4 1 5
0 0 0 1 4 3 0 0 0 1 0 0 2 0 2 7



2 2 1 0 2 2 1 0 0 0 1 0 10 10 1 4


0 1 1 3 0 1 1 3 0 0 0 1 4 3 0 0
as
6 4 1 5
2 0 2 7

[T ]BB21 =

10 10 1 4


4 3 0 0

c) Si la transformacion es un isomorfismo (biyectiva) determine


 1 B1
T B2
Solucion. ya sabemos que la transformacion es biyectiva, como
 1 B1  B2 1
T B2 = [T ]B1
se sigue
3 3 9 17

22
22 22 11
1
6 4 1 5
2 2 6

11 11 11
19
11

2 0 2 7

=

10 10 1 4

23 21 8 4
11 11 11 11

4 3 0 0



7 4 1 6
11 11 11 11
B
= T 1 B12


106
Apuntes Mat023 (Segundo semestre 2014)

2. Construir una transformacion lineal T : R3 R3 que cumpla las siguientes condicio-


nes (simultaneamente)

a) Ker(T ) = {(x, y, z) R3 : (x, y, z) = t (1, 0, 1) para t R}


b) Im(T ) = {(x, y, z) R3 : x 2y + z = 0}
Solucion. La transformacion tiene que cumplir las dos condiciones, notamos que
ker (T ) = h(1, 0, 1)i
y
(x, y, z) R3 : x 2y + z = 0

Im (T ) =
= (x, y, z) R3 : x + z = 2y

  
x+z 3
= x, , z R : x, z R
2
   
1 1
= 1, , 0 , 0, , 1
2 2
desde el punto de vista del teorema de las dimensiones estas dos condiciones no son
incompatibles puesto que
Dim (ker (T )) + Dim (Im (T )) = Dim R3 = 3


para la posible transformacion. Vamos a utilizar el teorema que nos permite construir
transformaciones si la conocemos en una base
 
1
T (1, 0, 0) = 1, , 0
2
 
1
T (0, 1, 0) = 0, , 1
2
T (1, 0, 1) = (0, 0, 0)
notamos que los vectores (1, 0, 0) , (0, 1, 0) y (1, 0, 1) son linealmente independientes
pues
1 0 1

0 1 0 = 1 6= 0

0 0 1
y luego forman una base de R3 estas condiciones determinan por completo una
transformacion lineal, note tambien que
Im (T ) = h{T (1, 0, 0) , T (0, 1, 0) , T (1, 0, 1)}i
    
1 1
= 1, , 0 , 0, , 1 , (0, 0, 0)
2 2
   
1 1
= 1, , 0 , 0, , 1
2 2
3

= (x, y, z) R : x 2y + z = 0

107
Apuntes Mat023 (Segundo semestre 2014)

y (1, 0, 1) ker (T ) que tiene dimension 1 (por el teorema de las dimensiones) as

ker (T ) = h{(1, 0, 1)}i


= (x, y, z) R3 : (x, y, z) = t (1, 0, 1) para t R


podemos tambien determinar en forma explcita esta transformacion

(x, y, z) = (1, 0, 0) + (0, 1, 0) + (1, 0, 1)

+ = x
= y
= z

de donde = x + z, = y, = z as

(x, y, z) = (x + z) (1, 0, 0) + y (0, 1, 0) + (z) (1, 0, 1)

luego

T (x, y, z) = T ((x + z) (1, 0, 0) + y (0, 1, 0) + (z) (1, 0, 1))


= (x + z) T (1, 0, 0) + yT (0, 1, 0) + (z) T (1, 0, 1)
   
1 1
= (x + z) 1, , 0 + y 0, , 1 + (z) (0, 0, 0)
2 2
 
1 1 1
= x + z, x + y + z, y
2 2 2

3. Sea T L (R3 , R2 ) con B = {(0, 0, 1) ; (0, 2, 1) ; (3, 2, 1)} y C = {(1, 3) ; (2, 5)}
bases ordenadas de R3 y R2 , respectivamente. Supongamos que:
 
C 1 3 2
[T ]B = M23 (R)
4 12 8

Con respecto a las hipotesis anteriores:

a) Hallar ker T sin calcular T (x, y, z).


Solucion. v ker (T ) [v]B es solucion de [T ]CB x = 0 luego
   
1 3 2 0 1 3 2 0

4 12 8 0 0 0 0 0
luego


[v]B =

108
Apuntes Mat023 (Segundo semestre 2014)

es solucion si y solo si
3 2 = 0
de esta forma

3 + 2
[v]B = = para , R

luego

v ker (T ) (x, y, z) = (3 + 2) (0, 0, 1) + (0, 2, 1) + (3, 2, 1)

para , R esto es

(x, y, z) = (3, 2 + 2, 4 + 3) para , R

as
ker (T ) = h(3, 2, 3) , (0, 2, 4)i
b) Determinar T (x, y, z).
Solucion. Notemos que

[T (x, y, z)]C = [T ]CB [(x, y, z)]B

luego
(x, y, z) = (0, 0, 1) + (0, 2, 1) + (3, 2, 1)
as

3 = x
2 + 2 = y
++ = z

luego = z 21 y, = 21 y 13 x, = 13 x de donde tenemos

z 12 y

[(x, y, z)] = 1 y 1 x
B 2 3
1
3
x
as
z 12 y

 
1 3 2 1y 1x
[T (x, y, z)]C = 2 3
4 12 8 1
3
x
2y 13 x z
 
= 4
3
x 8y + 4z

109
Apuntes Mat023 (Segundo semestre 2014)

as
   
1 4
T (x, y, z) = 2y x z (1, 3) + x 8y + 4z (2, 5)
3 3
 
7 17
= x 14y + 7z, 34y x 17z
3 3

x
c) Por que es falso que T (x, y, z) = [T ]CB y ?
z
Solucion. Esta igualdad no es posible ni por los ordenes de las matrices.
d ) Cuando es valida la igualdad anterior?
Solucion. Al considerar las bases canonicas en los espacios de partida y llegada
se cumple una desigualdad del tipo

x
t canonica
T (x, y, z) = [T ]
canonica y
z

(ademas la transformacion este definida de Rn en Rm )


e) Hallar la nulidad de T y el rango de T sin hallar la Im(T )?
Solucion. El rango de la transformacion lineal es igual al rango de la matriz
asociada en este caso es 1.
f ) Hallar Im(T ) y una base para Im(T ).
Solucion. Como ya tenemos la transformacion podemos determinar la imagen
como

Im (T ) = hT (1, 0, 0) , T (0, 1, 0) , T (0, 0, 1)i


  
7 17
= , , (14, 34) , (7, 17)
3 3
= h(7, 17)i

esto es una recta que pasa por el origen con direccion (7, 17).

4. Sean U = {(x, y, z) R3 : 2x 3y + 5z = 0} un espacio vectorial y A una base


ordenada de U y C la base canonica de R2 . Suponga que T : U R2 es una
transformacion lineal definida por T (x, y, z) = (x y, x z) tal que:
 
C 1 2
[T ]A = M22 (R)
1 1
Determine la base A.

110
Apuntes Mat023 (Segundo semestre 2014)

Solucion. Sea A = {(, , ) , (, , )} la base de U del ejercicio (el espacio tiene


dimension 2) entonces
 
1
[T (, , )]C =
1
 
2
[T (, , )]C =
1
ademas

2 3 + 5 = 0
2 3 + 5 = 0

pero
 

[T (, , )]C =

 

[T (, , )]C =

de donde tenemos el sistema de ecuaciones

= 1
= 1
= 2
= 1
2 3 + 5 = 0
2 3 + 5 = 0

que tiene solucion = 21 , = 49 , = 12 , = 54 , = 12 , = 14 as


   
1 1 1 1 9 5
A= , , , , ,
2 2 2 4 4 4

5. Considere la transformacion lineal


T : M22 (R) R4
   
x y x y
T = (x y, x z, x w, x)
z w z w
y suponga que
       
1 0 2 0 0 0 0 1
B1 = , , ,
0 1 0 0 1 0 0 0
B2 = {(1, 1, 1, 1) , (1, 1, 0, 0) , (1, 0, 0, 0) , (1, 1, 1, 0)}

111
Apuntes Mat023 (Segundo semestre 2014)

a) Calcular [T ]BB21 , muestre que T es invertible (isomorfismo) y calcule T 1 en forma


explcita.
Solucion. Como
 
x y
T = (x y, x z, x w, x)
z w

se sigue
 
1 0
T = (1, 1, 0, 1)
0 1
 
2 0
T = (2, 2, 2, 2)
0 0
 
0 0
T = (0, 1, 0, 0)
1 0
 
0 1
T = (1, 0, 0, 0)
0 0

luego tenemos que resolver los sistemas

(1, 1, 0, 1) = 1 (1, 1, 1, 1) + 1 (1, 1, 0, 0) + 1 (1, 0, 0, 0) + 1 (1, 1, 1, 0)


(2, 2, 2, 2) = 2 (1, 1, 1, 1) + 2 (1, 1, 0, 0) + 2 (1, 0, 0, 0) + 2 (1, 1, 1, 0)
(0, 1, 0, 0) = 3 (1, 1, 1, 1) + 3 (1, 1, 0, 0) + 3 (1, 0, 0, 0) + 3 (1, 1, 1, 0)
(1, 0, 0, 0) = 4 (1, 1, 1, 1) + 4 (1, 1, 0, 0) + 4 (1, 0, 0, 0) + 4 (1, 1, 1, 0)

resolveremos los 4 sistemas



1 1 1 1 1 2 0 1 1 0 0 0 1 2 0 0
1 1 0 1 1 2 1 0 0 1 0 0 1 0 1 0



1 0 0 1 0 2 0 0 0 0 1 0 0 0 1 1


1 0 0 0 1 2 0 0 0 0 0 1 1 0 0 0

se sigue
1 2 0 0
1 0 1 0

[T ]BB21 =

0 0 1 1


1 0 0 0
 1
b) Encontrar [T ]BB21

112
Apuntes Mat023 (Segundo semestre 2014)

Solucion.
 1 B
[T ]BB21 T 1 B12

=
1
1 2 0 0
1 0 1 0

=

0 0 1 1


1 0 0 0

0 0 0 1
1
0 0 12

= 2
0 1 0 1

0 1 1 1

c) Considere las bases canonicas deM22 (R) y R4


       
1 0 0 1 0 0 0 0
C1 = , , ,
0 0 0 0 1 0 0 1

C2 = {((1, 0, 0, 0)) , (0, 1, 0, 0) , (0, 0, 1, 0) , (0, 0, 0, 1)}


calcular [T ]CC21 usando la matrices de cambio de base adecuadas.
Solucion. Para calcular [T ]CC21 necesitamos hacer

[I]CB22 [T ]BB21 [I]BC11

as
         
1 0 2 0 0 0 0 1 2
+ + + =
0 1 0 0 1 0 0 0

de donde tenemos que resolver 4 sistemas



1 2 0 0 1 0 0 0 1 0 0 0 0 0 0 1
1
0 0 0 1 0 1 0 0 0 1 0 0 2 0 0 12


0 0 1 0 0 0 1 0 0 0 1 0 0 0 1 0

1 0 0 0 0 0 0 1 0 0 0 1 0 1 0 0

de aqu
0 0 0 1
1
2 0 0 12

[I]BC11 =

0 0 1 0


0 1 0 0

113
Apuntes Mat023 (Segundo semestre 2014)

 1
(note que es mas facil pensarlo como [I]CB11 = [I]BC11 si la canonica esta en la
llegada es escribir los vectores como columna) y

1 1 1 1
1 1 0 1

[I]CB22 =
1 0 0 1

1 0 0 0
as
[T ]CC21 = [I]CB22 [T ]BB21 [I]BC11

1 1 1 1 1 2 0 0 0 0 0 1
1
1 1 0 1 1 0 1 0 2 0 0 12

=

1 0 0 1 0 0 1 1 0 0 1 0


1 0 0 0 1 0 0 0 0 1 0 0

1 1 0 0
1 0 1 0

=

1 0 0 1


1 0 0 0

6. Considere la transformacion lineal T : R2 [x] R2 [x] dada por:


T ax2 + bx + c = (a + b) x2 + (a + b + c) x + c


a) Encuentre una base para ker (T )


b) Encuentre una base para Im (T )
c) Considere en el dominio la base B = {1 + x2 , 1 + x + x2 , 1} y en el codominio
la base D = {1 x, 1 + x, x2 } encontrar la matriz asociada a T respecto a estas
bases.

a) Por definicion
ker (T ) = ax2 + bx + c R2 [x] : T ax2 + bx + c = 0
 

= ax2 + bx + c R2 [x] : (a + b) x2 + (a + b + c) x + c = 0


= ax2 + bx + c R2 [x] : (a + b = 0) (a + b + c = 0) (c = 0)


1 1 0 a 0
2
= ax + bx + c R2 [x] : 1 1 1 b = 0

0 0 1 c 0
luego

1 1 0 1 1 0
1 1 1 0 0 1
0 0 1 0 0 0

114
Apuntes Mat023 (Segundo semestre 2014)

as
 2
ker (T ) = ax + bx + c R2 [x] : (a + b = 0) (c = 0)
= ax2 + bx + c R2 [x] : (b = a) (c = 0)


= ax2 ax R2 [x] : a R


 2
= x x

luego el kernel tiene dimension 1 (la nulidad es 1).


b) Notemos que T (ax2 + bx + c) = (a + b) x2 + (a + b + c) x + c y luego

(a + b) x2 + (a + b + c) x + c = a x2 + x + b x2 + x + c (x + 1)
 

as
T ax2 + bx + c x2 + x, x2 + x, x + 1



como hay un elemento repetido, el conjunto es L.D. as



 2
x + x, x2 + x, x + 1 = x2 + x, x + 1



se sigue
x2 + x, x + 1


Im (T )
por el teorema de las dimensiones se sigue

Dim (ker (T )) + Dim (Im (T )) = Dim (R2 [x])

as, de la parte a) se tiene

1 + Dim (Im (T )) = 3

de donde se obtiene
Dim (Im (T )) = 2
y as
x2 + x, x + 1


Im (T ) =
de esta forma, una base para la imagen es {x2 + x, x + 1}
c) Como T (ax2 + bx + c) = (a + b) x2 + (a + b + c) x + c se sigue que

T 1 + x2 = x2 + 2x + 1 = (1 x) + (1 + x) + x2


= x2 + ( ) x + (a + )

por determinar a, , .

T 1 + x + x2 = 2x2 + 3x + 1 = (1 x) + (1 + x) + x2


= x2 + ( ) x + (a + )

115
Apuntes Mat023 (Segundo semestre 2014)

por determinar a, , .

T (1) = x 1 = (1 x) + (1 + x) + x2
= x2 + ( ) x + (a + )

por determinar a, , .
Vamos a resolver esos 3 problemas de una sola vez, porque hay que resolver los
sistemas

= 1
( ) = 2
(a + ) = 1

= 2
( ) = 3
(a + ) = 1

= 0
( ) = 1
(a + ) = 1

en todas la matriz de coeficientes es la misma, formamos una tripe matriz


ampliada y escalonamos
1 0 0 21 1 0

0 0 1 1 2 0
Operaciones Elementales
1 1 0 2 3 1 0 1 0 3 2 1
2
1 1 0 1 1 1 0 0 1 1 2 0
1
2 1 0

[T ]D
B =
3
2
2 1
1 2 0
(notar que T (1 + x2 ) = x2 + 2x + 1 = 12 (1 x) + 32 (1 + x) + 1x2 ,
 

T (1 + x + x2 ) = 2x2 + 3x + 1 = (1) (1 x) + 2 (1 + x) + 2x2 y T (1) =


x 1 = 0 (1 x) + (1) (1 + x) + 0x2 )

7. Suponga que S : R2 [x] R2 [x] es una transformacion lineal con


1
2 1 0

[T ]D
B =
3
2
2 1
1 2 0

116
Apuntes Mat023 (Segundo semestre 2014)

B = {1 x, 1 + x, x2 } y D = {1 + x2 , 1 + x + x2 , 1} (No es el ejercicio anterior ver


las bases)

a) Encontrar ker (T )
b) Encontrar Im (T )
c) Calcule T (2 + x x2 )
d ) Encontrar una expresion para T (ax2 + bx + c)

a) Note que p ker (T ) si y solo si T (p) = 0 si y solo si



0
[T p]D = 0
0
pero
[T p]D = [T ]D
B [p]B

luego si resolvemos
1
2 1 0

0
0 = [T ]D [p] = 3 2 1
B B 2
0 1 2 0

estaremos encontrando las coordenadas




= [p]
B

de los vectores de ker (T ). Pues bien

12 1 0 1 0 2

3 2 1 0 1 1
2
1 2 0 0 0 0

se sigue que p ker (T ) si y solo si




= [p]
B

cumple
= 2 y =

117
Apuntes Mat023 (Segundo semestre 2014)

as p ker (T ) si y solo si

p (x) = (2) (1 x) + () (1 + x) + x2

para algun R es decir

3x + x2 + 1


ker (T ) =

b) De la matriz asociada tenemos

21

3
[T (1 x)]D = 2
as
1
   
1 2 3
1 + x + x2 + (1) (1)
 
T (1 x) = 1+x +
2 2
3
= x2 + x
2
tambien
1

[T (1 + x)]D = 2 as
2
T (1 x) = (1) 1 + x2 + (2) 1 + x + x2 + (2) (1)
 

= x2 + 2x 1

y


0
T x2 D = 1 as
 

0
T x2 = (0) 1 + x2 + (1) 1 + x + x2 + (0) (1)
  

= x2 x 1

luego  
3 2 2 2
Im (T ) = x + x, x + 2x 1, x x 1
2
 2 3
del teorema de las dimensiones x + 2 x, x2 + 2x 1, x2 x 1 debera ser
un conjunto L.D. en efecto

1 1 1 1 0 2

3 2 1 0 1 1
2
0 1 1 0 0 0

118
Apuntes Mat023 (Segundo semestre 2014)

 
3
(2) x + x + (1) x2 + 2x 1 = x2 x 1
2

2
se sigue  
2 3 2
Im (T ) = x + x, x + 2x 1
2
c) Para calcular T (2 + x x2 ) podemos hacer lo siguiente


2
 
2+xx B
=

donde
2 + x x2 = (1 x) + (1 + x) + x2

= 1
= 1
+ = 2

= 21 , = 23 , = 1 luego

21 1 0
1 7
4

2
2
  3
T 2+xx D = 2 2 1 2 = 19
3
4

7
1 2 0 1 2

se obtiene
     
2
 7 2
 19 2
 7
T 2+xx = 1+x + 1+x+x + (1)
4 4 2
19 1
= 3x2 + x
4 2

d ) Encontremos una expresion para

T ax2 + bx + c


notemos que
ax2 + bx + c = (1 x) + (1 + x) + x2
si y solo si

= a
= b
+ = c

119
Apuntes Mat023 (Segundo semestre 2014)

esto es
1
21 b

0 0 1 a 1 0 0 2
c
1
1 1 0 b 0 1 0
2
b + 12 c
1 1 0 c 0 0 1 a
luego
   
2 1 1 1 1
ax + bx + c = c b (1 x) + b + c (1 + x) + ax2
2 2 2 2

podemos encontrar T (ax2 + bx + c) en dos formas


   
2 1 1 1 1
b + c T (1 + x) + aT x2
 
T ax + bx + c = c b T (1 x) +
2 2 2 2
    
1 1 2 3 1 1
b + c x2 + 2x 1

= c b x + x +
2 2 2 2 2
2

+a x x 1
   
2 1 7 1 1
= (c a) x + a + b + c x + b c a
4 4 2 2

la otra forma es

12 1 0 1
12 b

2
c
T ax2 + bx + c
 
= 32 1
+ 21 c

D
2 1 2
b
1 2 0 a
14 b 34 c

= 14 b a + 47 c
1
2
b + 32 c

y as
   
2 1 3 2 1 7
b a + c 1 + x + x2
  
T ax + bx + c = b c 1+x +
4 4 4 4
 
1 3
+ b + c (1)
2 2
   
2 1 7 1 1
= (c a) x + a + b + c x + b c a
4 4 2 2

8. Muestre que T : M22 (R) R3 [x] dada por


 
a b
T = (a + b) x3 + cx2 + (a + d) x + (c d)
c d

es una transformacion lineal ademas determine el ker (T ) e Im(T ).

120
Apuntes Mat023 (Segundo semestre 2014)

Solucion. En general, si V y W son espacios vectoriales, una funcion F : V W


es llamada transformacion lineal si cumple: i) K,v V , F (v) = F (v) y ii)
v1 , v2 V , F (v1 + v2 ) = F (v1 ) + F (v2 ) (esto quiere decir, que enva combinaciones
lineales en combinaciones lineales). En nuestro ejercicio los elementos del espacio
de partida son matrices de orden 2 2 luego para probar la primera propiedad
necesitamos considerar un escalar arbitrario y una matriz arbitraria y en la segunda
propiedad, dos matrices arbitrarias como sigue:
 
a b
i) Sea R y M22 (R) se tiene
c d
    
a b a b
T = T
c d c d
 
A B
= T donde A = a, B = b, C = c, D = d
C D
= (A + B) x3 + Cx2 + (A + D) x + (C D)
= ((a) + (b)) x3 + (c) x2 + (a + d) x + (c d)
= (a + b) x3 + cx2 + (a + d) x + (c d)

 
a b
= T
c d

(Explicacion: tenemos que mostrar que el escalar se puede sacar de la funcion y para
    
a b a b
esto utilizamos solamente la definicion, en la linea T =T
c d c d
 
a b
no sabemos como actua la funcion T sobre el producto escalar es por
c d
eso que aplicamos la definicion de producto escalar de matrices y as
   
a b a b
=
c d c d
 
a b
y ahora podemos aplicar la definicion de T a la matriz (en este caso si
c d
sabemos como actua T ), para que se vea mas claro agregue la linea
   
a b A B
= donde A = a, B = b, C = c, D = d
c d C D
pero no es necesario, entonces por definicion
 
A B
T
C D
= (A + B) x3 + Cx2 + (A + D) x + (C D)

121
Apuntes Mat023 (Segundo semestre 2014)

y ahora reemplazamos para obtener la igualdad deseada)


   
a11 a12 b11 b12
ii) Sean , M22 (R) matrices arbitrarias, se tiene
a21 a22 b21 b22
     
a11 a12 b11 b12 a11 + b11 a12 + b12
T + = T
a21 a22 b21 b22 a21 + b21 a22 + b22
 
A B
= T
C D

donde A = a11 +b11 , B = a12 +b12 , C = a21 +b21 , D = a22 +b22 , aplicando la definicion
de T se sigue
 
A B
T = (A + B) x3 + Cx2 + (A + D) x + (C D)
C D
= ((a11 + b11 ) + (a12 + b12 )) x3 + (a21 + b21 ) x2 +
((a11 + b11 ) + (a22 + b22 )) x + ((a21 + b21 ) (a22 + b22 ))
= ((a11 + a12 ) + (b11 + b12 )) x3 + (a21 + b21 ) x2 +
((a11 + a22 ) + (b11 + b22 )) x + ((a21 a22 ) + (b21 b22 ))
= (a11 + a12 ) x3 + a21 x2 + (a11 + a22 ) x + (a21 a22 ) +


(b11 + b12 ) x3 + b21 x2 + (b11 + b22 ) x + (b21 b22 )



   
a11 a12 b11 b12
= T +T
a21 a22 b21 b22

de esto concluimos
   
a11 a12 b11 b12
T +
a21 a22 b21 b22
   
a11 a12 b11 b12
= T +T
a21 a22 b21 b22

de i) y ii) T es una transformacion lineal.


Vamos a buscar el ker (T )

ker (T ) = {v V : T (v) = W }
    
a b a b
= M22 (R) : T = 0 (el polinomio 0)
c d c d
  
a b 3 2
= M22 (R) : (a + b) x + cx + (a + d) x + (c d) = 0
c d
  
a b
= M22 (R) : (a + b) = 0 c = 0 (a + d) = 0 (c d) = 0
c d

122
Apuntes Mat023 (Segundo semestre 2014)

 
a b
de esta forma buscamos todas la matrices tales que
c d
a+b = 0
c = 0
a+d = 0
cd = 0
o matricialmente
1 1 0 0 a 0
0 0 1 0 b 0

=

1 0 0 1 c 0


0 0 1 1 d 0
resolvemos el sistema escalonando

1 1 0 0 0 1 1 0 0 0
0 0 1 0 0 0 1 0 1 0



1 0 0 1 0 0 0 1 0 0


0 0 1 1 0 0 0 0 1 0
de esto se obtiene a = b = c = d = 0 es decir
 
0 0
ker (T ) =
0 0
(se concluye que la transformacion es inyectiva) Por el teorema de las dimensiones se
tiene lo siguiente
Dim (ker (T )) + Dim (Im (T )) = Dim (M22 (R)) = 4
pero
Dim (ker (T )) = 0
as
Dim (Im (T )) = 4
como Dim (T ) < R3 [x] y R3 [x] tiene dimension 4, se sigue Im (T ) = R3 [x]. Otra
forma para determinar la imagen es utilizar el resultado que la imagenes de una base
generan la imagen de la transformacion, luego
        
1 0 0 1 0 0 0 0
Im (T ) = T ,T ,T ,T
0 0 0 0 1 0 0 1
pero notemos que
 
1 0
T = (1 + 0) x3 + 0x2 + (1 + 0) x + (0 0)
0 0
= x3 + x

123
Apuntes Mat023 (Segundo semestre 2014)

 
0 1
T = (0 + 1) x3 + 0x2 + (0 + 0) x + (0 0)
0 0
= x3
 
0 0
T = (0 + 0) x3 + 1x2 + (0 + 0) x + (1 0)
1 0
= x2 + 1
 
0 0
T = (0 + 0) x3 + 0x2 + (0 + 1) x + (0 1)
0 1
= x1

as
x3 + x, x3 , x2 + 1, x 1


Im (T ) =
y mostrar que {x3 + x, x3 , x2 + 1, x 1} es un conjunto L.I. luego Im (T ) tiene di-
mension 4 y por tanto tiene que ser R3 [x].

9. Encontrar por lo menos una transformacion lineal T : M22 (R) R3 tal que
  
a b
ker (T ) = M22 (R) : a + d + c = 0
c d
Solucion. Notemos que
  
a b
ker (T ) = M22 (R) : a + d + c = 0
c d
  
a b
= M22 (R) : a = c d
c d
  
c d b
= M22 (R) : b, c, d R
c d
       
1 0 0 1 1 0
= c +b +d M22 (R) : b, c, d R
1 0 0 0 0 1
     
1 0 0 1 1 0
, ,
1 0 0 0 0 1
luego ker (T ) tiene dimension 3 (no puede ser base de M22 (R) que tiene dimension
4), sabemos que para conocer completamente una T.L. necesitamos conocerla sobre
una base, buscamos algun elemento L.I. con las tres matrices que forman el kernel,
para ello,
         
1 0 0 1 1 0 a b 0 0
+ + + =
1 0 0 0 0 1 c d 0 0

124
Apuntes Mat023 (Segundo semestre 2014)

debe tener por unica solucion = = = = 0. Esto se traduce a

+ a = 0
+ b = 0
+ c = 0
+ d = 0

matricialmente la ampliada

1 0 1 a 0 1 0 1 a 0
0 1 0 b 0 0 1 0 b 0



1 0 0 c 0 0 0 1 a+c 0


0 0 1 d 0 0 0 0 a+c+d 0
6 0, de esta forma, podemos considerar la
la solucion es unica si y solo si a + c + d =
matriz  
1 1
1 1
pues 1 + 1 + 1 = 3 6= 0. Ahora para definir la transformacion (por ejemplo)
 
1 0
T = (0, 0, 0)
1 0
 
0 1
T = (0, 0, 0)
0 0
 
1 0
T = (0, 0, 0)
0 1
 
1 1
T = (1, 1, 1)
1 1
     
1 0 0 1 1 0
esto determina completamente quien es T (notar que , ,
1 0 0 0 0 1
 
1 1
son elementos del kernel pero no lo es). La T en forma explcita se determina
1 1
de la siguiente manera, Como
       
1 0 0 1 1 0 1 1
B= , , ,
1 0 0 0 0 1 1 1
 
x y
es base, dado una matriz arbitraria, deben existir escalares , , , tales
z w
que
         
x y 1 0 0 1 1 0 1 1
= + + +
z w 1 0 0 0 0 1 1 1

125
Apuntes Mat023 (Segundo semestre 2014)

para buscar tales escalares resolvemos el sistema



2
1 0 1 1 x 1 0 0 0 3
z 13 x 13 w
0 1 0 1 y 0 1 0 0 y 31 x 13 w 13 z



2
1 0 0 1 z 0 0 1 0 w 13 x 13 z

3

1
0 0 1 1 w 0 0 0 1 3
w + 31 x + 31 z
de donde se tiene
2 1 1
= z x w
3 3 3
1 1 1
= y x w z
3 3 3
2 1 1
= w x z
3 3 3
1 1 1
= w+ x+ z
3 3 3
as
    
x y 2 1 1 1 0
= z x w
z w 3 3 3 1 0
  
1 1 1 0 1
+ y x w z
3 3 3 0 0
  
2 1 1 1 0
+ w x z
3 3 3 0 1
  
1 1 1 1 1
+ w+ x+ z
3 3 3 1 1
luego
   
x y 2 1 1
T = z x w (0, 0, 0)
z w 3 3 3
 
1 1 1
+ y x w z (0, 0, 0)
3 3 3
 
2 1 1
+ w x z (0, 0, 0)
3 3 3
 
1 1 1
+ w + x + z (1, 1, 1)
3 3 3
 
1 1 1 1 1 1 1 1 1
= w + x + z, w + x + z , w + x + z
3 3 3 3 3 3 3 3 3
as
T : M22 (R) R3
     
x y x y 1 1 1 1 1 1 1 1 1
T = w + x + z, w + x + z , w + x + z
z w z w 3 3 3 3 3 3 3 3 3

126
Apuntes Mat023 (Segundo semestre 2014)

10. Sea B la base canonica de R2 y considere las transformaciones T : R2 R2 y


S : R2 R2 definidas por

T (a, b) = (2a + 3b, 4a 7b)

y
S (a, b) = (3a 2b, a b)
calcular [T ]B , [S]B y [T S]B comprobar que

[T ]B [S]B = [T S]B

Solucion. Al ser la matriz con respecto a la base canonica se tiene


   
2 3 3 2
[T ]B = y [S]B =
4 7 1 1

as     
2 3 3 2 9 7
[T ]B [S]B = =
4 7 1 1 5 1
por otro lado

T S (a, b) = T (3a 2b, a b)


= (2 (3a 2b) + 3 (a b) , 4 (3a 2b) 7 (a b))

= 9a 7b 5a b

as  
9 7
[T S]B =
5 1

11. Sea T : R4 R4 una transformacion lineal tal que

T (1, 1, 0, 0) = (0, 1, 0, 1)
T (1, 0, 1, 0) = (1, 1, 1, 0)

si ademas se tiene que T T = IR4 determinar la matriz asociada a T respecto a la


base canonica C de R4 . Es T un isomorfismo? Justifique.

Solucion. Por la propiedad T T = IR4 se sigue

T (T (1, 1, 0, 0)) = T (0, 1, 0, 1) = (1, 1, 0, 0)


T (T (1, 0, 1, 0)) = T (1, 1, 1, 0) = (1, 0, 1, 0)

127
Apuntes Mat023 (Segundo semestre 2014)

as
T (1, 1, 0, 0) = (0, 1, 0, 1)
T (1, 0, 1, 0) = (1, 1, 1, 0)
T (0, 1, 0, 1) = (1, 1, 0, 0)
T (1, 1, 1, 0) = (1, 0, 1, 0)
queremos determinar la matriz asociada a la base canonica
T (1, 1, 1, 0) T (1, 1, 0, 0) = T (0, 0, 1, 0)
= (1, 0, 1, 0) (0, 1, 0, 1)
= (1, 1, 1, 1)

T (0, 0, 1, 0) = (1, 1, 1, 1)
T (1, 0, 0, 0) = T (1, 0, 1, 0) T (0, 0, 1, 0)
= (1, 1, 1, 0) (1, 1, 1, 1)
= (0, 2, 0, 1)
T (0, 1, 0, 0) = T (1, 1, 0, 0) T (1, 0, 0, 0)
= (0, 1, 0, 1) (0, 2, 0, 1)
= (0, 1, 0, 0)
T (0, 0, 0, 1) = T (0, 1, 0, 0) T (0, 1, 0, 1)
= (0, 1, 0, 0) (1, 1, 0, 0)
= (1, 2, 0, 0)
as
0 0 1 1
2 1 1 2

[T ]CC =

0 0 1 0


1 0 1 0
es facil ver que es isomorfismo, pues
T T = I
[T T ]CC = I4
[T ]CC [T ]CC = I4
as [T ]CC es invertible y luego T es invertible. Otra forma

0
0 1 1
2 1 1 2

=1
0 0 1 0


1 0 1 0

128
Apuntes Mat023 (Segundo semestre 2014)

y luego T es invertible.
Otra forma es utilizar la matriz de cambio de base

1 1 0 1
1 0 1 1

[I]CB =

0 1 0 1


0 0 1 0

as 1
1 1 0 1 1 0 1 0
1 0 1 1 1 1 0 1

[I]BC = =

0 1 0 1 0 0 0 1


0 0 1 0 1 1 1 1
luego

[T ]CC = [T ]CB [I]BC



0 1 1 1 1 0 1 0
1 1 1 0 1 1 0 1

=
0 1 0 1 0 0 0 1

1 0 0 0 1 1 1 1

0 0 1 1
2 1 1 2

=

0 0 1 0


1 0 1 0

12. Sea T : M22 (R) R3 la funcion definida por:


 
a b 
T = a + b + d, b c, a + c + d
c d

a) Demuestre que T es una transformacion lineal.


b) Calcule el nucleo y la imagen de T .

129
Apuntes Mat023 (Segundo semestre 2014)

a b
 x y

a) Sean c d M2 (R) y R, entonces:
, z w

   !  
a b x y a + x b + y
T + =T
c d z w c + z d + w

= (a + x) + (b + y) + (d + w),
, (b + y) (c + z),

, (a + x) + (c + z) + (d + w)

= (a + b + d) + (x + y + z),
, (b c) + (y z),

, (a + c + d) + (x + z + w)

= a + b + d, b c, a + c + d +

+ x + y + w, y z, x + z + w
   
a b x y
= T +T
c d z w

Se concluye, por tanto, que T es una transformacion lineal.


b) Sabemos, por definicion, que el nucleo de T es el conjunto:
n   o
a b a b
ker T = c d M2 (R) : T c d = (0, 0, 0)

a b
  
Pero, T c d = a + b + d, b c, a + c + d . Entonces, ( ac db ker T , si y solo
si:

a + b + d, b c, a + c + d = (0, 0, 0)
(
a+b+d = 0
S: bc = 0
a + c + d = 0
As, mediante escalonamiento de la matriz asociada AS , se tiene:

1 1 0 1 1 0 1 1 1 0 0 0
AS = 0 1 1 0 0 1 1 0 0 1 0 1 = EAS
1 0 1 1 0 0 2 2 0 0 1 1

130
Apuntes Mat023 (Segundo semestre 2014)

de donde obtenemos que a = 0, b = d y c = d. Por lo tanto:


n   o
ker T = a b a b
c d : T c d = (0, 0, 0)
n  o
= a b : a = 0 d b = d c = d, d R
c d

n  o
0 d
= d d : d R
 
0 1
= 1 1

Finalmente, como dim M2 (R) = 4 y la nulidad de T es 1 tenemos, por el Teorema


de la Dimension, que el rango de T es 3. Por tanto, Im T = R3 .

13. Sean C1 y C2 las bases canonicas de R3 y R4 respectivamente (orden usual), T : R3


R4 , S : R4 R4 y L : R4 R3 transformaciones lineales tales que

1 1 0 1 1 0 1
1 1 0 1

1 1 0 1 1 0 1

[T ]CC21 = C
, [S]C22 =
C
y [L]C12 = 0 1 0 1
2 2 1 1 1 1 1
1 0 1 1
0 2 1 1 1 0 1

a) Determine explcitamente L S T : R3 R3
Notemos que

[L S T ]CC11 = [L]CC12 [S]CC22 [T ]CC21



1 1 0 1 1 1 0
1 1 0 1

1 1 0 1 1 1 0

= 0 1 0 1

1 1 1 1 2 2 1

1 0 1 1

1 1 0 1 0 2 1
2 0 1

= 0 4 2
2 4 0

se sigue que

2 0 1 2x z

x
[(L S T ) (x, y, z)]C1 = 0 4 2 y = 4y 2z
2 4 0 z 4y 2x

as
(L S T ) (x, y, z) = (2x z, 4y 2z, 4y 2x)

131
Apuntes Mat023 (Segundo semestre 2014)

b) Es S una transformacion biyectiva?


Como

1 1 0 1

1 1 0 1

= 4

1 1 1 1



1 1 0 1
se sigue que la transformacion es biyectiva.
c) Determine ker (T ) e Im(T ).
Hay varias formas de hacer esto, pero como las bases son las canonicas

1 1 0 1 0 0
1 1 0 0 1 0


2 2 1 0 0 1

0 2 1 0 0 0

as ker (T ) = {(0, 0, 0)} e


T T T
*
1 1 0

+
1 1 0

Im (T ) = , ,

2 2 1




0 2 1

es de dimension 3.

14. Sean u = (1, 0, 1), v = (1, 0, 1) y w = u v vectores en R3 y C la base canonica de


R3 :

a) Si T : R3 R3 es una transformacion lineal definida por T (u) = w, T (v) = v


y T (w) = u, determine [T ]CC y [T T ]CC
Notemos que

w= u v
= (1, 0, 1) (1, 0, 1)
= (0, 2, 0)

se sigue

T (1, 0, 1) = (0, 2, 0)
T (1, 0, 1) = (1, 0, 1)
T (0, 2, 0) = (1, 0, 1)

132
Apuntes Mat023 (Segundo semestre 2014)

notamos que
1 1 0

0 0 2 =4

1 1 0

luego B = {u, v, w} es base de R3 ademas



0 1 1
[T ]CB = 2 0 0
0 1 1

por otro lado



1 1 0
[I]CB = 0 0 2
1 1 0
y
1 1 1
 1 1 1 0 2
0 2
[I]CB = [I]BC = 0 0 2 = 21 0 12
1 1 0 0 21 0
se sigue

[T ]CC = [T ]CB [I]BC


1
0 21

0 1 1 2
= 2 0 0 12 0 21
0 1 1 0 12 0
1 1
21

2 2
= 1 0 1
12 12 21
y

[T T ]CC = [T ]CC [T ]CC


1 1 2
2 2
21
= 1 0 1
12 12 21

1 0 0
= 0 1 0
0 0 1

b) Determine explcitamente T 1 o argumente que no esta definida.

133
Apuntes Mat023 (Segundo semestre 2014)

Note que [T ]CC tiene inversa luego la transformacion tiene inversa o bien T (B) = B
se sigue que es sobre y por teorema de las dimensiones es biyectiva, ademas
1 1
12

 1 C 2 2
T C= 1 0 1
12 21 12

y as
1 1
12

2 2
x
 1 
T (x, y, z) C = 1 0 1 y
1 1 1
2 2 2 z
1 1 1
x + 2y 2z

2
= x+z
1 1 1
2
y 2x + 2z

esto es  
1 1 1 1 1 1 1
T (x, y, z) = x + y z, x + z, y x + z
2 2 2 2 2 2

c) Si B = {u, v, w} determine [T ]BB


Desarrollo:
0 0 1
[T ]BB = 0 1 0
1 0 0

d ) Encontrar una matriz A tal que A1 [T ]BB A = [T ]CC


Desarrollo: Podemos usar A = [I]BC la matriz de cambio de base:
 1
[I]BC [T ]BB [I]BC = [T ]CC

esto es
1 1
0 12
1
0 21
1 1
12

2
0 0 1 2 2 2
1

2
0 12 0 1 0 12 0 12 = 1 0 1
0 12 0 1 0 0 0 12 0 12 21 12

Ejercicios de la seccion

1. Determine si las siguientes funciones son transformaciones lineales:

a) T : R2 R, T (x, y) = |x + y|
b) T : R3 R2 , T (x, y, z) = (2x + y, z y)

134
Apuntes Mat023 (Segundo semestre 2014)

c) T : R R3 , T (x) = (x, 2x, x2 )


d ) T : R3 R3 , T (x, y, z) = (cos x, sin y, z)

2. Existe un transformacion lineal de R2 en R2 que transforma los vectores (1, 4) y


(3, 2) en los vectores (2, 1) y (4, 2), respectivamente? En caso afirmativo determine
la imagen del vector (2, a), con a R.

3. Sea U un espacio vectorial tal que dim U = n < . Suponga que u1 , u2 , . . . , un es


una base ordenada de U . Suponga que T : U U es tal que:

T (ui ) = ui , i = 1, 2, . . . , n (2.3)

Entonces:

a) Demuestre que existen funciones T : U U definidas por la condicion (2.3) que


son transformaciones lineales.
b) Verifique que si T es una transformacion lineal que satisface la condicion (2.3),
entonces T = IU , donde IU es la funcion identidad de U en U .

4. Sea T : R3 R2 la transformacion lineal definida por:

T (x, y, z) = (x + y 2z, x + y)

Calcule ker T e ImT .

5. Sea S L (R3 , M2 (R)) definida por:


 
x 3y y 2z
S (x, y, z) =
x z

a) Calcule condiciones sobre a R de modo que:

3e1 + ae2 e3 kerT

donde ei es el iesimo vector de la base canonica de R3 .


b) Calcule ImT

6. Sea T : R2 [x] M2 (R) una funcion definida por:


R1 !
 p00 (0) 0
p (x) dx
T p (x) =
p (1) 0

a) Demuestre que T es una transformacion lineal.

135
Apuntes Mat023 (Segundo semestre 2014)

b) Hallar una base para el kernel de T .


c) Hallar ImT y su dimension.

7. Sea T L (U, V ) tal que ker T = {0}. Suponga que B es un conjunto linealmente
independiente en U . Demuestre que:

T (B) = {T (u) : u B}

es linealmente independiente en V .

8. Sea T L (U, U ). Entonces, las siguientes proposiciones son equivalentes:

(i) ker T ImT = {0}.


(ii) Si T 2 (u) = 0, entonces T (u) = 0.

9. Sea T : R3 R3 la transformacion lineal definida por:

T (x, y, z) = (3x, x y, 2x + y + z)

Es T invertible? En caso afirmativo, hallar una formula para T 1 .

10. Sean U y V espacios vectoriales sobre el mismo cuerpo K, tales que dim U = dim V =
n < . Sea, ademas, T L (U, V ) tal que:

T u = 0 = u = 0

Entonces, existe una base u1 , u2 , . . . , un de U tal que T u1 , T u2 , . . . , T un es una base


de V .

11. Sea B = (1, 2, 1) ; (2, 1, 1) , (1, 1, 2) una base de R3 y T : R3 R3 una transfor-
macion lineal tal que:
3 3 5

[T ]BB = 1 1 1
2 2 4

a) Determinar una base del ker T .


b) Determinar una base del ker T 2 .
c) Determinar una base de ImT .

12. Sea T : R3 R3 tal que:



1 2 0
[T ]D
B =
1 1 1
3 1 1
donde B = {(1, 1, 2) ; (0, 1, 1) ; (1, 0, 0)} y D = {(1, 1, 1) ; (0, 1, 1) ; (0, 0, 1)} son
bases de R3 . Entonces:

136
Apuntes Mat023 (Segundo semestre 2014)

a) Encuentre T (1, 3, 1), sin calcular T (x, y, z).


b) Determine T 1 (x, y, z), si acaso existe.
c) Hallar [T ]F E , sin calcular T (x, y, z), si E = {(0, 1, 2) ; (1, 1, 1) ; (0, 1, 0)} y F =
{(1, 1, 0) ; (1, 1, 1) ; (1, 0, 0)} son, tambien, bases de R3 .

13. Sean T : R3 [x] R3 [x] una funcion definida por:

T p (x) = p0 (x) x p00 (x)




y B1 = {1 x2 , 1 + x2 , 1 x3 , 2 + x}, B2 = {1, x2 + x3 , x3 x2 , 1 2x} dos bases


ordenadas de R3 [x]. Entonces:

a) Pruebe que existe T es una transformacion lineal.


b) Hallar ker T y una base para ImT . Es T invertible?
c) Calcule [T ]BB21 .

14. Diremos que un espacio vectorial V es suma directa de los subespacios W1 y W2 ,


lo cual anotaremos, V = W1 W2 , si W1 W2 = {0}, y para cada v V , existen
w1 W1 y w2 W2 tales que v = w1 + w2 . Sean V un espacio vectorial de dimension
finita y T : V V una transformacion lineal. Pruebe que:

V = ker T ImT ker T 2 = ker T




15. Sean U y V espacios vectoriales sobre el mismo cuerpo K y sea S un isomorfismo de


U en V . Demuestre que:
T 7 S T S 1
es un isomorfismo de L (U, U ) en L (V, V ).

16. Hallar explcitamente, esto es, calculando T (ax3 + bx2 + cx + d), una transformacion
lineal T : R3 [x] R3 tal que:

ImT = (x, y, z) R3 : x + 2y z = 0


y cuya nulidad sea 2.

17. Sean:
B = {(1, 2, 1) , (0, 1, 2) , (1, 1, 1)}
y:        
1 2 0 0 1 2 2 1
D= , , ,
2 1 1 1 3 3 2 1

137
Apuntes Mat023 (Segundo semestre 2014)

bases ordenadas de R3 y M2 (R), respectivamente. Considere T : R3 M2 (R)


definida por:  
x + 2y x y + 3z
T (x, y, z) =
0 x+y+z

a) Hallar [T ]D
B.

b) Determine condiciones sobre a R para que (x, y, z) ker T , sabiendo que:

[(x, y, z)]B = a
2

c) Hallar ImT , usando [T ]D


B.

18. Sean T : M2 (R) M2 (R) una transformacion lineal y:


       
1 0 1 1 1 0 1 1
B= , , ,
0 1 0 1 1 1 2 2

una base ordenada de M2 (R) tal que:



1 0 1 0
1 2 3 0

[T ]BB =

1 1 2 0


1 1 2 1

Determinar una base D de M2 (R) de modo que [T ]D D sea una matriz diagonal.
(Ayuda: El polinomio caracterstico de T es fT () = ( 4) ( 1)2 )

19. Sean:
B = 1 + x x2 , 2x x2 , 1 + x2


y:
D = {(1, 2, 1) , (1, 3, 2) , (2, 1, 3)}
bases ordenadas de R2 [x] y R3 , respectivamente. Consideremos T : R2 [x] R3 tal
que:
2 2

5
[T ]D
B =
3 1 1
1 4 3

a) Demuestre que T es un isomorfismo.


b) Calcule ker T 1 e ImT 1 .

138
Apuntes Mat023 (Segundo semestre 2014)

c) Hallar explcitamente T 1 .

20. Sean B = {1 + x, 1 x2 , 1} y D = {1, 1 + x, 1 + x + x2 } bases ordenadas de R2 [x] .


Considere T : R2 [x] R2 [x] una transformacion lineal tal que:

1 0 1
[T ]D
B =
0 1 1
1 1 2

a) Hallar condiciones sobre a R para que:

1 (a + 1) x 2x2 ker T

b) Hallar condiciones sobre b R para que:

1 2x + (a 1) x2 ker T 1

21. Sea:
W = (x, y, z) R3 : 2x 3y + 5z = 0


un subespacio de R3 tal que B es una base W . Considere D = {(1, 1) , (2, 1)} una
base de R2 . Sea T L (W, R2 ) definida por T (x, y, z) = (x + y z, 3x y + 2z) tal
que:  
D 1 2
[T ]B =
3 1
Determine la base B.

22. Considere el plano en R3 dado por:

W = (x, y, z) R3 : ax + by + cz = 0


con a, b, c R. Para cualquier u R3 , denotaremos por PW (u) al punto del plano W


que se encuentra a menor distancia de u. Tal funcion PW (u) se llama la proyeccion
ortogonal de u sobre W . Entonces:

a) Dado u R3 , hallar una expresion para PW (u).


b) Demuestre que la funcion PW (u) : R3 R3 definida por:

u 7 PW (w)

es una transformacion lineal.


c) Calcule [PW ]CC , donde C es la base canonica de R3 , y verifique que:
 2  T
[PW ]CC = [PW ]CC [PW ]CC = [PW ]CC

139
Apuntes Mat023 (Segundo semestre 2014)

23. Considere el subespacio vectorial W < R2 [x] definido por:

W = 1 x2 , x + 2x2

Demuestre que existe una transformacion lineal T : R2 [x] R2 [x] diagonalizable tal
que 1 y 1 sean sus valores propios, y que el espacio propio asociado a 1 sea W .

Ecuaciones diferenciales lineales

Definiciones
Definicion 2.2.1. Sean I R un intervalo, A0 , A1 , . . . , An y R funciones continuas sobre
I tales que A (x) 6= 0, para cada x I. Una ecuacion diferencial de la forma:

An (x) y (n) + An1 (x) y (n1) + An2 (x) y (n2) + + A1 (x) y 0 + A0 (x) y = R (x) (2.4)

se denomina ecuacion diferencial lineal de orden n. Las funciones A0 , A1 , . . . , An son


llamados coeficientes de la ecuacion diferencial. Si R 0, la ecuacion lineal se denomina
ecuacion homogenea.
Observacion 2.2.1. En general, los puntos en los cuales An (x) = 0 se conocen como
puntos singulares y su estudio no sera considerado en este curso.
Observacion 2.2.2. La ecuacion (2.4) puede escribirse como:

y (n) + pn1 (x) y (n1) + pn2 (x) y (n2) + + p1 (x) y 0 + p0 (x) y = Q (x) (2.5)

al dividir la ecuacion (2.4) por An (x), de esta forma


Ai (x)
pi (x) = para i = 0, . . . , n 1
An (x)
y
R (x)
Q (x) =
An (x)
esta forma de la ecuacion es llamada forma normal.
Ejemplo 2.2.1. Son ecuaciones diferenciales lineales las siguientes ecuaciones:

1. y (4) + 3xy 00 2 (cos x) y 0 + y = e2x cos 3x

2. y 00 + y = 0

3. Las ecuaciones de Euler, definidas como:

(ax + b)n y (n) + A1 (ax + b)n1 y (n1) + + An1 (ax + b) y 0 + An y = Q (x)

con ax + b > 0.

140
Apuntes Mat023 (Segundo semestre 2014)

4. las ecuaciones de Legendre, definidas como:

1 x2 y 00 2xy 0 + ( + 1) y = 0


con R.

Ejemplo 2.2.2. Considere la ecuacion diferencial:

y 00 2y 0 + 2y = 0

1. Verifique que las funciones u1 (x) = ex cos x y u2 (x) = ex sin x son soluciones de la
ecuacion diferencial.

2. Sean C1 , C2 R. Verifique que y (x) = C1 u1 (x) + C2 u2 (x) es solucion de la ecuacion


diferencial.

3. Hallar una solucion y que satisfaga las condiciones iniciales y (0) = 1 e y 0 (0) = 4.

Solucion. Por partes:

1. Notemos que
d x
u01 (x) = (e cos x) = ex cos x ex sin x
dx
d x
u001 (x) = (e cos x ex sin x) = 2ex sin x
dx
se sigue

u001 2u01 + 2u1 = (2ex sin x) 2 (ex cos x ex sin x) + 2 (ex cos x)
= 0

de manera similar
d x
u02 (x) = (e sin x) = ex (cos x + sin x)
dx
d x
u002 (x) = (e (cos x + sin x)) = 2 (cos x) ex
dx
luego

u002 2u02 + 2u2 = (2 (cos x) ex ) 2 (ex (cos x + sin x)) + 2 (ex sin x)
= 0

141
Apuntes Mat023 (Segundo semestre 2014)

2. Notar que

y 0 (x) = C1 u01 (x) + C2 u02 (x)


y 00 (x) = C1 u001 (x) + C2 u002 (x)

luego

y 00 2y 0 + 2y = (C1 u001 (x) + C2 u002 (x)) 2 (C1 u01 (x) + C2 u02 (x)) + 2 (C1 u1 (x) + C2 u2 (x))
= (C1 u001 (x) 2C1 u01 (x) + 2C1 u1 (x)) + (C2 u002 (x) 2C2 u02 (x) + 2C2 u2 (x))
= C1 (u001 (x) 2u01 (x) + 2u1 (x)) + C2 (u002 (x) 2u02 (x) + 2u2 (x))
= C1 0 + C2 0
= 0

3. Sabemos que y (x) = C1 ex cos x + C2 ex sin x es solucion de la ecuacion, para cumplir


las condiciones y (0) = 1 e y 0 (0) = 4 las constantes las debemos escoger en forma
adecuada

1 = y (0) = C1
4 = y 0 (0) = C1 + C2

as

1 = C1
4 = C1 + C2

que tiene solucion C1 = 1, C2 = 3. As

y (x) = ex cos x + 3ex sin x

Observacion 2.2.3. Con el objeto de simplificar el estudio de las ecuaciones diferenciales


lineales y el de aprovechar la terminologa de las transformaciones lineales es que consi-
deraremos la nocion de operador diferencial asociado a una ecuacion diferencial lineal de
orden n. Para ello recordemos que C n (I) denota el espacio vectorial de todas las funciones
de clase C n definidas sobre I, esto es el conjunto de todas las funciones f : I R que son
n veces derivables con f (n) continua sobre I.

Definicion 2.2.2. Sean p0 , p1 , . . . , pn1 funciones continuas sobre un intervalo I R.


Llamaremos operador diferencial asociado a la ecuacion (2.5) a la funcion L : C n (I) C (I)
definida por:
L (f ) = f (n) + pn1 f (n1) + + p0 f (2.6)

142
Apuntes Mat023 (Segundo semestre 2014)

Observacion 2.2.4. El operador diferencial dado por la ecuacion (2.6) puede escribirse
como:
L = Dn + pn1 Dn1 + + p0 Id
en donde Dk representa la derivada de orden k y Id representa la transformacion lineal
identidad.

Teorema 2.2.1. Sea I R un intervalo cualquiera. El operador diferencial L : C n (I)


C (I) es una transformacion lineal.

Observacion 2.2.5. Con la introduccion del operador diferencial L toda ecuacion lineal:

y (n) + pn1 (x) y (n1) + + p0 (x) y = Q (x)

puede escribirse como:


L (y) = Q (2.7)
As, la ecuacion:
L (y) = 0
se llamara ecuacion homogenea asociada a la ecuacion (2.7).

Observacion 2.2.6. La ecuacion (2.7) tiene un analogo finito dimensional con sistemas
de ecuaciones lineales y consecuentemente con la respectiva ecuacion matricial asociada.
En efecto, sea S un sistema de m ecuaciones lineales con n incognitas. Luego, existe una
matriz A Mmn (K) que contiene los coeficientes del sistema, una matriz X Mm1 (K)
con las incognitas y ademas, una matriz B Mm1 (K) con las constantes. El producto
de matrices permite representar el sistema S como la ecuacion matricial siguiente:

AX = B

Observe que las propiedades del algebra de matrices nos permiten escribir X como:

X = X h + Xp

donde Xh representa la solucion general del sistema homogeneo AX = 0 y Xp representa


una solucion particular del sistema que se asume conocida. En efecto:

AX = A (Xh + Xp )
= AXh + AXp
= 0+B
= B

Ahora bien, mediante el isomorfismo LK (U, V ) ' Mmn (K), debe existir una transforma-
cion lineal T : U V tal que:
A = [T ]CB

143
Apuntes Mat023 (Segundo semestre 2014)

para ciertas bases ordenadas B y D, de U y V , respectivamente. Por tanto, el espacio


solucion de la ecuacion matricial es, salvo cambio de coordenadas, el nucleo o espacio nulo
de la transformacion lineal T . Por esta razon, nos interesara calcular el nucleo o kernel del
operador diferencial L.

Teorema 2.2.2. Suponga que yh representa la solucion de la ecuacion homogenea asociada


a L (y) = Q y que yp es una solucion particular (esto es, L (yp ) = Q) de la misma ecuacion.
Entonces, la solucion general de:
L (y) = Q
es de la forma:
y = yh + yp

En efecto, siguiendo los razonamientos de la observacion anterior, tenemos que:

L (y) = L (yh + yp )
= L (yh ) + L (yp )
= 0+Q
= Q

Observacion 2.2.7. Consideremos la ecuacion diferencial:

y (n) + pn1 (x) y (n1) + pn2 (x) y (n2) + + p1 (x) y 0 + p0 (x) y = 0

Por lo expuesto anteriormente, el operador lineal asociado L : C n (I) C (I) definido por:

L = Dn + pn1 Dn1 + + p1 D + p0 Id

permite expresar la ecuacion como L (y) = 0. Por tanto, una funcion y es solucion de la
ecuacion diferencial homogenea si y solo si y ker L, notamos ademas que:

ker L C n (I)

y el espacio vectorial C n (I) no es de dimension finita (basta ver que para cada n 1, el
conjunto 1, x, x2 , . . . , xn es linealmente independiente). Una aplicacion muy importante
del Teorema de Existencia y Unicidad para ecuaciones diferenciales sera la de mostrar que el
nucleo de un operador diferencial no solo es de dimension finita, sino que, ademas, es de
dimension n, en donde n es el orden de la ecuacion diferencial lineal. Es decir, si:

L = Dn + pn1 Dn1 + + p1 D + p0 Id

es un operador diferencial asociado a una ecuacion diferencial lineal, entonces:

dim ker L = n

144
Apuntes Mat023 (Segundo semestre 2014)

Teorema de Existencia y Unicidad

Observacion 2.3.1. El siguiente teorema juega un rol crucial en establecer que la dimen-
sion del nucleo del operador diferencial L asociado a una ecuacion diferencial lineal de
orden n del tipo:
y (n) + pn1 (x) y (n1) + + p0 (x) y = Q (x)
es de dimension finita. En particular, en esta seccion se utilizara el Teorema de Existencia
y Unicidad para demostrar que no solo es finita esa dimension, sino que ademas es de
dimension coincide con el orden de la ecuacion lineal. Introduzcamos primeramente la
siguiente definicion:
Definicion 2.3.1. Diremos que la funcion : I R R es solucion del problema de
valores iniciales de orden n:

y (n) = f x, y, y 0 , y 00 , . . . , y (n1)


y (x0 ) = y0 , y 0 (x0 ) = y1 , . . . , y (n1) (x0 ) = yn1

si : I R R es solucion de la ecuacion en su intervalo de definicion, x0 I y:

(x0 ) = y0 , 0 (x0 ) = y1 , . . . , (n1) (x0 ) = yn1

Observacion 2.3.2. Para mayor claridad de los conceptos los teoremas seran expuestos
para ecuaciones de orden 2 y notar que la extension se hace por induccion sobre n. As, los
resultados se indicaran para orden 2 u orden n segun convenga.
Teorema 2.3.1. Sean I R un intervalo abierto y p0 , p1 dos funciones continuas sobre I.
Considere el operador diferencial:

L (y) = y 00 + p1 y 0 + p0 y

Si x0 I e y0 , y1 son numeros reales cualesquiera, entonces existe una unica funcion


f : I R que es solucion de la ecuacion diferencial:

L (y) = 0

y que satisface las condiciones iniciales:

f (x0 ) = y0 f 0 (x0 ) = y1

Teorema 2.3.2. Consideremos el operador diferencial L (y) = y 00 + p1 y 0 + p0 y, con p0 y


p1 funciones continuas sobre un intervalo abierto I R. Suponga que u1 y u2 son dos
funciones no nulas, linealmente independientes sobre I, y que son solucion de la ecuacion
diferencial L (y) = 0. Entonces, para todo par de constantes c1 y c2 , la combinacion lineal:

c1 u1 + c2 u2

145
Apuntes Mat023 (Segundo semestre 2014)

es solucion de la ecuacion diferencial L (y) = 0 en I. Recprocamente, si y es una solucion


de L (y) = 0 en I, entonces existen constantes c1 y c2 tales que:
y = c1 u 1 + c2 u 2
Demostracion. En primer lugar, note que como L es una transformacion lineal, se tiene
que:
C1 u1 + C2 u2 ker L
luego, C1 u1 + C2 u2 es solucion de L (y) = 0.
Por otro lado, como u1 y u2 son linealmente independientes en I, bastara verificar que:
ker L = hu1 , u2 i
En efecto, sean f ker L y x0 I. Por el Teorema de Unicidad, es suficiente verificar que
existen constantes C1 y C2 tales que:

C1 u1 (x0 ) + C2 u2 (x0 ) = f (x0 )
C1 u01 (x0 ) + C2 u02 (x0 ) = f 0 (x0 )
es decir, que las funciones C1 u1 + C2 u2 y f coinciden en las condiciones iniciales. Entonces,
para que tales constantes existan, es suficiente a su vez que el determinante:

u1 (x0 ) u2 (x0 ) 0 0
u0 (x0 ) u0 (x0 ) = u1 (x0 ) u2 (x0 ) u2 (x0 ) u1 (x0 )

1 2

sea no nulo. Consideremos la funcion W : I R R definida por:



u1 (x) u2 (x)
W (x) = 0
= u1 (x) u02 (x) u2 (x) u01 (x)
u1 (x) u02 (x)
Supongamos que W (x) = 0, para todo x I, entonces:
u1 (x) u02 (x) u2 (x) u01 (x)
 
d u2
(x) =
dx u1 u21 (x)
W (x)
=
u21 (x)
= 0
luego, por el Teorema del Valor Medio el cociente u2 /u1 debe ser constante, pues I es un
intervalo. Esto contradice la hipotesis de que u1 y u2 son linealmente independientes sobre
I. Por tanto, debe existir al menos un x0 I tal que W (x0 ) 6= 0, y esto ultimo implica
que existen constantes C1 y C2 tales que:
f (x) = C1 u1 (x) + C2 u2 (x)
en otras palabras, u1 y u2 forman una base para el nucleo del operador L, y as:
dim ker L = 2

146
Apuntes Mat023 (Segundo semestre 2014)

Observacion 2.3.3. As, entonces, el operador L asociado a la ecuacion diferencial lineal:

y (n) + pn1 (x) y (n1) + + p0 (x) y = Q (x)

tiene nucleo ker L con dimension finita dado por el orden de la ecuacion diferencial. Es
decir:
dim ker L = n

vemos entonces que para determinar la solucion general de una ecuacion diferencial
lineal homogenea, necesitamos una familia de n funciones linealmente independientes si
la ecuacion es de orden n. Antes de continuar, veamos algunos ejemplos de funciones
linealmente independientes:

Ejemplo 2.3.1. Sean r1 , r2 , . . . , rn R tales que ri =


6 rj , para todo i 6= j. Considere la
familia de n funciones definidas por:

ui (x) = eri x , i = 1, 2, . . . , n

con x U R. Note que si n = 2, entonces u1 y u2 son linealmente independientes, pues:


u1 (x)
= e(r1 r2 )x
u2 (x)
no es constante. Supongase que el resultado es cierto para k funciones exponenciales. Sea:
k+1
X
Ci eri x = 0, x U (2.8)
i=1

Luego, multiplicando por erk+1 x y derivando la ecuacion anterior respecto de x, obtenemos:


k
X
Ci (ri rk+1 ) e(ri rk+1 )x = 0
i=1

Los k numeros ri rk+1 , con 1 i k son distintos. Utilizando la hipotesis de induccion,


las k exponenciales de la ecuacion anterior son linealmente independientes en U . Por tanto,
se debe tener que:
Ci (ri rk+1 ) = 0, i = 1, 2, . . . , k
6 rk+1 , para cada i k, se tiene que Ci = 0 para i k. Utilizando esto ultimo
pero ri =
junto con la ecuacion (2.8) se concluye tambien que Ck+1 = 0.

Ejemplo 2.3.2. Sea r R. Las n funciones:

ui (x) = xi1 erx , i = 1, 2, . . . , n

son linealmente independientes en todo intervalo I R.

147
Apuntes Mat023 (Segundo semestre 2014)

Observacion 2.3.4. Si el cociente entre dos funciones u y v es identicamente una constante,


entonces u y v son linealmente dependientes. En efecto, suponga que:
u
, constante
v
entonces u = v. Es decir, u hvi, y por tanto, no puede ser linealmente independiente.
Ejemplo 2.3.3. Tres polinomios de primer grado cualesquiera son linealmente dependientes
en (, +).
Ejemplo 2.3.4. Cuatro polinomios de segundo grado cualesquiera son linealmente depen-
dientes en (, +).
Definicion 2.3.2. Diremos que las funciones u1 , u2 , . . . , un forman un sistema funda-
mental de soluciones de la ecuacion diferencial L (y) = 0, de orden n, si:

1. L (ui ) = 0, para todo i = 1, 2, . . . , n.

2. u1 , u2 , . . . , un son linealmente independientes (sobre I).

Observacion 2.3.5. Note que un sistema fundamental de soluciones u1 , u2 , . . . , un es una


base para el nucleo de L. Es decir:

ker L = h{u1 , u2 , . . . , un }i C n (I)

Ejemplo 2.3.5. Considere la ecuacion y 00 +y = 0. Verifique que las funciones u1 (x) = cos x
y u2 (x) = sin x, forman un sistema fundamental de soluciones de la ecuacion diferencial.
Solucion. Las funciones sin x y cos x son linealmente independientes y son soluciones de
la ecuacion y 00 + y = 0, se sigue que ker (D2 + 1) = h{sin x, cos x}i.

El wronskiano

Observacion 2.4.1. En el teorema anterior, se demostro que el operador diferencial L,


asociado a la ecuacion diferencial lineal homogenea de orden 2 siguiente:

y 00 + p1 (x) y 0 + p0 (x) y = 0

tiene nucleo:
ker L = h{u1 , u2 }i
en el cual el determinante:
u1 (x0 ) u2 (x0 )

u0 (x0 ) u0 (x0 )
1 2

juega un papel importante en relacion con la independencia lineal de las funciones, tenemos
la siguiente definicion:

148
Apuntes Mat023 (Segundo semestre 2014)

Definicion 2.4.1. Llamaremos wronskiano de las funciones u1 (x) , u2 (x) , . . . , un (x) a:



u1 (x)
u2 (x) un (x)
u0 (x) u02 (x) u0n (x)
1
W (x) =

.. .. .. ..
. . . .
(n1) (n1) (n1)

u
1 (x) u 2 (x) un (x)
nn

Se anota tambien W [u1 , u2 , . . . , un ] (x).

Ejemplo 2.4.1. Calcule el wronskiano de u1 (x) = cos x y u2 (x) = sin x.



cos x sin x
Solucion. W (x) =
= cos2 x + sin2 x = 1
sin x cos x

Teorema 2.4.1. Sean u1 (x) y u2 (x) dos soluciones de la ecuacion diferencial:

y 00 + p1 (x) y 0 + p0 (x) y = 0 (2.9)

Entonces, el wronskiano:
u (x) u2 (x)
W (x) = 10


u1 (x) u02 (x)

satisface la ecuacion diferencial:

W 0 + p1 (x) W = 0

Demostracion. Note que:


0
W 0 = (u1 u02 u01 u2 )
= u01 u02 + u1 u002 u001 u2 u01 u02
= u1 u002 u001 u2

luego:

W 0 + p1 (x) W = u1 u002 u001 u2 + p1 (x) {u1 u02 u01 u2 }


= u1 {u002 + p1 (x) u02 } u2 {u001 + p1 (x) u01 }
= u1 {u002 + p1 (x) u02 + p0 (x) u2 } u2 {u001 + p1 (x) u01 + p0 (x) u1 }
= u1 0 u2 0
= 0

Por tanto, W 0 + p1 (x) W = 0, como se quera demostrar.

149
Apuntes Mat023 (Segundo semestre 2014)

Observacion 2.4.2. En vista del teorema anterior, si u1 (x) y u2 (x) son dos soluciones
de la ecuacion (2.9), la ecuacion diferencial asociada al wronskiano:

W 0 + p1 (x) W = 0

es de variable separable, luego:


Z Z
dW
= p1 (x) dx + C
W
Por tanto: R
W (x) = W (x0 ) e p1 (x)dx

para algun x0 I. Por tanto se tiene el siguiente teorema:

Teorema 2.4.2 (Formula de Abel para el wronskiano). Sea W = W (x) el wronskiano


asociado a dos soluciones linealmente independientes de la ecuacion diferencial de segundo
orden:
y 00 + p1 (x) y 0 + p0 (x) y = 0
entonces: R
W (x) = W (x0 ) e p1 (x)dx

Teorema 2.4.3. Sean u1 , u2 , . . . , un soluciones de la ecuacion diferencial:

y (n) + pn1 (x) y (n1) + + p0 (x) y = 0

donde las funciones p1 , p2 , . . . , pn1 son continuas en un intervalo I R. Entonces,


6
u1 , u2 , . . . , un son linealmente independientes sobre I si y solamente si W [u1 , u2 , . . . , un ] (x) =
0 en I.
1
Ejemplo 2.4.2. Sean u1 (x) = x y u2 (x) = x
soluciones de la ecuacion diferencial:

1 1
y 00 + y 0 2 y = 0
x x
definida para x > 0. Hallar la solucion general.

Solucion. Para hallar la solucion general, es suficiente probar que u1 (x) y u2 (x) son
linealmente independientes. Por el teorema anterior:
1
 
1 x
x

= 2 6= 0
W x, =
x 1 x12

Por tanto:
1
y (x) = C1 x + C2
x

150
Apuntes Mat023 (Segundo semestre 2014)

Teorema 2.4.4 (Formula de Abel para una segunda solucion). Sea u1 (x) una solucion no
trivial de la ecuacion diferencial:

y 00 + p1 (x) y 0 + p0 (x) y = 0

entonces, una segunda solucion u2 (x), linealmente independiente con u1 (x), esta dada por:
R
e p1 (x)dx
Z
u2 (x) = u1 (x) dx
u21 (x)

Demostracion. Como:

u (x) u2 (x)
W (x) = 10


u1 (x) u02 (x)
= u1 u02 u01 u2

y: R
W (x) = C e p1 (x)dx

obtenemos la ecuacion diferencial lineal de primer orden en la variable u2 siguiente:


R
u1 u02 u01 u2 = C e p1 (x)dx

Dividiendo la ecuacion anterior por u21 , se tiene que:

u1 u02 u01 u2 C R p1 (x)dx


= e
u21 u21

Por consiguiente:  
d u2 C R p1 (x)dx
= e
dx u1 u21
integrando y eligiendo valores adecuados para las constantes de integracion, se obtiene
finalmente que: Z R p1 (x)dx
e
u2 (x) = u1 (x) dx
u21 (x)

Ejemplo 2.4.3. Hallar la solucion general de la ecuacion de Legendre, con = 1:

1 x2 y 00 2xy 0 + 2y = 0,

|x| < 1

Solucion. Notamos que u1 (x) = x es una solucion particular de la ecuacion diferencial.


Luego, de:
2x 0 2
y 00 y + y=0
1 x2 1 x2

151
Apuntes Mat023 (Segundo semestre 2014)

y de la formula de Abel, se obtiene que:


2x
R
Z dx
e 1x2
y2 (x) = x dx
x2
Z ln(1x2 )
e
= x dx
x2
Z
dx
= x
x (1 x2 )
2
Z  
1 1 1
= x + + dx
x2 2 (1 x) 2 (1 + x)

mediante un desarrollo en fracciones parciales del penultimo integrando. Por tanto:



1 1 + x
1
y2 (x) = x ln

2 1 x

As, la solucion general de la ecuacion diferencial es:


 
x 1 + x
y (x) = C1 x + C2 ln 1
2 1 x

Ejemplo 2.4.4. Hallar la solucion general de la ecuacion diferencial:

xy 00 2 (x + 1) y 0 + (x + 2) y = 0

para x > 0, bajo el supuesto de que la ecuacion homogenea tiene una solucion de la forma
y = emx , con m R una constante adecuada.

Solucion. Por hipotesis, la ecuacion diferencial posee una solucion de la forma y = emx .
As, al reemplazar en:
xy 00 2 (x + 1) y 0 + (x + 2) y = 0
obtenemos la ecuacion:

xm2 emx 2m (x + 1) emx + (x + 2) emx = 0

para todo x > 0. Pero:

xm2 2m (x + 1) + (x + 2) = xm2 2m (x + 1) + (x + 1) + 1
= xm2 + (x + 1) (1 2m) + 1
= xm2 + (x + 1) (1 2m) + 1 m2 + m2
= (x + 1) m2 + (x + 1) (1 2m) + 1 m2
= (x + 1) m2 2m + 1 + 1 m2
 

= (x + 1) (m 1)2 + 1 m2


152
Apuntes Mat023 (Segundo semestre 2014)

entonces:
(x + 1) (m 1)2 + 1 m2 = 0


para todo x > 0. Por tanto, m = 1. As, y1 (x) = ex es una solucion de la ecuacion. Para
hallar una segunda solucion linealmente independiente con y1 (x) = ex , utilizaremos la
formula de Abel. Primeramente escribimos la ecuacion en su forma normal:
   
00 1 0 2
y 2 1+ y + 1+ y=0
x x

entonces, si p1 (x) = 2 1 + x1 , tenemos que:




R
e p1 (x)dx
Z
y2 (x) = y1 (x) dx
[y1 (x)]2
Z 2 R (1+ 1 )dx
x e x
= e dx
e2x
Z 2(x+ln x)
x e
= e dx
e2x
Z 2 2x
x xe
= e dx
e2x
Z
= ex x2 dx
1 3 x
= xe
3
por tanto, la solucion general de la ecuacion diferencial esta dada por:

y (x) = C1 ex + C2 x3 ex

Ecuaciones diferenciales a coeficientes constantes

Definiciones
Definicion 2.5.1. Una ecuacion diferencial lineal a coeficientes constantes es una ecuacion
de la forma:

y (n) + an1 y (n1) + an2 y (n2) + + a1 y 0 + a0 y = Q (x)

en donde a0 , a1 , . . . , an1 R y Q es una funcion continua en R. Las constantes ai R,


i = 0, 1, . . . , n 1, se llaman coeficientes de la ecuacion.

Observacion 2.5.1. Comenzamos el estudio de este tipo de ecuaciones con el caso n = 2.

153
Apuntes Mat023 (Segundo semestre 2014)

La ecuacion de orden 2
Observacion 2.5.2. Consideremos la ecuacion diferencial de segundo orden siguiente:

y 00 + ay 0 + by = 0 (2.10)

Supongamos que la ecuacion (2.10) posee una solucion y = y (x) de la forma y (x) = ex ,
con R. Note que:
y 0 (x) = ex y 00 (x) = 2 ex
Por tanto, reemplazando lo anterior en la ecuacion (2.10) obtenemos:

2 ex + aex + bex = 0

As, en vista de la ecuacion anterior, las condiciones sobre R de modo que y (x) = ex
sea solucion es que:
2 + a + b = 0 (2.11)
Esto es, que sea raz de la ecuacion de segundo grado anterior. La ecuacion (2.11) se
llama ecuacion caracterstica asociada a la ecuacion (2.10).
Por tanto, tenemos tres casos dados por = a2 4b, el discriminante de (2.11). En
efecto:

1. Si = a2 4b > 0, entonces la ecuacion (2.11), tiene dos soluciones reales 1 y 2


distintas, dadas por:

a + a2 4b a a2 4b
1 = 2 =
2 2
las que a su vez definen las funciones:

u1 (x) = e1 x u2 (x) = e2 x

Luego, como u2 /u1 no es constante, la solucion general de la ecuacion diferencial


lineal homogenea de orden 2 es:

y (x) = C1 e1 x + C2 e2 x

Ejemplo 2.5.1. Resuelva la ecuacion diferencial:

y 00 + y 0 2y = 0

Solucion. En este caso la ecuacion caracterstica es

2 + 2 = 0

( + 2) ( 1) = 0

154
Apuntes Mat023 (Segundo semestre 2014)

luego tenemos dos soluciones u1 (x) = e2x y u1 (x) = ex de y 00 + y 0 2y = 0 las


cuales son linealmente independientes pues

 e2x

x
2x x e x
W e , e = 2x x = 3e

2e e

la solucion general de la ecuacion sera entonces

y (x) = c1 ex + c2 e2x

2. Si = a2 4b = 0, entonces la ecuacion (2.11) tiene una raz real de multiplicidad


2, dada por:
a
=
2
x
Luego, u1 (x) = e es una solucion, pero dim ker L = 2 por tanto, debemos hallar
otra solucion u2 (x) linealmente independiente con u1 (x), podemos usar la formula
de Abel para la segunda solucion
Z R adx
a e
u2 (x) = e 2 x dx
eax
a
= xe 2 x
a
se sigue que la segunda solucion es u2 (x) = xe 2 x y as la solucion general es

y (x) = c1 ex + c2 xex

donde es la raz repetida de la ecuacion caracterstica.

Ejemplo 2.5.2. Resuelva la ecuacion diferencial:

y 00 + 4y 0 + 4y = 0

Solucion. La ecuacion caracterstica es

2 + 4 + 4 = 0

la cual tiene raz repetida = 2 se sigue que la solucion general de la ecuacion es

y (x) = c1 e2x + c2 xe2x

3. Si = a2 4b < 0, entonces la ecuacion (2.11) no tiene races reales. Sin embargo,


sea:
= + i
una raz compleja de (2.11). Recordemos el siguiente teorema:

155
Apuntes Mat023 (Segundo semestre 2014)

Teorema 2.5.1 (Formula de Euler). Sea R, entonces:

ei = cos + i sin

Entonces:

ex = e(+i)x
= ex eix
= ex {cos (x) + i sin (x)}
= ex cos (x) + i {ex sin (x)}

Consideremos ahora:

Teorema 2.5.2. Sean L (y) = 0 una ecuacion diferencial lineal de orden 2 e y (x) =
6 v. Entonces, u (x) y
u (x) + iv (x) una solucion compleja de la ecuacion tal que u =
v (x) son dos soluciones reales y linealmente independientes de L (y) = 0.

De:
ex = ex cos (x) + i ex sin (x)
se obtiene que:

u1 (x) = ex cos (x) u2 (x) = ex sin (x)

son dos soluciones reales y linealmente independientes de la ecuacion diferencial


lineal:
y 00 + ay 0 + by = 0

Observacion 2.5.3. Debemos notar que ex y ex son efectivamente funciones linealmente


independientes como C espacio vectorial.

Ejemplo 2.5.3. Resuelva la ecuacion:

y 00 + 2y 0 + 5y = 0

Solucion. La ecuacion caracterstica es

2 + 2 + 5 = 0

la cual tiene races


= 1 2i
se sigue que la solucion general es

y (x) = c1 ex cos (2x) + c2 ex sin (2x)

156
Apuntes Mat023 (Segundo semestre 2014)

Ahora daremos una segunda mirada a la ecuacion de segundo orden, en relacion al


operador diferencial asociado. En general los operadores diferenciales no conmutan, esto es,
si L1 y L2 son dos operadores diferenciales en general no es cierto que
L1 L2 = L2 L1
sin embargo, si los operadores diferenciales son con coeficientes constantes, estos se com-
portan igual que polinomios. Por ejemplo, considere los operadores diferenciales D 1 y
D + 2 entonces
(D 1) (D + 2) y = (D 1) (y 0 + 2y)
= y 00 + 2y 0 y 0 2y
= y 00 + y 0 2y
D2 + D 2 y

=
por otro lado
(D + 2) (D 1) y = (D + 2) (y 0 y)
= y 00 y 0 + 2y 0 2y
= y 00 + y 0 2y
D2 + D 2 y

=
se sigue
(D 1) (D + 2) = (D + 2) (D 1) = D2 + D 2


note que si (D 1) y = 0 entonces (D + 2) (D 1) y = 0 y as y ker (D2 + D 2) en


otras palabras una solucion de la ecuacion
y0 y = 0
tambien es solucion de
y 00 + y 0 2y = 0
de manera similar, si (D + 2) y = 0 entonces (D 1) (D + 2) y = 0 y as y ker (D2 + D 2)
en otras palabras una solucion de la ecuacion
y 0 + 2y = 0
tambien es solucion de
y 00 + y 0 2y = 0
pero la soluciones y 0 y = 0 son u1 (x) = cex y las soluciones de y 0 + 2y = 0 son
u2 (x) = ke2x (son de primer orden de variables separadas) as podemos obtener las
soluciones ex y e2x de la ecuacion y 00 + y 0 2y = 0 y como son linealmente independientes,
se sigue que la solucion general de la ecuacion es dada por
y (x) = c1 ex + c2 e2x
son estas ideas las que utilizaremos para resolver la ecuacion diferencial lineal de orden n.

157
Apuntes Mat023 (Segundo semestre 2014)

La ecuacion de orden superior

Observacion 2.5.4. En general, respecto de la ecuacion diferencial:

y (n) + an1 y (n1) + an2 y (n2) + + a1 y 0 + a0 y = Q (x)

tenemos:

Definicion 2.5.2. Sean I R un intervalo abierto y L : C (n) (I) C (I) un operador


diferencial de orden n a coeficientes constantes, es decir:

L = Dn + an1 Dn1 + + a1 D + a0 1 (2.12)

El polinomio caracterstico de la ecuacion diferencial L (y) = 0 es el polinomio en R []


siguiente:
fL () = n + an1 n1 + a1 + a0
Diremos, ademas, que la ecuacion caracterstica asociada a la ecuacion diferencial
L (y) = 0 es la ecuacion siguiente:
fL () = 0
O bien:
n + an1 n1 + a1 + a0 = 0

Ejemplo 2.5.4. La ecuacion diferencial:

y (4) 3y 00 + y 0 3y = 0

tiene polinomio caracterstico:

f () = 4 32 + 3

Note que, el operador diferencial asociado es:

L = D4 3D2 + D 3

Ejemplo 2.5.5. La ecuacion diferencial:

y (5) + 2y (4) 3y 000 y 00 2y 0 + 3y = 0

tiene polinomio caracterstico:

f () = 5 + 24 33 2 2 + 3

Note que:

D5 + 2D4 3D3 D2 2D + 3 = D2 + D + 1 (D 1)2 (D + 3)




158
Apuntes Mat023 (Segundo semestre 2014)

Teorema 2.5.3. Sean L y S dos operadores a coeficientes constantes con polinomios


caractersticos fL y fS , respectivamente, y sea R, entonces:

1. L = S, si y solo si, fL = fS

2. fL+S = fL + fS

3. fLS = fL fS

4. fL = fL

Observacion 2.5.5. Del teorema anterior se deduce que toda relacion algebraica consti-
tuda por sumas, productos y productos por escalares de polinomios caractersticos son
tambien validas para los operadores L y S. En particular, se tienen las leyes conmutativas:

L+S =S+L LS = SL (L) = (L)

Note, ademas, que si L y S son operadores tales que LS = SL, entonces:

ker L + ker S ker (LS) (2.13)

Por otro lado, recordemos que en el conjunto R [x] los elementos irreducibles monicos
son los polinomios de la forma x c y x2 + x + , con 2 4 < 0. Luego, el Teorema
Fundamental del Algebra implica el siguiente resultado:

Teorema 2.5.4. Sea p (x) R [x]. Entonces, p (x) se factoriza como


r s
Y ni
Y mj
p (x) = (x i ) x2 + j x + j (2.14)
i=1 j=1

donde i son las races reales de p (x) con multiplicidad ni , para cada i = 1, 2, . . . , r,
j2 4j < 0 y mj es la multiplicidad de la raz compleja asociada, para cada j = 1, 2, . . . , s.

Obtenemos lo siguiente para los operadores diferenciales dados por la factorizacion de


la ecuacion caracterstica en (2.14):

1. ker (D )m = ex , xex , x2 ex , . . . , xm1 ex





2.
m
ker (D )2 + 2 ex cos (x) , xex cos (x) , . . . , xm1 ex cos (x) ; ex sin (x) ,


=

, xex sin (x) , . . . , xm1 ex sin (x)


Por tanto, mediante el uso del wronskiano y la ecuacion (2.14), la solucion general de la
ecuacion diferencial esta dada por las combinaciones lineales (con las constantes indexadas
adecuadamente) de los sistemas fundamentales de soluciones de cada factor.

159
Apuntes Mat023 (Segundo semestre 2014)

Ejemplo 2.5.6. Resuelva la ecuacion diferencial:


y 000 y 00 8y 0 + 12y = 0
Solucion. Consideremos la ecuacion caracterstica:
3 2 8 + 12 = 0
Como:
3 2 8 + 12 = ( + 3) ( 2)2
se sigue que
ker (D 2)2 ker D3 D2 8D + 12


ker (D + 3) ker D3 D2 8D + 12


pero ker (D 2)2 = h{e2x , xe2x }i y ker (D + 3) = h{e3x }i luego


 2x 2x 3x
ker D3 D2 8D + 12

e , xe , e
sabemos que el espacio ker (D3 D2 8D + 12) tiene dimension tres luego, la solucion
general de la ecuacion
y 000 y 00 8y 0 + 12y = 0
es dada por
yG (x) = c1 e2x + c2 xe2x + c3 e3x
Ejemplo 2.5.7. Suponga que L (y) = 0 es una ecuacion diferencial con operador:
L = D5 + 2D4 3D3 D2 2D + 3
Hallar la solucion general.
Solucion. Note que:
L = D5 + 2D4 3D3 D2 2D + 3
= D2 + D + 1 (D 1)2 (D + 3)


Ahora bien:
* ! !+
3x 3x
ker D2 + D + 1 = ex/2 cos ; ex/2 sin

2 2

ker (D 1)2 = hex ; xex i


y ademas:
ker (D + 3) = e3x

Por tanto, la solucion general de la ecuacion L (y) = 0 esta dada por:


! !
3x 3x
y (x) = C1 ex/2 cos + C2 ex/2 sin + C3 ex + C4 xex + C5 e3x
2 2

160
Apuntes Mat023 (Segundo semestre 2014)

Ejemplo 2.5.8. Hallar la solucion general de una ecuacion diferencial lineal a coeficientes
constantes cuya ecuacion caracterstica es:
5 24 + 63 92 + 8 4 = 0
 
sabiendo que y = ex/2 cos 23 x es una solucion de dicha ecuacion.
Solucion. Sea una ecuacion diferencial lineal a coeficientes constantes homogenea de la
cual se sabe que:
5 24 + 63 92 + 8 4 = 0 (2.15)
es la ecuacion caracterstica asociada, y que ademas:
!
x/2 3
y = e cos x
2
 
x/2 3
es solucion de la ecuacion. Ahora bien, como y = e cos x
es solucion:
2

1 3
= + i
2 2
es una raz de la ecuacion caracterstica (2.15), entonces:

1 3
= i
2 2
tambien es raz, luego la expresion:
( !) ( !)
1 3 1 3
+ i i = 2 + 1
2 2 2 2
factoriza a la ecuacion caracterstica. As:
5 24 + 63 92 + 8 4 = 3 2 + 4 4 2 + 1
 

Notamos, ademas, que = 1 es raz de la ecuacion 3 2 + 4 4 = 0. Entonces, por el


teorema del factor, se tiene que:
3 2 + 4 4 = 2 + 4 ( 1)


entonces
5 24 + 63 92 + 8 4 = 2 + 1 2 + 4 ( 1)
 

Por tanto, la ecuacion (2.15) tiene las siguientes races:


( )
1 3 1 3
+ i, i, 1, 2i, 2i
2 2 2 2
Finalmente, la solucion general de la ecuacion esta dada por:
( ! !)
3 3
y (x) = C1 ex + ex/2 C2 cos x + C3 sin x + {C4 cos 2x + C5 sin 2x}
2 2

161
Apuntes Mat023 (Segundo semestre 2014)

Metodo de variacion de parametros

Observacion 2.6.1. En las secciones anteriores obtuvimos metodos para encontrar el


espacio solucion de ecuaciones diferenciales, para los siguientes casos:

1. Es conocida una solucion particular y1 (x) de la ecuacion:

y 00 + p1 (x) y 0 + p0 (x) y = 0

En particular, para la ecuacion anterior, obtuvimos que la solucion general esta dada
por:
y (x) = C1 y1 (x) + C2 y2 (x)
donde: R
e p1 (x)dx
Z
y2 (x) = y1 (x) dx
y12 (x)
2. La ecuacion diferencial es de la forma:

y (n) + an1 y (n1) + + a1 y 0 + a0 y = 0

con ai R, para cada i = 0, 1, . . . , n 1. En este caso, las soluciones dependen de


las races del polinomio caracterstico:

fL () = n + an1 n1 + + a1 + a0

Por otro lado, dada la estructura lineal de la ecuacion L (y) = Q, donde L es el operador
diferencial asociado a la ecuacion y Q es una funcion continua, entonces la solucion general
de la ecuacion lineal se descompone como:

y = yh + yp

donde yh es la solucion general de la ecuacion L (y) = 0. El objetivo de esta seccion es


hallar una solucion particular yp para la ecuacion L (y) = Q.

Observacion 2.6.2. En este curso, se ven dos de tales metodos: el metodo de variacion de
parametros y el metodo de los anuladores. Como se acostumbra, haremos los razonamientos
para n = 2, y luego se presentara el resultado para n 2. Veamos primeramente:

Teorema 2.6.1 (Variacion de parametros). Sean u1 y u2 dos soluciones linealmente


independientes de la ecuacion:

L (y) = y 00 + p1 (x) y 0 + p0 (x) y = 0

162
Apuntes Mat023 (Segundo semestre 2014)

en un intervalo I R. Entonces, la ecuacion no homogenea L (y) = Q tiene una solucion


particular de la forma:
yp (x) = C1 (x) u1 (x) + C2 (x) u2 (x)
donde:
Z Z
u2 (x) Q (x) u1 (x) Q (x)
C1 (x) = dx C2 (x) = dx (2.16)
W (x) W (x)

y W (x) es el wronskiano de u1 (x) y u2 (x).

Demostracion. Supongamos que yp (x) = C1 (x) u1 (x) + C2 (x) u2 (x), luego:

yp0 = C1 u01 + C2 u02 + (C10 u1 + C20 u2 )


0
yp00 = C1 u001 + C2 u002 + (C10 u01 + C20 u02 ) + (C10 u1 + C20 u2 )

Ahora bien, al evaluar yp en L se obtiene que:

L (yp ) = yp00 + p1 (x) yp0 + p0 (x) yp


0
= (C10 u01 + C20 u02 ) + (C10 u1 + C20 u2 ) + p1 (x) (C10 u1 + C20 u2 )

pues todos los terminos que contienen a C1 (x) y C2 (x) desaparecen dado que L (u1 ) =
L (u2 ) = 0. Como deseamos que L (yp ) = Q, entonces de la ultima ecuacion imponemos
que:
C10 u1 + C20 u2 = 0 C10 u01 + C20 u02 = Q
As, las funciones C1 (x) y C2 (x), deben satisfacer que:
 0
C1 (x) u1 (x) + C20 (x) u2 (x) = 0
0 0 0 0
C1 (x) u1 (x) + C2 (x) u2 (x) = Q (x)
para cada x I. Sin embargo, note que el ultimo sistema tiene solucion si y solo si la matriz
de coeficientes de el tiene determinante no nulo. Es decir, si:

u1 (x) u2 (x)
u0 (x) u0 (x) 6= 0

1 2

para cada x I. Pero el determinante anterior es el wronskiano W (x) de u1 (x) y u2 (x), y


como u1 , u2 son linealmente independientes W (x) = 6 0, para cada x I. Por lo tanto, el
sistema tiene unica solucion, y ademas, por la regla de Cramer, obtenemos finalmente:

0 u2 (x)

Q (x) u0 (x) u2 (x) Q (x)
2
C10 (x) = =
u1 (x) u2 (x) W (x)

u0 (x) u0 (x)
1 2

163
Apuntes Mat023 (Segundo semestre 2014)

y:

u1 (x) 0
u01 (x) Q (x) u1 (x) Q (x)
C10 (x) = =
u1 (x) u2 (x) W (x)

u0 (x) u0 (x)
1 2

Integrando, se obtiene el resultado deseado.

Observacion 2.6.3. En base a los razonamientos en la demostracion del teorema anterior,


tenemos:

Teorema 2.6.2. Sean u1 , u2 , . . . , un una familia de n soluciones linealmente independientes


de la ecuacion lineal de orden n:

L (y) = y (n) + pn1 (x) y (n1) + + p1 (x) y 0 + p0 (x) y = 0

definida en un intervalo abierto I R. Entonces, la ecuacion no homogenea L (y) = Q,


tiene una solucion particular:
n
X
yp (x) = Ci (x) ui (x)
i=1

donde Ci (x), con i = 1, 2, . . . , n son funciones que se obtienen como soluciones del sistema
de ecuaciones:



C10 u1 + C20 u2 + + Cn0 un = 0
0 0 0 0 0 0
C1 u1 + C2 u2 + + Cn un = 0



0 00 0 00 0 00
C1 u1 + C 2 u2 + + Cn un = 0 (2.17)


.
.. .
..


C0 u (n1) (n1) (n1)
+ C20 u2 + + Cn0 un

1 1 = Q (x)

Ejemplo 2.6.1. Resuelva la ecuacion diferencial:

y 00 + y = tan x

Solucion. Consideremos un intervalo en el que el segundo miembro sea continuo, sea este
(a, a), siendo 0 < a < /2. La ecuacion homogenea tiene solucion general:

yh (x) = C1 cos x + C2 sin x

pues la ecuacion caracterstica de dicha ecuacion homogenea es:

2 + 1 = 0

164
Apuntes Mat023 (Segundo semestre 2014)

En particular, notamos que:



cos x sin x
W (x) = = cos2 x + sin2 x = 1
sin x cos x

Entonces, en vista de las ecuaciones en (2.16) las funciones C1 (x) y C2 (x) satisfacen:
Z
sin x tan x
C1 (x) = dx
W (x)
Z
= sin x tan xdx
= sin x ln |sec x + tan x|

y ademas:
Z
cos x tan x
C2 (x) = dx
W (x)
Z
= cos x tan xdx
= cos x

Por tanto, por el metodo de variacion de parametros, una solucion particular yp viene dada
por:

yp = C1 (x) cos x + C2 (x) sin x


= sin x cos x cos x ln |sec x + tan x| sin x cos x
= cos x ln |sec x + tan x|

y la solucion general es:

y = C1 cos x + C2 sin x + yp
= C1 cos x + C2 sin x cos x ln |sec x + tan x|

Solucion. Hallar la solucion general de la ecuacion diferencial:


ex
y 00 2y 0 + y =
(1 x)2
Solucion. Considere la ecuacion diferencial:
ex
y 00 2y 0 + y =
(1 x)2
Notamos que la ecuacion anterior es una ecuacion diferencial lineal a coeficientes constantes
no homogenea. As, si y = y (x) es la solucion general, entonces:

y (x) = yh (x) + yp (x)

165
Apuntes Mat023 (Segundo semestre 2014)

donde yh (x) es la solucion de la ecuacion homogenea asociada e yp (x) es una solucion


particular. Consideremos, entonces, la ecuacion homogenea asociada:
y 00 2y 0 + y = 0
Luego, la ecuacion caracterstica:
2 2 + 1 = 0
tiene raz = 1 con multiplicidad 2. Por tanto, la solucion homogenea yh (x) esta dada por:
yh (x) = C1 ex + C2 xex
Ahora bien, por el metodo de variacion de parametros, tenemos que la solucion particular
yp (x) tiene la forma:
yp (x) = C1 (x) ex + C2 (x) xex
donde C1 (x) y C2 (x) son soluciones del sistema de ecuaciones:
(
C10 (x) ex + C20 (x) xex = 0
0 x 0 x ex
C1 (x) e + C2 (x) (1 + x) e = (1x)2

Ahora bien, como el wronskiano:


x
e xex
= e2x
W (x) = x
e (1 + x) ex

es no nulo, por la regla de Cramer, se tiene que:



0
xex

e x
(1x)2 (1 + x) ex

0
x
C1 (x) = =
W (x) (x 1)2
y ademas:
ex 0

x ex
e

(1x)2
1
C20 (x) = =
W (x) (x 1)2
As: Z
x 1
C1 (x) = 2 dx = {ln (x 1) x ln (x 1) + 1}
(x 1) x1
y: Z
1 1
C2 (x) = 2 dx =
(x 1) x1
Por tanto, la solucion general y (x) de la ecuacion diferencial esta dada por:
ex xex
y (x) = C1 ex + C2 xex + {ln (x 1) x ln (x 1) + 1}
x1 x1
x x x
= C1 e + C2 xe e {ln (x 1) + 1}

166
Apuntes Mat023 (Segundo semestre 2014)

Ejemplo 2.6.2. Hallar la solucion general de la ecuacion


 
2 00 0 2 1
x y + xy + x y = x3/2
4
en el intervalo (0, +) sabiendo que y1 (x) = x1/2 cos x es solucion de la ecuacion ho-
mogenea asociada.
Solucion. Usando la formula de Abel obtenemos una segunda solucion de a homogenea
dx
Z R
e x
y2 (x) = x1/2 cos x 1
dx
x cos2 x
e ln x
Z
1/2
= x cos x dx
x1 cos2 x
Z
1/2 1
= x cos x dx
cos2 x
= x1/2 cos x tan x
= x1/2 sin x
ahora buscamos la solucion particular usando variacion de parametros
yp (x) = x1/2 cos x C1 (x) + x1/2 sin x C2 (x)
 

donde

0
x1/2 sin x

1/2 0
x x1/2 sin x

x1 sin x
C10 (x) = = 1

x1/2 cos x x1/2 sin x x
d
x1/2 cos x d
x1/2 sin x
 
dx dx
= sin x
as C1 (x) = cos x
x1/2 cos x


0
d
x1/2 cos x x1/2

x1 cos x
C20 (x) = dx
=
x1/2 cos x x1/2 sin x x1

d x1/2 cos x d
x 1/2
sin x

dx dx
= cos x
as C2 (x) = sin x se sigue
x1/2 cos x cos x + x1/2 sin x sin x
 
yp (x) =
= x1/2
la solucion general de la ecuacion no homogenea es
yg (x) = x1/2 cos x + x1/2 sin x + x1/2

167
Apuntes Mat023 (Segundo semestre 2014)

Observacion 2.6.4. Las integrales que aparecen en las formulas (2.16) o al despejar
C1 (x) y C2 (x) del sistema (2.17), son integrales indefinidas, luego las funciones quedan
determinadas salvo una constante de integracion. Suponga entonces que tenemos:
vi (x) = Ci (x) + Ki
con Ki R, i = 1, 2, constantes de integracion. Luego, si u1 y u2 son las soluciones
linealmente independientes de L (y) = 0, considere:
y1 (x) = A1 u1 (x) + A2 u2 (x) + v1 (x) u1 (x) + v2 (x) u2 (x)
= (A1 + K1 ) u1 (x) + (A2 + K2 ) u2 (x) + C1 (x) u1 (x) + C2 (x) u2 (x)
Entonces:
y1 (x) = 1 u1 (x) + 2 u2 (x) + C1 (x) u1 (x) + C2 (x) u2 (x)
que era la solucion que ya tenamos.

Metodo del anulador

Definicion 2.7.1. Diremos que el operador diferencial L es un anulador o aniquilador de


si se cumple L () = 0.
Ejemplo 2.7.1. El operador diferencial L = (D 1)2 es un anulador de (x) = 2ex +5xex
pues ker (D 1)2 = h{ex , xex }i, es decir, toda combinacion lineal de ex y xex esta en el
nucleo de (D 1)2 . Notar que (D 1)2 no es un anulador de x2 pues
(D 1)2 x2 = (D 1) (D 1) x2
 

= (D 1) 2x x2


= (2 2x) 2x x2


= x2 4x + 2
6 0
Observacion 2.7.1. Recordemos que ker L + ker L ker (LS) para los operadores con
coeficientes constantes.
2 2
Ejemplo 2.7.2. (D2 + 1) es anulador de x sin x y D2 es anulador de x luego D2 (D2 + 1)
es un anulador de 5x sin x 3x

Suponga que buscamos resolver la ecuacion L (y) = Q donde L es un operador con


coeficientes constantes y LQ es un anulador de Q (con coeficientes constantes tambien)
entonces
L (y) = Q

LQ L (y) = LQ (Q) = 0

168
Apuntes Mat023 (Segundo semestre 2014)

luego la solucion de L (y) = Q se encuentra entre las soluciones de la ecuacion homogenea


LQ L (y) = 0.
Ejemplo 2.7.3. Resolver la ecuacion diferencial:

y 00 5y 0 + 6y = xex

Solucion. Notamos que el operador asociado a la ecuacion diferencial es:

L = D2 5D + 6
= (D 2) (D 3)

As, la ecuacion diferencial puede escribirse como L (y) = Q, con Q (x) = xex . Por otro lado,
la ecuacion homogenea L (y) = 0 tiene soluciones linealmente independientes u1 (x) = e2x
y u2 (x) = e3x . Sabemos que la solucion general de la ecuacion es de la forma:

y (x) = C1 e2x + C2 e3x + yp (x)

donde yp es una solucion de la ecuacion no homogenea L (y) = xex . Notamos que:

xex ker (D 1)2

por tanto, aplicamos el operador diferencial (D 1)2 a la ecuacion diferencial para obtener
:
(D 1)2 (D 2) (D 3) (y) = (D 1)2 (xex ) = 0
As, hemos obtenido la ecuacion diferencial homogenea:

S (y) = (D 1)2 (D 2) (D 3) (y) = 0

por tanto, la solucion general de S (y) = 0 es:

yS (x) = aex + bxex + ce2x + de3x

donde a, b, c y d son constantes reales. Ahora bien, se deben elegir los valores de a, b, c y d
de tal modo que:
L aex + bxex + ce2x + de3x = xex


sin embargo, ce2x + de3x ker L, luego bastara elegir a y b tales que:

L (aex + bxex ) = xex

as, la ecuacion:

L (aex + bxex ) = (D 2) (D 3) (aex + bxex )


= (2a 3b) ex + 2bxex
= xex

169
Apuntes Mat023 (Segundo semestre 2014)

implica que a = 3/4 y b = 1/2. Finalmente:


3 1
yp (x) = ex + xex
4 2
y por tanto:
yp
y
z }|h { z3 }|1 {
y (x) = C1 e + C2 e + ex + xex
2x 3x
4 2
Observacion 2.7.2. Recalcamos que el operador (D 1)2 se eligio de tal modo que:

xex ker (D 1)2

Luego, al aplicar dicho operador a la ecuacion diferencial L (y) = xex , vemos que el
termino de la derecha se anula bajo la aplicacion de (D 1)2 . Por este hecho, el metodo
anterior se llama el metodo del anulador. El metodo del anulador, funciona correctamente
para ecuaciones que son anulables, en el sentido de que son funciones que aparecen como
soluciones de ecuaciones diferenciales a coeficientes constantes. En particular, funciones
que aparecen como combinaciones lineales de funciones de la forma:

xm1 ex xm1 ex cos x xm1 ex sin x

donde m es un numero natural y y son constantes reales. A continuacion detallamos


algunas de las funciones anteriores con sus respectivos anuladores:
Funcion Anulador

y = xm1 Dm
y = ex D
y = xm1 ex (D )m
y = cos x o bien y = sin x D2 + 2
m
y = xm1 cos x o bien y = xm1 sin x (D2 + 2 )
y = ex cos x o bien y = ex sin x D2 2D + (2 + 2 )
m
y = xm1 ex cos x o bien y = xm1 ex sin x {D2 2D + (2 + 2 )}
Observacion 2.7.3. Un resultado que utiliza las propiedades lineales del operador dife-
rencial asociado a una ecuacion, es el principio de superposicion, que facilita en algunos
casos el calculo de la solucion particular. Tenemos, entonces:
Teorema 2.7.1 (Principio de superposicion). Sean L un operador diferencial no necesaria-
mente a coeficientes constantes y f1 , f2 funciones continuas. Suponga que yi es solucion de
la ecuacion diferencial L (y) = fi , con i = 1, 2. Entonces:

y = y1 + y2

es solucion de la ecuacion L (y) = f1 + f2 .

170
Apuntes Mat023 (Segundo semestre 2014)

Este principio nos permite separar la busqueda de la solucion particular.

Ejemplo 2.7.4. Resolver


(D 1) (D 2) y = ex + x
o en otras palabras
d2 y dy
2
3 + 2y = ex + x
dx dx

Solucion. Sabemos que


(D 1) (ex ) = 0 y D2 (x) = 0
se sigue que
(D 1) D2 (ex + x) = 0
as

(D 1) D2 (D 1) (D 2) y = 0
D2 (D 1)2 (D 2) = 0

luego
yp (x) = c1 + c2 x + c3 ex + c4 xex + c5 e2x
y como
(D 1) (D 2) yp (x) = ex + x
se sigue

(D 1) (D 2) c1 + c2 x + c3 ex + c4 xex + c5 e2x
 

= (D 1) (D 2) [c1 + c2 x] + (D 1) (D 2) [c4 xex ]


= D2 3D + 2 [c1 + c2 x] + (D 1) (D 1 1) [c4 xex ]


= 2c1 3c2 + 2c2 x (D 1) [c4 xex ]


= 2c1 3c2 + 2c2 x (c4 ex + c4 xex c4 xex )
= 2c1 3c2 + 2c2 x c4 ex

de esto obtenemos que las constantes cumplen

2c1 3c2 = 0
2c2 = 1
c4 = 1

se sigue
c4 = 1, c2 = 1/2, c1 = 3/4

171
Apuntes Mat023 (Segundo semestre 2014)

as

yG (x) = (3/4) + (1/2) x xex + c3 ex + c5 e2x


= [(3/4) + (1/2) x xex ] + c3 ex + c5 e2x
 

= yp (x) + yh (x)

es la solucion general.
Note que
d2
(D 1) (D 2) ((3/4) + (1/2) x) = ((3/4) + (1/2) x)
dx2
d
3 ((3/4) + (1/2) x) + 2 ((3/4) + (1/2) x)
dx
= x
d2 d
(D 1) (D 2) (xex ) = 2
(xex ) 3 (xex ) + 2 (xex )
dx dx
= 3ex (x + 1) ex (x + 2) 2xex
= ex

luego vale el principio de superposicion.

Veamos como podemos trabajar, en algunos casos, si no conocemos el anulador:


Ejemplo 2.7.5. Hallar la solucion general de la ecuacion diferencial:
2
y 00 3y 0 + 2y = xex+x

Solucion. Notamos que:

L = D2 3D + 2
= (D 1) (D 2)

Buscamos, primeramente, una solucion particular yp . Considere el cambio de variables:

u = (D 2) y (2.18)

la ecuacion queda:
2
(D 1) u = xex+x
Mediante la formula de Leibnitz, obtenemos:
Z
x 2 1 2
ue = xex dx + C = ex + C
2
Ahora bien, como buscamos yp , podemos elegir C = 0. As:
1 2
u = ex+x
2
172
Apuntes Mat023 (Segundo semestre 2014)

Sustituyendo en (2.18), se obtiene:


1 2
(D 2) y = ex+x
2
Luego, nuevamente por la formula de Leibnitz:
Z
1 2x 2
yp = e ex x dx
2
Finalmente, la solucion general de la ecuacion diferencial es:
Z
1 2x 2
x 2x
y = C1 e + C2 e + e ex x dx
2

Ejemplo 2.7.6. Resuelva la ecuacion diferencial:

y (6) + 2y (5) y (4) + 4y 000 5y 00 + 2y 0 3y = x

sabiendo que =3 es una raz de la ecuacion caracterstica y que u1 (x) = cos x es una
solucion de la ecuacion homogenea asociada.

Solucion. Consideremos la ecuacion diferencial:

y (6) + 2y (5) y (4) + 4y 000 5y 00 + 2y 0 3y = x (2.19)

En primer lugar, notamos que la ecuacion caracterstica de la ecuacion homogenea asociada


a (2.19) es:
f () = 6 + 25 4 52 3 = 0 (2.20)
Ahora bien, como u1 (x) = cos x es una solucion de la ecuacion homogenea, entonces = i
y = i son races de la ecuacion (2.20). Ademas, por hipotesis, = 3 es una raz de la
ecuacion (2.20). Por tanto, por el Teorema del Factor, el polinomio f () es divisible por

2 + 1 ( + 3). As, se obtiene la siguiente factorizacion:

f () = (2 + 1)( + 3) 3 2 + 1


Por otro lado, por inspeccion, notamos que = 1 es raz de 3 2 + 1, entonces


3 2 + 1 = (2 + 1)( 1). Por tanto, la factorizacion completa de f () es:
2
f () = 2 + 1 ( 1)( + 3) (2.21)

As, la solucion de la ecuacion homogenea esta dada por:

yh (x) = C1 cos x + C2 sin x + C3 x cos x + C4 x sin x + C5 ex + C6 e3x

173
Apuntes Mat023 (Segundo semestre 2014)

Debemos, ahora, calcular una solucion particular de la ecuacion:


2
D2 + 1 (D 1)(D + 3) y(x) = x

Un anulador para la funcion Q(x) = x es el operador diferencial D2 . Entonces, por el


metodo del anulador, sabemos que una solucion particular yp (x) de la ecuacion anterior,
satisface:
2
yp ker D2 D2 + 1 (D 1)(D + 3)
Luego:
yp (x) = a + bx + (x)
2
donde (x) ker D2 + 1 (D 1)(D + 3). Entonces:
2 2
D2 + 1 (D 1)(D + 3) yp (x) = D2 + 1 (D 1)(D + 3) a + bx + (x)
 
2
= D2 + 1 (D 1)(D + 3) a + bx
 
2
= D2 + 1 (D 1) b + 3a + 3bx
 
2 
= D2 + 1 2b 3a 3bx


= 2b 3a 3bx
2
Ahora bien, como D2 + 1 (D 1)(D + 3) yp (x) = x, obtenemos el sistema de ecuaciones:

3b = 1
2b 3a = 0

de donde a = 2/9 y b = 1/3. As, la solucion particular de la ecuacion (2.19) es


yp (x) = 29 x 13 .
Finalmente, la solucion y(x) = yh (x) + yp (x) de la ecuacion diferencial en (2.19) es:

2 1
y(x) = C1 cos x + C2 sin x + C3 x cos x + C4 x sin x + C5 ex + C6 e3x x
9 3
Finalmente, como una aplicacion de los metodos de variacion de parametros y del
anulador resolveremos la ecuacion de Euler.

Definicion 2.7.2. Una ecuacion de Euler es una ecuacion de la forma:

(ax + b)n y (n) + An1 (ax + b)n1 y (n1) + + A1 (ax + b) y 0 + A0 y = Q (x) (2.22)

donde ax + b > 0 y A0 , A2 , . . . , An1 son constantes cualesquiera.

Observacion 2.7.4. El metodo de resolucion de la ecuacion de Euler consiste en considerar


el cambio de variables:
ax + b = et

174
Apuntes Mat023 (Segundo semestre 2014)

Como se puede observar, el cambio de variable es un cambio en la variable independiente.


As, por la regla de la cadena:
dy dy dt dy
= = aet
dx dt dx dt 
d2 y
    2 
d dy d t dy dt 2 2t d y dy
= = ae =a e
dx2 dx dx dt dt dx dt2 dt
3
 2   2  3
d2 y
  
dy d dy d 2 2t d y dy 3 3t dy dy
= = ae =a e 3 2 +2
dx3 dx dt dt dt2 dt dt3 dt dt
y as sucesivamente. Reemplazando todas las derivadas de orden superior en la ecuacion de
Euler en (2.22) se obtiene la siguiente ecuacion a coeficientes constantes:

y (n) (t) + Bn1 y (n1) (t) + + B1 y 0 (t) + B0 y (t) = Q (t)

Ejemplo 2.7.7. Resuelva la ecuacion diferencial:

x2 y 00 + xy 0 + y = 1

Solucion. Utilizando el cambio de variables x = et , se tiene que:


d2 y
 2 
dy t dy 2t d y dy
=e =e
dx dt dx dt2 dt
al reemplazar en la ecuacion original, obtenemos:
d2 y
+y =1
dt2
luego:
y = C1 cos t + C2 sin t + 1
donde yp = 1, se obtiene inmediatamente por medio del metodo del anulador. Por tanto:

y = C1 cos (ln x) + C2 sin (ln x) + 1

es la solucion general de la ecuacion diferencial.

Ejemplo 2.7.8. Resolver las ecuacion de Euler

x2 y 00 + 2xy 0 2y = 0

Solucion. Haciendo el cambio de variables x = et se sigue


 2 
2 00 d y dy
xy =
dt2 dt
dy
xy 0 =
dt
175
Apuntes Mat023 (Segundo semestre 2014)

entonces la ecuacion se transforma en

D (D 1) y + 2Dy 2y = 0

donde y = y (et ) entonces

D2 + D 2 y = 0 (D + 2) (D 1) y = 0


se sigue
y et = c1 e2t + c2 et


se sigue
c1
y (x) = + c2 x
x2
Ejemplo 2.7.9. Determine una solucion y = y(x) de la ecuacion diferencial:

x3 y 00 + 5x2 y 0 + 4xy = x2 + 1, x>0

de modo que la recta tangente a esta solucion en el punto (1, 1) tenga una inclinacion de
45 con respecto al eje X.

Solucion. Consideremos la ecuacion:

x3 y 00 + 5x2 y 0 + 4xy = x2 + 1, x>0 (2.23)

Como x > 0, tenemos que la ecuacion (2.23) se escribe de forma equivalente como la
ecuacion:
1
x2 y 00 + 5xy 0 + 4y = x + , x>0 (2.24)
x
la cual es una ecuacion de Euler. En efecto, consideramos x = et , y entonces:

t dy dy dt d2 y d  dy 
i) dx = e dt ii) = iii) =
dt dt dx dx2 dx dx
dy d  t dy  dt
= et = e
dt dt dt dx
 2 
2t d y dy
=e
dt2 dt

Luego, al cambiar la variable y reemplazar ii) y iii) en la ecuacion (2.24), obtenemos:

d2 y dy
2
+ 4 + 4y = 2 cosh t (2.25)
dt dt
Ahora bien, la ecuacion caracterstica asociada a la ecuacion anterior:

2 + 4 + 4 = 0

176
Apuntes Mat023 (Segundo semestre 2014)

tiene raz = 2 con multiplicidad 2. Luego, la solucion de la ecuacion homogenea asociada


a (2.25) es:
yh (t) = C1 e2t + C2 te2t
Ahora, por el metodo de variacion de parametros, la solucion particular de la ecuacion a
coeficientes constantes (2.25) esta dada por:

yp (t) = C1 (t) e2t + C2 (t) te2t

donde C1 (t) y C2 (t) estan dadas por el sistema de ecuaciones:

C10 (t)e2t + C20 (t)te2t = 0




2C10 (t)e2t + C20 (t)(1 2t)e2t = 2 cosh t

As, entonces, por la regla de Cramer, tenemos que:


2t

0 te

2 cosh t (1 2t)e 2t
C10 (t) =
W (t)
y: 2t
e 0

22t 2 cosh t
C10 (t) =
W (t)
donde:
2t
te2t

e
W (t) =

2e2t (1 2t)e2t

1 t
= e4t


2 1 2t
= e4t

Luego:
e3t
Z
te3t + tet dt = (1 t)et +

C1 (t) = (1 3t)
9
y:
e3t
Z
e3t + et dt = et +

C2 (t) =
3
Por tanto:
e3t e3t  2t
yp (t) = (1 t)et + (1 3t) e2t + et +
  
te
9 3
et
= + et
9

177
Apuntes Mat023 (Segundo semestre 2014)

As, la solucion general (en la variable t) de la ecuacion (2.25) esta dada por:

et
y(t) = C1 e2t + C2 te2t + + et
9
Ahora bien, como x = et , se tiene que t = ln x, luego:
1 ln x x 1
y(x) = C1 + C 2 + +
x2 x2 9 x
Finalmente, como la curva y = y(x) debe pasar por el punto (1, 1), y ademas en este
punto la recta tangente a la curva forma un angulo de 45 con respecto al eje X, se tiene,
entonces, que las condiciones iniciales son y(1) = 1 e y 0 (1) = 1. Por tanto, C1 = 19
9
y
55
C2 = 9 .
As, y finalmente, la curva y = y(x) del problema es:
19 55 ln x x 1
y(x) = 2
+ + +
9x 9x2 9 x
En las ecuaciones diferenciales de orden superior (al igual que en las de primer orden)
podemos hacer los cambios de variables que consideremos convenientes, sin embargo,
cualquier cambio de variables no asegura el poder resolver la ecuacion:

Ejemplo 2.7.10. Haciendo el cambio de variables x = t3 resolver la ecuacion


d2 y 2/3 dy
9x4/3 + 2y = x2/3

+ 6 3
x 9x
dx2 dx
Solucion. Usando el cambio de variables
dy dy dt
=
dx dt dx
2
 2
dy d2 y dt dy d2 t
= +
dx2 dt2 dx dt dx2
pero
dt 1 2/3
= x
dx 3
d2 t 2 5/3
= x
dx2 9
se sigue

dy dy 1 2
= t
dx dt 3
2
d2 y d2 y 1 2
  
dy 2 5
= t + t
dx2 dt2 3 dt 9

178
Apuntes Mat023 (Segundo semestre 2014)

reemplazando
2 !
d2 y
    
4 1 2 dy 2 5 2 dy 1 2
+ 2y = t2

9t t + t + 6t 9t t
dt2 3 dt 9 dt 3

esto es
d2 y dy 1 1 dy dy
+ 2y = t2

+ 2t + 2t 3
dt2 dt dt dt
es decir,
d2 y dy
3 + 2y = t2
dt2 dt
esta ecuacion es
(D 2) (D 1) y = t2
aplicamos anuladores
D3 (D 2) (D 1) y = 0
la solucion es de la forma

y = e2t + et + C1 + C2 t + C3 t2

determinamos las constantes C1 , C2 y C3

D2 3D + 2 C1 + C2 t + C3 t2 = t2


2C3 3C2 6C3 t + 2C1 + 2C2 t + 2C3 t2 = t2

obtenemos el sistema

2C3 3C2 + 2C1 = 0


6C3 + 2C2 = 0
2C3 = 1

que tiene solucion C1 = 47 , C2 = 32 , C3 = 1


2
as
7 3 1
y = e2t + et + + t + t2
4 2 2
volvemos a la variable original y la solucion es

3
3 7 3 1
y (x) = e2 x
+ e x
+ + 3 x + x2/3
4 2 2
con , R.

Ejemplo 2.7.11. Use z = sin (x) para resolver la ecuacion

y 00 + tan (x) y 0 + cos2 (x) y = sin (x) cos2 (x)

179
Apuntes Mat023 (Segundo semestre 2014)

Solucion. Las derivadas son con respecto a x la idea es cambiar las derivadas ahora con
respecto a z, para ello usamos la regla de la cadena: Notemos que
dy dy dx
=
dz dx dz
o bien
dy dz dy
=
dz dx dx
reemplazando
dy

dy dx dy
= = sec x
dz (cos x) dx
luego
 
d dy d
= ((sec x) y 0 )
dz dz dz
    2 
dx dy d y dx
= sec x tan x + sec x
dz dx dx2 dz
1 d2 y
 
sin x 1 dy
= +
cos2 x cos x dx cos2 x dx2

se sigue
d2 y dy d2 y
 
2
cos x = tan x +
dz 2 dx dx2
reemplazando en la ecuacion original
 d2 y
 
2
+ cos2 x y = 0

cos x 2
dz

luego
d2 y
+y =0
dz 2
se sigue
y (z) = c1 sin z + c2 cos z
es decir la solucion general de la es

y (x) = c1 sin (sin x) + c2 cos (sin x) para c1 , c2 R

180
Apuntes Mat023 (Segundo semestre 2014)

Movimiento vibratorio

Observacion 2.8.1. Consideremos un sistema mecanico compuesto por un resorte de


longitud inicial l0 con su extremo superior sujeto firmemente y una masa m atada en su
extremo inferior. Nos interesa hallar la posicion x (t) de la masa en cualquier instante
t. Mas precisamente, nos interesa hallar la ecuacion que describa el movimiento de la
partcula a la cual se le ha dado un desplazamiento x0 y una velocidad inicial v0 . Para ello
consideramos los siguientes supuestos:

1. La masa se mueve a lo largo de la vertical que pasa por el centro de gravedad de la


masa y en el sistema de referencia x = 0 indica la posicion de reposo de la masa en
el resorte.

2. En cualquier tiempo t la magnitud de la fuerza ejercida sobre la masa es proporcional


a la diferencia entre la longitud l del resorte y su posicion de equilibrio l0 . La constante
positiva de proporcionalidad k se llama constante del resorte, y el principio anterior
se conoce como la ley de Hooke.

Recordemos que la segunda ley del movimiento de Newton establece que la sumatoria
de fuerzas que actua sobre la partcula es igual a la variacion instantanea del momemtum
mv respecto del tiempo, pero como la masa m es constante, se tiene:
X
Fi = m a
i

2
donde a (t) = d dtx(t)
2 .

Suponemos, por ahora, que la unica fuerza que actua sobre la partcula es la fuerza
restauradora inducida por el resorte, es decir, no se consideran las fuerzas de friccion, por
ejemplo. Por la ley de Hooke, tenemos que:

d2 x
m 2 = kx
dt
y el signo negativo es debido a que la fuerza restauradora del resorte se opone al movimiento.
Asumiendo, que la posicion inicial (es decir, en t = 0) de la masa es x0 y que su velocidad
inicial es v0 , obtenemos el siguiente problema de valor inicial:
k
x00 + x = 0, x (0) = x0 , x0 (0) = v0 (2.26)
m
La ecuacion caracterstica de la ecuacion diferencial anterior es:
k
2 + =0
m

181
Apuntes Mat023 (Segundo semestre 2014)

As:
= 0
p
en donde 0 = k/m, luego:

x (t) = C1 cos 0 t + C2 sin 0 t

Usando las condiciones iniciales, obtenemos que C1 = x0 y C2 = v0 /0 . Por tanto:

x (t) = x0 cos 0 t + (v0 /0 ) sin 0 t


= A sin (0 t + )
q
donde A = x20 + (v0 /0 )2 y es tal que tan = x0 0 /v0 .

Definicion 2.8.1. El movimiento de la masa m bajo las condiciones anteriores, se denomina


movimiento armonico simple. La frecuencia natural f del movimiento es el numero
de oscilaciones completas por unidad de tiempo:
0
f=
2
Ejemplo 2.8.1. Considere un resorte, sujeto en su extremo superior, que sostiene una
pesa de 10 libras en su extremo inferior. Suponga que la pesa estira el resorte 6 pulgadas.
Halle la ecuacion del movimiento de la pesa si esta se lleva a una posicion de 4 pulgadas
por debajo de su punto de equilibrio y se suelta. Calcule la ecuacion del movimiento su
amplitud y la frecuencia natural. h i
pie
Ayuda: Considere que 1 [pie] = 12 [pulgadas] y que g = 32 seg 2 .

Observacion 2.8.2. En los supuestos que dan origen al movimiento armonico simple
no se consideran las fuerzas, que en una situacion mas realista, actuan sobre la masa.
Considerando por ejemplo resistencia del medio, ya sea por accion del aire o realizacion
de la oscilacion en un medio viscoso, es que se tienen las vibraciones amortiguadas. El
supuesto es simple, la fuerza amortiguadora del movimiento se presupone proporcional a
la velocidad del movimiento, as el problema de valor inicial de la ecuacion (2.26) queda
como:
k c
x00 = x x0 (2.27)
m m
con las condiciones iniciales x (0) = x0 y x0 (0) = v0 . La ecuacion caracterstica de la
ecuacion (2.27) es:
c k
2 + + =0
m m
cuya solucion es:
c c2 4km
=
2m

182
Apuntes Mat023 (Segundo semestre 2014)

Por tanto, el comportamiento del movimiento depende del valor que tome el discriminante:

= c2 4km

Tenemos, por tanto, tres casos:

Caso 2.8.1 (c2 4km > 0). La solucion de la ecuacion (2.27) en este caso esta dada por:
   
c + c2 4km c c2 4km
x (t) = C1 exp + C2 exp
2m 2m

Note que 4km < c2 , lo que implica que c2 4km < c, y entonces:

c + c2 4km < 0 c c2 4km < 0

As, en este caso tenemos que:


lm x (t) = 0
t

Caso 2.8.2 (c2 4km = 0). La solucion de la ecuacion (2.27) es:

x (t) = (C1 + C2 t) e(c/2m)t

y al igual que en el caso anterior:


lm x (t) = 0
t

Observacion 2.8.3. Entonces, se observa por la forma de las soluciones que si:

c2 4km 0

no se producen oscilaciones en el sistema mecanico masa-resorte considerando amortiguacion.


Este tipo de movimiento tpicamente ocurre en un medio viscoso como agua o aceite.

Caso 2.8.3 (c2 4km < 0). Como (c/2m) 6= 0, obtenemos:


 c  
4km c2 4km c2
x (t) = exp C1 cos t + C2 sin t
2m 2m 2m
Observacion 2.8.4. Finalmente, podemos suponer una fuerza g = g (t) aplicada sobre
extremo superior del resorte (que inicialmente se supuso sujeto firmemente, esto es con
g 0) en la direccion del movimiento de la masa m. El movimiento de la masa obtenido bajo
este supuesto se conoce como vibracion forzada. Una vibracion forzada tambien puede
contener fuerzas amortiguadoras. Este es el caso mas general del movimiento vibratorio, el
problema de valor inicial queda en su forma mas general como:
c 0 k g (t)
x00 + x + x= (2.28)
m m m
con x (0) = x0 e x0 (0) = v0 .

183
Apuntes Mat023 (Segundo semestre 2014)

Ejemplo 2.8.2. Considere un resorte que en su extremo superior esta sometido a una
fuerza externa:
10
f (t) = sin t
32
y sostiene, en su parte inferior una masa de 10 libras. Suponga que la masa estira el resorte
6 pulgadas. Hallar la ecuacion del movimiento de la masa si esta se lleva a una posicion de
4 pulgadas por debajo del punto de equilibrio y se suelta.

Observacion 2.8.5. Un caso muy importante es cuando g es una funcion periodica, es


decir, supongamos que:
g (t) = F0 sin t
As, la ecuacion (2.28) queda:
c 0 k F0
x00 + x + x= sin t (2.29)
m m m
Sabemos que el anulador de g (t) = F0 sin t es el operador:

D2 + 2

Aplicando este ultimo operador al operador D2 + mc D + m


k
, obtenemos:
 
2 2 2 c k  F0
x = D2 + 2

D + D + D+ sin t = 0
m m m

Por tanto, la ecuacion (2.29) tiene una solucion particular de la forma:

xp = b1 cos t + b2 sin t (2.30)

donde b1 y b2 son soluciones del sistema:

(02 2 ) b1 + c

m 2
b = 0
c 2 2 F0
m b1 + (0 ) b2 = m
q
k
donde 0 = m
. As:

F0 c
b1 = 2
m2 2 ) + (c)2
(02
F0 m (02 2 )
b2 = 2
m2 (02 2 ) + (c)2
As, a partir de lo anterior y mediante el procedimiento de transformacion a sinusoide de
la ecuacion (2.30) encontramos que:

xp = A sin (t + )

184
Apuntes Mat023 (Segundo semestre 2014)

donde la amplitud A esta dada por:


F0 /k
A = s
 2 2  2
c
1 0 + 2 c0 0

y el angulo de fase satisface:


2 cc0
tan =  2

0
1
con c0 = 2m0 . Notamos que de la ecuacion que define la amplitud los coeficientes
importantes son:

1. c/c0 llamado el cociente de amortiguacion, y

2. /0 llamado el cociente de frecuencia del movimiento.

Se observa que el comportamiento de la solucion particular xp depende entonces del


cociente de amortiguacion si este es cero o despreciable, entonces tenemos el movimiento
armonico simple:
x00 + 02 x = 0
y por tanto, hay dos casos para el cociente de frecuencia:

Caso 2.8.4 ( 2 6= 02 ). En este caso se tiene:

F0 /k
x (t) = C1 cos 0 t + C2 sin 0 t + sin t
1 (/0 )2
donde:
v0 (F0 /k) (/0 )
C 1 = x0 C2 =
0 1 (/0 )2
Caso 2.8.5 ( 2 = 02 ). En este caso debemos buscar una solucion particular de la forma:

xp = b1 t cos t + b2 t sin t

pues la ecuacion (2.30) se anula con el operador D2 + 2 , luego el anulador a considerar es


2
(D2 + 2 ) . As, luego de los calculo habituales del metodo del anulador se obtiene:
F0
b1 = b2 = 0
2m
Por tanto, la solucion general tiene la forma:
F0
x (t) = C1 cos t + C2 sin t t cos t
2m

185
Apuntes Mat023 (Segundo semestre 2014)

Observacion 2.8.6. Observe en el caso anterior que al aumenta t, las vibraciones causadas
por el ultimo termino de la ecuacion anterior aumentaran sin cota. En este caso, se dice
que la fuerza externa esta en resonancia con la masa en vibracion.

Ejemplo 2.8.3. Una masa de 2 [kg] se sujeta de un resorte suspendido del techo. Esto
ocasiona que el resorte se estire 49
65
[mt] al llegar a su estado de equilibrio. En el instante
t = 0 la masa se desplaza 1[mt] hacia abajo y se suelta. En ese mismo instante se aplica
una fuerza externa 5 sin(2t) [newton] al sistema. Si la constante de amortiguacion es de
seg
 
8h newton
i mt
. Determine el desplazamiento x(t) de la masa para t > 0. Considere g=9, 8
mt
seg2
.

Solucion.
mx00 + cx0 + kx = 5 sin(2t)
donde
49
2 (9,8) = k
65
de donde k = 26 se sigue

2x00 + 8x0 + 26x = 5 sin(2t)


x (0) = 1
x0 (0) = 0

la solucion de la ecuacion es
9 4
x (t) = sin 2t cos 2t + C1 (cos 3t) e2t + C2 e2t (sin 3t)
58 29
reemplazando la C.I
4
1= + C1
29
33
C1 = 29
9 4 33
x (t) = sin 2t cos 2t + (cos 3t) e2t + C2 e2t (sin 3t)
58 29 29
derivando y reemplazando
9 66
0= + 3C2
29 29
C2 = 19
29
se sigue
        
9 4 33 19
x (t) = sin 2t cos 2t + cos 3t + sin 3t e2t
58 29 29 29

186
Apuntes Mat023 (Segundo semestre 2014)

Ejercicios del captulo

1. En los problemas siguientes se muestra que el principio de superposicion generalmente


no se cumple para ecuaciones no lineales

a) Muestre que y = 1/x es solucion de la ecuacion y 0 + y 2 = 0 pero si c 6= 0 y c 6= 1


entonces y = c (1/x) no es solucion.

b) Muestre que 1 (x) = 1 e 2 (x) = x son soluciones de yy 00 + (y 0 )2 = 0 pero la
suma no es solucion.

2. Busque una solucion polinomial de la ecuacion de Legendre

1 x2 y 00 2xy 0 + 2y = 0


y usando la formula de Abel determine la solucion general de esta ecuacion.

3. Verificar que 1 (x) = x1/2 cos x es solucion (para x > 0) de la ecuacion de Bessel
de orden 12  
2 00 0 2 1
x y + xy + x y=0
4
y obtener su solucion general.

4. Muestre que y1 (x) = x + 1 es solucion de la ecuacion

1 2x x2 y 00 + 2 (1 + x) y 0 2y = 0


y resolverla completamente (encontrar solucion general).

5. Considere la ecuacion diferencial:

y 00 + P1 (x) y 0 + P2 (x) y = Q (x) (2.31)

y sean u1 (x) , u2 (x) soluciones linealmente independientes de la ecuacion homogenea


asociada. Pruebe que la funcion:

f (x) = C1 (x) u1 (x) + C2 (x) u2 (x)

es una solucion particular de la ecuacion (2.31), si se satisface el sistema de ecuaciones:


 0
C1 (x) u1 (x) + C20 (x) u2 (x) = 0
0 0 0 0
C1 (x) u1 (x) + C2 (x) u2 (x) = Q (x)

6. Compruebe que: Z x
1
y= f (t) sin (x t) dt
0
es una solucion particular de y 00 + 2 y = f (x).

187
Apuntes Mat023 (Segundo semestre 2014)

7. Suponga que ex y ex son soluciones de una ecuacion diferencial lineal ordinaria


homogenea, muestre que las funciones sinh x y cosh x tambien son soluciones de esa
ecuacion.

8. Muestre que la solucion particular de una EDO lineal de segundo orden no homogenea

y 00 + a1 (x) y 0 + a0 (x) y = f (x)

se puede escribir en la forma


Z x
yp (x) = G (x, t) f (t) dt
x0

donde G es una funcion adecuada.

9. Hallar la solucion general de


d y4 2
d y
1) y 000 + 3y 00 y 0 3y = 0 2) y (4) + 16y = 0 3) dx 4 + dx2 + y = 0

4) y 00 36y = 0 5) y 00 + 2y 0 3y = 0 6) y 00 + 8y 0 + 16y = 0
7) y 0000 + 4y 00 + 3y = 0 8) y 00 4y 0 + 5y = 0 9) y 000 5y 00 + 3y 0 + 9y = 0

10. Resuelva la ecuacion diferencial sujeta a las condiciones indicadas.


0
a) y 00 + 16y = 0 ; y(0) = 2, y (0) = 2
b) y 00 + 6y 0 + 25y = 0 ; y(0) = 2, y 0 (0) = 6
c) y 00 4y = 0 ; y(0) = 4, y 0 (0) = 16
d ) y 000 + 2y 00 5y 0 6y = 0 ; y(1) = 0, y 0 (1) = 0, y 00 (1) = 1

11. Resuelva las siguientes ecuaciones por anuladores.

a) y 00 + y 0 = 2t + sin t
b) y 00 + 4y 0 + 2y = te2t

c) y 00 4y 0 + 5y = 4 + sin( 7t)
d ) y 00 + 3y = x2 e3x
e) y 00 2y 0 + 2y = e2x (cos x 3 sin x)
f ) y 000 y 00 4y 0 + 4y = 5 ex + e2x
g) y 000 2y 00 + 2y 0 y = ex + cos x

12. Resuelva la ecuacion diferencial sujeta a las condiciones indicadas.

a) 2y 00 + 3y 0 2y = 14x2 4x 11 , y(0) = 0, y 0 (0) = 0

188
Apuntes Mat023 (Segundo semestre 2014)

b) y 00 + 4y 0 + 4y = (3 + x)e2x , y(0) = 2, y 0 (0) = 5


c) y 00 2y 0 + 2y = 2x 2 , y() = 0, y 0 () = 0
d2 x
d) + 2 x = F0 cos t , x(0) = 0, x0 (0) = 1
dt2
d2 x
e) 2
+ 2 x = F0 sin t , x(0) = 0, x0 (0) = 0
dt
13. Hallar una EDO lineal con coeficientes constantes que tenga por solucion general:
1) y (x) = c1 xex + c2 ex + x + 1
   
2) y (x) = c1 e2x cos 3x + c2 e2x sin 3x + c3 xe2x cos 3x + c4 xe2x sin 3x + ex

14. Encontrar un anulador de


1) f (x) = x cos 2x + 3

2
2) f (x) = (x2 + 1) ex + x sinh (x)

2
3) f (x) = (x2 1) (1 + sin x) e 3x

15. Encontrar la solucion general de


1) x2 y 00 + xy 0 + 9y = 0
2) x2 y 00 + xy 0 p2 y = 0 con p R

16. Encontrar la solucion general de


00 0
1) x2 y 00 + xy 0 9y = x2 + 1 2) x2 y + xy 9y = x3 + 1
00 0
3) x2 y 00 + 4xy 0 + 2y = 2 ln x 4) x2 y + xy + 9y = sin(ln x3 )
5) x3 y 000 + 4x2 y 00 + xy 0 y = x 6) x2 y 00 xy 0 + 2y = 16
7) y 00 + x1 y 0 x12 y = x2 +x
1
3 8) y 00 x2 y 0 + x22 y = lnxx
9) x4 y 00 + x3 y 0 4x2 y = 1 10) x3 y 000 + 6x2 y 00 + 4xy 0 4y = x
17. Encontrar una solucion general de cada una de las ecuaciones diferenciales, usando el
metodo de variacion de parametros.

a) y 000 y 00 + y 0 y = 4xex
b) y 000 y 0 = sin x
c) y 000 2y 00 = 4(x + 1)
d ) y 000 3y 00 y 0 + 3y = 1 + ex
e) y 000 7y 0 + 6y = 2 sin x
f ) 3y 00 6y 0 + 30y = ex tan 3x
ex
g) y 00 2y 0 + y = 4x2 3 +
x
189
Apuntes Mat023 (Segundo semestre 2014)

h) y 000 2y 0 4y = 2ex sec x (puede dejar integrales finales sin calcular)

18. Una solucion y = u(x) de la ecuacion diferencial y 00 3y 0 4y = 0 intersecta a una


solucion de la ecuacion diferencial y 00 + 4y 0 5y = 0 en el origen. Determinar las
funciones u y v si las dos soluciones tienen la misma pendiente en el origen y si
(v(x))4 5
lm = .
x u(x) 6

19. Sean y1 = u(x) y y2 = xu(x) dos soluciones de la ecuacion y 00 + 4xy 0 + p(x)y = 0. Si


u(0) = 1, determine u(x) y p(x) explcitamente.

20. Se sabe que para x > 0 la ecuacion homogenea asociada a



4xy 00 + 2 8 x y 0 5y = 1

tiene una solucion


y1 (x) = e x

determine la solucion general.

21. Determine la solucion general de la ecuacion diferencial


d2 y dy
(2x 1)2 2
+ 4 (2x 1) 8y = 2x 1
dx dx
Ind.: Considere el cambio de variables u = 2x 1.

22. Dada la ecuacion diferencial


d2 y dy
(x a) 2
x + a2 y = a (x 1)2 ex
dx dx
determine el valor del parametro de forma que ex sea una solucion de la homogenea
asociada y determine la solucion general de la ecuacion no homogenea para tales
valores del parametro.

23. Use z = sin (x) para resolver la ecuacion

y 00 + tan (x) y 0 + cos2 (x) y = sin (x) cos2 (x)

24. Considere la ecuacion diferencial

x2 y 00 2xy 0 + 2y = 6

con condiciones iniciales y (0) = 3, y 0 (0) = 1 tiene soluciones yc (x) = cx2 + x + 3


para todo c R, este problema no tiene solucion unica, contradice esto el teorema
de existencia y unicidad.?

190
Apuntes Mat023 (Segundo semestre 2014)

25. Resolver
(xD + 1) (D x) (xD) y = x
d
donde D = dx
.

26. Un cuerpo con masa m = 12 kilogramo (kg) esta unido en el extremo de un resorte
estirado 2 metros (m) debido a una fuerza de 100 newtons (N) y es puesto en
movimiento a partir de la posicion inicial x0 = 1 (m) y velocidad inicial v0 = 5 (m/s).
(Notese que estas condiciones iniciales indican que el cuerpo se desplaza a la derecha
y a la izquierda en el tiempo t = 0). Encuentrese la funcion de la posicion del cuerpo,
as como su amplitud, frecuencia, periodo de oscilacion y el tiempo de retardo de su
movimiento.

27. Determine el periodo y la frecuencia del movimiento armonico simple de una masa
de 4 kg unida al extremo de un resorte con constante de 16 N/m.

28. Establezca el periodo y la frecuencia del movimiento armonico simple de un cuerpo


con una masa de 0.75 kg unida al extremo de un resorte con constante de 48 N/m.

29. Un cuerpo con masa de 250 g esta unido al extremo de un resorte estirado 25 cm por
una fuerza de 9 N. En el tiempo t0 el cuerpo es movido 1 m a la derecha, estirando el
resorte y aplicando un movimiento con una velocidad inicial de 5 m/s a la izquierda.
(a) Encuentre x(t). (b) Obtenga la amplitud y el periodo de movimiento del cuerpo.

30. Asuma que la ecuacion diferencial de un pendulo simple de longitud L es L00 +g = 0,


donde g = GM/R2 es la aceleracion gravitacional en el lugar donde este se encuentra
(a una distancia R del centro de la Tierra; M significa la masa de la Tierra). Dos
pendulos de longitudes L1 y L2 ubicados a una distancia R1 y R2 respecto del centro
de la Tierra tienen periodos p1 y p2 . Muestre que

p1 R1 L1
=
p2 R2 L2

191
Captulo 3 : Sistemas de ecuaciones diferenciales

Definiciones

Considere los dos tanques que se ilustran en la figura. Suponga que el tanque A contiene
50 galones de agua en los que hay disueltas 25 libras de sal. Suponga que el tanque B
contiene 50 galones de agua pura. A los tanques entra y sale lquido como se indica en
la figura; se supone que tanto la mezcla intercambiada entre los tanques como el lquido
bombeado hacia fuera del tanque B estan bien mezclados. Se desea construir un modelo
matematico que describa la cantidad de libras x1 (t) y x2 (t) de sal en los tanques A y B
respectivamente en el tiempo t.

se tiene
            
dx1 gal lb gal x2 lb gal x1 lb
= 3 0 +1 4
dt min gal min 50 gal min 50 gal
2 x2
= x1 +
25 50
y
dx2 x1 x2 x2
= 4 3 1
dt 50 50 50
2 2
= x1 x2
25 25
as obtenemos el sistema
dx1 2 1
= x1 + x2
dt 25 50
dx2 2 2
= x1 x2
dt 25 25
192
Apuntes Mat023 (Segundo semestre 2014)

y las condiciones x1 (0) = 25 y x2 (t) = 0


Un sistema de ecuaciones de la forma
dX
= A (t) X (t) + B (t)
dt
esto es
x01 (t)

a11 (t) a12 (t) a1n (t) x1 (t) b1 (t)
x02 (t) a21 (t) a22 (t) a2n (t) x2 (t)
b2 (t)
= +

.. .. .. .. .. .. ..
. . . . . . .
x0n (t) an1 (t) an2 (t) ann (t) xn (t) bn (t)

donde X (t) = (x1 (t) , x2 (t) , . . . , xn (t))T , A (t) = (aij (t)) es una matriz de orden n n
y B (t) = (b1 (t) , b2 (t) , . . . , bn (t))T es llamado sistema de ecuaciones diferenciales
lineal de primer orden (se supone que las funciones aij (t) y bi (t) son continuas en un
intervalo abierto I). Si adicionalmente se debe cumplir X (t0 ) = X0 para t0 I, X0 Rn
entonces tenemos el P.V.I
dX
= A (t) X (t) + B (t)
dt
X (t0 ) = X0

Ejemplo 3.1.1. Son sistemas de ecuaciones lineales:


dx
dt
= x + 2y + et
1.
dy
dt
= x + 2y + cos t
dx
dt
= et x + et y + t
2.
dy
dt
= x + (sin t) y
dx
dt
= x+y+z

dy
3. dt
= x + 2y + 3z

dz
dt
= 2x + 3y + z

mientras que el sistema


dx
= x2 + e t y 2
dt
dy p
= cos t + x2 + y 2
dt
no es lineal.

193
Apuntes Mat023 (Segundo semestre 2014)

Teorema 3.1.1. Si las funciones aij (t) y bi (t) son continuas en el abierto I R, t0 I,
X0 Rn entonces el problema de valores iniciales
dX
= A (t) X (t) + B (t)
dt
X (t0 ) = X0

tiene solucion unica.

Ejemplo 3.1.2. Resolver el sistema


dx1
= x2
dt
dx2
= x1
dt
con x1 (0) = 1, x2 (0) = 0.

Solucion. Note que derivando la primera ecuacion respecto a t obtenemos

d 2 x1 dx2
2
= = x1
dt dt
de donde x1 debe cumplir
d 2 x1
+ x1 = 0
dt2
as x1 (t) = c1 cos t + c2 sin t para algunas constantes c1 y c2 ahora bien, x1 (0) = 1 = c1 se
sigue
x1 (t) = cos t + c2 sin t
ademas
dx1
x2 (t) = = sin t + c2 cos t
dt
luego
0 = x2 (0) = c2
se sigue

x1 (t) = cos t
x2 (t) = sin t

194
Apuntes Mat023 (Segundo semestre 2014)

Observacion 3.1.1. Toda ecuacion diferencial ordinaria lineal de orden n puede ser
transformada en un sistema de ecuaciones lineales, en efecto si tenemos
y (n) + an1 (t) y (n1) + + a1 (t) y 0 + a0 (t) y = h (t)
poniendo
x1 (t) = y (t)
x2 (t) = y 0 (t)
..
.
xn (t) = y (n1) (t)
entonces

0 1 0 0 0

x1 x1 0
x2
0 0 1 0 0
x2 0
d .. .. .. .. .. .. .. ..

..

= . . . . . . +

. . .
dt

...

xn1
0 0 0 0 1 xn1
0
xn a0 a1 an1 xn h (t)
Ejemplo 3.1.3. Considere la ecuacion
d2 x dx
+ (cos t) + 2x = et
dt2 dt
esta se transforma en el sistema
      
d x1 0 1 x1 0
= +
dt x2 2 cos t x2 et
donde x1 (t) = x (t) y x2 (t) = x0 (t)

La teora de los sistemas lineales es similar a la que hemos visto hasta ahora en el caso
de una ecuacion. La ecuacion Homogenea asociada a
dX
= A (t) X (t) + B (t)
dt
es
dX
= A (t) X (t)
dt
su conjunto solucion
 
1 dX
n
S= X (t) C (I, R ) : = A (t) X (t)
dt
es un espacio vectorial de dimension n (si el sistema es de orden n) y la solucion de la no
homogenea sera una solucion particular mas una combinacion lineal arbitraria de elementos
de la base de S.

195
Apuntes Mat023 (Segundo semestre 2014)

Ecuacion con coeficientes constantes

Suponga que trabajamos con el sistema


dX
= AX
dt
donde A Mnn (R) (coeficientes constantes).

Teorema 3.2.1. Si es un valor propio de A con vector propio v entonces

x (t) = vet
dX
es solucion de dt
= AX.

Matriz A diagonalizable

Si A es una matriz diagonalizable entonces

P 1 AP = D

donde D es una matriz diagonal, la ecuacion


dX
= AX
dt
la podemos ver como
dX
= P DP 1 X
dt
multiplicando la ecuacion por P 1 por la izquierda
d
P 1 X = DP 1 X

dt
cambiamos la variable
Y = P 1 X
nos queda
dY
= DY
dt
que es un sistema de la forma
dyi
= i yi para i = 1, 2, . . . , n
dt

196
Apuntes Mat023 (Segundo semestre 2014)

tiene soluciones
yi (t) = ci ei t
luego la solucion es
n
X
X (t) = P Y (t) = ci vi ei t
i=1

donde vi son los vectores propios asociados a los valores propios correspondientes.

Observacion 3.2.1. Si el valor propio i es complejo vi ei t y vi ei t apareceran como


 
soluciones, al buscar soluciones reales se tiene que Re vi ei t y Im vi ei t son soluciones
l.i. reales correspondientes a combinaciones lineales de las dos complejas luego generan lo
mismo.

Ejemplo 3.2.1. Resolver

1 1 1

x1 x1
d
x2 = 1 1 1 x2
dt
x3 1 1 1 x3

Solucion. Primero buscamos el polinomio caracterstico de la matriz

1 1 1

A = 1 1 1
1 1 1

esta dado por



1 1 1

PA () = |A I| =
1 1 1 = ( 2) ( + 2) ( + 1)

1 1 1

luego tiene 3 valores propios distintos y es una matriz de orden 3 luego es diagonalizable,
ahora buscamos los vectores propios:
* 1 + * 1 + * 1 +
2
1 1, 1 2, 1 2
2
1 0 1

as la solucion del sistema es


c e c2 e2t c1 et
1 1 2t
1 1

2 2 3
X (t) = c1 1 et + c2 1 e2t + c3 12 e2t = c2 e2t c1 et + 21 c3 e2t
1 0 1 c1 et + c3 e2t

197
Apuntes Mat023 (Segundo semestre 2014)

Ejemplo 3.2.2. Resolver



x1 5 2 2 x1
d
x2 = 2 2 4 x2
dt
x3 2 4 2 x3
Si
5 2 2
A = 2 2 4
2 4 2
entonces es diagonalizable por ser una matriz simetrica

5 2 2
2

|A I| = 2 2 4 = ( + 3) ( 6)
2 4 2
luego hay 1 valor propio de multiplicidad 2 buscamos los vectores propios
* 1 + * 2 2 +
2
1 3, 1 , 0 6
1 0 1
se sigue que la solucion del sistema es
2c2 e6t 21 c1 e3t + 2c3 e6t
1
2

2 2
X (t) = c1 1 e3t + c2 1 e6t + c3 0 e6t = c1 e3t + c2 e6t
3t 6t
1 0 1 c1 e + c3 e

Ejemplo 3.2.3. Resolver



x1 0 2 0 x1
d
x2 = 2 0 0 x2
dt
x3 0 0 1 x3
Solucion. Si
0 2 0
A = 2 0 0
0 0 1
entonces la ecuacion caracterstica es ( 1) (2 + 4) = 0 de donde tenemos que la matriz
es diagonalizable, 3 valores propios distintos aunque hay dos complejos, veamos los espacios
propios asociados
* 0 + * i + * i +
0 1, 1 2i, 1 2i
1 0 0

198
Apuntes Mat023 (Segundo semestre 2014)

el resultado nos dice que


i

i
1 e2it y 1 e2it
0 0
son soluciones pero buscamos soluciones reales y para ello

i i
Re 1 e2it y Im 1 e2it
0 0

generan lo mismo que las dos anteriores



i i
2it
1 e = 1 (cos (2t) i sen (2t))
0 0

i cos (2t) + sen (2t)
= cos (2t) i sen (2t)
0

sen (2t) cos (2t)
= cos (2t) + i sen (2t)
0 0

se sigue

i sen (2t)
Re 1 e2it = cos (2t)
0 0
y

i cos (2t)
Im 1 e2it = sen (2t)
0 0
luego la solucion es

0 sin (2t) cos (2t) c3 cos 2t + c2 sin 2t
X (t) = c1 0 et + c2 cos (2t) + c3 sin (2t) = c2 cos 2t c3 sin 2t
1 0 0 c1 et

Si la matriz no es diagonalizable no hemos desarrollado una teora para poder resolverla


(mas adelante resolveremos estos sistemas utilizando la transformada de Laplace) sin
embargo podemos enfrentar directamente el sistema a traves de eliminacion, considere el
siguiente ejemplo:

199
Apuntes Mat023 (Segundo semestre 2014)

Ejemplo 3.2.4. Resolver el sistema


    
d x 2 1 x
=
dt y 1 4 y

Solucion. En este caso el polinomio es



2 1 = 2 + 6 + 9 = ( + 3)2

|A I| =
1 4

se sigue que hay un solo valor propio de multiplicidad algebraica 2, Si calculamos vemos
que la multiplicidad geometrica (dimension del espacio propio asociado) es 1 luego la matriz
no es diagonalizable  
1
W=3 =
1
Lo resolveremos directamente

x0 = 2x y
y 0 = x 4y

si derivamos la primera ecuacion obtenemos

x00 = 2x0 y 0

pero de la segunda ecuacion sabemos

y 0 = x 4y

reemplazando

x00 = 2x0 (x 4y)


= 4y x 2x0

pero de la primera ecuacion


y = x0 2x
as

x00 = 4 (x0 2x) x 2x0


= 9x 6x0

as
x00 + 6x0 + 9x = 0 (D + 3)2 x = 0
luego
x (t) = c1 e3t + c2 te3t

200
Apuntes Mat023 (Segundo semestre 2014)

ademas

y (t) = x0 2x
d
c1 e3t + c2 te3t 2 c1 e3t + c2 te3t
 
=
dt
= e3t (c1 c2 + tc2 )
= c1 e3t + c2 (1 + t) e3t

luego

c1 e3t + c2 te3t
   
x (t)
X (t) = =
y (t) c1 e3t + c2 (1 + t) e3t
   
1 3t t
= c1 e + c2 e3t
1 1 + t

es la solucion. Note que la estructura de la solucion es similar a las ecuaciones con raz
repetida.

Variacion de parametros en sistemas

Suponga que queremos determinar la solucion particular de la ecuacion


dX
= AX + B (t)
dt
donde la solucion de la homogenea es

Xh (t) = c1 X1 (t) + c2 X2 (t) + + cn Xn (t)

entonces proponemos una solucion particular de la forma


n
X
Xp (t) = ci (t) Xi (t)
i=1

se sigue que
n
X n
X
Xp0 (t) = c0i (t) Xi (t) + ci (t) Xi0 (t)
i=1 i=1
Xn Xn
= c0i (t) Xi (t) + ci (t) AXi (t)
i=1 i=1
n
X
= c0i (t) Xi (t) + AXp (t)
i=1

201
Apuntes Mat023 (Segundo semestre 2014)

entonces n
X
c0i (t) Xi (t) = B (t)
i=1

se sigue

X1 (t) X2 (t) Xi1 (t) B (t) Xi+1 (t) Xn (t)
c0i (t) =
X1 (t) X2 (t) Xi1 (t) Xi (t) Xi+1 (t) Xn (t)

es un cociente de determinantes por la regla de Cramer.

Ejemplo 3.3.1. Determine la solucion general del sistema


 dx       2t 
dt
1 3 x e
dy = +
dt
3 1 y e4t

Solucion. Primero resolvemos el sistema homogeneo, la matriz


 
1 3
3 1

tiene valores propios 2, 4 y


 
1
W=2 =
1
 
1
W=4 =
1

se sigue que la solucion general del sistema homogeneo es


   
1 2t 1
Xh (t) = e + e4t para , R
1 1

para determinar la solkucion particular usamos variacion de parametros, existe una solucion
de la forma    
1 2t 1
Xp (t) = C1 (t) e + C2 (t) e4t
1 1
donde
2t
e e4t

e4t e4t e6t + e8t
C10 (t) = 2t =
2e2t

e
e4t
e2t e4t
1 4t 1 6t
= e + e
2 2
202
Apuntes Mat023 (Segundo semestre 2014)

se sigue Z  
1 4t 1 6t 1 1
C1 (t) = e + e dt = e4t + e6t
2 2 8 12
por otro lado
2t 2t
e e

e2t e4t e2t 1
C20 (t) = 2t =
e e4t 2e2t

e2t e4t
1 1 2t
= e
2 2
se sigue
Z 
1 1 2t
C2 (t) = e dt
2 2
1 1
= t + e2t
2 4
as
   
1 2t 1
Xp (t) = C1 (t) e + C2 (t) e4t
1 1
     
1 4t 1 6t 1 2t 1 1 2t 1
= e + e e + t+ e e4t
8 12 1 2 4 1
 1 2t 1 4t
  1 4t 1 2t
 
8
e + 12
e 2
te + e
4 
= 1 2t 1 4t + 1 4t 1 2t
8
e + 12
e 2
te + 4
e
 1 2t 1 4t 1 4t 1 2t

8
e + 12 e 2 te 4 e
= 1 2t 1 4t
8
e + 12 e + 12 te4t + 41 e2t
 1 2t 1 2t 1 4t 1 4t

8
e 4
e + 12
e 2
te
= 1 2t 1 2t 1 4t 1 4t
8
e + 4
e + 12
e + 2
te
la solucion general es
81 e2t + 1
12 t e4t
      
1 2t 1 4t 12
XG = e + e + 3 2t 1
+ 2t e4t

1 1 8
e + 12
para , R

Ejemplo 3.3.2. Resolver el sistema


dx
1 4 0

dt
x 1
dy = 4 1 0 y + 0
dt
dz
dt
0 0 2 z t

203
Apuntes Mat023 (Segundo semestre 2014)

Solucion. Este sistema tiene la forma


dX
= AX + B (t)
dt
donde
1 4 0

1
B (t) = 0 y A = 4 1 0
t 0 0 2
primero resolvemos el sistema homogeneo
dX
= AX
dt
es decir
dx
1 4 0

dt
x
dy = 4 1 0 y
dt
dz
dt
0 0 2 z
buscamos los valores y vectores propios asociados, en este caso son:

0 i i
0 2, 1 1 4i, 1 1 + 4i

1 0 0

se sigue que las solucion general del sistema homogeneo es de la forma

i i

0
Xh (t) = c1 0 e2t + c2 Re 1 e(14i)t + c3 Im 1 e(14i)t
1 0 0

donde
i i

1 e(14i)t = 1 et e4ti
0 0
i

= 1 et (cos 4t i sin 4t)


0
iet cos 4t et sin 4t

= et (cos 4t i sin 4t)


0
sin 4t cos 4t

= et cos 4t + iet sin 4t


0 0

204
Apuntes Mat023 (Segundo semestre 2014)

entonces
sin 4t cos 4t

0
Xh (t) = c1 0 e2t + c2 et cos 4t + c3 et sin 4t
1 0 0
ahora buscamos una solucion particular de la ecuacion no homogenea
dX
= AX + B (t)
dt
de la forma
Xp (t) = c1 (t) X1 (t) + c2 (t) X2 (t) + c3 (t) X3 (t)
(mediante variacion de parametros) donde
et sin 4t et cos 4t

0
X1 (t) = 0 , X2 (t) = et cos 4t y X3 (t) = et sin 4t
e2t 0 0
se sigue las funciones c1 , c2 y c3 deben cumplir
0
0 et sin 4t et cos 4t

c1 (t) 1
t t 0
0 e cos 4t e sin 4t c2 (t) = 0
e2t 0 0 c03 (t) t
sistema que podemos resolver por ejemplo mediante determinantes:
1 et sin 4t et cos 4t


0 et cos 4t et sin 4t

te2t
t 0 0
0
c1 (t) = = = te2t
0 et sin 4t et cos 4t e4t


0 et cos 4t et sin 4t

e2t 0 0
:
0 1 et cos 4t


0 0 et sin 4t

e2t t 0 e3t sin 4t

c02 (t) = = = et sin 4t
0 et sin 4t et cos 4t e4t


0 et cos 4t et sin 4t

e2t 0 0
y
0 et sin 4t 1


0 et cos 4t 0

e2t 0 t e3t (cos 4t)

c03 (t) = = = (cos 4t) et
0 et sin 4t et cos 4t e4t


0 et cos 4t et sin 4t

e2t 0 0

205
Apuntes Mat023 (Segundo semestre 2014)

integramos : Z
1
c1 (t) = te2t dt = e2t (2t + 1)
4
Z
1
c2 (t) = et sin 4tdt = et (4 cos 4t + sin 4t)
17
Z
1
c3 (t) = (cos 4t) et )dt = et (cos 4t 4 sin 4t)
17
entonces la solucion particular es

 et sin 4t

 0
 
1 1 t
Xp (t) = e2t (2t + 1) 0 + e (4 cos 4t + sin 4t) et cos 4t
4 17
e2t 0
et cos 4t

1 t
+ e (cos 4t 4 sin 4t) et sin 4t
17
0
1
17

4
= 17

12 t 14

verifiquemos que es correcto



0
Xp0 (t) = 0
21
1
1 4 17

0 1
4
= 4 1 0 17
+ 0
0 0 2 12 t 1
4
t
1

1
= 0 + 0
t 21 t

se sigue que la solucion del sistema es

X (t) = Xh (t) + Xp (t)


1
sin 4t cos 4t 17

0
= c1 0 e2t + c2 et cos 4t + c3 et sin 4t + 4
17

1 0 0 21 t 1
4

206
Apuntes Mat023 (Segundo semestre 2014)

Analisis cualitativo de sistemas

Vamos a analizar el comportamiento de los sistemas de orden 2 de la forma


dx
= ax + by
dt
dy
= cx + dy
dt
donde a, b, c y d son constantes, basandonos en los valores propios de la matriz asociada al
sistema  
a b
A=
c d
la cual supondremos diagonalizable y adicionalmente que det (A) = ad bc 6= 0 de esta
forma la unica solucion constante del sistema corresponde al origen, esta solucion es llamada
solucion de equilibrio, las demas soluciones seran entonces curvas parametricas (x (t) , y (t))
y segun su comportamiento clasificaremos la solucion de equilibrio.

Valores propios reales y distintos (no nulos)

1. Punto silla: En el caso en que los valores propios son reales y distintos con signos
contrarios, el origen se dice que es un punto de silla. Considere el sistema
    
d x 1 1 x
=
dt y 3 1 y

En este caso la matriz de coeficientes es


 
1 1
A=
3 1
la cual tiene determinante distinto de cero. Buscamos sus valores propios y vectores
propios asociados, en este caso son:
   1 
1 3
2, 2
1 1
luego las soluciones del sistema son de la forma
 1   
1
X (t) = c1 3 e + c22t
e2t
1 1
Note que si c1 = 1 y c2 = 0 obtenemos una solucion recta
 1 
X1 (t) = 3 e2t
1

207
Apuntes Mat023 (Segundo semestre 2014)

en esta solucion x (t) = 13 e2t y y (t) = e2t se sigue que los puntos de esta curva estan
sobre la recta
y e2t
= 1 2t = 3
x 3
e

 cuando t + por el primer cuadrante. De


es decir y = 3x y se alejan delorigen
1
manera similar, X1 (t) = 3 e2t es la curva parametrica que esta sobre la
1
recta y = 3x y si t + la solucion se aleja del origen por el tercer cuadrante.
Existe otra solucion recta, al poner c1 = 0 y c2 = 1 se obtiene
 
1
X2 (t) = e2t
1

es decir la curva parametrica

x = e2t
y = e2t

esta curva esta sobre la recta


y
= 1 es decir y = x
x
note que cuando t + los puntos se acercan al origen sobre tal recta por el
segundo cuadrante. Note que X2 (t) tambien es solucion, en este caso

x = e2t
y = e2t

esta curva tambien esta sobre la recta y = x los puntos se acercan al origen pero
por el cuarto cuadrante. Las solucion general es
 1   
1
X (t) = c1 3 2t
e + c2 e2t
1 1
 
1
note que si t + entonces c2 e2t 0 y la solucion se comporta como
1
 1 
c1 3 e2t esto es, cuando el tiempo avanza la curva solucion se acerca a la recta
1
y = 3x (el cuadrante depende de c1 ) y si analizamos el comportamiento cuando
 
1
t las curvas soluciones tienden a c2 e2t esto se interpreta como que
1
las soluciones parten cercana a esa recta y se aproximan a y = 3x. Recuerde que

208
Apuntes Mat023 (Segundo semestre 2014)

por teorema de existencia y unicidad dos soluciones distintas no se pueden cortar.


Grafiquemos algunas soluciones y el campo de direcciones asociado a este sistema

2. Sumidero: Si los valores propios son reales y distintos, ambos negativos, el origen
es llamado sumidero. Considere el sistema
   6 2  
d x 5 5 x
= 2 9
dt y 5
5 y

en este caso la matriz de coeficientes es


 6 2

5 5
A= 2
5
95

que tiene vectores y valores propios asociados


   1 
2 2
1, 2
1 1

luego las soluciones son de la forma


   1 
2 t 2
X (t) = c1 e + c2 e2t
1 1

209
Apuntes Mat023 (Segundo semestre 2014)

grafiquemos el campo de direcciones y diagrama de fase asociados. Note que en este


caso tenemos dos soluciones rectas y que las soluciones tienden al origen si t +

3. Fuente: Si los valores propios son distintos pero ambos positivos entonces el origen
es llamado fuente (las soluciones se alejan del origen) Considere el sistema
    
d x 0 6 x
=
dt y 1 5 y

en este caso la matriz de coeficientes es


 
0 6
A=
1 5

que tiene vectores y valores propios


   
3 2
2, 3
1 1

luego las soluciones son de la forma


   
3 2t 2
X (t) = c1 e + c2 e3t
1 1

210
Apuntes Mat023 (Segundo semestre 2014)

note que si t entonces kX (t)k se sigue que las soluciones se alejan del
origen y tenemos dos soluciones rectas.

Valores propios complejos

1. Sumidero espiral: En el caso en que los valores propios son complejos y la parte
real del valor propio es negativa, el origen es un sumidero espiral. Considere el sistema
    
d x 1 2 x
=
dt y 4 3 y

en este caso la matriz asociada es


 
1 2
A=
4 3

tiene por vectores y valores propios


 1 1   1 1 
2 + 2i 2 2i
1 2i, 1 + 2i
1 1

se sigue que la solucion del sistema esta generada por


 1 1    1 1  
2 + 2i (12i)t 2 + 2i (12i)t
Re e e Im e
1 1

211
Apuntes Mat023 (Segundo semestre 2014)

entonces
1
sin 2t 12 cos 2t 1
sin 2t + 21 cos 2t
   
t 2 t 2
X (t) = c1 e + c2 e
cos 2t sin 2t

ahora el campo de vectores y algunas curvas (note que en este caso no hay soluciones
rectas) las soluciones se acercan al origen

2. Fuente espiral: En el caso en que los valores propios son complejos con parte real
positiva las soluciones se alejan del origen y este es llamado fuente espiral. Considere
el sistema     
d x 4 3 x
=
dt y 6 2 y
en este caso la matriz asociada es
 
4 3
A=
6 2

los valores y vectores propios son


 1 1   1 1 
2 + 2i 2 2i
1 3i, 1 + 3i
1 1

212
Apuntes Mat023 (Segundo semestre 2014)

luego la solucion es generada por


 1 1    1 1  
2 + 2i (13i)t 2 + 2i (13i)t
Re e e Im e
1 1

es decir las soluciones son de la forma


 1
sin 3t 12 cos 3t 1
sin 3t + 12 cos 3t
  
t 2 t 2
X (t) = c1 e + c2 e
cos 3t sin 3t

grafiquemos el campo de direcciones y algunas soluciones el comportamiento es el


mismo pero las soluciones se alejan del origen. (no tenemos soluciones rectas)

3. Centro: Cuando los valores propios son complejos con parte real igual a cero el
origen es llamado centro, en este caso las soluciones no se acercan al origen sino que
se mantienen alrededor de el, el origen es estable en este caso pero no asintoticamente
estable. Considere el sistema
    
d x 3 3 x
=
dt y 6 3 y

en este caso la matriz asociada es


 
3 3
A=
6 3

213
Apuntes Mat023 (Segundo semestre 2014)

que tiene por valores y vectores propios


 1 1   1 1 
2 + 2i 2 2i
3i, 3i
1 1
luego la solucion del sistema es generada por
 1 1    1 1  
2 + 2i (3i)t 2 + 2i (3i)t
Re e e Im e
1 1
es decir las soluciones son de la forma
 1 1 1
sin 3t + 21 cos 3t
  
sin 3t cos 3t
X (t) = c1 2 2 + c2 2
cos 3t sin 3t
grafiquemos el campo de vectores y algunas curvas solucion.

Valores propios repetidos (no nulos)

1. Fuente: Cuando hay solo un valor propio y es positivo el origen es llamado fuente,
las soluciones se alejan del origen. Considere el sistema
    
d x 2 0 x
=
dt y 0 2 y

214
Apuntes Mat023 (Segundo semestre 2014)

en este caso la matriz asociada es


 
2 0
A=
0 2

y el valor propio es = 2 se ve que las soluciones son de la forma


   
1 2t 0
X (t) = c1 e + c2 e2t
0 1

es decir
c1 e2t
 
X (t) =
c2 e2t
las soluciones son todas rectas que se alejan del origen.

2. Sumidero: Si hay solo un valor propio y es negativo las curvas se acercan al origen
y este es llamado sumidero. Considere el sistema
    
d x 2 0 x
=
dt y 0 2 y

en este caso la matriz asociada es


 
2 0
A=
0 2

los valores propios son iguales a 3 pero la matriz no es diagonalizable, las soluciones
son de la forma
c1 e2t
 
X (t) =
c2 e2t
son rectas que se acercan al origen.

Teorema 3.4.1. Para un sistema lineal X0 = AX en el cual det (A) =


6 0, sea X = X (t)
la solucion que satisface X (0) = X0 con X0 6= 0.

1. lmt+ X (t) = 0 si y solo si los valores propios de A tienen partes reales negativas.

2. X (t) es periodica si y solo si las valores propios de A son imaginarios puros.

Ejemplo 3.4.1. Sea R {1, 2}. Considere el sistema de ecuaciones


    
d x 2 x
=
dt y 1 y

215
Apuntes Mat023 (Segundo semestre 2014)

1. Clasificar la solucion de equilibrio (silla, atractor, repulsor, etc.) para los distintos
valores de .
Primero notemos que


2
= 22 3 + 2 > 0
1

para todo R, luego la unica solucion de equilibrio es el origen. Para clasificar


calculamos los valores propios

2 2 + 22 3 + 2


luego
p
42 4 (22 3 + 2)
2
=
p 2
2 4 4 (22 3 + 2)
2
=
p 2
= ( 1) ( 2)

a) Si ( 1) ( 2) > 0 es decir ]1, 2[ tenemos soluciones reales y notemos


que el producto de las dos es 22 3 + 2 > 0 luego las dos son positivas o
negativas,
p
( 1) ( 2) > 0
(para > 0)
2 > ( 1) ( 2)

2
+ ( 1) ( 2) > 0

lo que tiene solucion todo R+ pero por la restriccion se sigue ]1, 2[ . As,
para ]1, 2[ tenemos dos valores propios reales y positivos, el punto es un
repulsor.
b) Si ( 1) ( 2) < 0 esto es ], 1[ ]2, +[ tenemos races complejas
conjugadas. Para = 0 tenemos un centro, para < 0 tenemos un atractor
espiral y para ]0, 1[ ]2, +[ tenemos un repulsor espiral.
3
2. Para = 2
determine la solucion general del sistema y bosquejar el diagrama de
fases.
3
si = 2
entonces
3 1
=
2 2
216
Apuntes Mat023 (Segundo semestre 2014)

es decir, los valores propios son 2, 1 buscamos los espacios propios


 3 3
  3 1

2 2
2 2
2
=
1 23 3
2
1
2
3
2

se sigue  


1
W=1 =
1
y  
1
W=2 =
1
la solucion general es
     
x (t) 1 t 1
= C1 e + C2 e2t
y (t) 1 1

el diagrama de fases es

Ejemplo 3.4.2. Considere el sistema de ecuaciones diferenciales lineales


 0    
x a a x
0 =
y 1 a y

1. Bosquejar los diagramas de fases para todo a R {0, 1}


Buscamos los valores propios, determinamos si son reales y complejos para clasificar.
El polinomio caracterstico es

2 + 2a + a2 a


luego como a2 a = a (a 1) 6= 0 se sigue que los valores propios son no nulos


y la unica solucion de equilibrio es el cero. El discriminante de esta cuadratica es

217
Apuntes Mat023 (Segundo semestre 2014)

4a2 4 (a2 a) = 4a se sigue que para a < 0 las races son complejas y para a > 0
las races son reales.
Supongamos a < 0 entonces las races son

2a 4a
= a a
2
= a i a

complejas con parte real positiva es una fuente espiral.


Para a > 0 las races son reales, su producto es a (a 1) el cual es positivo si a > 1
y negativo si 0 < a < 1. Este signo nos dice que para 0 < a < 1 los valores propios
tienen distinto signo (es una silla) y para a > 1 tienen igual signo veamos si es
positivo o negativo

a + a y a a
son negativas por la segunda, se sigue que para a > 1 es un sumidero.
Los graficos son los siguientes: Para a < 0

218
Apuntes Mat023 (Segundo semestre 2014)

para 0 < a < 1

para a > 1

2. Resolver el sistema no homogeneo


 0      t 
x 4 4 x e
0 = +
y 1 4 y e2t

Resolvemos el sistema homogeneo, la matriz


 
4 4
1 4

219
Apuntes Mat023 (Segundo semestre 2014)

   
2 2
tiene valores propios 2 y 6 y vectores propios asociados y respec-
1 1
tivamente, se sigue que la solucion del sistema homogeneo es
   
2 2t 2
X (t) = e + e6t
1 1
la solucion particular la determinamos usando variacion de parametros
   
2 2t 2
Xp (t) = C1 (t) e + C2 (t) e6t
1 1
donde
e 2e6t
t


e2t e6t 2e4t + e5t
C10 (t) = 2t =
2e 2e6t 4e8t

e2t e6t
1 4t 1 3t
= e + e
2 4
as
Z  
1 4t 1 3t
C1 (t) = e + e dt
2 4
1 3t 1 4t
= e + e
12 8
de manera similar
2t
2e
et

e2t e2t 1 1
C20 (t) = 2t = e8t e7t
2e
2e6t

2 4
e2t e6t

luego
Z  
1 8t 1 7t
C2 (t) = e e dt
2 4
1 8t 1
= e e7t
16 28
se sigue
     
1 t 1 2t 2 1 2t 1 2
Xp (t) = e + e + e et
12 8 1 16 28 1
5 t
e + 81 e2t
 
= 21
1 t 3 2t
21
e + 16 e

220
Apuntes Mat023 (Segundo semestre 2014)

la solucion general es
5 t
e + 18 e2t
     
2 2t 2 6t 21
XG (t) = e + e + 1 t 3 2t
1 1 21
e + 16 e
con , R.

Ejemplo 3.4.3. Resolver el sistema de ecuaciones

x0 = 3x + 25y
y 0 = x 3y

y bosquejar el diagrama de fases.

Solucion. El sistema se puede escribir matricialmente como


 0    
x 3 25 x
0 =
y 1 3 y
buscamos los valores propios y vectores propios. La ecuacion caracterstica es

3 25
P () =
1 3
= 2 + 6 + 34
= 0

que tiene soluciones 1 = 3 + 5i y 2 = 3 5i.


       
a 2 3 (3 + 5i) 25 a 0
W=3+5i = R : =
b 1 3 (3 + 5i) b 0
       
a 5i 25 a 0
= R2 : =
b 1 5i b 0
  
a
= R2 : a + 5ib = 0
b
 
5i
=
1
as las soluciones base son
   
5i (3+5i)t 5i
e y e(35i)t
1 1
para las soluciones reales calculamos
     
5i (3+5i)t 5i (3+5i)t
Re e e Im e
1 1

221
Apuntes Mat023 (Segundo semestre 2014)

donde
   
5i (3+5i)t (5i) (cos (5t) + i sin (5t))
3t
e = e
1 (cos (5t) + i sin (5t))
 
3t 5 sin 5t 5i cos 5t
= e
cos 5t + i sin 5t
   
5 sin 5t 3t 5 cos 5t
= e +i e3t
cos 5t sin 5t

as la solucion general es
   
5 sin 5t 3t 5 cos 5t
X= e + e3t
cos 5t sin 5t

para , R. El diagrama de fases es

Ejemplo 3.4.4. Considere el sistema de ecuaciones


d
X = AX + B
dt
     t 
x (t) 2 4 e
donde X = ,A= yB=
y (t) 1 3 e2t

222
Apuntes Mat023 (Segundo semestre 2014)

d
1. Resolver el sistema homogeneo dt X = AX, bosquejar el diagrama de fases y clasificar
   
x (0) 3
la solucion de equilibrio. Si = determine la ecuacion de la recta a
y (0) 2
la cual tiende la curva solucion cuando t +.
Buscamos los valores propios

2 4
= 2 + 2
1 3
= ( + 2) ( 1)

luego los valores propios son = 2 y = 1. Como los valores propios son reales
con distinto signo, la solucion de equilibrio corresponde a un punto silla. Buscamos
los vectores propios
       
x 4 4 x 0
W=2 = : =
y 1 1 y 0
 
1
1
y
       
x 1 4 x 0
W=1 = : =
y 1 4 y 0
 
4
1

se sigue que la solucion del sistema homogeneo es


     
x (t) 1 2t 4
= c1 e + c2 et
y (t) 1 1

Note que independiente del punto por donde pase las soluciones se aproximan a la
recta    
x 4
= c2 et
y 1
 
4
(a menos que sea una solucion de la forma c2 et ) se sigue que la recta es
1

x = 4c2 et
y = e t c2

as
y 1
=
x 4
223
Apuntes Mat023 (Segundo semestre 2014)

esto es
x
y=
4

d
2. Resolver el sistema no homogeneo dt
X = AX + B
d
Dado que tenemos la solucion de dt X = AX, necesitamos la solucion particular la
cual tiene la forma
   
1 2t 4
Xp (t) = c1 (t) e + c2 (t) et
1 1

(por variacion de parametros) donde


t
t
e
4e
e2t et 1 3t 4
c01 (t) = = e +
e2t 4et 3 3

e2t et
Z  
1 3t 4
c1 (t) = e + dt
3 3
4 1
= t + e3t
3 9

224
Apuntes Mat023 (Segundo semestre 2014)

de manera similar
e2t et


e2t e2t e3t 1
c02 (t) = =
e2t 4et

3 3

e2t et
Z  3t 
e 1 1 1
c2 (t) = dt = e3t t
3 3 9 3
as
     
4 1
1 2t 1 3t 1 4
Xp (t) = t + e3t e + e t et
3 91 9 3 1
34 te2t + 19 et 4 19 e2t 31 tet
     
= 4 2t 1 t
 + 1 2t 1 t

3
te + 9
e 9
e 3
te
4 t 4 2t 4 2t 1 t
3
te 9 e 3 te 9 e
=
1 t 1 2t 4 2t 1 t
9
e + 9 e + 3 te 3 te
as la solucion es
4 t
94 e2t 43 te2t 91 et

      3
te
x (t) 1 4
= c1 e2t + c2 et +
y (t) 1 1 1 t
9
e + 91 e2t + 43 te2t 31 tet

Ejercicios del captulo

1. Escribir los siguientes sistemas en forma matricial


dx dx
a) dt
= 3x 2y b) dt
= x 2ty + t2
dy dy
dt
= 4x + 8y dt
= 4t2 x + 8y + sin t

dx dx
c) dt
= 3x y + 3z d) dt
= et x cos ty + 3tz + cos t
dy dy
dt
= 4x + 3y + z dt
= x + et
dz dz
dt
= 2x + y z dt
= 2x + y z 3t + 1

2. Escribir los siguientes sistemas sin usar matrices


      
d x 2 1 x sin t
a) dt = +
y t 0 y et
1 1 1 1 1 + t

b) X0 = 0 1 2 X+ 0 et + 1 et sin t
1 1 1 1 1

225
Apuntes Mat023 (Segundo semestre 2014)

3. En los siguientes ejercicios verificar si el vector X dado es solucion del sistema


dx
  
1 5t dt
= 3x 4y
a) X = e de dy
2 dt
= 4x 7y
dx
1 dt = x + 2y + z
dy
b) X = 6 de dt
= 6x y
dz
13 dt
= x 2y z

sin t 1 0 1
c) X = 21 sin t 21 cos t de X0 = 1 1 0 X
sin t + cos t 2 0 1

4. Determine si los vectores dados son linealmente independientes


     
1 t 2 t 8
a) X1 = e y X2 = e + tet
1 6 8

1 1 2
b) X1 = 6 , X2 = 2 e4t y X3 = 3 e3t
13 1 2
   
1 t 1
5. Muestre que la funcion Xp = e+ tet es solucion particular del sistema
1 1
   
0 2 1 1
X= X et
1 1 7

6. Demostrar que la solucion general del sistema



0 6 0
X0 = 1 0 1 X
1 1 0

es de la forma

3

6 2
t 2t
X =c1 1 e + c2 1 e + c2 1 e3t
5 1 1

7. Determinar la solucion general de los sistemas:

226
Apuntes Mat023 (Segundo semestre 2014)

dx dx
a) dt
= x + 2y b) dt
= 2x + 2y
dy dy
dt
= 4x + 3y dt
= x + 3y

dx dx
c) dt
= x+yz d) dt
= 2x 7y
dy dy
dt
= 2y dt
= 5x + 10y + 4z
dz dz
dt
= yz dt
= 5y + 2z

dx dx
e) dt
= 6x y f) dt
= 2x + y + 2z
dy dy
dt
= 5x + 2y dt
= 3x + 6z
dz
dt
= 4x 3z
8. Resolver los siguientes sistemas usando variacion de parametros
dx dx
a) dt
= 3x 3y + 4 b) dt
= 2x y
dy dy
dt
= 2x 2y 1 dt
= 3x 2y + 4t

dx dx
c) dt
= x+yz+t d) dt
= y + sec t
dy dy
dt
= 2y + t dt
= x
dz
dt
= yz+t

dx dx
e) dt
= x + 2y + et csc t f) dt
= 3x y z
dy dy
dt
= 21 x + y + et sec t dt
= x+yz+t
dz
dt
= x y + z + 2et
9. Sea X = X (t) la solucion al sistema de ecuaciones
x0 = x y
y 0 = x + y
que satisface la condicion inicial X (0) = X0 . Determine condiciones sobre , que
aseguren  
0
lm X (t) =
t+ 0
10. Determine condiciones sobre tal que (0, 0) sea un centro para el sistema lineal
x0 = x + y
y 0 = x + y

11. Sea A Mnn (R) defina


2 3
At 2t 3t
e = I + At + A +A +
2! 3!
k
kt
X
= A
k=0
k!

227
Apuntes Mat023 (Segundo semestre 2014)

 
At a c
a) Calcular e para A =
0 b
b) Muestre que si A = PDP1 con D =diag(1 , 2 , . . . , n ) diagonal entonces

etA = P diag e1 t , e2 t , . . . , en t P1


c) Muestre que la solucion general de


 
d 4 3
X= X
dt 2 1
es  
tA c1
X =e
c2
 
4 3
donde A =
2 1
d ) Conjeturar una formula para la solucion general de

X0 = AX + B (t)

en terminos de etA .

12. Clasificar la solucion de equilibrio de los siguientes sistemas y bosquejar el diagrama


de fases:
a) dx dt
= x 5y b) dx dt
= 4x + 3y
dy dy
dt
= x + 3y dt
= 2x 3y

dx dx
c) dt
= x + 2y d) dt
= x + 4y
dy dy
dt
= x 4y dt
= x + y

dx dx
e) dt
= 3x + y f) dt
= 2x + y
dy dy
dt
= 2x y dt
= 3x + 6y

228
Captulo 4 : Transformacion integral de Laplace

Definiciones y teoremas fundamentales

Observacion 4.1.1. Las ecuaciones diferenciales describen formas en las cuales ciertas
cantidades cambian respecto al tiempo, cantidades tales como la corriente en un circuito
electrico, las oscilaciones de una membrana, flujos de calor, etc. esas ecuaciones generalmente
estan acompanadas de condiciones iniciales que describen el estado del sistema en el
tiempo t = 0. Una herramienta muy poderosa para resolver este tipo de problemas es la
transformada de Laplace la cual transforma la ecuacion diferencial original en una ecuacion
algebraica la cual puede ser transformada nuevamente en la solucion del problema original.
Esta tecnica es conocida como el metodo de la transformada de Laplace. La clave del
proceso anterior radica en la formula de integracion por partes. Si f, g C 1 recordemos
que la formula de integracion por partes corresponde a:
Z b b Z b
0
f (x) g (x) dx = f (x) g (x) f 0 (x) g (x) dx

a a a

y se puede interpretar del modo siguiente: En un producto de funciones bajo el smbolo de


la integral es posible intercambiar la derivada presente en una funcion. Sin embargo, tal
procedimiento implica al final el tener que evaluar el producto de tales funciones en los
extremos del intervalo y trabajar con una nueva derivada.
La idea, entonces, es trabajar con una funcion que tenga un buen comportamiento de la
derivada y la evaluacion en los extremos del intervalo de integracion, esto se puede lograr
mediante una funcion que no cambie esencialmente al derivarla i.e. una exponencial y que
se pueda anular en los extremos. Lo anterior motiva la siguiente definicion:

Definicion 4.1.1. Una funcion f : [0, +) R se dice localmente integrable si para


RT
cada T > 0, la integral 0 f (t) dt existe. Llamaremos transformacion integral de
Laplace o simplemente la transformada de Laplace a la funcion L [f (t)] : D R R
definida mediante la integral impropia:
Z
s L [f (t)] (s) = f (t) est dt
0

para todos los valores de s R para los cuales la integral sea convergente. El conjunto de
los valores de s R para los cuales la integral converge se llamara dominio de L [f (t)] y se
denotara por D (L [f ]).

Ejemplo 4.1.1. Sea f : R R, t f (t) = 1. Calcular su transformada de Laplace.

229
Apuntes Mat023 (Segundo semestre 2014)

Solucion. Note que:


Z Z T
st
1e dt = est dt
lm
0 T 0
 
1 st T
= lm e
s

T 0
1
=
s
para cada s > 0. Por tanto:
1
L [1] (s) = , s>0
s
Ejemplo 4.1.2. Sea f (t) = eat , con a una constante real. Calcular su transformada de
Laplace

Solucion. Por definicion:


Z Z T
at at st
e(as)t dt
 
L e (s) = dt = lm e e
0 T 0
1 (as)t T
= lm e
T a s

0
1
e(as)T 1

= lm
T a s

1 1

as, si a s < 0, entonces lmT as e(as)T 1 = sa , luego:

1
L eat (s) =
 
Para s > a
sa
Observacion 4.1.2. Con respecto al ejemplo anterior, dos observaciones deben hacerse.
En primer lugar, respecto a lo notacional, en lo sucesivo escribiremos:
T
lm f (t) = f (t)

T 0 0

para evitar anotar cada vez que se calcule una transformada de Laplace, el lmite relacionado
con la integral impropia. En segundo lugar, notamos que D (L [eat ]) = ]a, +[, luego la
transformada no necesariamente estara definida para todos los valores de s R.

Ejemplo 4.1.3. Calcule, si acaso existe, la transformada de Laplace de f (t) = 1t , con


t > 0.

Solucion. Sea t > 0. Luego:


Z 1
est est est
Z Z
dt = dt + dt
0 t 0 t 1 t

230
Apuntes Mat023 (Segundo semestre 2014)

pero si 0 < t 1 y s > 0, entonces es est y la integral:


Z 1 s
e
dt
0 t
diverge. Por tanto, la transformada de Laplace de f (t) = 1/t no existe para ningun valor
de s R.

En vista del ejemplo anterior, trataremos de dar respuesta a la siguiente pregunta:


Para que tipo de funciones podemos asegurar la existencia de la transformada de Laplace?

Definicion 4.1.2. Sean f una funcion real. Anotaremos:

f a+ = lm+ f (t) f a = lm f (t)


 
ta ta

si acaso estos lmites existen. Diremos que una funcion f tiene una discontinuidad de
salto en a si:
f a+ f a <
 

o bien si f (a+ ) = f (a ), pero estos valores son distintos de f (a) (puede ser incluso que
f (a) no este definida). Una funcion f : [0, +) R se dice seccionalmente continua
si para cada T > 0 la funcion f |[0,T ] es continua y tiene a lo sumo un numero finito de
discontinuidades de salto.

Ejemplo 4.1.4. La funcion 


1/t t 6= 0
f (t) =
0 t=0
no es seccionalmente continua en [0, +), lo mismo para

cos (1/t) t > 0
f (t) =
0 t0

mientras que g (t) = btc si es seccionalmente continua.

Definicion 4.1.3. Diremos que una funcion f : [0, +) R es de orden exponencial


si existen constantes M > 0, R y un valor t0 0 tales que:

|f (t)| M et

para todo t t0 .

Ejemplo 4.1.5. Las siguientes funciones son seccionalmente continuas y de orden expo-
nencial:

1. f (t) = sinh t, pues:


et et et
sinh t = <
2 2

231
Apuntes Mat023 (Segundo semestre 2014)

2. f (t) = tn . En efecto, sabemos que et = tn tn


< et . Por
P
n=0 n!
. Luego, para t > 0 , n!
tanto:
tn n! et

Teorema 4.1.1 (Existencia). Sea f : [0, +) R una funcion seccionalmente continua y


de orden exponencial entonces la transformada de Laplace L [f (t)] (s) existe para s > .

Demostracion. En primer lugar note que, como f : [0, +) R es seccionalmente


continua, la funcion t 7 f (t) est es localmente integrable en [0, +). Por otro lado, por
la desigualdad triangular tenemos que:
Z Z t0 Z
st st
|f (t)| est dt


f (t) e dt
f (t) e dt +
0 0 t0
R
t0 st
Note que 0 f (t) e dt < y ademas, como f : [0, +) R es de orden exponencial,

existen constantes M, > 0 y un valor t0 > 0 tales que:

|f (t)| M et

para todo t t0 . Luego:


Z
M e(s)
Z Z
st t st
|f (t)| e dt M e e dt = M e(s)t dt =
s t0

t0 t0 t0

en donde el ultimo termino converge si y solo si s < 0. Por tanto, como:


Z Z
st st

f (t) e dt < f (t) e dt
0 0

se tiene que L {f (t)} (s) existe para cada s > .

 
2 2
Observacion 4.1.3. La funcion f (t) = 2tet cos et es continua en [0, +[ pero no de
orden exponencial sin embargo su transformada de Laplace existe para s > 0, por otro
lado la funcion g (t) = 1t no es seccionalmente continua en [0, +[ sin embargo posee
transformada de Laplace.

Teorema 4.1.2 (Linealidad de la transformada). Sean f, g : [0, +) R dos funciones


localmente integrables y R, entonces para cada s D (L [f ]) D (L [g]) se tiene que:

L [f + g] (s) = L [f ] (s) + L [g] (s)

232
Apuntes Mat023 (Segundo semestre 2014)

Calculo de transformadas

Observacion 4.2.1. En esta seccion haremos el calculo de transformadas de Laplace


basicas, para as obtener una tabla elemental de transformadas.

Ejemplo 4.2.1. Transformadas de f (t) = sin t y f (t) = cos t.

Solucion. Integrando por partes obtenemos

cos (x) ex sin (x) ex


Z
ex cos (x) dx = + +C
2 + 2 2 + 2
cos (x) ex sin (x) ex
Z
ex sin (x) dx = + +C
2 + 2 2 + 2
luego
Z
L [sin t] (s) = sin (t) est dt
0
+
cos (t) est s sin (t) est
=
s2 + 2 s2 + 2 0

= 2
s + 2
si s > 0. De manera similar
Z
L [cos t] (s) = cos (t) est dt
0
+
s cos (t) est sin (t) est
= +
s2 + 2 s2 + 2 0
s
= 2
s + 2
para s > 0.

Ejemplo 4.2.2. Usemos variable compleja para calcular las mismas transformadas: Por la
formula de Euler:
eit = cos t + i sin t
y la linealidad de la transformada, se obtiene que:

eit + eit
cos t =
2
y:
eit eit
sin t =
2i

233
Apuntes Mat023 (Segundo semestre 2014)

Entonces:
eit + eit
 
L [cos t] (s) = L (s)
2
1
L eit (s) + L eit (s)
    
=
2 
1 1 1
= +
2 s i s + i
 
1 s + i + s i
=
2 s2 (i)2
s
=
s + 2
2

para s > 0. Analogamente, se obtiene que:



L [sin t] (s) = , s>0
s2 + 2
Ejemplo 4.2.3. Transformadas de f (t) = sinh t y f (t) = cosh t. Sabemos que:
et + et et et
cosh t = sinh t =
2 2
Por la linealidad de la transformada de Laplace se obtiene que:
s
L [cosh t] (s) = 2 , s > ||
s 2
y:

L [sinh t] (s) = 2 , s > ||
s 2
Ejemplo 4.2.4. Transformada de f (t) = t. Notemos que:
Z
1
Z
st
0
te dt = t est dt
0 s 0
1
 Z 
st st
= te e dt
s 0 0
 
1 1
= 0
s s
1
= 2
s
para todo s > 0. Por tanto:
1
L [t] (s) = , s>0
s2
usando integracion por partes, es facil mostrar que
n!
L [tn ] (s) = , s>0
sn+1

234
Apuntes Mat023 (Segundo semestre 2014)

Observacion 4.2.2. Resumimos las funciones con sus respectivas transformadas de Laplace
y sus dominios en la siguiente tabla:

f (t) L [f (t)] (s) Dominio

1
1 s>0
s
1
t s>0
s2
1
et s>
s
s
cos t s>0
s2 + 2

sin t s>0
s + 2
2
s
cosh t s > ||
s 2
2

sinh t s > ||
s2 2
n!
tn n+1
s>0
s

Ejemplo 4.2.5. Calcule la transformada de Laplace de la funcion:

f (t) = 3 2t + 4 cos 2t

Solucion. Por la linealidad de la transformada y la tabla anterior, se obtiene:

L [f (t)] (s) = L [3 2t + 4 cos 2t] (s)


= 3L [1] (s) 2L [t] (s) + 4L [cos 2t] (s)
3 2 4s
= 2+ 2
s s s +4
Ejemplo 4.2.6. si f (t) = a0 + a1 t + + an tn entonces
n
X
ak L tk (s)
 
L [f (t)] (s) =
k=0
n
X k!
= ak
k=0
sk+1

Teorema 4.2.1. Si f es seccionalmente continua en [0, +) y de orden exponencial


entonces
lm L [f (t)] (s) = 0
s+

235
Apuntes Mat023 (Segundo semestre 2014)

El resultado anterior se debe a la desigualdad


Z
st
M
e f (t) dt

0 s
tomando s + se obtiene el resultado.

s
Observacion 4.2.3. Las funciones s1 , e no pueden ser transformadas de una funcion
s+1 s
seccionalmente continua de orden exponencial.

Primer Teorema de la Traslacion

Observacion 4.3.1. El siguiente resultado nos permite calcular la transformada de Laplace


de et f (t) cuando es conocida la transformada de Laplace de f (t), en este resultado se
pone de manifiesto la primera propiedad de la traslacion de la transformada de Laplace.
En efecto, tenemos el siguiente teorema:
Teorema 4.3.1. Suponga que F (s) = L [f (t)] (s) existe para cada s > . Si R,
entonces:
L et f (t) (s) = F (s )
 

para todo s > + .


Ejemplo 4.3.1. Calcule L [e3t cos 2t] (s)
s
Solucion. Notamos que F (s) = L [cos 2t] (s) = s2 +4
. Luego:
s+3
L e3t sin 2t (s) = F (s + 3) =
 
(s + 3)2 + 4
Observacion 4.3.2. La primera propiedad de traslacion de la transformada de Laplace,
deja nuestra tabla basica de transformadas como sigue:
f (t) L [f (t)] (s) Dominio

n!
et tn s>
(s )n+1
s
et cos t s>
(s )2 + 2

et sin t s>
(s )2 + 2
s
et cosh t s > + ||
(s )2 2

et sinh t s > + ||
(s )2 2

236
Apuntes Mat023 (Segundo semestre 2014)

Transformada de la derivada

Observacion 4.4.1. El origen y el objetivo de la transformada de Laplace es la resolucion


de ecuaciones diferenciales. Para lograr lo anterior, debemos calcular la transformada de
Laplace de la derivada de una funcion. Entonces, sea y = y (t) una funcion derivable, note
que:
Z Z
0 st st
0
y (t) e dt = y (t) e y (t) est dt
0 0
Z 0
= y (0) + s y (t) est dt
0

bajo el supuesto que:


lm y (t) est = 0
t

As, si y = y (t) es una funcion derivable y de orden exponencial, se tiene que y 0 (t) existe
para cada t 0 y:
|y (t)| M et , t t0
Entonces:
M e(s)t y (t) est M e(s)t
donde los extremos convergen a cero si s < 0, cuando t . As, bajo las condiciones
anteriores se obtiene que:

L [y 0 (t)] (s) = sL [y (t)] (s) y 0+




Por tanto, tenemos el siguiente teorema:

Teorema 4.4.1. Suponga que f es continua en ]0, [ y de orden exponencial , supongamos


ademas que f 0 es seccionalmente continua en [0, +) entonces:

L [f 0 (t)] (s) = sL [f (t)] (s) f 0+




Corolario 4.4.1. Suponga que f (t) , f 0 (t) , , f (n1) (t) son continuas en ]0, [ y de
orden exponencial, suponga tambien que f (n) (t) es seccionalmente continua en [0, +).
Entonces

L f (n) (t) (s) = sn L [f (t)] (s) sn1 f 0+ sn2 f 0 0+ f (n1) 0+


    

Ejemplo 4.4.1. Calcule la transformada de Laplace de f (t) = tn , para n 2.

Solucion. Note que:


f 0 (t) = ntn1

237
Apuntes Mat023 (Segundo semestre 2014)

as, por la transformada de la derivada, se tiene que:


L ntn1 (s) = L [f 0 (t)] (s) = sL [f (t)] (s) f 0+ = sL [tn ] (s)
  

as:
n  n1 
L [tn ] (s) = L t (s) , n2
s
entonces:
1

n s2
n=1
L [t ] (s) = n n1 s>0
s
L [t ] (s) n > 1
se sigue:
n!
L [tn ] (s) = , s>0
sn+1
Ejemplo 4.4.2. Como aplicacion de la transformada de la derivada, calcular la transfor-
mada de Laplace de sin2 (t)
Solucion. Derivando
f 0 (t) = 2 sin (t) cos (t)
= sin (2t)
luego
sL [f (t)] (s) f (0) = L [ sin (2t)] (s)
as
22
sL [f (t)] (s) =
s2 + 42
finalmente
22
L [f (t)] (s) =
s (s2 + 42 )
Ejemplo 4.4.3. Sea f : [0, +) R una funcion continua. Se define:
Z t
F (t) = f (u) du
0

Calcule L [F (t)] (s).


Solucion. Como f es continua, por el Teorema Fundamental del Calculo, F es derivable
con:
F 0 (t) = f (t)
Entonces, por la transformada de la derivada tenemos que:
L (F 0 (t)) (s) = sL [F (t)] (s) F 0+


As: Z t 
1
L f (t) dt (s) = L [F (t)] (s) = L [f (t)] (s)
0 s
pues F (0) = 0.

238
Apuntes Mat023 (Segundo semestre 2014)

Teorema 4.4.2. Sea n N, suponga que f es seccionalmente continua en [0, +) de


orden exponencial y que F (s) = L [f (t)] (s) entonces:

n dn
n
L [t f (t)] (s) = (1) F (s) (4.1)
dsn
para s > .

Observacion 4.4.2. El siguiente teorema, habla sobre la inyectividad de la transformacion


de Laplace, lo que nos permite hablar de inversa.

Teorema 4.4.3. Dos funciones con la misma transformada de Laplace no pueden diferir
en todo un intervalo de longitud positiva. Es decir, si L [f (t)] (s) = L [g (t)] (s) entonces
f (t) = g (t), para cada t tal que f y g simultaneamente sean continuas.
Observacion 4.4.3. Funciones continuas diferentes tienen transformadas de Laplace
diferentes.
Definicion 4.4.1. Sea F : [0, +) R una funcion tal que lms F (s) = 0. Diremos
que una funcion f : [0, +) R, para la cual existe su transformada de Laplace, es una
transformada de Laplace inversa de F si satisface:

L [f (t)] (s) = F (s)

Por el Teorema de Lerch, tal transformada inversa esta unicamente determinada solo sobre
sus puntos de continuidad. Abusando del lenguaje, se escribe:

f (t) = L1 [F (s)] (t)

Observacion 4.4.4. El operador L1 tambien es lineal. Es decir:

L1 [f + g] = L1 [f ] + L1 [g]

Ejemplo 4.4.4. Calcule L1 s2 +6s+13


 s+9 
(t).
Solucion. Observe que:
s+9 s+9
=
s2 + 6s + 13 (s + 3)2 + 4
s+3 6
= +
(s + 3) + 4 (s + 3)2 + 4
2

Entonces:
     
1 s+9 1 s+3 1 2
L (t) = L (t) + 3L (t)
s2 + 6s + 13 (s + 3)2 + 4 (s + 3)2 + 4
= e3t cos 2t + 3e3t sin 2t

239
Apuntes Mat023 (Segundo semestre 2014)

Ejemplo 4.4.5. Calcular


 
1 s
L (s)
(s + 1) (s + 2) (s + 3)

Solucion. Usando la tecnica de fracciones parciales


s 2 1 3
=
(s + 1) (s + 2) (s + 3) s + 2 2 (s + 1) 2 (s + 3)

luego
 
1 s
L (s)
(s + 1) (s + 2) (s + 3)
 
1 2 1 3
= L (s)
s + 2 2 (s + 1) 2 (s + 3)
1 3
= 2e2t et e3t
2 2
Ejemplo 4.4.6. Calcular   
1 s+1
L ln (s)
s+2
Solucion. Queremos determinar f (t) tal que
 
s+1
L [f (t)] (s) = ln
s+2
notamos que
L [f (t)] (s) = ln (s + 1) ln (s + 2)
derivando
1 1
L [f (t)]0 (s) =
s+1 s+2
pero
L [f (t)]0 (s) = L [tf (t)] (s)
se sigue
L [tf (t)] (s) = L et e2t (s)
 

as
tf (t) = et e2t
de donde obtenemos
e2t et
f (t) =
t
Ahora veremos algunos ejemplos de como funciona el metodo de la transformada de
Laplace para resolver ecuaciones:

240
Apuntes Mat023 (Segundo semestre 2014)

Ejemplo 4.4.7. Resuelva la ecuacion diferencial:

ty 00 ty 0 y = 0, y (0) = 0, y 0 (0) = 3 (4.2)

Solucion. Sea Y (s) = L [ y (t)] (s). Aplicando la transformada de Laplace a la ecuacion


(4.2), obtenemos:
L [ty 00 ] (s) L [ty 0 ] (s) L [y] (s) = 0
pero:
d  2
L [ty 00 ] (s) = s L [y] (s) sy (0) y 0 (0) = s2 Y 0 2sY

ds
y
d
L [ty 0 ] (s) =
{sL [y] (s) y (0)} = sY 0 Y
ds
reemplazando estas expresiones en la ecuacion original obtenemos:

s2 Y 0 2sY + sY 0 + Y Y = 0

rs decir:
s (s 1) Y 0 + 2sY = 0
entonces:
2
Y0+ Y =0
s1
la cual es una ecuacion de variable separable. Separando, entonces, las variables, obtenemos:
dY 2
= ds
Y s1
de donde se sigue que:
C
Y (s) =
(s 1)2
se sigue que
C  t

L [y (t)] (s) = Y (s) = 2 = L Cte (s)
(s 1)
usando el teorema de Lerch, se sigue

y (t) = Ctet

(note que se busca una funcion derivable, luego continua). Ahora bien, como y 0 (0) = 3, se
obtiene que C = 3. Por tanto, la solucion de la ecuacion diferencial es:

y (t) = 3tet

241
Apuntes Mat023 (Segundo semestre 2014)

Ejemplo 4.4.8. Resolver el P.V.I.


ty 00 ty 0 + y = 2 et 1


y (0) = 0
y 0 (0) = 1
Solucion. Aplicando la transformada de Laplace
 
d 2 d 1 1
s Ly sy (0) y 0 (0) +

(sLy y (0)) + Ly = 2
ds ds s1 s
reemplazando las C.I.
d 2  d 2
s Ly + 1 + (sLy ) + Ly =
ds ds s (s 1)
se sigue
2
2sLy + s2 L0y + Ly + sL0y + Ly
 
=
s (s 1)

0 0 2
2sLy + s2 Ly Ly + sLy Ly
 
=
s (s 1)

2
2
L0y sL0y

s + 2sLy 2Ly =
s (s 1)

2
L0y s2 s + 2 (s 1) Ly

=
s (s 1)
se sigue
2
s (s 1) L0y + 2 (s 1) Ly =
s (s 1)

2 2
L0y + Ly =
s s2 (s 1)2
2
R
ds
es una EDO lineal con factor integrante (s) = e s = e2 ln s = s2
d 2  2
s Ly =
ds (s 1)2
integrando
2
Z
2 2
s Ly = 2 ds = +C
(s 1) s1
se sigue
2 C
Ly = 2
+ 2
(s 1) s s
2 2 2 C
= 2+ 2
s1 s s s
242
Apuntes Mat023 (Segundo semestre 2014)

as
y (t) = 2et 2 2t + Ct
como y (0) = 0 e y 0 (0) = 1 se sigue

1 = C

as
y (t) = 2et 2 3t
Ejemplo 4.4.9. Utilizando la transformada de Laplace, resuelva la siguiente ecuacion
diferencial:
y 00 4y = 0, y (0) = 1, y 0 (0) = 2
Solucion. Aplicando la transformada de Laplace

s2 L [y] (s) sy (0) y 0 (0) 4L [y] (s) = 0

reemplazando los valores iniciales

s2 4 L [y] (s) = s + 2


as
s+2 1
= L e2t (s)
 
L [y] (s) = 2
=
s 4 s2
finalmente la solucion de la ecuacion es:

y (t) = e2t

Funciones escalonadas y Segundo Teorema de la Traslacion

Observacion 4.5.1. Cuando se estudio el movimiento vibratorio en el caso mas general,


se obtuvo una ecuacion diferencial de la forma:
c k
x00 + x0 + x = F (t)
m m
para constantes fsicas c, k y m. La funcion forzadora F (t), que actuaba como fuerza
externa al sistema, se supuso continua. En las aplicaciones, no siempre se puede pedir tal
condicion; muchas veces, tal funcion es una funcion escalonada o con discontinuidades de
salto.
Definicion 4.5.1. La funcion de Heaviside con salto en 0 se define como la funcion:

1 t0
H (t) =
0 t<0
La funcion de Heaviside con salto en a > 0 esta dada por Ha (t) = H (t a), con
t R.

243
Apuntes Mat023 (Segundo semestre 2014)

Observacion 4.5.2. Para las funciones anteriores, se tiene:


Z
1
L [H (t)] (s) = H (t) est dt = , s>0
0 s
y para todo a > 0:
Z
L [Ha (t)] (s) = Ha (t) est dt
Z0
= est dt
a
eas
=
s
para s > 0.

Observacion 4.5.3. Note que si a < b la funcion:

f (t) = Ha (t) Hb (t)

corresponde a la funcion 
1 t [a, b[
f (t) =
0 t [a, b[c
Observacion 4.5.4. Para introducir la segunda propiedad de traslacion de la transformada
de Laplace, requerimos truncar una funcion. Mas precisamente, consideremos f : R R
una funcion y a > 0. Entonces, Ha fa : R R es la funcion definida como:

f (t a) t a
(Ha fa ) (t) =
0 t<a

Note que (Ha fa ) (t) = f (t a) H (t a).


Tenemos as el siguiente teorema, que muestra que multiplicar por una funcion escalon
unitario tiene el efecto de multiplicar su transformada por una funcion exponencial:

Teorema 4.5.1 (Segundo Teorema de la Traslacion). Sea a > 0 y f una funcion con
transformada de Laplace, entonces:

L [(Ha fa ) (t)] (s) = eas L [f (t)] (s)


aebs
Ejemplo 4.5.1. L [sin a (t b) H (t b)] (s) = ebs L [sin at] (s) = s2 +a2
.

Ejemplo 4.5.2. Calcule L [f (t)] (s), si:


 t
e 0 t < 2
f (t) =
et + cos t t > 2

244
Apuntes Mat023 (Segundo semestre 2014)

Solucion. Note que

f (t) = et + H (t 2) (cos t)
= et + H (t 2) (cos (t 2 + 2))
= et + H (t 2) (cos (t 2))

as
1 se2s
L [f (t)] (s) = + 2
s1 s +1
Ejemplo 4.5.3. Calcule:
1 es/2
 
1
L (t)
1 + s2
Solucion. Notamos que

1 es/2 1 1
2
= 2 es/2 2
1+s s +1 s +1
s/2
= L [sin t] (s) e L [sin t] (s)
h    i
= L [sin t] (s) L H t sin t (s)
2 2
h    i
= L sin t H t sin t (s)
2 2
luego
1 es/2
 
1
 
L (t) = sin t + H t cos t
1 + s2 2
Ejemplo 4.5.4. Resolver el P.V.I. dado por

y 00 2y = f (t) , y (0) = y 0 (0) = 0

donde 
t 0t<1
f (t) =
t2 t>1

Solucion. Usando la funcion salto podemos escribir la ecuacion como

y 00 2y = t + H (t 1) t2 t


para poder usar el segundo teorema de la traslacion manipulamos la funcion

t2 t = (t 1 + 1)2 (t 1 + 1)
= (t 1)2 + 2 (t 1) + 1 (t 1) 1
= (t 1)2 + (t 1)

245
Apuntes Mat023 (Segundo semestre 2014)

as
y 00 2y = t + H (t 1) (t 1)2 + H (t 1) (t 1)
se sigue
1 2 1
s2 2 L [y] (s) = 2 + es 3 + es 2

s s s
luego
1 2 1
L [y] (s) = + es + es
s2 (s2 2) s3 (s2 2) s2 (s2 2)
usando fracciones parciales
 
1 1 1 1
2 2
= 2
2
s (s 2) 2 s 2 s
y
2 1 s 1 1
=
s3 (s2 2) 2 s2 2 2s s3
luego
 
1 1 1 1
2 2
=
s (s 2) 2 s2 2 s2

1   t 
= L sinh 2t (s)
2 2 2
y
2 1 s 1 1
= 3
s3 (s2 2) 2
2 s 2 2s s

1   1 t2 
= L cosh 2t (s)
2 2 2
se sigue
1 2 1
L [y] (s) = + es + es
s2 (s2
2) 2) s3 (s2 2) s2 (s2

 
1   t
= L sinh 2t (s)
2 2 2

1   1 t2 
s
+e L cosh 2t (s)
2 2 2

1   t 
s
+e L sinh 2t (s)
2 2 2

1   t 
= L sinh 2t (s)
2 2 2
" !#
1   1 (t 1)2
+L H (t 1) cosh 2 (t 1) (s)
2 2 2
 
1   (t 1) 
+L H (t 1) sinh 2 (t 1) (s)
2 2 2

246
Apuntes Mat023 (Segundo semestre 2014)

la solucion es entonces
1   t
y (t) = sinh 2t
2 2 2
!
1   1 (t 1)2 1   (t 1)
+H (t 1) cosh 2 (t 1) + sinh 2 (t 1)
2 2 2 2 2 2

La Transformada de integrales de convolucion

Definicion 4.6.1. Sean f y g funciones seccionalmente continuas en [0, +). Se define la


convolucion de f y g como la funcion:
Z t
(f g) (t) = f (t u) g (u) du
0

Ejemplo 4.6.1. La convolucion entre f (t) = t y g (t) = sin t, esta dada por:
Z t
t sin t = (t u) sin u du
0
Z t Z t
= t sin u du u sin u du
0 0
= t t cos t sin t + t cos t
= t sin t

Ejemplo 4.6.2. Calcular t t2


Solucion.
Z t
2
tt = (t u) u2 du
0
1 4
= t
12
Observacion 4.6.1. Dentro de las propiedades algebraicas del producto de convolucion,
destacaremos la propiedad de conmutatividad. En efecto, observe que:
Z t
(f g) (t) = f (t u) g (u) du
0
Z 0
= f (z) g (t z) dz z = t u
t
Z t
= g (t z) f (z) dz
0
= (g f ) (t)

otras de sus propiedades basicas son:

247
Apuntes Mat023 (Segundo semestre 2014)

1. (f g) = (f ) g = f (g) para constante.

2. f (g h) = (f g) h

3. f (g + h) = (f g) + (f h)

Teorema 4.6.1. Sean f y g funciones seccionalmente continuas en [0, +) y de orden


exponencial entonces:

L [(f g) (t)] (s) = L [f (t)] (s) L [g (t)] (s)

o bien
L1 [L [f ] (s) L [g] (s)] (t) = (f g) (t)

Observacion 4.6.2. El resultado anterior, desde una perspectiva practica se utiliza


diciendo que la transformada de Laplace inversa de un producto de transformadas es la
convolucion.

Ejemplo 4.6.3. Calcule:


( )
s+1
L1
(s 2) (s + 1)2 + 32
2 

Solucion. Note que:


s+1 1 s+1
2 2 =
(s 2) (s + 1) + 32 (s 2) (s + 1)2 + 32
2

Luego:
" #    
s+1 1 s+1
L1  (t) = L 1
(t) L 1
(s)
(s 2)2 (s + 1)2 + 32 (s 2)2 (s + 1)2 + 32
= te2t et cos (3t)
Z t
= (t u) e2(tu) eu cos (3u) du
0
1 2t 1
= te et sin 3t
6 18
Ejemplo 4.6.4. Calcular
 
1 1
L (t)
(s 2) (s + 2) (s 1)

248
Apuntes Mat023 (Segundo semestre 2014)

Solucion. Esto lo podemos calcular con fracciones parciales pero


1 1 1 1
=
(s 2) (s + 2) (s 1) s2s+2s1
= L e2t e2t et (s)
 

donde t
e2t
Z
2t 1
e e2t
= e2(tu) e2u du = e2t
0 4 4
y
t
1 2u e2u
Z  
2t 2t t tu
e e e = e e du
0 4 4
2t
e 1 1
= et + e2t
12 3 4
se sigue
e2t 1 t 1 2t
 
1 1
L (t) = e + e
(s 2) (s + 2) (s 1) 12 3 4
Definicion 4.6.2. Sean f (t) , g (t) funciones continuas. Llamaremos ecuacion de Volte-
rra a una ecuacion de la forma:
Z t
x (t) = f (t) + g (t u) x (u) du
0

Observacion 4.6.3. Note que, mediante el producto de convolucion, podemos escribir la


ecuacion anterior como:
x (t) = f (t) + (g x) (t)
Ahora bien, si ponemos X (s) = L [x (t)] (s) , F (s) = L [f (t)] (s) y G (s) = L [g (t)] (s) y
aplicamos luego la transformada de Laplace a la ecuacion anterior, obtenemos:

X (s) = F (s) + G (s) X (s)

de donde se obtiene finalmente:


F (s)
X (s) =
1 G (s)
de donde podemos obtener
 
1 F (s)
x (t) = L (t)
1 G (s)
Ejemplo 4.6.5. Resuelva la ecuacion integral:
Z t
2
x (t) = t + sin (t u) x (u) du
0

249
Apuntes Mat023 (Segundo semestre 2014)

Solucion. Supongamos que X (s) = L [x (t)] (s). Entonces, aplicando la transformada de


Laplace a la ecuacion anterior, tenemos:
2 1
X (s) = 3
+ 2 X (s)
s s +1
Es decir:
2 2
X (s) = 3
+ 5
s s
2 2 4!
= 3+
s 4! s5
Por lo tanto:
   
2
1 2 1 4!
x (s) = L (t) + L (t)
s3 4! s5
t4
= t2 +
12
Ejemplo 4.6.6. Resolver el sistema de ecuaciones

x0 = x y + t
y 0 = x + y + et

con las condiciones iniciales x (0) = y (0) = 0.


Solucion. Pongamos X (s) = L [x (t)] (s) y Y (s) = L [y (t)] (s) entonces
1
sX = X Y +
s2
1
sY = X +Y +
s1
luego
2s 1
X =
2s2
4s3 + 3s4 s5
s 12 1 1
= 2 2
s 2s + 2 s 1 2s
s2 + 1
Y =
2s2 2s3 + s4
1
1 s 32 1
= 22 + 2
2s s 2s + 2 2s
de donde obtenemos
1 1
x (t) = et cos t + et sin t et t
2 2
1 1 1 t
y (t) = t + e (cos t 2 sin t)
2 2 2

250
Apuntes Mat023 (Segundo semestre 2014)

Ejemplo 4.6.7. Resolver el problema


t
d2 y
Z
= t+2 e(tu) y (u) du
dt2 0
y (0) = 0
y 0 (0) = 0
Solucion. Aplicando la transformada de Laplace
1 1
s2 L [y] (s) = 2
+2 L [y] (s)
s s+1
luego  
2 2 1
s L [y] (s) =
s+1 s2
despejando
1
s2 1 s+1
L [y] (s) = 2
=
s2 s+1
s2 s3 + s2 2
s+1
=
s2 (s 1) (2s + s2 + 2)
hacemos fracciones parciales
s+1
s2
(s 1) (2s + s2 + 2)
1 3
2 s + 10 1 1
= + 210 2
5 (s 1) s + 2s + 2 2s 2s
y reconocemos las transformadas
 
2 2
= L et (s)
5 (s 1) 5
1 3  
10
s + 10 1 t
= L e (cos t + 2 sin t) (s)
s2 + 2s + 2 10
 
1 1 1 1
2 = L t (s)
2s 2s 2 2
se sigue por Lerch
2 1 1 1
y (t) = et + et (cos t + 2 sin t) t
5 10 2 2
otra forma es usar convoluciones, note que
s+1 1 1 s+1
= 2
s2 2
(s 1) (2s + s + 2) s (s 1) (s + 1)2 + 1
= L [t] (s) L et (s) L et cos t (s)
   

= L t et et cos t (s)
  

251
Apuntes Mat023 (Segundo semestre 2014)

de donde
y (t) = t et et cos t
 

calculando
1  t
= L [t] (s) L e (s)
s2 (s 1)
= L t et (s)
 

as
Z t
t
te = (t u) eu du
0
= et t 1

luego
1 s+1  t   t 
= L e t 1 (s) L e cos t (s)
s2 (s 1) (s + 1)2 + 1
as
Z t
etu t + u 1 eu cos udu

y (t) =
0
2 t 1 1 1 1
= e t+ (cos t) et + (sin t) et
5 2 10 5 2
Observacion 4.6.4. Como apendice calcularemos la transformada de la funcion delta
de Dirac. Muy informalmente, la funcion delta de Dirac se describe como una funcion que
es cero en todas partes excepto en t = 0 y cumple ademas con:
Z
(t) dt = 1

Claramente, ninguna funcion real puede satisfacer tales condiciones. Sin embargo, considere
la familia de funciones:  1
2
, < t <
f (t) =
0 , |t| >
Note que, para cada > 0 las funciones f (t) son seccionalmente continuas, y ademas:
Z Z
f (t) dt = f (t) dt = 1

De este modo, podemos pensar a como:

(t) = lm+ f (t)


0

252
Apuntes Mat023 (Segundo semestre 2014)

Definicion 4.6.3. Sea a > 0. La funcion delta de Dirac con salto en t = a se define
como:
a (t) = (t a)
Observacion 4.6.5. Notemos que
1 1
f (t a) = H (t a + ) H (t a )
2 2
as
e(a+)s e(+a)s
L [f (t a)] (s) =
2s
se sigue

L [ (t a)] (s) = lm L [f (t a)] (s)


0
es es
= eas lm
0 2s
ses
+ ses
= eas lm
0 2s
as
= e

Teorema 4.6.2. Sea a > 0, entonces:

L [a (t)] (s) = eas s>0

y
L [ (t)] (s) = 1

Ejemplo 4.6.8. Resolver el P.V.I.

y 00 y = 2 (t)
y (0) = 0
y 0 (0) = 0

Solucion. Aplicando la transformada de Laplace

s2 1 L [y] (s) = e2s




as
e2s
L [y] (s) = = e2s L [sinh (t)] (s)
s2 1
= L [H (t 2) sinh (t 2)] (s)

as
y (t) = H (t 2) sinh (t 2)

253
Apuntes Mat023 (Segundo semestre 2014)

Observacion 4.6.6. Una aplicacion interesante es estudiar la ecuacion diferencial:


dI Q
L + R I + = a (t)
dt C
que es la ecuacion diferencial asociada a un circuito RLC en serie. Es decir, un circuito
electrico compuesto por una resistencia R [], un capacitor C [F], un inductor L [H] y
una fuente de voltaje modelada por la funcion delta de Dirac como funcion forzadora.
Naturalmente, las herramientas de las ecuaciones diferenciales a coeficientes constantes no
bastan para resolver este sistema.

254
Apuntes Mat023 (Segundo semestre 2014)

Ejercicios del captulo

1. Sean , > 0. Encontrar la transformada de Laplace de:


 
1) sin (t + ) + cos2 (t) 2) (t + ) e5t + t cos 2t +
4

3) t2 cos t + 2e3t sin (2t) 4) t3 e2t cos (6t) + 7e4t sin t cos2 t
Z t   Z t
2t d 3u 2
5) te u e cos u du 6) t cos2 (u) du
0 du 3

 
t si t < 2 cos t si 0 t < 2
7) f (t) = 8) f (t) =
0 si t > 2 sin t si t > 2

cos t si 0 t 2 Z t
9) f (t) = sin t si 2 < t 4 10) sinh [2 (t u)] cosh (2u) du
t 0
e si 4 < t
Z t Z t
3
11) (t u) sin u du 12) eut cos (3u) du
0 0

2. Si > 0, calcular las transformadas de:


Z t
sin t sin (u) cos t 1
1) 2) du 3)
t 0 u t
t t
1 cosh t 1 cos (u) 1 cosh (u)
Z Z
4) 5) du 6) du
t 0 u 0 u
Z t
2 2
X
7) erf (t) = eu du 8) H (t n)
0 n=0

255
Apuntes Mat023 (Segundo semestre 2014)

3. Encontrar la transformada inversa de las siguientes funciones:


2 3 s+1
1) 2) 3 3)
s (s + 1) (s + 1) s (s + 4)2

2s s 2s
4) 5) 6)
s (s 7) s2 + 6s + 25 (s2 + 4)2

es e3s 1
7) 8) 9)
(s2 + 1)2 (s 1) (s + 2) s3 + 23

n
s2 eks s2 + 1
X  
10) 11) 12) ln
s3 + 2 3 k=1
s s (s 3)

es s
   
2s 1 1 1
13) e 2
+ 4 14) arctan 15)
s s s s2 + 3

e3s
 
s 1 1
16) 17) 18) arctan
s2 9 (s + 3) (s2 + 1)
2 s s

52
   
1 s
19) 20) ln 21) ln 1 + 2
(s2 + 1)3 s+1 s

4. Suponga que , R, n, m N. Calcular explcitamente las siguientes convoluciones:

a) exp (t) exp (t)


b) tn exp (t)
c) tn tm
d ) exp (t) sin (t)
e) exp (t) cos (t)
f ) sin (t) sin (t)
g) sin (t) cos (t)
h) cos (t) cos (t)
i ) tn cos (t)
j ) tn sin (t)

5. Escriba las transformadas inversas de las siguientes funciones en terminos de convo-


luciones:

256
Apuntes Mat023 (Segundo semestre 2014)

s3
a)
(s2 + 4) (s2 + 9) (s2 + 16)
s
b)
(s 2) (s + 2) (s 1)
     
s+1 1 1
c) ln ln 1 + 2 arctan
s+2 s s

6. Resuelva las siguientes ecuaciones integrales o integro-diferenciales.


Z t
t
a) x (t) = e 2 cos (t u) x (u) du
0
Z t
1
b) x (t) = t + (t u)3 x (u) du
6 0
Z t
c) x (t) = t + sin (t u) x (u) du
0
Z t
d ) x (t) = sin t + cos (t u) x (u) du
0
Z t
0
e) x + 2x + x (u) du = f (t) donde
0

cos t si 0 t < 2
f (t) =
sin t si t > 2

con la condicion inicial x (0) = 1


Z t
f) sin (t u) y (u) du = y (t) + et cos (3t)
0

7. Suponga que f, f 0 : [0, +[ R Zsonlocalmente integrables y de orden exponen-


t Z u 
df
cial: Calcular la transformada de f (v) dv du y tet en terminos de la
0 0 dt
transformada de f .

8. La funcion Gamma esta definida por:


Z
(x) = eu ux1 du, x>0
0
R
a) Demuestre que (x + 1) = 0
eu ux dx.
b) Integrando por partes, muestre que:

(x + 1) = x (x)

c) Demuestre que (1) = 1

257
Apuntes Mat023 (Segundo semestre 2014)

d ) Usando los resultados de (b) y (c), pruebe que si n es un entero positivo,


entonces:
(n + 1) = n!
e) Calcule, usando la sustitucion u = st en la parte (a), demuestre que:
(x + 1)
L [tx ] (s) = , s > 0, x > 1
sx+1
1

f ) Se sabe que 2
= . Calcule:
n o
1) L 1t

2) L t
 5/2
3) L t

9. Resuelva las siguientes ecuaciones diferenciales mediante Transformada de Laplace:

a) y 00 + y = 0, y (0) = 1, y 0 (0) = 3
b) y 00 4y = 0, y (0) = 2, y 0 (0) = 5
c) y 00 + 4y 0 + 4y = 0, y (0) = 1, y 0 (0) = 2
d ) y 00 + 2y 0 + 2y = H (t 3) , y (0) = y 0 (0) = 0
e) y 000 + y = 0, y (0) = y 00 (0) = 1, y 0 (0) = 1
f ) y (4) y = 0, y (0) = y 00 (0) = 0, y 0 (0) = y 000 (0) = 1
g) y (4) y = sinh t, y (0) = y 00 (0) = 0, y 0 (0) = y 000 (0) = 1

10. Resolver los siguientes problemas de valores iniciales

a) y 00 4y = f (t) donde

t2

si 0 t < 1
f (t) = 1 t si 1 < t < 3
et

3<t

con las condiciones iniciales y (0) = 0, y 0 (0) = 1.


b) y 00 + 4y 0 + 13y = et cos t con valores iniciales y (0) = 1, y 0 (0) = 0.
c) y 00 + y = 1 con y (1) = y 0 (1) = 1.
d)

y 00 + y = 3 sin (4t) H (t 2) sin (8t)


y (0) = 1
y 0 (0) = 1

258
Apuntes Mat023 (Segundo semestre 2014)

e)

y 00 + 3y 0 + 2y = 3e4t H (t 2)
y (0) = 2
y 0 (0) = 1

f)

ty 00 + 2 (t 1) y 0 2y = 0
y (0) = 0
y 0 (0) = 0

g)

ty 00 + 2 (t 1) y 0 + (t 2) y = 0
y (0) = 1
y 0 (0) = 1

h)

ty 00 ty 0 + y = 0
y (0) = 0
y 0 (0) = 1

11. Resolver el siguiente sistema utilizando transformada de Laplace:

2x01 + x1 + x002 = e6t


2x1 + x02 = 0

donde x1 (0) = 1, x2 (0) = 2 y x02 (0) = 2.

12. Resolver los siguientes sistemas utilizando transformada de Laplace:


 0
x 4x y + t = 0
1) con x (0) = 0, y (0) = 1
y 0 + 2x y + et = 0

x0 + x + y 0 + cos t = 0

2) con x (0) = 0, y (0) = 0
y 0 + x y + et = 0

x0 + x + y = cos t

3) con x (0) = 1, y (0) = 2
y 0 + x y = sin t

x00 + y 00 + y 0 = t2

4) con x (0) = 1, x0 (0) = 0, y (0) = 0, y 0 (0) = 1
x00 y 00 = 2t

259
Apuntes Mat023 (Segundo semestre 2014)

13. Calcular la transformada de Laplace de f (t) = abtc donde btc es la parte entera de t
y a R+ y con esa informacion resolver la ecuacion

y (t + 2) 3y (t + 1) + 2y (t) = 0

si se sabe que y (t) es constante e igual a 0 en el intervalo [0, 1[ y es constante e igual


a 1 en [1, 2[.

14. Resolver el problema de valores iniciales

y 00 + ty 0 2y = 4
y (0) = 1
y 0 (0) = 0

15. Resolver la ecuacion diferencial


d2 y dy
2
+ 2t 4y = 2 2t
dt dt
con y (0) = 0 y y 0 (0) = 1.

16. Muestre que si f : R R es una funcion periodica de periodo T > 0 entonces


Z T
1
L [f (t)] (s) = est f (t) dt
1 esT 0
RT R +
Ind.: L [f (t)] (s) = 0
f (t) est dt + T
f (t) est dt y cambie variable.

17. Calcular la transformada de las funciones f (t) = |sin t| y f (t) = t btc

260
Apuntes Mat023 (Segundo semestre 2014)

18. Calcular las transformadas de las funciones de la figura

261
Captulo 5 : Series de Fourier

Definiciones

El espacio de las funciones continuas en un intervalo [a, b] a valores reales representado


por C [a, b], es un espacio de dimension infinita, pues para todo m N se cumple

1.x.x2 , . . . , xm C [a, b]


y el conjunto {1.x.x2 , . . . , xm } es linealmente independiente. Sabemos que si un espacio V


tiene una base B = {v1 , v2 , . . . , vm } entonces todo elemento del espacio se puede escribir
como combinacion lineal, esto es,
Xm
v= j vj
j=1

al ser un espacio de dimension infinita, debemos esperar que los elementos sean mas
complicados que un polinomio, por ejemplo sabemos que ex no es un polinomio. Es natural
preguntarse entonces, si todos los elementos se pueden escribir como una combinacion
lineal infinita de los elementos de este conjunto, esto es, una serie de la forma
+
X
f (x) = ak xk = a0 + a1 x + a2 x2 +
k=0

lo que llamamos serie de Taylor. Sabemos de Mat022 que las funciones que se pueden
escribir de esta forma son funciones de clase C en su intervalo de convergencia, luego
no es de esperar que toda funcion continua se pueda escribir como una serie de Taylor
existira otra forma de escribir estas funciones en forma de serie? La respuesta a esta
pregunta es afirmativa y viene dada por las series de Fourier, en algun sentido que vamos a
especificar en breve, toda funcion continua f C [a, b] se puede escribir en la forma
+   +  
a0 X 2nx X 2nx
+ an cos + bn sin
2 n=1
ba n=1
ba

para elecciones adecuadas de las constantes a0 , an y bn .

El espacio SC [a, b]

Definicion 5.1.1. Una funcion a valores reales f es llamada funcion seccionalmente


continua en [a, b] si:

1. f esta definida y es continua en todos los puntos de [a, b] salvo quizas un numero
finito de ellos.

262
Apuntes Mat023 (Segundo semestre 2014)

2. Los lmites

f x+

0 = lm f (x)
xx+
0

f x

0 = lm f (x)
xx
0

existen en cada punto de [a, b] (note que en los extremos solo un lmite es relevante).

Z b
Observacion 5.1.1. Si f SC [a, b] entonces f (x) dx existe y es independiente de
a
los valores (si los toma) de la funcion en los puntos de discontinuidad. En particular, si
f, g SC [a, b] son iguales salvo en los puntos de discontinuidad entonces
Z b Z b
f (x) dx = g (x) dx
a a

diremos entonces que dos funciones en SC [a, b] son iguales si f (x) = g (x) salvo quizas en
los puntos de discontinuidad.

Observacion 5.1.2. Si R, f, g SC [a, b] entonces f , f + g y f g son funciones en


SC [a, b].

Observacion 5.1.3. C [a, b] SC [a, b].

Definicion 5.1.2. Sea V un espacio vectorial real. Diremos que la funcion h, i : V V


R es un producto interior si cumple:

1. f V , hf, f i 0, Ademas hf, f i = 0 f = 0

2. f, g V , hf, gi = hg, f i

3. R, f V , hf, gi = hf, gi

4. f, g, h V , hf + g, hi = hf, hi + hg, hi

Ejemplo 5.1.1. En V = Rn , la funcion h, i : V V R definida por


n
X
< x, y >= xi y i
i=1

es un producto interior.

263
Apuntes Mat023 (Segundo semestre 2014)

Ejemplo 5.1.2. En SC[a, b] la funcion h, i : SC[a, b] SC[a, b] R definida por


Z b
(f, g) hf, gi = f (x)g(x) dx
a

es un producto interior. Llamaremos a este producto, el producto usual en SC[a, b].

Ejemplo 5.1.3. En C[a, b], si C [a, b] es tal que (x) > 0 para x [a, b] la funcion
h, i : C[a, b] C[a, b] R definida por
Z b
(f, g) hf, gi = (x) f (x)g(x) dx
a

es un producto interior. Este producto es llamado producto con peso .

En todo espacio vectorial con producto interior es posible definir una norma

k k : V R

como p
kf k = hf, f i
y esta a su vez, nos permite definir una funcion distancia en el espacio V mediante la
formula

d(f, g) = kf gk

Teorema 5.1.1. Sea (V, h, i) un espacio vectorial real con producto interior, para toda
f, g V se tiene p p
|hf, gi| hf, f i hg, gi

Ejemplo 5.1.4. Si f (x) = ex , g(x) = ex son funciones en C [0, 1], calcular d (f, g), con
la distancia inducida por el producto usual del ejemplo.
Tenemos
p
d (f, g) = hf g, f gi
s
Z 1
= (f (x) g (x))2 dx
0
s
Z 1
= (ex ex )2 dx
0

= sinh 2 2

264
Apuntes Mat023 (Segundo semestre 2014)

Proposicion 5.1.1. Propiedades basicas del producto interior h, i en un espacio vectorial


real V :

1. f, g V, R, hf, gi = hf, gi

2. f, g, h V, hf, g + hi = hf, gi + hf, hi

3. fk , gj V, con k = 1, 2, . . . , n y j = 1, 2, . . . , m y k , j R, con k = 1, 2, . . . , n y
j = 1, 2, . . . , m * n +
X Xm X n Xm
k fk , j gj = k j hfk , gj i
k=1 j=1 k=1 j=1

Definicion 5.1.3. Sea V un espacio con producto interior < , >.

1. Diremos que f y g en V son ortogonales si < f, g >= 0

2. Diremos que {fk }kI es un conjunto ortogonal si < fk , fj >= 0, para k 6= j

3. Diremos que {fk }kI es un conjunto ortonormal si es ortogonal y la norma de cada


elemento es 1

Ejemplo 5.1.5. En SC[0, L], la familia sin nx


 
L
, n N es ortogonal.
En efecto, si n 6= m entonces
Z  nx   mx 
sin sin dx
L L
 
L (m n) x (m + n) x
= (m + n) sin + (n m) sin
2 (m2 n2 ) L L
(usar prostaferesis) luego
Z L  nx   mx 
sin sin dx
0 L L
  L
L (m n) x (m + n) x
= (m + n) sin + (n m) sin
2 (m2 n2 ) L L
 0
L (m n) L (m + n) L
= 2 2
(m + n) sin + (n m) sin
2 (m n ) L L
L
= ((m + n) sin [(m n) ] + (n m) sin [(m + n) ])
2 (m2 n2 )
= 0

265
Apuntes Mat023 (Segundo semestre 2014)

Ejemplo 5.1.6. En SC[a, b] la familia


           
2x 2x 4x 4x 6x 6x
{1, cos , sin , cos , sin , cos , sin ,...}
ba ba ba ba ba ba
=      
2nx 2nx
1, cos , sin con n N
ba ba

es una familia ortogonal.

Ejemplo 5.1.7. Si (V, h, i) es un espacio con producto interior y {u1 , u2 , u3 } V es un


conjunto ortonormal, determine la norma del vector

u1 u2 + u3

Solucion. Usando las propiedades del producto punto y la ortogonalidad

ku1 u2 + u3 k2 = hu1 u2 + u3 , u1 u2 + u3 i
= ku1 k2 + ku2 k2 + ku3 k2
= 3

(los vectores tienen norma 1) entonces



ku1 u2 + u3 k = 3
n o
Ejemplo 5.1.8. Muestre que el conjunto B = 12 , cos
x , sin

2x

es ortonormal en C [, ]
y calcular
Z  2 !1/2
1 cos x sin 2x
+ dx
2

Solucion.
  Z
1 cos x cos x
, = dx
2 2
Z
1
= cos xdx
2

1
= sin (x) = 0
2

y   Z
1 sin x sin 2x
, = dx = 0
2 2

266
Apuntes Mat023 (Segundo semestre 2014)

(funcion impar)   Z
sin 2x cos x sin 2x cos x
, = dx = 0

funcion impar. Es facil ver que

1
= 1 k1k = 2 = 1
2 2 2
Z 1/2
sin 2x
= 1 2
sin 2xdx

 Z 1/2
1 1 cos 4x
= 2 dx
0 2
1 1/2
= =1

Z 1/2
cos x 1 2
= cos xdx

 Z 1/2
1 2
= 2 cos xdx
0
 Z 1/2
1 1 + cos 2x
= 2 dx
0 2
= 1

usando el ejercicio anterior


2 !1/2

Z 
1 cos x sin 2x
+ dx = 3
2

Teorema de la mejor aproximacion

Ahora vamos a analizar cuando tenemos familia ortonormales como se logra la mejor
aproximacion de un elemento en un espacio dado. La idea es obtener una aproximacion
con distancia cero.

Teorema 5.1.2. Sea {fk }N


k=1 un conjunto ortogonal en V . Si k R, k = 1, . . . N
entonces 2
XN N
X
f = |k |2 kfk k2

k k

k=1 k=1

267
Apuntes Mat023 (Segundo semestre 2014)

Demostracion.
N 2 * N N
+
X X X
k f k = k f k , j f j



k=1 k=1 j=1
N X
X N
= k j hfk , fj i
k=1 j=1
N
X
= 2k kfk k2
k=1

Note que si el conjunto {fk }N


k=1 es ortogonal (y funciones no nulas) en V entonces es
linealmente independiente
XN
k fk = V
k=1

implica
* N +
X
0 = hV , fi i = k fk , fi
k=1
N
X
= k hfk , fi i
k=1
= i hfi , fi i

de donde obtenemos i = 0.
Sea f V dado, Cual es la mejor aproximacion que podemos hacer en V mediante un
elemento de de la forma
XN
k fk
k=1

esto es un elemento del subespacio generado por {f1 , f2 , ..., fN }

S = h{f1 , f2 , ..., fN }i

donde {fk } es un conjunto ortonormal.

268
Apuntes Mat023 (Segundo semestre 2014)

N
!2 N
2
X X
d f, k f k = f k f k


k=1 k=1
N
X N
X
2
= kf k 2 k hf, fk i + |k |2 kfk k2
k=1 k=1
XN XN
= kf k2 2 k hf, fk i + |k |2
k=1 k=1
N
X N
X
= kf k2 hf, fk i2 + (hf, fk i k )2
k=1 k=1

se sigue, independiente de los escalares 1 , 2 , . . . , N

N
! v u N
X u 2
X
d f, k fk kf k
t hf, fk i2
k=1 k=1

luego el mnimo valor de esta distancia corresponde a


v
u
u XN
tkf k2 hf, fk i2
k=1

que se alcanza en la combinacion lineal de la forma


N
X
hf, fk i fk
k=1

se sigue que
N
X
hf, fk i fk
k=1

es la mejor aproximacion de f V en el espacio.

S = h{f1 , f2 , ..., fN }i
y
! v
N
X
u
u N
X
mn d f, k fk = tkf k2 < f, fk >2
(1 ,...,N )Rn
k=1 k=1

269
Apuntes Mat023 (Segundo semestre 2014)

Teorema 5.1.3. Sea {fk }N k=1 un conjunto ortonormal en un espacio vectorial V con
producto interior, f V y S = h{f1 , f2 , . . . , fN }i se tiene
N
!
X
d (f, S) = mn n d f, k f k
(1 ,...,N )R
k=1
v
u
u N
X
2
= tkf k hf, fk i2
k=1

y esta distancia mnima se logra para i = hf, fi i con i = 1, 2, . . . , N . Es decir, el elemento


de S a menor distancia de f es
XN
hf, fk i fk
k=1

de este resultado obtenemos:

Teorema 5.1.4 (Desigualdad de Bessel). Si {fk }Nk=1 es un conjunto ortonormal en V


entonces f V
XN
hf, fk i2 kf k2
k=1

Demostracion. De los calculos anteriores


N
!2 N N
X 2
X 2
X
0 d f, k fk = kf k hf, fk i + (hf, fk i k )2
k=1 k=1 k=1

si tomamos hf, fk i k = 0 entonces


N
X
2
0 kf k hf, fk i2
k=1

que es la desigualdad deseada.

Teorema 5.1.5. Si {fk }


k=1 es un conjunto ortonormal en V entonces para cada f en V
se cumple
lm hf, fk i = 0
k

Demostracion. Por la desigualdad de Bessel se tiene N N


N
X
hf, fk i2 kf k2
k=1

270
Apuntes Mat023 (Segundo semestre 2014)

luego la serie + 2
P
k=1 hf, fk i converge (sucesion de sumas parciales creciente y acotada
superiormente) luego
lm hf, fk i = 0
k

n o
Ejemplo 5.1.9. Muestre que el conjunto 1 , sin
x
es ortonormal en C [, ] y deter-
2
minar el elemento de la forma
sin x
+
2
(con , R) mas cercano a f (x) = x. Cual es la mnima distancia?.
Solucion. Primero calculamos las normas
Z 1/2
1 1
= dx
2
2
 Z 1/2
1
= dx
2
 Z 1/2
1
= dx
2
= (1)1/2
= 1

y
Z 2
1/2
sin x sin x
= dx


 Z 1/2
1 1 cos 2x
= dx
2
 1/2
1
=

= 1

que son ortogonales es facil


  Z
1 sin x 1
, = sin xdx
2 2
= 0

(por imparidad). El elemento mas cercano es


   
1 1 sin x sin x
g (x) = , x + , x
2 2

271
Apuntes Mat023 (Segundo semestre 2014)

donde
  Z
1 x
,x = dx = 0
2 2
Z

 
sin x x sin x
,x = dx = 2

as
g (x) = 2 sin x
y la distancia mnima es
2
q
d = kxk2 02 2
donde Z
2 2
kxk = x2 dx = 3
3
entonces r
2 3
d= 4
3
Observacion 5.1.4. El conjunto
( )
2kx 2kx
 
1 cos ba
sin ba
B= ,  ,
2kx
2kx
 con k N
k1k cos ba
sin
ba

es un conjunto ortonormal en SC[a, b], notemos que


Z b 1/2
p
dx = (b a)
a

!
b b1 + cos 4kx
Z   Z
2kx ba
cos2 dx = dx
a ba a 2
1 b
   
ba
Z
4kx
= + cos dx
2 2 a ba
     b
ba 1 4kx b a
= + sin
2 2 ba 4k a
        
ba 1 4kb 4ka ba
= + sin sin
2 2 ba ba 4k
 
ba
=
2
donde hemos usado la formula de prostaferesis
   
xy x+y
sin (x) sin (y) = 2 sin cos
2 2

272
Apuntes Mat023 (Segundo semestre 2014)

para ver    
4kb 4ka
sin sin =0
ba ba
ahora bien
Z b   Z b  
2 2kx 2kx 2
sin dx = 1 cos dx
a ba a ba
 
ba
= (b a)
2
 
ba
=
2

as

k1k = ba
  r
cos 2kx =
ba
ba 2
  r
sin 2kx =
ba
ba 2

Definicion 5.1.4. Sea f SC [a, b]. Llamaremos serie de Fourier de f a la serie de la


forma   X  
a0 X 2k 2k
+ ak cos x + bk sin x
2 k=1
b a k=1
b a
donde los coeficientes son dados por la mejor aproximacion.

Calculemos los coeficientes en forma explcita:

hf (x) , 1i
*   X   +
a0 X 2k 2k
= + ak cos x + bk sin x ,1
2 k=1
b a k=1
b a
*   + *X   +
Da E 2k 2k
0
X
= ,1 + ak cos x ,1 + bk sin x ,1
2 k=1
b a k=1
b a
    X    
a0 X 2k 2k
= h1, 1i + ak cos x ,1 + bk sin x ,1
2 k=1
b a k=1
b a
a0
= (b a)
2
esto es Z b
2
a0 = f (x) dx
ba a

273
Apuntes Mat023 (Segundo semestre 2014)

de manera similar
       
2n 2n 2n
f, cos x = an cos x , cos x
ba ba ba

as
2n


f, cos ba x
an =
2n
 2n

cos ba x , cos ba x
Z b  
2 2n
= f (x) cos x dx para n N
ba a ba
y
2n


f, sin ba x
bn =
2n
 2n

sin ba x , sin ba x
Z b  
2 2n
= f (x) sin x dx para n N
ba a ba

Teorema 5.1.6. Si f SC [a, b] entonces


!
N   XN 
a 0
X 2k 2k
lm f (x) + ak cos x + bk sin x =0

N + 2 k=1
ba ba
k=1

si a0 , ak y bk corresponden a los coeficientes de Fourier de f .

Podemos entonces escribir


  X  
a0 X 2k 2k
f (x) = + ak cos x + bk sin x
2 k=1
ba k=1
ba

pero la convergencia de la serie no es necesariamente al evaluar en los puntos, sino en el


sentido de la distancia definida.

Ejemplo 5.1.10. Determinar la serie de Fourier de f (x) = x en C [, ]

Solucion. En este intervalo la serie tiene la forma



a0 X X
+ ak cos (kx) + bk sin (kx)
2 k=1 k=1

274
Apuntes Mat023 (Segundo semestre 2014)

note que al ser una funcion impar a0 = ak = 0 y


1
Z
bk = x sin (kx) dx

2
Z
= x sin (kx) dx
0
2
= 2
((sin kx kx cos kx)|0 )
k
2  k

= k (1)
k 2
2 (1)k+1
=
k
as

!
X 2 (1)k+1
x= sin (kx)
k=1
k

275
Apuntes Mat023 (Segundo semestre 2014)

N
!
X 2 (1)k+1
en media. Veamos algunos graficos de la aproximacion sin (kx)
k=1
k
   
P3 2(1)k+1 P6 2(1)k+1
k=1 k
sin (kx) k=1 k
sin (kx)

P15  2(1)k+1  P60  2(1)k+1 


k=1 k
sin (kx) k=1 k
sin (kx)

Ejemplo 5.1.11. Determinar la serie de Fourier de

f (x) = ex

para x [1, 1] .

Solucion. La serie de Fourier de f en este intervalo es de la forma


   
a0 X 2nx X 2nx
+ an cos + an sin
2 n=1
2 n=1
2

a0 X X
= + an cos (nx) + an sin (nx)
2 n=1 n=1

276
Apuntes Mat023 (Segundo semestre 2014)

donde
Z 1
2
a0 = ex dx = 2 sinh 1
2 1
1
(2 sinh 1) (1)n
Z
an = ex cos (nx) dx = para n N
1 2 n2 + 1
y
1
(2n sinh 1) (1)n+1
Z
x
bn = e sin (nx) dx = para n N
1 2 n2 + 1
se sigue

x
X (2 sinh 1) (1)n X (2n sinh 1) (1)n+1
e = (sinh 1) + cos (nx) + sin (nx)
n=1
2 n2 + 1 n=1
2 n2 + 1

!
X 2 (1)n X 2n (1)n+1
= (sinh 1) 1 + 2 n2 + 1
cos (nx) + 2 n2 + 1
sin (nx)
n=1
n=1

as por ejemplo, la grafica de la aproximacion


10 10
!
X 2 (1)n X 2n (1)n+1
(sinh 1) 1 + cos (nx) + sin (nx)
n=1
2 n2 + 1 n=1
2 n2 + 1
es:

en rojo esta la grafica de la exponencial.

Teorema 5.1.7 (Identidad de Parseval). Si f SC [a, b] entonces


Z b
2 a2 X 2 X 2
f 2 (x) dx = 0 + an + bn
ba a 2 n=1 n=1

donde Z b  
2 2n
an = f (x) cos x dx para n N {0}
ba a ba

277
Apuntes Mat023 (Segundo semestre 2014)

y Z b  
2 2n
bn = f (x) sin x dx para n N
ba a ba

Ejemplo 5.1.1. Calcular la serie de Fourier de f (x) = |x| para x [1, 1] y usando
Parseval calcular
X 1
n=1
(2n + 1)4

Convergencia Puntual de series de Fourier

Si bien hemos enunciado que la serie de Fourier converge en norma la funcion dada, se
presenta la siguiente situacion: Determinemos la serie de Fourier de

1 si x [0, ]
f (x) =
1 si x [, 0[

se tiene:
1
Z
a0 = f (x) dx = 0 (la funcion es impar)

1
Z
an = f (x) cos (nx) dx = 0 (la funcion f (x) cos (nx) es impar) para cada n N

1
Z
1
bn = f (x) sin (nx) dx = (2 (1)n 2) para n N
n

se sigue que
 
X 1 n
f (x) = (2 (1) 2) sin (nx) (en media)
n=1
n
 
2X 1 n
= ((1) 1) sin (nx) (en media)
n=1 n

pero
f (0) = 1
pero si evaluamos
 
2X 1 n
((1) 1) sin (nx)
n=1 n
en x = 0 nos queda
 
2X 1 n
((1) 1) sin (n0) = 0
n=1 n

278
Apuntes Mat023 (Segundo semestre 2014)

luego el valor al cual converge la serie no es el valor de la funcion en el punto. Como se ve


en las siguiente graficas, los combinaciones trigonometricas
N  
2X 1 n
((1) 1) sin (nx)
n=1 n

cuando N crece se va pareciendo mas a la grafica pero todos estas funciones pasan por el 0.
El siguiente teorema nos dice a que valor converge la serie de Fourier
  X  
a0 X 2k 2k
+ ak cos x + bk sin x
2 k=1
b a k=1
b a

de f SC [a, b] cuando evaluamos en un x0 [a, b].

Teorema 5.2.1 (Convergencia puntual). Sea f, f 0 SC [a, b] entonces la serie de Fourier


converge en todos los puntos del intervalo [a, b], ademas si x0 [a, b] la serie de Fourier
converge a

f x+
 
0 + f x0
2
cuando x0 ]a, b[ y a
f (b ) + f (a+ )
2
si x0 = a o x0 = b.

Por convergencia puntual se entiende


N     !
a0 X 2k 2k
lm + ak cos x0 + bk sin x0 = lm SN (x0 )
N 2 k=1
ba ba N


f x+
 
0 + f x0
=
2
(no es la convergencia en norma, la cual esta garantizada sin mirar la derivada).

Ejemplo 5.2.1. Sabemos que



!
n n+1
X 2 (1) X 2n (1)
ex = (sinh 1) 1 + 2 n2 + 1
cos (nx) + 2 n2 + 1
sin (nx) (en media)
n=1
n=1

para x [1, 1]. Usando esa serie, determinar el valor de



X (1)n
n=1
2 n2 + 1

279
Apuntes Mat023 (Segundo semestre 2014)

Solucion. Como

!
n n+1
X 2 (1) X 2n (1)
ex = (sinh 1) 1 + cos (nx) + sin (nx)
n=1
2 n2 + 1 n=1
2 n2 + 1

evaluando en x = 0 que es un punto de continuidad se obtiene


+
!
X 2 (1)n
1 = (sinh 1) 1 +
n=1
2 n2 + 1

as
+
(1)n
  X
1 1
1 =
2 sinh 1 n=1
2 n2 + 1

Note que si f, f 0 SC [a, b] la serie de Fourier


  X  
a0 X 2k 2k
+ ak cos x + bk sin x
2 k=1
ba k=1
b a

converge en cada punto del intervalo, si denotamos por


  X  
a0 X 2k 2k
SF (x) = + ak cos x + bk sin x
2 k=1
ba k=1
ba

del resultado anterior se tiene que SF (x0 ) = f (x0 ) en los puntos en los cuales f es continua,
2k 2k
 
pero se obtiene una propiedad adicional, dada que las funciones 1, cos ba x , sin ba x
son periodicas de periodo (b a) se sigue que
  X  
a0 X 2k 2k
SF (x) = + ak cos x + bk sin x
2 k=1
ba k=1
ba

es una funcion periodica de periodo (b a) y as SF (x) esta bien definida en todo R,

SF : RR
x SF (x)

donde SF : R R es una funcion periodica de periodo (b a) tal que



f x+
 
0 + f x0
SF (x0 ) = para x0 ]a, b[
2
f (b ) + f (a+ )
= para x0 = a, b
2

280
Apuntes Mat023 (Segundo semestre 2014)

Ejemplo 5.2.2. La serie de Fourier de



1 si x [0, ]
f (x) =
1 si x [, 0[
es  
2X 1 n
((1) 1) sin (nx)
n=1 n
entonces
 
2X 1 n
1 = ((1) 1) sin (nx) para x ]0, [
n=1 n
 
2X 1 n
1 = ((1) 1) sin (nx) para x ], 0[
n=1 n
1 + (1)
= 0 para x = 0, ,
2
y por periodicidad podemos decir por ejemplo que
 
2X 1 n
  
((1) 1) sin n 2 +
n=1 n 4
 
2X 1 n
 n 
= ((1) 1) sin
n=1 n 4
= 1
Ejemplo 5.2.3. Determinar la serie de Fourier de f C [2, 2] definida por

1 si 0 x 2
f (x) =
1 si 2 x < 0
Determine el valor al cual converge la serie en x = 6 y bosquejar un grafico de la serie
poniendo especial cuidado en los puntos de discontinuidad.
Solucion. La funcion es impar, esto permitira simplifica los calculos, la serie de Fourier
tiene la forma
  X  
a0 X 2nx 2nx
F (x) = + an cos + bn sin
2 n=1
4 n=1
4

a0 X  nx  X  nx 
= + an cos + bn sin
2 n=1
2 n=1
2
donde
2 2
Z
a0 = f (x) dx = 0
4 2
1 2
Z  nx 
an = f (x) cos dx = 0
2 2 2

281
Apuntes Mat023 (Segundo semestre 2014)

1 2
Z  nx 
bn = f (x) sin dx
2 2 2
Z 2  nx 
= sin dx
0 2
2  nx  2
= cos
n 2 0
2
= (cos (n) 1)
n
(1)n 1
= 2
n
n+1
(1) +1
= 2
n
as

!
n+1
X (1) +1  nx 
F (x) = 2 sin
n=1
n 2
por los teoremas vistos en clases, la serie es periodica de periodo 4 se sigue que

1 + (1)
F (6) = F (2) = =0
2
la grafica de la serie es:

1.

Series de Fourier de senos y cosenos

Definicion 5.3.1. Si f : [0, L] R es una funcion en SC [0, L], llamaremos:

282
Apuntes Mat023 (Segundo semestre 2014)

1. Extension par de f a la funcion



f (x) si x [0, L]
fp (x) =
f (x) si x [L, 0]

2. Extension impar de f a la funcion



f (x) si x [0, L]
fI (x) =
f (x) si x [L, 0[

Observacion 5.3.1. fp SC [L, L] es una funcion par tal que

fp (x) = f (x) para x [0, L]

y fI (x) SC [L, L] es una funcion impar tal que

fI (x) = f (x) para x [0, L]

Al desarrollar en serie de Fourier la funcion fp en [L, L] se tiene


   
a0 X kx X kx
fp (x) = + ak cos + bk sin
2 k=1
L k=1
L

donde
1 L
Z
a0 = fp (x) dx
L L
1 L
Z  
kx
ak = fp (x) cos dx para k N
L L L
1 L
Z  
kx
bk = fp (x) sin dx para k N
L L L

notemos que al ser fp una funcion par fp (x) cos kx kx


 
L
es par y fp (x) sin L
es impar,
luego

1 L 2 L
Z Z
a0 = fp (x) dx = fp (x) dx
L L L 0
2 L
Z
= f (x) dx
L 0

283
Apuntes Mat023 (Segundo semestre 2014)

1 L
Z  
kx
ak = fp (x) cos dx para k N
L L L
2 L
Z  
kx
= fp (x) cos dx para k N
L 0 L
2 L
Z  
kx
= f (x) cos dx para k N
L 0 L
y
Z L  
1 kx
bk = fp (x) sin dx para k N
L L L
= 0

as la serie es de la forma
 
a0 X kx
fp (x) = + ak cos
2 k=1
L

con
2 L
Z
a0 = f (x) dx
L 0
2 L
Z  
kx
ak = f (x) cos dx para k N
L 0 L

de manera similar, si al desarrollar en serie de Fourier la funcion fI en [L, L] se tiene


   
a0 X kx X kx
fI (x) = + ak cos + bk sin
2 k=1
L k=1
L

donde
1 L
Z
a0 = fI (x) dx
L L
1 L
Z  
kx
ak = fI (x) cos dx para k N
L L L
1 L
Z  
kx
bk = fI (x) sin dx para k N
L L L

notemos que al ser fI una funcion impar fI (x) cos kx kx


 
L
es impar y fI (x) sin L
es par,
luego
1 L
Z
a0 = fI (x) dx = 0
L L

284
Apuntes Mat023 (Segundo semestre 2014)

1 L
Z  
kx
ak = fI (x) cos dx para k N
L L L
= 0 para k N

1 L
Z  
kx
bk = fI (x) sin dx para k N
L L L
2 L
Z  
kx
= fI (x) sin dx para k N
L 0 L
2 L
Z  
kx
= f (x) sin dx para k N
L 0 L
as la serie es de la forma  
X kx
fI (x) = bk sin
k=1
L
con Z L  
2 kx
bk = f (x) sin dx para k N
L 0 L
como fp (x) = fI (x) = f (x) para x [0, L] se obtiene que es posible desarrollar f
SC [0, L] en series de la forma
 
a0 X kx
f (x) = + ak cos
2 k=1
L

donde
2 L
Z
a0 = f (x) dx
L 0
2 L
Z  
kx
ak = f (x) cos dx para k N
L 0 L
llamada serie cosenoidal de f o serie de cosenos de f y
 
X kx
f (x) = bk sin
k=1
L

donde Z L  
2 kx
bk = f (x) sin dx para k N
L 0 L
llamada serie senoidal de f o serie de senos de f .

285
Apuntes Mat023 (Segundo semestre 2014)

Ejemplo 5.3.1. Obtener la serie senoidal de


f (x) = cos x
en [0, ].
Solucion. Hacemos uso de la extension impar de f al intervalo [, ] entonces

cos x si x [0, ]
fI (x) =
cos (x) si x [, 0[

entonces
a0 X X
fI (x) = + an cos (nx) + bn sin (nx)
2 n=1 n=1
donde
1
Z
a0 = fI (x) = 0

Z
1
an = fI (x) cos (nx) dx = 0

2
Z
bn = cos x sin (nx) dx
0
si n = 1 entonces Z
2
b1 = cos x sin xdx = 0
0
pero
sin (x + nx) = sin x cos nx + sin nx cos x
sin (x nx) = sin x cos nx sin nx cos x
as
sin (x + nx) sin (x nx) = 2 cos x sin nx
luego
cos ((n + 1) x) cos ((1 n) x)
Z
+ =2 cos x sin nxdx
n+1 1n
luego
 
cos ((n + 1) x) cos ((1 n) x)
Z
+ = 2 cos x sin nxdx
n+1 1n
0
   0
cos ((n + 1) ) cos ((1 n) )
Z
1 1
+ + = 2 cos x sin nxdx
n+1 1n n+1 1n 0
(1)n (1)n
Z
2n
+ + 2 = 2 cos x sin nxdx
n+1 n1 n 1 0
n Z
((1) + 1) 2n
= 2 cos x sin nxdx
n2 1 0

286
Apuntes Mat023 (Segundo semestre 2014)

se sigue
+ 
((1)n + 1) n
 X 
2
fI (x) = sin (nx)
n=2 n2 1
as en [0, ] obtenemos la representacion
+ 
((1)n + 1) n
 X 
2
cos x = sin (nx)
n=2 n2 1

la grafica de la aproximacion (en verde)


100 
((1)n + 1) n
 X 
2
sin (nx)
n=2 n2 1
es

en rojo esta la grafica de coseno en el intervalo deseado.

Ejemplo 5.3.2. Obtener la serie cosenoidal de la funcion

f (x) = x

para x [0, 4] .
Solucion. En este caso hacemos uso de la extension par de f al intervalo [4, 4] se tiene

x si x [0, 4]
fp (x) =
(x) si x [4, 0[

287
Apuntes Mat023 (Segundo semestre 2014)

entonces    
a0 X 2nx X 2nx
fp (x) = + an cos + an sin
2 n=1
8 n=1
8
donde
2 4
Z
a0 = fp (x) dx
8 4
1 4
Z
= xdx = 4
2 0

1 4
Z  nx 
an = fp (x) cos dx
4 4 4
1 4
Z  nx 
= x cos dx
2 0 4
8 ((1)n 1)
=
2 n2
y Z 4
1  nx 
bn = fp (x) sin dx = 0
4 4 4
se sigue

8 ((1)n 1)
X   nx 
fp (x) = 2 + cos
n=1
2 n2 4
en particular en [0, 4] se tiene

((1)n 1)
X   nx 
x=2+8 cos
n=1
2 n2 4

Ejemplo 5.3.3. Sea f : [0, ] R definida por f (x) = x ( x)

1. Encontrar una funcion F periodica y par tal que F (x) = f (x) para x [0, ] y
desarrollar en serie de Fourier F .

2. Encontrar una funcion G periodica e impar tal que G (x) = f (x) para x [0, ] y
desarrollar en serie de Fourier G.

3. Use los puntos anteriores para probar



X (1)n+1 2 X (1)n+1 3
= y =
n=1
n2 12 n=1 (2n 1)3 32

288
Apuntes Mat023 (Segundo semestre 2014)

Solucion. Por punto pero calculando las sumas pedidas.

1. F es la serie de cosenos

a0 X
F (x) = + an cos (nx)
2 n=1

donde
2
Z
2
a0 = x ( x) dx =
0 3

2 ((1)n + 1)
Z
2
an = x ( x) cos (nx) dx =
0 n2
as
2 X ((1)n + 1)
F (x) = 2 cos (nx)
6 n=1
n2
si consideramos la suma sobre los pares (en los impares los coeficientes son nulos)

2 X 2
F (x) = 2 cos (2nx)
6 n=1
(2n)2

2 X cos (2nx)
=
6 n=1
n2

evaluando en 2
obtenemos


    2 X cos 2n 2
=
2 2 6 n=1
n2

2 X (1)n
=
6 n=1
n2

esto es

2 2 X (1)n+1
=
4 6 n=1
n2

2 X (1)n+1
=
12 n=1
n2

2. G es la serie senoidal
X
G (x) = bn sin (nx)
n=1

289
Apuntes Mat023 (Segundo semestre 2014)

donde
Z
2
bn = x ( x) sin (nx) dx
0
2
= (2 (1)n 2)
n3
4 ((1)n 1)
=
n3
as 
4 ((1)n 1)
X 
G (x) = sin (nx)
n=1
n3
note que ahora en los pares los coeficientes son nulos, al considerar en los impares
tenemos
8 X sin ((2n 1) x)
G (x) =
n=1 (2n 1)3

evaluando en 2
tenemos

8 X sin (2n 1) 2
   

=
2 2 n=1 (2n 1)3

8 X (1)n+1
=
n=1 (2n 1)3

pues  
sin (2n 1) = (1)n+1
2
as
2 8 X (1)n+1
=
4 n=1 (2n 1)3
de donde obtenemos
3 X (1)n+1
=
32 n=1 (2n 1)3

Ejemplo 5.3.4. Sea f (x) = exp ( [x]) definida en [0, 2]. Obtenga su serie de Fourier de
cosenos y use la serie para calcular el valor de

X (1)n1
n=1
2n 1

Obs.: [x] es la parte entera de x.

290
Apuntes Mat023 (Segundo semestre 2014)

Solucion. Tenemos que encontrar la serie de cosenos, usamos la extension par


 
a0 X 2nx
fp (x) = + an cos
2 n=1
4

donde
2 2
Z
a0 = fp (x) dx
4 2
Z 2
= f (x) dx
0
Z 2
= exp ( [x]) dx
0
Z 1 Z 2
= 1dx + edx
0 1
1
= e +1

y
2 2
Z  
2nx
an = fp (x) cos dx
4 2 4
Z 2  nx 
= exp ( [x]) cos dx
0 2
Z 1  nx  Z 2  nx 
= cos dx + e1 cos dx
0 2 1 2
 
1 1 1 1 2 1
= 2e sin n 2e sin n + sin n
n 2 n 2
2  n 
e1 1

= sin
n 2
luego

e1 + 1 X 2  n   nx 
1 e1 cos

fp (x) = + sin
2 n=1
n 2 2
evaluando en x = 0 se tiene

e1 + 1 X 2  n 
1 e1

1= + sin
2 n=1
n 2

de donde obtenemos
e1 + 1 2 (1 e1 ) X sin n
2
1 =
2 n=1
n
luego

12 12 e1 sin n

X
2
=
2 (1 e1 ) n=1
n

291
Apuntes Mat023 (Segundo semestre 2014)

luego

X sin n2
=
4 n=1
n
si n es par sin n
2
es cero, se sigue
(2n1)
X sin 2
=
4 n=1
2n 1
pero
(2n 1)
sin = (1)n+1
2
es decir
X (1)n+1
=
4 n=1
2n 1

Derivacion e integracion de Series de Fourier

Si para x ], [

X 2 (1)k+1
x= sin (kx)
k=1
k
es cierto que

X
1= 2 (1)k+1 cos (kx)
k=1

para x ], [? En otras palabras es posible derivar termino a termino una serie de


Fourier y asegurar que converge a la derivada de la funcion?, la respuesta es no, ya que
2 (1)k+1 no corresponde al coeficiente de Fourier de la funcion, el cual tiene que cumplir

lm Ck = 0
k

Para que sea valida la derivacion termino a termino tenemos el siguiente resultado:

Teorema 5.4.1. Si f C [a, b] , f (a) = f (b) y f 0 SC [a, b] entonces la serie de Fourier


para f 0 puede ser obtenida derivando termino a termino la serie de f . La serie obtenida
converge puntualmente a f 0 (x) en los puntos en los cuales f 00 existe.

para la integracion de las series tenemos el siguiente resultado:

Teorema 5.4.2. Si f SC [a, b] y


    
a0 X 2nx 2nx
f (x) = + ak cos + bk sin (media)
2 n=1
b a b a

292
Apuntes Mat023 (Segundo semestre 2014)

entonces
x
(x a)
Z
f (t) dt = a0
a 2

X Z x  
2nt
+ ak cos dt
n=1 a ba
Z x  
X 2nt
+ bk sin dt (media)
n=1 a b a

(Esto es, es posible integrar la serie termino a termino)

Ejercicios del captulo

1. Suponga que
 
X kx
x (L x) = ak sin
k=1
L
para x [0, L]. Haciendo uso de la propiedad de ortogonalidad de la familia
sin kx
 
L kN
en C[0, L] encontrar los coeficientes de la serie.

2. Muestre que en C[a, b] = V se cumple

a) f, g V
kf gk2 + kf + gk2 = 2 kf k2 + 2 kgk2
b) Si hf, gi = hf, hi para todo f V entonces g = h.

3. Muestre que para todo f C [, ]


Z
lm sin (nx) f (x) dx = 0
n

Ind.: Identidad de Parseval.

4. (Calculando Productos) Calcular para n, m N


Z Z
sin (mx) sin (nx) dx, sin (nx) dx

Z Z
cos (nx) dx, cos (nx) cos (mx) dx

Z
cos (nx) sin (mx) dx

Que dicen estos calculos del conjunto B = {1, cos (nx) , sin (nx) con n N} en
SC[, ]?

293
Apuntes Mat023 (Segundo semestre 2014)

5. (De lo general a lo particular) En SC [a, b] hemos definido el producto interior


Z b
hf, gi = f (x) g (x) dx
a

y enunciado que el conjunto


     
2nx 2nx
C = 1, cos , sin con n N
ba ba

es una conjunto ortogonal. Escribir en los casos especiales SC[, ], SC[L, L] y


SC[0, L]

6. (Calculando series de Fourier) Encontrar las series de Fourier para las funciones
siguientes en SC[, ]

a) x + sin x
b) ex

1 si x 0
c)
x2 si x > 0

x si x 0
d)
x3 si x > 0
e) sin3 x

7. (La identidad de Parseval) Utilizar la serie de Fourier de f (x) = x en [, ]


para calcular el valor de

X 1
k=1
k2

8. (Funciones pares e impares) Clasificar las siguientes funciones en par, impar o


ninguna de las anteriores

A) tan x
2
B) xex
x+1
C)
x1
D) ln |x|
E) arcsin x
F) f (|x|) definida en [1, 1] donde f : R R es una funcion cualquiera.
G) x cos x cos 2x

294
Apuntes Mat023 (Segundo semestre 2014)

9. (Parte par e impar de una serie) En Mat021 se mostro que toda funcion f :
[a, a] R (donde a R+ ) se puede descomponer en su parte par mas su parte
impar, esto es
f (x) = fp (x) + fi (x)
donde
f (x) + f (x)
fp (x) =
2
f (x) f (x)
fi (x) =
2
Suponiendo que no hay problemas de convergencia, encontrar las partes par e impar
de
a0 X
+ (ak cos (kx) + bk sin (kx))
2 k=1
donde ak , bk son constantes.

10. (Convergencia puntual) Encontrar el desarrollo en serie de Fourier de:



1 < x < 0
f (x) =
1 0<x<
y mostrar que la serie evaluada en x = 0 converge a 0. Utilizando esta serie mostrar
que
1 1 1
= 1 + +
4 3 5 7
11. (Series de Fourier en intervalos arbitrarios) Encontrar el desarrollo en serie de
Fourier para las siguientes funciones:

A) f (x) = x para x ], [
B) f (x) = ex para x ]0, 2[

1 < x < 0
C) f (x) =
1/2 0 < x <
D) f (x) = |sin x| para x ]2, 2[
E) f (x) = x (L x) para x ]0, L[

12. (Series de Fourier y series numericas) Encontrar el desarrollo en serie de Fourier


de la funcion
f (x) = cos (x) para x ], [
para todos los valores de R. Usar esta serie para demostrar que

!
1 1 X 2
cot () =
k=1 k 2 2
siempre que 6 Z.

295
Apuntes Mat023 (Segundo semestre 2014)

13. (Combinaciones lineales de series) Si f, g SC[a, b] son tales que


    
a0 X 2kx 2kx
f (x) = + ak cos + bk sin
2 k=1
b a ba

y
    
A0 X 2kx 2kx
g (x) = + Ak cos + Bk sin
2 k=1
b a ba

A) Determinar la serie de Fourier de f + g.


B) Es posible mostrar que las series

1 2 X sin (2k 1) x
+
2 k=1 2k 1

y

2 X (1)k+1
sin (kx)
k=1 k
son las series de Fourier de las funciones

0 < x < 0
f1 (x) =
1 0<x<
y
x
f2 (x) = para x ], [

respectivamente, usar estas series y el la parte anterior del ejercicio para obtener
las series de las siguientes funciones:

1/2 < x < 0
1) f (x) =
1/2 0 < x <

x + < x < 0
2) f (x) =
x 0<x<

x < x < 0
3) f (x) =
x + 2 0 < x <

14. (Ojo serie truncada) Cual es el desarrollo en serie de Fourier de f (x) = 2 +


5 cos (3x) 4 sin (7x) en SC[, ]?.

15. (Para trigonometricas mejor identidades) Encontrar la serie de Fourier de


f (x) = cos4 x en SC[, ]

16. (Extensiones pares e impares) Encontrar las extensiones pares e impares de las
siguientes funciones y trazar su grafica:

296
Apuntes Mat023 (Segundo semestre 2014)

A) f (x) = 1 definida en ]0, 5[


B) f (x) = x2 definida en ]0, [
C) f (x) = sin (x) definida en 0, 4
 

D) f (x) = ex definida en ]0, [

17. (Series de senos) Encontrar la serie de Fourier de senos de las funciones

A) f (x) = cos x para x ]0, [


B) f (x) = ex para x ]0, [
C) f (x) = x2 para x ]0, [
D) f (x) = x2 x para x ]0, 1[

18. (Series de cosenos) Encontrar la serie de Fourier de cosenos de las funciones

A) f (x) = sin x para x ]0, [


B) f (x) = ex para x ]0, [
C) f (x) = x2 para x ]0, [
D) f (x) = x x3 para x [0, 4]

19. (Derivando Series) Si para x ], [



X 2 (1)k+1
x= sin (kx)
k=1
k

Es cierto que

X
1= 2 (1)k+1 cos (kx)
k=1

para x ], [? En otras palabras es posible derivar termino a termino una serie de


Fourier y asegurar que converge a la derivada de la funcion?. (Obs: Para poder derivar
una serie de Fourier y garantizar convergencia basta que la funcion sea continua
f (a) = f (b) y f 0 , f 00 SC[a, b] derivar por ejemplo la serie de Fourier de |x| para
x ], [)

20. (Cual serie usar?) Calcular el valor de



X 1 X 1 X 1
, ,
k=1
k 2 k=1 k 4 k=1 k 6

usando series de Fourier adecuadas.

297
Parte II

Calculo diferencial en varias variables

298
Captulo 6 : Elementos de topologa de Rn

El espacio euclidiano Rn

Definicion 6.1.1. Se define el espacio ndimensional sobre el conjunto de los numeros


reales por:
Rn = {x = (x1 , x2 , . . . , xn ) : xi R, i = 1, 2, . . . , n}
junto con la suma en Rn dada por:

x + y = (x1 , x2 , . . . , xn ) + (y1 , y2 , . . . , yn )
= (x1 + y1 , x2 + y2 , . . . , xn + yn )

y la multiplicacion por escalar definida por:

x = (x1 , x2 , . . . , xn )
= (x1 , x2 , . . . , xn )

para todos x, y Rn y R.

Teorema 6.1.1. El espacio Rn es un espacio vectorial real de dimension n.

Producto interno y norma

Definicion 6.2.1. Sean x = (x1 , . . . , xn ) e y = (y1 , . . . , yn ) en Rn . Se define el producto


interno o producto escalar de x e y como el numero real:
n
X
hx, yi = xi yi
i=1

Observacion 6.2.1. El producto interno es el que confiere la nocion de distancia y de


perpendicularidad al espacio Rn .

Proposicion 6.2.1. Sean x, y, z Rn y R, entonces:

1. Bilinealidad: hx + y, z i = hx, zi + hy, zi y hx, y + zi = hx, yi + hx, zi

2. Simetra: hx, yi = hy, xi

3. Definido positivo: hx, xi 0 y hx, xi = 0 x = 0.

299
Apuntes Mat023 (Segundo semestre 2014)

Definicion 6.2.2. Sea x = (x1 , . . . , xn ) Rn . Se define la norma (euclidiana) de x


como el numero real:
p
kxk = hx, xi
( n )1/2
X
= x2i
i=1

Proposicion 6.2.2. Sean x Rn y R, entonces:

1. kxk 0

2. kxk = || kxk

Teorema 6.2.1 (Desigualdad de Cauchy-Schwarz). Sean x, y Rn , entonces:

|hx, yi| kxk kyk

Demostracion. Consideremos : R R definida por:

() = hx + y, x + yi

Note que:
() = kxk2 + 2 hx, yi + 2 kyk2
Ahora bien, () 0 es equivalente a que el discriminante de la expresion cuadratica
anterior sea negativa o cero. Es decir:

4 hx, yi2 4 kxk2 kyk2 0

Luego, extrayendo raz cuadrada se obtiene:

|hx, yi| kxk kyk

Observacion 6.2.2. La nocion de perpendicularidad caracterstica de los espacios eucli-


dianos se obtiene de la desigualdad de Cauchy-Schwarz. En efecto, sean x, y Rn r {0},
luego:
|hx, yi| hx, yi
|hx, yi| kxk kxk 1 1 1
kxk kyk kxk kyk
Por tanto, debe existir un angulo [0, ) tal que:
hx, yi
cos =
kxk kyk
O bien, en su forma clasica:
hx, yi = kxk kyk cos

300
Apuntes Mat023 (Segundo semestre 2014)

Definicion 6.2.3. Sean x, y Rn . Se define el angulo entre x e y como:


hx, yi
(x, y) = arc cos
kxk kyk

Ademas, diremos que x e y son perpendiculares u ortogonales si (x, y) = 0. En tal


caso, se anota x y.

Teorema 6.2.2 (Pitagoras). Sean x, y Rn tales que x y, entonces:

kx + yk2 = kxk2 + kyk2

Teorema 6.2.3. Sean x1 , x2 , . . . , xm Rn r {0} con xi xj , i =


6 j y m n, entonces
x1 , x2 , . . . , xm son linealmente independientes.

Observacion 6.2.3. Otra consecuencia de la desigualdad de Cauchy-Schwarz es la de-


sigualdad triangular. La desigualdad triangular es una propiedad muy importante de la
norma euclidiana y es la piedra angular de la nocion de distancia sobre el espacio Rn .

Teorema 6.2.4 (Desigualdad triangular). Sean x, y Rn , entonces:

kx + yk kxk + kyk

Demostracion. Por la desigualdad de Cauchy-Schwarz:

kx + yk2 = hx + y, x + yi
= kxk2 + 2 hx, yi + kyk2
kxk2 + 2 kxk kyk + kyk2
(kxk + kyk)2

Lo cual implica el resultado.

Definicion 6.2.4. Sean x, y Rn . Se define la distancia entre x e y como el numero


real dado por:
d (x, y) = kx yk

Teorema 6.2.5. Sean x, y, z Rn , entonces:

1. d (x, y) 0; d (x, y) = 0 x = y

2. d (x, y) = d (y, x)

3. d (x, z) d (x, y) + d (y, z) (desigualdad triangular)

Observacion 6.2.4. La nocion de norma euclidiana se puede generalizar al concepto de


norma, como una funcion mas general N : Rn R la cual debe cumplir:

301
Apuntes Mat023 (Segundo semestre 2014)

1. N (x) 0

2. N (x) = || N (x)

3. N (x + y) N (x) + N (y)
A modo de ejemplo, las normas mas comunes en Rn son:

kxk = max {|x1 | , |x2 | , . . . , |xn |}


n
X
kxk1 = |xi |
i=1
( n )1/p
X
kxkp = |xi |p
i=1

Ademas, se tiene que:


lm kxkp = kxk
p

As:

Teorema 6.2.6. Sean N y N1 dos normas cualesquiera sobre Rn . Entonces, existen


constantes , > 0 tales que:

N1 (x) N (x) N1 (x)

para todo x Rn .

Elementos de topologa de Rn

Definicion 6.3.1. Sean a Rn y > 0. Llamaremos bola abierta de radio con


centro en a al conjunto definido por:

B (a, ) = {x Rn : kx ak < }

Ejemplo 6.3.1. Si n = 1, entonces:

B (a,) = (a , a + )

302
Apuntes Mat023 (Segundo semestre 2014)

Si n = 2 y a = (x0 , y0 ), entonces:
 q 
2 2 2
B (a, ) = (x, y) R : (x x0 ) + (y y0 ) <

y si n = 3 y a = (x0 , y0 , z0 ) entonces
 q 
3 2 2 2
B (a, ) = (x, y, z) R : (x x0 ) + (y y0 ) + (z z0 ) <

303
Apuntes Mat023 (Segundo semestre 2014)

Definicion 6.3.2. Sea U Rn . Diremos que a es un punto interior de U si:

> 0, B (a, ) U

Llamaremos interior de U al conjunto de todos los puntos interiores de U , este conjunto


es representado por U .

Ejemplo 6.3.2. Muestre que (1, 2) es punto interior de

U = (x, y) R2 : x + y 1


pero (1,0) no lo es.

304
Apuntes Mat023 (Segundo semestre 2014)

Solucion. El conjunto U corresponde a la region de la figura

notemos que la distancia de (1, 2) a la recta x + y = 1 es

|1 + 2 1| 2 4
= 5>
1+4 5 5
4
se sigue que existe = 5
tal que
 
4
B ((1, 2) ; ) = B (1, 2) ; U
5

305
Apuntes Mat023 (Segundo semestre 2014)

para el punto (1, 0) note que (1, 0) U pero para todo > 0, B ((1, 0) ; ) 6 U toda bola
abierta con centro (1, 0) contiene puntos que no pertenecen a U, por ejemplo el punto
1, 2


Definicion 6.3.3. Sea U Rn . Diremos que U es un conjunto abierto en Rn , o


simplemente abierto en Rn si U = U .
Observacion 6.3.1. U es abierto si:
a U, > 0, B (a, ) U
Ejemplo 6.3.3. Los conjuntos y Rn son abiertos.
Ejemplo 6.3.4. Un conjunto finito A Rn no es abierto.
Teorema 6.3.1. Toda bola abierta es un conjunto abierto.
Demostracion. Sean a Rn y > 0. Considere x B (a, ) y:
= kx ak > 0
Sea, ahora, y B (x, ). Luego, ky xk < y ademas:
ky ak = k(y x) + (x a)k
ky xk + kx ak
< + kx ak
= kx ak + kx ak =
As, ky ak < , y por tanto, y B (a, ). Esto implica que, B (x, ) B (a, ). Es decir,
que todo punto x B (a, ) es interior.

306
Apuntes Mat023 (Segundo semestre 2014)

Ejemplo 6.3.5. Todo intervalo abierto en R es un conjunto abierto.


Ejemplo 6.3.6. Sea:
U = (x, y) R2 : x > 2


Entonces, U es abierto.
Ejemplo 6.3.7. Si U = {(x, y) R2 : x2 y}

entonces U = {(x, y) R2 : x2 < y}

se sigue que el conjunto no es abierto.

307
Apuntes Mat023 (Segundo semestre 2014)

Ejemplo 6.3.8. Considere el conjunto

A = (x, y) R2 : y > x2



muestre que (0, 2) A y (0, 0) A.

Solucion. El conjunto A es la parte achurada del dibujo siguiente

En esta figura se muestran los puntos (2, 0) y (0, 0)


Primero mostraremos que (0, 2) A. Consideremos r = 1 y mostremos

B ((0, 2) ; 1) A

si (x, y) B ((0, 2) ; 1) entonces x2 + (y 2)2 < 1 entonces

x2 x2 + (y 2)2 < 1 |x| < 1

tambien
(y 2)2 x2 + (y 2)2 < 1 |y 2| < 1

308
Apuntes Mat023 (Segundo semestre 2014)

se sigue 1 < y < 3 entonces


|x|2 = x2 < 1 < y
as
x2 < y

de donde obtenemos (x, y) A, as B ((0, 2) ; 1) A luego (0, 2) A

Ahora mostremos que (0, 0) A, sea r > 0 se tiene

B ((0, 0) , r) A 6=
B ((0, 0) , r) Ac 6=

esto es claro pues el punto (0, r/2) B ((0, 0) , r) A y (0, r/2) B ((0, 0) , r) Ac


Ejemplo 6.3.9. Sea U = {(x, y) : y = 2x} muestre que U = .

Solucion. Sabemos que U U , mostremos que un (x0 , y0 ) U no puede ser punto interior.
Si r > 0 entonces
B ((x0 , y0 ) , r) 6 U

309
Apuntes Mat023 (Segundo semestre 2014)

r r
 
note que el punto x0 , y0 + 2
B ((x0 , y0 ) , r) pero x0 , y0 + 2
6 U pues
r r
y0 + = 2x0 = 0
2 2
lo que es una contradiccion

Proposicion 6.3.1. Los conjuntos abiertos de Rn poseen las siguientes propiedades:


1. y Rn son abiertos.

2. La union arbitraria de conjuntos abiertos en Rn es un conjunto abierto en Rn .

3. La interseccion de un numero finito de conjuntos abiertos en Rn es un conjunto


abierto en Rn .
Ejemplo 6.3.10. Sea Hn = a n1 , b + n1 R, para cada n N. Notamos que cada Hn


es un conjunto abierto en R, pero:



\
H= Hn = [a, b]
n=1

310
Apuntes Mat023 (Segundo semestre 2014)

el cual no es abierto.

Definicion 6.3.4. Sean U Rn y a Rn . Diremos que a es un punto de adherencia


a U si:
> 0, B (a, ) U 6=
El conjunto de todos los puntos de adherencia de U se denomina la clausura de U y se
representa por U .

Observacion 6.3.2. Para todo U Rn , U U U .

Ejemplo 6.3.11. La bola cerrada con centro en a y radio esta dada por:

B (a, ) = {x Rn : kx ak }

Definicion 6.3.5. Sea U Rn . Diremos que U es un conjunto cerrado si U = U .

Ejemplo 6.3.12. Los conjuntos y Rn son cerrados.

Teorema 6.3.2. Sea U Rn . Entonces, U es cerrado en Rn , si y solo si, U C es un conjunto


abierto.

Definicion 6.3.6. Sean U Rn y a Rn . Diremos que a es un punto de acumulacion


de U si:
> 0, (B (a, ) r {a}) U 6=
El conjunto de todos los puntos de acumulacion de U se denomina el conjunto derivado
de U y se representa por U 0 .

Ejemplo 6.3.13. Sea U = (0, 1] {2, 3}. Luego, U 0 = [0, 1].

Ejemplo 6.3.14. Sea X = n1 : n N . Luego, X 0 = {0}. Note que lo anterior quiere




decir que hay infinitos puntos distintos de X suficientemente cercanos a 0.

Observacion 6.3.3. Si U Rn tal que U 0 6= , entonces U es infinito. As, para todo


conjunto finito A Rn , se tiene que A0 = .

Teorema 6.3.3. Sea U Rn . Entonces, U es cerrado en Rn , si y solo si, U 0 U .

Definicion 6.3.7. Sean U Rn y a Rn . Diremos que a es un punto frontera de U si:

> 0, B (a, ) U 6= B (a, ) U C 6=

El conjunto de todos los puntos frontera de U se llama la frontera de U y se anota como


U .

Observacion 6.3.4. Note que, desde la definicion, se obtiene que U = U U C .

311
Apuntes Mat023 (Segundo semestre 2014)

Ejemplo 6.3.15. Considere el conjunto

A = ]1, 2[ ]1, 3[

1. Muestre que A es abierto en R2 .


El conjunto corresponde a un rectangulo sin los bordes como se muestra en la figura.

Supongamos que (u0 , v0 ) A entonces 1 < u0 < 2 y 1 < v0 < 3 sea


1
r= mn {u0 1, 2 u0 , v0 1, 3 v0 }
2
(la menor distancia a los bordes dividida por dos) entonces

B ((u0 , v0 ) , r) ]1, 2[ ]1, 3[

312
Apuntes Mat023 (Segundo semestre 2014)

en efecto, si (x, y) B ((u0 , v0 ) , r) entonces


q
(x u0 )2 + (y v0 )2 < r

luego
|x u0 | < r x ]u0 r, u0 + r[
y
|y v0 | < r y ]v0 r, v0 + r[
como r = 21 mn {u0 1, 2 u0 , v0 1, 3 v0 } se sigue

u0 1
r < u0 1 1 < u0 r
2
2 u0
r < 2 u0 r + u0 < 2
2
v0 1
r < v0 1 2 < v0 r
2
3 v0
r < 3 v0 v0 + r < 3
2
se sigue

]u0 r, u0 + r[ ]1, 2[
]v0 r, v0 + r[ ]1, 3[

313
Apuntes Mat023 (Segundo semestre 2014)

as
(x, y) ]1, 2[ ]1, 3[
se sigue que todo punto de A es un punto interior luego es un conjunto abierto.

2. Determine A y A.
Desarrollo:
A = [1, 2] [1, 3]
corresponde al rectangulo con el borde y

A = {1} [1, 3] {2} [1, 3]


[1, 2] {1} [1, 2] {3}

son los bordes del rectangulo.

Ejemplo 6.3.16. La esfera con centro en a y radio > 0 se obtiene como:

S (a, ) = B (a, )
= {x Rn : kx ak = }

As, la esfera unitaria S n1 en Rn se obtiene como:

S n1 = B (0, 1)
= {x Rn : kxk = 1}

Definicion 6.3.8. Diremos que un conjunto X Rn es acotado si existe un numero real


M > 0 tal que:
kxk M
para todo x X.

Observacion 6.3.5. Note que la cota anterior es uniforme, es decir: sup kxk : x

X M.

Ejemplo 6.3.17. El conjunto

U = (x, y, z) R3 : x2 + (y 1)2 + z 2 1 (x 1)2 + y 2 + z 2 1




es acotado pues si (x, y, z) U entonces, de la primera ecuacion se obtiene

x2 + z 2 x2 + (y 1)2 + z 2 1

y de la segunda
y 2 (x 1)2 + y 2 + z 2 1

314
Apuntes Mat023 (Segundo semestre 2014)

se sigue
x2 + z 2 + y 2 2


as p
k(x, y, z)k = x2 + z 2 + y 2 2
se sigue
sup k(x, y, z)k 2
(x,y,z)U

en otras palabras h i
U B (0, 0, 0) , 2

U esta contenido en la bola cerrada de centro (0, 0, 0) y radio 2.

Definicion 6.3.9 (Heine-Borel). Un conjunto K Rn se dice compacto si es cerrado y


acotado.

Ejemplo 6.3.18. La bola cerrada B (a, ) es un conjunto compacto sobre en Rn . En efecto,


como B (a, ) es cerrado, basta verificar que es acotado. Note que para todo x B (x, ),
se tiene que:
kxk kx ak + kak < + kak

Ejemplo 6.3.19. Sean L Rn y K Rm conjuntos compactos, respectivamente. Entonces,


L K es compacto en Rn+m = Rn Rm

Definicion 6.3.10. Un conjunto A Rn se llama disconexo si existen dos conjuntos


abiertos U y V de Rn tales que:

1. U A 6= V A 6= .

2. (U A) (V A) = .

3. (U A) (V A) = A

Por otro lado, un conjunto A Rn se dice conexo si no es disconexo.

Ejemplo 6.3.20. N y Q son disconexos.

Ejemplo 6.3.21. R es conexo.

Ejemplo 6.3.22. Todo intervalo real es conexo.

Teorema 6.3.4. Sean A Rn un conjunto conexo y C Rn tales que A C A,


entonces C es conexo.

315
Apuntes Mat023 (Segundo semestre 2014)

Ejercicios del captulo

1. Muestre que un plano en R3 es cerrado y su interior es vaco.

2. En los siguientes conjuntos, determine cuales son: Abiertos, Cerrados. Determine en


cada caso el interior, la clausura y frontera del conjunto.
a) {(x, y) R2 / y = 2x} b) {x Rn : kxk 1}
c) {(x, y, z) R3 / z < x2 + y 2 } d) {(x, y, z) R3 / x = 0}
e) Una recta en R2 o R3 f) Un plano en R3
g) {(x, y, z) R3 / x = y = z = (1)n , n N} h) {(x, y, z) R3 / x < y}
1
i) {(x, y) R2 / x = + 1, n N} j) {x Rn : kxk = 1}
n

3. Demuestre rigurosamente que:

a) (0, 1) no es un punto interior de A = {(x, y) R2 / |x| + |y| < 1}.


b) (0, 0) es un punto interior de A.
c) (1, 0) es un punto de acumulacion de A.
 
1 1
d) , es un punto de frontera de A.
2 2

4. Probar que B (a; r) = {x Rn : kx ak < r} es abierto en Rn .

5. Si A es un conjunto cerrado que contiene cada numero racional r [0, 1], probar que
[0, 1] A.

6. Encuentre un conjunto en R2 que no sea Abierto ni Cerrado.

7. Sea R = [1, 2] [3, 4[

a) Es Abierto?, Es Cerrado?
b) Determine R.

8. Si K = [0, 1] [0, 1] {(2, 2)}

a) Es Abierto?
b) Es cerrado?

9. Sea {Ai : i I} una coleccion de subconjuntos abiertos de Rn muestre que:

a) iI Ai es un conjunto abierto (la union arbitraria de abiertos es un abierto)

316
Apuntes Mat023 (Segundo semestre 2014)

b) iI 0 Ai es un conjunto abierto, donde I 0 I es un conjunto finito (la interseccion


finita de abiertos es un abierto)
c) Muestre que la interseccion arbitraria de abiertos no es necesariamente un
abierto, para ello considere en Rn la coleccion de bolas abiertas B ; n1 con


n N.

10. Encontrar A , A, A0 y A si

A = (x, y) R2 : x > 0, y > 0 {(1, 1)}




11. Si T : Rn Rm es una transformacion lineal y P Rn , r > 0 En que consiste el


conjunto T (B (P ; r))?Es abierto?

12. Muestre que p A si y solo si existe una sucesion de puntos en A, digamos {an }nN
tal que an p (esto es
lm d (an , p) = 0
n

en el sentido de los limites de sucesiones en R)

13. Considere el subconjunto de R


 
1 1
U = xR:x= + para algun n, m N
n m

encontrar U , U , U 0 y U .

14. Muestre que si U y V son abiertos en R entonces U V es abierto en R2 .

15. Sean U, V subconjuntos de Rn probar:

a) U V U V
b) U V = U V

16. Si U Rn muestre que:

a) U y U son disjuntos y U = U U
b) Muestre que la frontera es vaca si y solo si el conjunto es abierto y cerrado.
c) Muestre que U es abierto si y solo si U = U U

17. Si U es abierto Es verdad que U = U ?

18. Muestre que todo abierto de R2 es una union numerable de cajas abiertas de la forma
]a, b[ ]c, d[ y tambien es una union numerable de bolas abiertas.

317
Captulo 7 : Funciones de varias variables

Definiciones basicas

Definicion 7.1.1. Consideremos una funcion f : U Rn Rm . Diremos que:

1. f es una funcion real de varias variables si n 2 y m = 1.

2. f es una funcion vectorial de una variable real si n = 1 y m 2.

3. f es una funcion vectorial de varias variables si n 2 y m 2.

En general, las funciones reales de varias variables se denotan con letras minusculas,
como por ejemplo: f, g, h, etc. y las funciones vectoriales se anotan con letras mayusculas,
tales como: F, G, H, etc.

Ejemplo 7.1.1. Son funciones reales de varias variables las siguientes funciones:

1. f : R2 R, (x, y) 7 f (x, y) = x2 + 2y.

2. g : R3 R, (x, y, z) 7 g (x, y, z) = sin (xyz).


1
(x, y, z) 7 h (x, y, z) = e 2 (x ).
2 +y 2 +z 2
3. h : R3 R,

Ejemplo 7.1.2. Un ejemplo importante de funcion de varias variables corresponde a la


kesima proyeccion sobre R de un vector x = (x1 , x2 , . . . , xn ) Rn . Mas precisamente, se
define la kesima proyeccion de x Rn como la funcion k : Rn R definida por:

k (x) = k (x1 , x2 , . . . , xn ) = xk

Esta funcion, en particular, nos permitira representar funciones tales como:


2x2 + y 3
f (x, y, z) =
x2 + z 2 + 1
como algebra de proyecciones. Es decir, podemos escribir f (x, y, z) en la forma:

2 12 (x, y, z) + 23 (x, y, z)
f (x, y, z) =
12 (x, y, z) + 32 (x, y, z) + 1
Ejemplo 7.1.3. Son funciones vectoriales las siguientes funciones:

1. F : R R2 , t 7 F (t) = (C1 et , C2 et ), con C1 , C2 R.

2. G : R R3 , t 7 G (t) = ( cos t, sin t, t), con , > 0.

3. H : R2 R3 , (x, y) 7 H (x, y) = (ln (x2 + y 2 + 1) , cos (x + y) , x + 3y).

318
Apuntes Mat023 (Segundo semestre 2014)

Ejemplo 7.1.4. Notamos que, si en el ejemplo (3) anterior definimos:

f1 (x, y) = ln x2 + y 2 + 1


f2 (x, y) = cos (x + y)
f3 (x, y) = x + 3y

entonces:
H (x, y) = (f1 (x, y) , f2 (x, y) , f3 (x, y))
donde
fi (x, y) = (i H) (x, y)

Ejemplo 7.1.5. Sean fi : Ui Rn R funciones de varias variables con dominio Ui , con


i = 1, 2, . . . , m. Suponga que:
m
\
U= Ui
i=1

Entonces, la transformacion F : U R Rm definida por:


n

F (x) = (f1 (x) , f2 (x) , . . . , fm (x)) , x = (x1 , x2 , . . . , xn ) U

es una funcion vectorial de varias variables. Las funciones reales de varias variables fi se
llaman funciones componentes de F.

Definicion 7.1.2. Sea f : D Rn R, (x1 , x2 , . . . , xn ) f (x1 , x2 , . . . , xn ). Llamaremos


dominio maximo de f al conjunto:

Dom ( f ) = {x Rn : f (x) R}

As mismo, el dominio maximo de F : D Rn Rm , x F (x) = (f1 (x) , f2 (x) , . . . , fm (x))


es dado por:
\m
Dom (F) = Dom ( fi )
i=1

Ejemplo 7.1.6. Hallar el dominio maximo de:


p
x2 + y 2 1
f (x, y) =
ln (|x| y)

Solucion. El dominio maximo en R2 para que esta expresion represente una funcion a
valores reales en tal conjunto es

Dom ( f ) = (x, y) R2 : x2 + y 2 1 0 |x| y > 0 |x| y 6= 1




319
Apuntes Mat023 (Segundo semestre 2014)

Ejemplo 7.1.7. Si   
sin (xyz) 1
F (x, y, z) = , y cos
xyz x
Determine el dominio maximo de F.

Solucion. El dominio maximo corresponde a

Dom (F) = (x, y, z) R3 : xyz 6= 0




Definicion 7.1.3. El recorrido de F : U Rn Rm se define como:

Rec ( F) = {y Rm : x U, y = F (x)}

Observacion 7.1.1. En general, para funciones de varias variables es difcil calcular el


recorrido. Sin embargo, puede ser util tomar restricciones. Esto es imagenes directas de
conjuntos adecuados.

Ejemplo 7.1.8. El recorrido de f : R2 R, (x, y) f (x, y) = xy 2 + yx2 es todo R. En


efecto, si

(x, y) R2 : x = y

T =
= {(x, x) : x R}

Entonces, f T = x x2 + x x2 = 2x3 , se sigue que f (T ) = R luego R =f (T ) Rec(f ) esto
implica rec (f ) = R.
x3 y 2
Ejemplo 7.1.9. El recorrido de f : R2 {(0, 0)} R, (x, y) f (x, y) = x2 +y 2
es R. En
efecto f (1, 0) = 0 luego 0 Rec(f ). Sea

T = (x, y) R2 : y = x, x 6= 0


entonces
x5 x3
f (x, y) T = 2 =

2x 2
se sigue
R {0} = f (T )
as
R {0} = f (T ) Rec (f ) R
y 0 Rec(f ) se sigue
Rec (f ) = R

320
Apuntes Mat023 (Segundo semestre 2014)

Graficos, conjuntos de nivel y trazas

Definicion 7.2.1. Sea f : U Rn R una funcion. Llamaremos grafico de f al conjunto


definido por:
Graf (f ) = (x, f (x)) Rn+1 : x U


Observacion 7.2.1. Si f : U R2 R es una funcion real de dos variables reales,


diremos que:
z = f (x, y) , (x, y) U
es una superficie en R3 .
Observacion 7.2.2. Respecto de lo anterior, hay algunas superficies clasicas que es
conveniente reconocer. Tales superficies son conocidas como superficies cuadricas.
Definicion 7.2.2. Una superficie cuadrica es una ecuacion del tipo:
P (x, y, z) = 0
donde P (x, y, z) es un polinomio de segundo grado en tres variables. En particular, se
consideran la superficies cuadricas en su forma normal, es decir, considerando las ecuaciones:
Ax2 + By 2 + Cz 2 + D = 0
o bien:
Ax2 + By 2 + Cz = 0
Observacion 7.2.3. Las superficies cuadricas mas importantes son las siguientes:
1. Esfera:
x2 + y 2 + z 2 = r 2

321
Apuntes Mat023 (Segundo semestre 2014)

2. Elipsoide:
x2 y2 z2
a2
+ b2
+ c2
= 1, a, b, c 6= 0

3. Hiperboloide de una hoja:


x2 y2 z2
a2
+ b2
c2
= 1, a, b, c 6= 0

322
Apuntes Mat023 (Segundo semestre 2014)

4. Hiperboloide de dos hojas:


2 y2 z2
xa2 + b2
c2
= 1, a, b, c 6= 0

5. Paraboloide:
x2 y2
a2
+ b2
= cz, a, b 6= 0 c > 0

323
Apuntes Mat023 (Segundo semestre 2014)

6. Paraboloide hiperbolico:
x2 y2
a2
b2
= cz, a, b 6= 0 c > 0

Definicion 7.2.3. Sea f : U Rn R y c R. Llamaremos conjunto de nivel c de f


al conjunto definido por:

Lc (f ) = {x U : f (x) = c} Rn

En particular, si n = 2 diremos que Lc (f ) es la curva de nivel c de f . Si n = 3, diremos


que Lc (f ) es la superficie de nivel c de f.

Observacion 7.2.4. Note que:

Lc (f ) = f 1 ({c})

Observacion 7.2.5. De los conjuntos anteriores, nos interesa el aspecto grafico para n = 2
y n = 3. La coleccion de los graficos superpuestos de un numero adecuado de curvas de
nivel o superficies de nivel para una funcion permite esbozar el grafico de la funcion dada.

Ejemplo 7.2.1. Las curvas de nivel c de la funcion f (x, y) = x2 + y 2 son circunferencias



de centro en (0, 0) y radio c para c > 0. Para c = 0 es solo un punto y para c < 0 es
conjunto vaco.
p
Ejemplo 7.2.2. Las curvas de nivel c de la funcion f (x, y) = x2 + y 2 son circunferencias
de centro en (0, 0) y radio c para c > 0. Para c = 0 es solo un punto y para c < 0 es
conjunto vaco.

324
Apuntes Mat023 (Segundo semestre 2014)

Observacion 7.2.6. Se puede observar que para los ejemplos anteriores las curvas de nivel
son, basicamente, las mismas, es decir, crculos concentricos desde el origen. En este caso,
las curvas de nivel no son suficiente para determinar el grafico de las funciones anteriores.
Debemos considerar la nocion de traza.

Definicion 7.2.4. Llamaremos traza de una superficie S : z = f (x, y) a la interseccion


de dicha superficie con alguno de los planos coordenados. Denotaremos las trazas de una
superficie S mediante los smbolos Ty y Tx , si la interseccion se efectua con los planos xz e
yz, respectivamente.

Ejemplo
p 7.2.3. Por ejemplo, las trazas de las superficies S1 : z = x2 + y 2 y S2 : z =
x2 + y 2 estan dadas por:

(x, y, z) Ty (x, y, z) S1 {(x, y, z) : y = 0} z = x2 , y=0

para S1 , y para S2 por:

(x, y, z) Ty (x, y, z) S1 {(x, y, z) : y = 0} z = |x| , y=0

En particular, son algunas trazas las que nos permiten distinguir los graficos de las
superficies S1 y S2 .

Ejemplo 7.2.4. Sea f : D R2 R definida por


p
x2 + y 2 1
f (x, y) = p
4 x2 y 2

1. Determine el dominio D de f .

(x, y) R2 : x2 + y 2 1 0 4 x2 y 2 > 0

D =
= (x, y) R2 : x2 + y 2 1 4 > x2 + y 2


= (x, y) R2 : 1 x2 + y 2 < 4


3
 
2. Demostrar que 2
D, 1, 3 D y (0, 0) 6 D.
,0
Claramente B 32 , 0 ; 41 D luego el punto es punto interior. Si r > 0 la bola
 
 
B 1, 3 ; r contiene puntos del conjunto y puntos del complemento y finalmente
B (0, 0) ; 12 D = .


3. Describir los conjuntos de nivel de f y graficar la funcion.

325
Apuntes Mat023 (Segundo semestre 2014)

Los conjuntos de nivel de f son


( p )
2
x +y 1 2
Nc = (x, y) R2 : p =c
4 x2 y 2
n p p o
2 2 2 2 2
= (x, y) R : x + y 1 = c 4 x y
= (x, y) R2 : x2 + y 2 1 = c2 4 x2 y 2
 

= (x, y) R2 : 1 + c2 x2 + 1 + c2 y 2 = 1 + 4c2
  

1 + 4c2
 
2 2 2
= (x, y) R : x + y =
1 + c2
q
2
son circunferencias de radio 1+4c 1+c2
y centro (0, 0). Por otro lado, si y = 0 se tiene

x2 1
z=
4 x2
que tiene por grafica

as la grafica es

326
Apuntes Mat023 (Segundo semestre 2014)

Ejercicios del captulo

1. Encontrar los dominios de las siguientes funciones:


p
2 y x2
a) f : D R R (x, y) 2
x y2
p
y x2 ln (1 x2 + y 2 )
b) f : D R2 R (x, y)
ln (x2 y 2 )
!
x x x2
c) f : D R2 R3 (x, y) , , p
x + y x + y 2 |x| x2 + y 2

2. Esbozar curvas de nivel y graficas de las siguientes funciones:


a) f : R2 R (x, y) x y + 2
b) f : R2 R (x, y) x2 + 9y 2
c) f : R2 R (x, y) max {|x| , |y|}

3. Describir el comportamiento de las curvas de nivel f (x, y) = c para c en R con:


a) f (x, y) = x2 + 4y 2 + 5 b) f (x, y) = 4 x2 y 2
f (x, y) = (x2 + y 2 ) e(x +y )
2 2
c) f (x, y) = |8 |x2 + y 2 4|| + 1 d)

4. Considere la funcion f (x, y) = x2 + y 2 y defina para 0, 2 el conjunto


 

S = (x, y, z) R3 : y = (tan ) x


ver la forma de la curva G (f ) S donde G (f


p) corresponde2 a la grafica de la funcion
2 2 x +y 2
f . Hacer lo mismo con la funcion f (x, y) = x + y + e .

5. Describir los conjuntos de nivel de las funciones:


a) f : R3 R (x, y, z) x2 y 2 z 2
b) f : R3 R (x, y, z) 4x2 y 2 + 9z 2
c) f : R3 R (x, y, z) z x2 4y 2

6. Representar en R3 los conjuntos:

y2
 
3 2
a) A = (x, y, z) R : z + =1
4
b) B = (x, y, z) R3 : x = y 2


c) B = (x, y, z) R3 : x2 + y 2 = 1 x + y + z = 0


327
Apuntes Mat023 (Segundo semestre 2014)

7. Utilizando coordenadas polares describir las curvas de nivel de la funcion


2xy

x2 + y 2 si (x, y) 6= (0, 0)


f (x, y) =


0 si (x, y) = (0, 0)

328
Captulo 8 : Lmites y continuidad

Definiciones

Definicion 8.1.1. Sean U Rn , a U 0 y f : U R una funcion. Diremos que el numero


real L es el lmite de f (x) cuando x tiende a a si:

> 0, > 0, 0 < kx akRn < = |f (x) L| < (8.1)

Lo anterior se representa mediante el smbolo:

lm f (x) = L
xa

Observacion 8.1.1. En particular, si f : U R2 R y (a, b) U 0 , la definicion de lmite


en (8.1) queda como:
q
> 0, > 0, 0 < (x a)2 + (y b)2 < = |f (x, y) L| <

y en smbolos:
lm f (x, y) = L
(x,y)(a,b)

Usualmente, a los lmites del tipo anterior, se les conoce como lmites dobles.

Ejemplo 8.1.1. Demuestre que:

lm (3x + 2y) = 4
(x,y)(2,1)

Solucion. Por demostrar que, dado cualquier > 0, existe > 0 tal que si:
q
0 < (x 2)2 + (y + 1)2 <

implica que:
|3x + 2y 4| <
En efecto, note que:

|3x + 2y 4| = |3 (x 2) + 2 (y + 1)|
3 |x 2| + 2 |y + 1|
< 3 + 2 = 5
q
Por tanto, dado > 0, existe = /5 tal que si 0 < (x 2)2 + (y + 1)2 < , entonces
|3x + 2y 4| < . Por tanto, se concluye que lm(x,y)(2,1) (3x + 2y) = 4.

329
Apuntes Mat023 (Segundo semestre 2014)

Ejemplo 8.1.2. Demuestre que:

x2 cos (x2 + y 2 )
lm p =0
(x,y)(0,0) x2 + y 2

Solucion. Note que si (x, y) 6= (0, 0)



x2 cos (x2 + y 2 ) |x|2
cos x2 + y 2

0 =

p p
x2 + y 2 x2 + y 2
|x| |x|
p
x2 + y 2
p p
x2 + y 2 x 2 + y 2
p
x2 + y 2
p
= x2 + y 2 =
q
por lo tanto, dado > 0 existe = tal que si 0 < (x 0)2 + (y 0)2 < entonces

x2 cos (x2 + y 2 )
0 < .

p 2
x + y2

Observacion 8.1.2. Es importante destacar que la nocion de lmite es un concepto que


puede tratarse de manera mas general que en el caso de funciones reales de varias variables.
En particular, el concepto de lmite se puede extender a funciones vectoriales considerando
la norma correspondiente al espacio de llegada. Mas precisamente, tenemos:

Definicion 8.1.2. Sean U Rn , a U 0 y F : U Rm una funcion vectorial. Diremos


que el vector L Rm es el lmite de F (x) cuando x tiende a a si:

> 0, > 0, 0 < kx akRn < = kF (x) LkRm <

Sin embargo, para nuestro propositos de calculo, es util el siguiente teorema:

Teorema 8.1.1. Sean U Rn , x U 0 y F : U Rm una funcion vectorial tal que:

F (x) = (f1 (x) , f2 (x) , . . . , fm (x))

Entonces, el vector L = (L1 , L2 , . . . , Lm ) Rm es el lmite de F (x) cuando x tiende a a,


si y solo si:
lm fi (x) = Li
xa

para todo i = 1, 2, . . . , m. En otras palabras, F (x) tiende al vector L cuando x tiende a


a, si y solo si, la convergencia se da en cada coordenada de F (x) a la correspondiente
coordenada de L.

330
Apuntes Mat023 (Segundo semestre 2014)

La demostracion de este resultado se sigue de la desigualdad

|fi (x) Li | k(f1 (x) , f2 (x) , . . . , fm (x)) (L1 , L2 , . . . , Lm )k


m
X
m |fi (x) Li |
i=1

Observacion 8.1.3. En vista del resultado anterior, centraremos nuestra atencion en


funciones reales de varias variables.
Teorema 8.1.2. Sean F : U Rn Rm una funcion vectorial y a U 0 . Suponga que:

lm F (x) = L lm F (x) = M
xa xa

entonces, L = M.
Demostracion. Sea > 0. Por hipotesis, existen 1 , 2 > 0 tales que:

0 < kx ak < 1 = kf (x) Lk <
2
y:

0 < kx ak < 2 = kf (x) M k <
2
Considerando, = mn {1 , 2 }, se tiene que:

kL M k = k(L f (x)) + (f (x) M )k


kf (x) Lk + kf (x) M k

< + =
2 2
Lo anterior implica que L = M .

Este resultado nos entregara una tecnica para demostrar la no existencia de ciertos
lmites.
Ejemplo 8.1.3. Determine si acaso existe el siguiente lmite:
sin (xy) + z
lm
(x,y,z)(0,0,0) x2 + y 2 + |z|

Solucion. Anotemos f (x, y, z) = xsin(xy)+z


2 +y 2 +|z| y consideremos el conjunto:

T = (x, y, z) R3 : x = y = 0 z =

6 0

Ahora bien: 
z 1 , z>0
f T = =
|z| 1 , z<0
sin(xy)+z
Por unicidad del lmite, se concluye que el lmite lm(x,y,z)(0,0,0) x2 +y 2 +|z|
no existe.

331
Apuntes Mat023 (Segundo semestre 2014)

Ejemplo 8.1.4. Sea f : R2 r {(0, 0)} R una funcion de dos variables definida por:
xy
f (x, y) =
x2 + y2
Considere:
Tm = (x, y) R2 : y = mx


con m 6= 0. Note que:


 
f Tm (x, y) = f (x, mx)
mx2
=
x2 + m 2 x2
m
=
1 + m2
Notamos que si (x, y) (0, 0) a traves de Tm , es decir, si consideramos el lmite
lm(x,y)(0,0) f (x, y), se tiene que:
(x,y)Tm

lm f (x, y) = lm f (x, mx)


(x,y)(0,0) x0
(x,y)Tm
m
= lm
x0 1 + m2
m
=
1 + m2
y por tanto, se puede concluir que el lmite de f (x, y) no existe, pues depende directamente
de la pendiente de la recta de aproximacion al origen y = mx.

Ejemplo 8.1.5. Considere el lmite


x2 y
lm
(x,y)(0,0) x4 + y 2

note que
x2 y x2 0
lm = lm = lm 0 = 0
(x,y)(0,0) x4 + y 2 x0 x4 + 02 x0
y=0

x2 y 02 y
lm = lm = lm 0 = 0
(x,y)(0,0) x4 + y 2 y0 04 + y 2 y0
x=0

si tomamos otra recta de la forma y = mx (con m 6= 0) entonces


x2 y x2 (mx)
lm = lm
(x,y)(0,0) x4 + y 2 x0 x4 + (mx)2
y=mx
mx
= lm 2 =0
x0 x + m2

332
Apuntes Mat023 (Segundo semestre 2014)

se sigue que por todas las rectas que contienen al origen el lmite es cero, note que estas
rectas cubren todos los puntos del plano pero esto NO implica que el lmite sea cero,
la razon de esto es que las rectas no son la unicas trayectorias por las cuales podemos
acercarnos al origen, en efecto, al tomar la trayectoria y = x2 se tiene
x2 y x2 x2
lm = lm
(x,y)(0,0) x4 + y 2 x0 x4 + x4
y=x2
1
= lm
x0 2
1
=
2
se sigue que el lmite no existe.
Lo anterior se puede generalizar introduciendo la idea de camino en Rn . Consideremos:
Definicion 8.1.3. Un camino en Rn es una funcion : I Rn , cuyo dominio es un
intervalo I R. Es decir:

(t) = (1 (t) , 2 (t) , . . . , n (t)) , tI

y cada funcion componente i : I R R es continua en I.


Definicion 8.1.4. Sean U Rn , a U 0 y un camino de la forma : [0, 1] U . Diremos
que (t) converge propiamente a a, cuando t 0+ si:

lm (t) = a
t0+

p
y (t) 6= a, para todo t (0, 1]. Anotamos lo anterior mediante el smbolo (t) a.
Teorema 8.1.3. Sean L R, f : U Rn R y a U 0 tales que:

lm f (x) = L
xa

entonces:
lm f ( (t)) = L
t0+
p
para todo camino : [0, 1] U tal que (t) a.
se sigue de este teorema, que si por dos caminos distintos el limite es diferente o bien,
por un camino no existe entonces el limite general no existe.
Ejemplo 8.1.6. Muestre que el lmite:
x3 + y 3
lm
(x,y)(0,0) x y

no existe

333
Apuntes Mat023 (Segundo semestre 2014)

Solucion. Note que:

x3 + y 3 x3 y 3 + 2y 3
f (x, y) = =
xy xy
2 2
 2y 3
= x + xy + y +
xy
Ahora bien, suponga que:
2y 3
=r
xy
luego:
2
x = y + y3
r
Defina:  
2 3
(t) = t + t , t
r
entonces
lm (t) = (0, 0)
t0
y
   2 !
2 3
2 2 3
lm f ( (t)) = lm t + t t + t + t + t +r
t0 t0 r r
= r

luego tomando r = 1 y r = 2 obtenemos lmites distintos. El lmite no existe. La tecnica


empleada en este caso se conoce como la tecnica de los conjuntos de nivel.

Observacion 8.1.4. Otro procedimiento habitual para investigar acerca de la existencia


o inexistencia de un lmite es transformar mediante un cambio de variables adecuado el
espacio R2 completo. Es decir, considerando el cambio de coordenadas:

x = r cos
y = r sin
xy
la funcion f (x, y) = x2 +y 2
queda como:

f (x (r, ) ; y (r, )) = f (r cos , r sin )


r2 cos sin
= 2
r cos2 + sin2


= cos sin

Ahora bien, notando que si (x, y) (0, 0) implica que r 0, se observa que el lmite
anterior no existe, pues depende del angulo de entrada al origen (0, 0).

334
Apuntes Mat023 (Segundo semestre 2014)

Ejemplo 8.1.7.
x3 y 2
lm 2 = 0
(x,y)(0,0) (x2 + y 2 )

Solucion. Usando coordenadas polares

x = r cos
y = r sin

se tiene
(r3 cos3 ) r2 sin2

x3 y 2
=
(x2 + y 2 )2 (r2 )2
= r cos3 sin2

se sigue
x3 y 2
= lm r cos3 sin2 = 0

lm 2
(x,y)(0,0) (x2 + y 2 ) r0
[0,2[

Note tambien que


x3 y 2
= r cos3 sin2
(x2 + y 2 )2
implica
x3 y 2

p
= r cos3 sin2 r = x2 + y 2
(x2 + y 2 )2

luego podemos aplicar el teorema de acotamiento para demostrar que el lmite existe (ver
calculo de lmites).

Definicion 8.1.5. Sean f : U R2 R una funcion de dos variables y (a, b) U 0 . Se


definen los lmites iterados de f como los lmites univariados:
 
lm lm f (x, y)
xa yb

y  
lm lm f (x, y)
yb xa

Teorema 8.1.4. Sean f : U R2 R una funcion y (a, b) U 0 tales que:

lm f (x, y) = L
(x,y)(a,b)

Entonces, lmxa (lmyb f (x, y)) y lmyb (lmxa f (x, y)) existen, y ademas:
   
lm lm f (x, y) = lm lm f (x, y) = L
xa yb yb xa

335
Apuntes Mat023 (Segundo semestre 2014)

Ejemplo 8.1.8. Calcule los lmites iterados en (0, 0) de la funcion:


xy
f (x, y) = , (x, y) 6= (0, 0)
x2 + y 2
Solucion. Note que:
   
lm lm f (x, y) = lm 0 = 0 = lm lm f (x, y)
x0 y0 x0 y0 x0

Por tanto, ambos lmites iterados existen y tienen valor cero, sin embargo, sabemos que
f (x, y) no posee lmite en (0, 0).

Ejemplo 8.1.9. Determine la existencia del lmite:


sin (x + y)
lm
(x,y)(0,0) x + 3y
Solucion. Usando lmites iterados
   
sin (x + y) sin x
lm lm = lm =1
x0 y0 x + 3y x0 x
   
sin (x + y) sin y 1
lm lm = lm =
y0 x0 x + 3y y0 3y 3
como los lmites son distintos el lmite no existe.

Observacion 8.1.5. Este resultado se puede extender a funciones con mas variables.

Calculo de lmites

Observacion 8.2.1. En la seccion anterior, se establecieron procedimientos para indicar


que una funcion no posee lmite. Nuestro interes, ahora, se centra en aquellas funciones
que s lo poseen. Por tanto, debemos entregar algunos elementos de calculo:

Algebra de lmites

Teorema 8.2.1. Sean f, g : U Rn R funciones tales que:

lm f (x) = L lm g (x) = M
xa xa

para x U 0 entonces:

1. R, (f (x)) = lm f (x) = L,
xa

2. lm (f (x) + g (x)) = lm f (x) + lm g (x) = L + M


xa xa xa

336
Apuntes Mat023 (Segundo semestre 2014)

  
3. lm (f (x) g (x)) = lm f (x) lm g (x) = LM
xa xa xa

4. Si M 6= 0, lm (f (x) /g (x)) = lm f (x) / lm g (x) = L/M


xa xa xa

Ejemplo 8.2.1. Calcule:


sin (3xy)
lm
(x,y)(0,0) sin x sin y

sin u
Solucion. Usaremos el lmite fundamental de una variable lmu0 u
=1
sin(3xy)
sin (3xy) 3xy
lm = lm
(x,y)(0,0) sin x sin y (x,y)(0,0) sin x sin y
3xy
sin(3xy)
3xy
= lm   sin y 
(x,y)(0,0) 1 sin x
3 x y
= 3

Ejemplo 8.2.2. Calcule:


1 cos xy
lm
(x,y)(0,0) x2 y sin y

Solucion. Recordemos que


1 cos u 1
lm 2
=
u0 u 2
luego
1 cos xy 1 cos xy
lm = lm  
(x,y)(0,0) x2 y sin y (x,y)(0,0) 2 2 sin y
xy y
1cos xy
(xy)2
= lm  
(x,y)(0,0) sin y
y
1
=
2

Desigualdades y Teorema del Sandwich o teorema de acotamiento

Observacion 8.2.2. Una herramienta muy importante para el calculo de lmites el teo-
rema del sandwich o teorema del acotamiento. Este teorema requiere el uso adecuado de
desigualdades notables, las mas usuales son:

1. Sea x = (x1 , x2 , . . . , xn ) Rn , entonces |xi | kxk, para todo i = 1, 2, . . . , n.

2. a) (a + b)2 2 (a2 + b2 )

337
Apuntes Mat023 (Segundo semestre 2014)

b) 2ab a2 + b2

3. |sin x| 1, para todo x R.

4. |sin x| |x|, para todo x R.


a a
5. a, b, c > 0 = b+c
b

Teorema 8.2.2. Sean f, g, h : U Rn R y a U 0 tales que:

f (x) g (x) h (x) , x U

Suponga que lmxa f (x) = lmxa h (x) = L, entonces:

lm g (x) = L
xa

Ejemplo 8.2.3. Verificaremos que:


sin2 y
lm =0
(x,y)(0,0) x2 + |y|

Solucion. En efecto, basta notar que:


2
sin2 y |y|2

|y| = |y|
x2 + |y| x2 + |y| |y|
se sigue que
sin2 y
|y| 2 |y|
x + |y|
como lm(x,y)(0,0) |y| = 0 se sigue que
sin2 y
lm =0
(x,y)(0,0) x2 + |y|

Ejemplo 8.2.4. Calcular


x sin3 y
lm
(x,y)(0,0) |x| + y 2

Solucion. Note que


x sin3 y |x| |sin y|3 |x| |y|3

=
|x| + y 2 |x| |y|
|x| + y 2 |x| + y 2
as
x sin3 y
|x| |y| |x| |y|
|x| + y 2
se sigue por el teorema de acotamiento que
x sin3 y
lm =0
(x,y)(0,0) |x| + y 2

338
Apuntes Mat023 (Segundo semestre 2014)

Ejemplo 8.2.5. Demostrar que


x3 y 2
lm 2 = 0
(x,y)(0,0) (x2 + y 2 )

Solucion. Usando coordenadas polares


x3 y 2
= r cos3 sin2
(x2 + y 2 )2
luego
x3 y 2

p
= r cos3 sin2 r = x2 + y 2
(x2 + y 2 )2
as
p x3 y 2 p
x2 + y 2 2 x2 + y 2
(x2 + y 2 )
y como p
lm x2 + y 2 = 0
(x,y)(0,0)

se sigue por el teorema de acotamiento que


x3 y 2
lm 2 = 0
(x,y)(0,0) (x2 + y 2 )

Continuidad

Definicion 8.3.1. Sean U Rn , a U 0 U y f : U R una funcion real de varias


variables. Diremos que f es continua en a si:

> 0, > 0, kx akRn < = |f (x) f (a)| < (8.2)

Ademas, diremos que f es continua en U si f es continua en cada punto de U .


Observacion 8.3.1. Note que la definicion dada anteriormente y por la proposicion en

(8.2) , dice que f es continua en a si:

lm f (x) = f (a)
xa

Definicion 8.3.2. Diremos que f : U Rn R es discontinua en a U U 0 , si f no


es continua en a.
Ejemplo 8.3.1. Sea k : Rn R la kesima proyeccion de x Rn . Entonces, k es
continua en Rn . En efecto, para cada a Rn , tenemos que:

|k (x) k (a)| = |xk ak | kx ak

bastara tomar = .

339
Apuntes Mat023 (Segundo semestre 2014)

Ejemplo 8.3.2. Demuestre que f : R2 R definida por:


2

xy
, (x, y) 6= (0, 0)
f (x, y) = x4 + y 2
0 , (x, y) = (0, 0)

es continua en (0, 0).

Solucion. Basta notar que:


xy 2 |x| y 2

x4 + y 2 y 2 = |x|

As, si (x, y) (0, 0), entonces |x| 0. Por el Teorema del Sandwich se concluye que:

xy 2
lm =0
(x,y)(0,0) x4 + y 2

como f (0, 0) = 0, se tiene que f es continua en (0, 0).

Ejemplo 8.3.3. Sean U = {(x, y) : |x| < y 2 } R2 y f : U R definida por:


( xy
, (x, y) U
f (x, y) = x2 + y2
0 , (x, y)
/U

Es continua en (0, 0)?

Solucion. Note que si (x, y) U

|y|3

xy |x| |y|
|f (x, y) 0| = 2
= 2
x + y 2 x2 + y 2 x + y2
|y|3
= |y|
|y|2

y si (x, y) 6 U entonces
|f (x, y) 0| = 0 |y|
se sigue que, para todo (x, y) R2

|f (x, y) 0| |y|

y por el teorema de acotamiento

lm f (x, y) = 0
(x,y)(0,0)

La funcion es continua en (0, 0).

340
Apuntes Mat023 (Segundo semestre 2014)

Definicion 8.3.3. Se dice que una funcion f : U Rn Rm es Lipschitziana en U si:

K > 0, x, y U, kf (x) f (y)k K kx yk

La constante K se llama constante de Lipschitz.

Toda funcion lipschitziana es continua en U . En efecto, supongamos que f : U Rn


Rm es una funcion lipschitziana en U y que > 0, entonces:

kf (x) f (a)k K kx ak < K

As, tomando = K , se verifica que f es continua en a. Como a U es cualquiera, se


obtiene, finalmente, la continuidad de f en U .

Ejemplo 8.3.4. Toda transformacion lineal T : Rn Rm es una funcion lipschitziana.

Algebra de funciones continuas

El siguiente teorema facilita la identificacion de funciones continuas:

Teorema 8.4.1 (Algebra de funciones continuas). Sean f, g : U Rn R funciones


continuas en a U 0 U y R, entonces f, f + g, f g y f g son continuas en a. Si
g (a) 6= 0, entonces f /g es continua en a.

Ejemplo 8.4.1. Son funciones continuas en R3 :

1. f (x, y, z) = 3xy 2 + z 2
sin x+sin y+sin z
2. g (x, y, z) = x2 +y 2 +z 2 +1

2
3. h (x, y, z) = ex sin (y) + z 2

Teorema 8.4.2. Sean f : U Rn Rm y g : V Rm Rp tales que f (U ) V .


Suponga, ademas, que f es continua en a U 0 U y que g es continua en b = f (a) V 0 ,
entonces:
g f : U Rn Rp
es continua en a.

Demostracion. Sea > 0. Como g es continua en b = f (a), existe > 0 de modo que si
y V , entonces:
ky bk < = kg (y) g (b)k <

341
Apuntes Mat023 (Segundo semestre 2014)

Ahora bien, para y = f (x), existe > 0 tal que:

kx ak < = kf (x) f (a)k <

As, para x U tal que:

kx ak < = f (x) V kf (x) f (a)k < = kg (f (x)) g (f (a))k <

Ejemplo 8.4.2. Una aplicacion inmediata del teorema anterior es el cambio de variables.
Ilustramos lo anterior con el siguiente ejemplo: sea f : R2 R definida por:
(
sin(x2 +y 2 )
f (x, y) = x2 +y 2
, si x2 + y 2 6= 0
1 , si x2 + y 2 = 0

Estudie la continuidad de f en (0, 0).

Solucion. Naturalmente, haciendo u = x2 + y 2 , tenemos que:

(x, y) (0, 0) = u 0

As:
sin (x2 + y 2 ) sin u
lm 2 2
= lm =1
(x,y)(0,0) x +y u0 u

Como f (0, 0) = 1, obtenemos que f es continua en (0, 0).

Ejemplo 8.4.3. Usando compuesta de continuas podemos garantizar la continuidad de:


p
1. f (x, y) = x2 + sin2 y + 1

|x|+y 2 +z 2
2. g (x, y, z) = z2 +y2 +x|y|

Continuidad de funciones vectoriales

Observacion 8.5.1. La continuidad para una funcion vectorial de varias variables F : U


Rn Rm en a U U 0 se define por:

> 0, > 0, kx akRn < = kF (x) F (a)kRm <

Observacion 8.5.2. Al igual que en los lmites para funciones vectoriales, tenemos que la
continuidad de F : U Rn Rm definida por x 7 F (x) = (f1 (x) , f2 (x) , . . . , fm (x)) en
a U 0 U se obtiene a traves de la continuidad de las funciones componentes en x. Mas
precisamente:

342
Apuntes Mat023 (Segundo semestre 2014)

Teorema 8.5.1. Sean F : U Rn Rm una funcion vectorial definida por:

F (x) = (f1 (x) , f2 (x) , . . . , fm (x))

y a U U 0 . Entonces, F es continua en a U U 0 , si y solo si, fi es continua en a, para


cada i = 1, 2, . . . , m.

Ejemplo 8.5.1. La funcion F : R2 R3 definida por:

F (x, y) = x2 y, sin x, 3x2 cos y




es continua en el plano.

Ejemplo 8.5.2. Sea f : R2 R la funcion definida por:


( 2xy2 +2
, si x2 + y 2 < 1
1x 2 y 2
f (x, y) =
0 , si x2 + y 2 1

Analizar, completamente, la continuidad de f (x, y).

Solucion. Consideremos los siguientes subconjuntos del plano:

A = (x, y) R2 : x2 + y 2 > 1


B = (x, y) R2 : x2 + y 2 = 1


C = (x, y) R2 : x2 + y 2 < 1


Tenemos los siguientes casos:

1. Si (x, y) A, entonces f (x, y) = 0. Luego, f es continua en A por ser una funcion


constante en A.
2
2. Si (x, y) C, entonces f (x, y) = 2xy 2+2 2 , con 1 x2 y 2 > 0. Luego, f es continua
1x y
en C por algebra de funciones continuas y composicion de funciones continuas.

3. Si (a, b) B, entonces, f (x, y) no puede ser continua en (a, b) B si:

2a b2 + 2 6= 0

pues, si:

(x, y) (a, b) = 1 x2 y 2 0 2x y 2 + 2 k, con k 6= 0




2
con lo cual la expresion 2xy 2+2 2 se indefine. Por tanto, los puntos (a, b) B tales
1x y
que:
2a b2 + 2 = 0

343
Apuntes Mat023 (Segundo semestre 2014)

estan dados por el sistema:


2a b2 + 2 = 0


a2 + b 2 = 1
Es decir, (a, b) = (1, 0). Por tanto, investigamos el lmite:
2x y 2 + 2
lm p
(x,y)(1,0) 1 x2 y 2
Note que:
2x y 2 + 2 1 x2 y 2 + x2 + 2x + 1
p = p
1 x2 y 2 1 x2 y 2
p (x + 1)2
= 1 x2 y 2 + p
1 x2 y 2
Por tanto, para que el lmite:
2x y 2 + 2
lm p
(x,y)(1,0) 1 x2 y 2
exista es necesario y suficiente que el lmite:
(x + 1)2
lm p
(x,y)(1,0) 1 x2 y 2
exista. Sin embargo, note que:
( )
2
(x + 1) (x + 1)2
lm lm p = lm =0
x 1 y0 1 x2 y 2 x 1 1 x2
y que ademas, al considerar la trayectoria:
= (x, y) C : x4 + 4x3 + 7x2 + 4x = y 2
 

vemos que:
(x + 1)2 (x + 1)2
lm p = lm =1
(x,y)(1,0) 1 x2 y 2 x1 1 x2 + x4 + 4x3 + 7x2 + 4x
Por tanto:
(x + 1)2
lmp
(x,y)(1,0) 1 x2 y 2
no existe. En este caso, el lmite:
2x y 2 + 2
lm p
(x,y)(1,0) 1 x2 y 2
no existe. As, f no es continua en (1, 0). Por lo tanto, f es continua en:
R2 (x, y) R2 : x2 + y 2 = 1 .


344
Apuntes Mat023 (Segundo semestre 2014)

Ejemplo 8.5.3. Determine el mayor subconjunto de R2 en el cual la funcion


(
xy
x3 y
si x3 6= y
f (x, y) =
0 si x3 = y

es continua.
Solucion. Si (a, b) R2 es tal que a3 6= b entonces
 
xy
lm f (x, y) = lm
(x,y)(a,b) (x,y)(a,b) x3 y
ab
=
a3 b
= f (a, b)

luego f es continua en (a, b). Si a3 = b entonces


xy
lm f (x, y) = lm
(x,y)(a,a3 ) (x,y)(a,a3 ) x3 y

a a3
3
a a3
si a a3 6= 0 este lmte no existe (es un numero distinto de cero sobre cero). Si a a3 = 0
entonces tenemos una forma indetermnada 00 . aa3 = 0 a = 1, 0, 1 de donde obtenemos
que los unicos puntos de la curva y = x3 donde f puede ser continua son (1, 1), (0, 0) y
(1, 1). Vamos a analizar estos lmites
 
xy
1. lm
(x,y)(1,1) x3 y
Note queoir la trayectoria y = 1 se tiene
 
x1 1
lm 3
=
x1 x 1 3
pero por la trayectoria x = 1 se tiene
 
1y
lm =1
y1 1y
luego el lmite no existe. f no es continua en (1, 1)
 
xy
2. lm
(x,y)(0,0) x3 y
Note que por la trayectoria y = 0 se tiene
x
lm 3 = @
x0 x

as el lmite no existe. f no es continua en (0, 0)

345
Apuntes Mat023 (Segundo semestre 2014)

 
xy
3. lm
(x,y)(1,1) x3 y
usamos limites iterados
 
xy
lm lm
y1 x1 x3 y
 
1 y
= lm =1
y1 1 y
y
 
xy
lm lm
x1 y1 x3 y
 
x+1
= lm
x1 x3 + 1
1
=
3
se sigue que el lmite no existe y la funcion no es continua en (1, 1).
El mayor subconunto de R2 en el cual f es continua es

U = (x, y) R2 : x3 6= y


Ejemplo 8.5.4. Determine el mayor subconjunto de R2 en el cual la funcion



sin(xy)

y si (x, y) R2 y =
6 0
f (x, y) =
si (x, y) R2 y = 0

0

es continua.

Solucion. Si (a, b) R2 es tal que b 6= 0 entonces f es continua en (a, b) por algebra de


continuas. Supongamos que (a, b) R2 es tal que b = 0 entonces
sin (xy)
lm f (x, y) = lm
(x,y)(a,0) (x,y)(a,0) y
 
sin (xy)
= lm x
(x,y)(a,0) (xy)
= a

pero f (a, 0) = 0 luego la funcion es continua solo si a = 0 es decir en el punto (0, 0). El
mayor subconjunto de R2 en el cual f es continua es

U = (x, y) R2 : y 6= 0 {(0, 0)}




346
Apuntes Mat023 (Segundo semestre 2014)

Finalmente enunciamos un teorema que nos habla sobre el comportamiento de las


funciones continuas sobre los conjuntos compactos y conexos:
Teorema 8.5.2. Sea f : U Rn Rm una funcion continua en U , entonces:

1. Si A U es conexo, entonces f (A) es conexo.

2. Si K U es compacto, entonces f (K) es compacto.

Ejercicios del captulo

1. Determine el valor de R de modo que el lmite:



x x cos xy (x2 xy) 1 2
 
lm +
(x,y)(0,0) y sin2 (3x) x2 y 2
exista.

2. Sea f : R2 {(0, 0)} R la funcion definida por


 
2 1
x sin x+y
f (x, y) = 2
x + |x| + 2
calcular lm f (x, y)
(x,y)(0,0)

3. Considere la funcion definida por


x

si x 6= y 2
x + y2


f (x, y) =

si x = y 2

0

analizar la continuidad de f en R2 .

4. Sean f y g funciones definidas en R2 {(0, 0)} por


p 
sin 2
x +y 2
f (x, y) = p
x2 + y 2 + x2 + y 2
 3
x y3

g (x, y) =
|x| + |y|
que valor debe tener para que

lm {f (x, y) + g (x, y)}


(x,y)(0,0)

exista.

347
Apuntes Mat023 (Segundo semestre 2014)

5. Estudiar la continuidad de la funcion

x3 + sin (y 2 )

yx
f (x, y) =
2
x xy + y 3 + cos (y 2 ) y>x

6. Estudiar la continuidad de la funcion


2 2
x y
x2 +y2 1
x2 + y 2 6= 1
f (x, y) =
x2 + y 2 = 1

0

7. Determine si los siguientes lmites existen


|sin x sin y| x2 y 2
1) lm 2) lm 2
(x,y)(0,0) |xy| (x,y)(0,0) x2 y 2 + (y x)

xy sin (y 3 ) sin (xy 4 )


3) lm 4) lm
(x,y)(0,0) x4 + y 4 (x,y)(0,0) x2 + y 8

p
y 3 |x| x3 + y 2
5) lm 6) lm
(x,y)(0,0) |x| + y 4 (x,y)(0,0) |x| + |y|

p 
sin x + sin y cos |xy| 1
7) lm 8) lm
(x,y)(0,0) x+y (x,y)(1,0) y

(x 1)4 y 2
p
x3 y x2 + y 2
9) lm 8 10) lm
(x,y)(1,0) (x 1) + y 4 (x,y)(0,0) sin (xy)

x2 y 2x2 y 2
11) lm 12) lm
(x,y)(0,0) x2 + y (x,y)(0,0) x2 + 2y 2

x4 + 2x2 y 2 + 2y 4 x6 + y 6
13) lm 14) lm
(x,y)(0,0) (x2 + y 2 )2 (x,y)(0,0) x4 + y 4

348
Captulo 9 : Diferenciacion en varias variables

En secciones anteriores hemos estudiado algunos metodos para dibujar la grafica de


funciones de varias variables. Utilizando solo esos metodos es muy difcil obtener suficiente
informacion para captar las caractersticas generales de una funcion con expresion algo
complicada. Sabemos del calculo de una variable que la derivada nos puede ayudar mucho
en esta tarea; por ejemplo, nos permite cuantificar los cambios en la grafica, localizar
maximos, mnimos, estudian concavidad, etc. La derivada tambien tiene otras muchas
aplicaciones como el estudiante habra descubierto en calculo de una variable. Una funcion
diferenciable de R2 en R debera ser una funcion con grafica suave, que no tenga dobleces
bruscos.
Para hacer precisas estas ideas necesitamos una definicion de lo que entenderemos por
diferenciable en el caso de funciones de varias variables. Comenzamos con una nocion de
derivada que descansa en nuestros conocimientos de una variable.

Derivadas parciales

Observacion 9.1.1. En lo que sigue denotaremos por ei , con i = 1, 2, . . . , n, a los n


vectores de la base canonica de Rn , mas precisamente, ei representa el vector de Rn que
tiene todas su componentes cero salvo la i-esima componente la cual es igual a uno.

Definicion 9.1.1. Sean f : U Rn R y a U . Se define la derivada parcial de f en a
respecto a la i-esima variable como el lmite:

f f (a + tei ) f (a)
(a) = lm
xi t0 t
f (a1 , a2 , . . . , ai + t, . . . , an ) f (a1 , a2 , . . . , an )
= lm
t0 t
si este existe.

Observacion 9.1.2. Para f : U Rn R, las derivadas parciales se denotan por:


f
xi
, fxi , Di f , etc.

Observacion 9.1.3. Si n = 2, f : U R2 R, (x, y) f (x, y) y a = (a, b) U ,
entonces:
f f (a + te1 ) f (a)
(a, b) = lm
x t0 t
f ((a, b) + t (1, 0)) f (a, b)
= lm
t0 t
f (a + t, b) f (a, b)
= lm
t0 t

349
Apuntes Mat023 (Segundo semestre 2014)

y
f f (a + te2 ) f (a)
(a, b) = lm
y t0 t
f ((a, b) + t (0, 1)) f (a, b)
= lm
t0 t
f (a, b + t) f (a, b)
= lm
t0 t
note que corresponden a lmites de una variable, a saber, la variable t.
f
Observacion 9.1.4. En particular, si n = 2, 3 y 4, anotamos: x 1
= f , f = f
x x2
, f = f
y x3 z
f f
y x4 = w , segun corresponda respecto al numero de variables de f y el nombre y orden
que a estas asignemos.
f
Ejemplo 9.1.1. Sea f : R2 R, (x, y) f (x, y) = x3 y + 2x2 y 3 + 2x, calcular x
(1, 1)
y f
y
(1, 2) si estas existen.
Solucion. Por definicion
f f (1 + t, 1) f (1, 1)
(1, 1) = lm
x t0 t
(1 + t) 1 + 2 (1 + t)2 13 + 2 (1 + t) (1)3 1 + 2 (1)2 13 + 2 (1)
3  
= lm
t0 t
2
t (t t + 1)
= lm
t0 t
= lm t2 t + 1

t0
= 1
y
f f (1, 2 + t) f (1, 2)
(1, 2) = lm
y t0 t
1 (2 + t) + 2 (12 ) (2 + t)3 + 2 (1) (13 2 + 2 (12 ) (23 ) + 2)
3

= lm
t0 t
2
t (2t + 12t + 25)
= lm
t0 t
= lm 2t2 + 12t + 25

t0
= 25
note que en estos lmites, una de las variables se deja fija y se calcula la derivada usual de
una variable respecto a la otra, en otras palabras
f f (x + t, y) f (x, y)
(x, y) = lm
x t0 t
h (x + t) h (x)
= lm
t0 t
= h0 (x)

350
Apuntes Mat023 (Segundo semestre 2014)

donde hemos pensado h (x) = f (x, y), al estar y fija esta es solo una funcion de la variable
x. En el ejemplo
f
x3 y + 2x2 y 3 + 2x

(x, y) =
x x
= 3x2 y + 4xy 3 + 2

evaluando en (1, 1)
f
(1, 1) = 3 (1)2 + 4 (1) + 2 = 1
x
y
f
x3 y + 2x2 y 3 + 2x

(x, y) =
y y
= x3 + 6x2 y 2

luego
f
(1, 2) = 13 + 6 22 = 25

y
Ejemplo 9.1.2. Si f (x, y, z) = 2xyz 2 + sin (xy 2 ) + 2x entonces
f
2xyz 2 + sin xy 2 + 2x = 2yz 2 + y 2 cos xy 2 + 2
  
(x, y, z) =
x x
f
2xyz 2 + sin xy 2 + 2x = 2xz 2 + 2xy cos xy 2
  
(x, y, z) =
y y
f
2xyz 2 + sin xy 2 + 2x = 4xyz
 
(x, y, z) =
z z
Ejemplo 9.1.3. Verifique que:
u u u
+ + =1
x y z
si:
xy
u=x+
yz
Solucion. Derivando
 
xy yz+1
x+ =
x yz yz
 
xy xz
x+ =
y yz (y z)2
 
xy xy
x+ =
z yz (y z)2
se sigue      
yz+1 xz xy
+ + =1
yz (y z)2 (y z)2

351
Apuntes Mat023 (Segundo semestre 2014)

Ejemplo 9.1.4. Verifique que:

xzx + yzy = xy + z

si:
y
z = xy + xe x
Solucion. Derivando
 y
 1  1y 1

xy + xe x = xe x ye x y + xy
x x
 y
 1
xy + xe x = x + exy
y
luego

xzx + yzy
  
1 1
y 1
y
 1
y

= x xe ye + xy
x x +y x+e x
x
y y y
= xe x ye x + xy + xy + ye x
y
= xe x + 2xy
y
= xy + xy + xe x
= xy + z

Ejemplo 9.1.5. Sea f : R2 R la funcion definida por:


(
xy
x2 +y 2
, si (x, y) 6= (0, 0)
f (x, y) =
0 , si (x, y) = (0, 0)

Calcule las derivadas parciales de f en (0, 0).


Solucion. Es importante recordar que f no es continua en (0, 0). Sin embargo, la conti-
nuidad no afecta la existencia de las derivadas parciales. En efecto:
f f (t, 0) f (0, 0)
(0, 0) = lm
x t0 t
00
= lm =0
t0 t
Analogamente, se tiene que
f f (0, t) f (0, 0)
(0, 0) = lm
y t0 t
0t
2 2 0
= lm 0 +t
t0 t
= lm 0
t0
= 0

352
Apuntes Mat023 (Segundo semestre 2014)

Ejemplo 9.1.6. Sea f : R2 R la funcion definida por:


 2
x , yx
f (x, y) =
xy , y > x
f
Calcule y
(1, 1).
f
Solucion. Calculamos y
(1, 1). Note que:

f (1, 1 + t) f (1, 1) 1 (1 + t) 1
lm+ = lm+
t0 t t0 t
t
= lm+
t0 t
= 1

y, analogamente, se obtiene que:

f (1, 1 + t) f (1, 1)
0, si t 0
t
f
Por tanto, y
(1, 1), no existe.

en la figura se muestra el dominio y en dos colores distintos la parte y x e y > x, al


aproximarnos al punto (1, 1) por los puntos (1, 1 + t) se obtienen dos valores distintos de
lmite para t > 0 y t < 0.

Observacion 9.1.5. La sola existencia de las derivadas parciales no es un buen concepto


de ser derivable en el caso de funciones de varias variables. Por ejemplo, la funcion

1 si xy = 0
f (x, y) =
0 si xy 6= 0

353
Apuntes Mat023 (Segundo semestre 2014)

no es continua en (0, 0) pero las derivadas parciales existen en ese punto, luego la existencia
de las derivadas parciales no implica continuidad, si la existencia de las derivadas parciales
fuera el concepto de ser derivable no podramos recuperar el teorema de una variable,
derivable en un punto implica continua en el punto. Veremos que el concepto de ser
derivable estudia las variaciones de la funcion en una forma mas global y no en una
direccion especfica como las derivadas parciales.

Ejemplo 9.1.7. Considere la siguiente ecuacion en derivadas parciales:


2 2

u = u, 0 < x < 1, t > 0
t2 x2 (9.1)
u(0, t) = u(1, t) = 0, t>0

Demuestre que la funcion:


n
X o
u(x, t) = n cos(nt) + n sin(nt) sin(nx) (9.2)
n=1

es solucion del problema (9.1).


Indicacion: Asuma las hipotesis necesarias para la derivacion de la serie.
Suponga que la solucion u(x, t) dada por (9.2) satisface las ecuaciones:

u(x, 0) = sin(7x), 0<x<1


u
(x, 0) = x(1 x), 0<x<1
t
Calcular los coeficientes n y n de la solucion u(x, t).

Solucion. Sean 0 < x < 1 y t > 0. Asumiendo las condiciones necesarias para la
convergencia de la serie, tenemos que:

X 
u 
= n cos(nt) + n sin(nt) sin(nx)
x x n=1
 
X  
= n cos(nt) + n sin(nt) sin(nx)
n=1
x

X 
= n cos(nt) + n sin(nt) sin(nx)
n=1
x

X  
= n n cos(nt) + n sin(nt) cos(nx)
n=1

354
Apuntes Mat023 (Segundo semestre 2014)

As:

2u
X 
 
= n n cos(nt) + n sin(nt) cos(nx) (9.3)
x2 x n=1

X  
2 2
= n n cos(nt) + n sin(nt) sin(nx)
n=1

Por otro lado:



X 
u 
= n cos(nt) + n sin(nt) sin(nx) (9.4)
t t n=1
 
X  
= n cos(nt) + n sin(nt) sin(nx)
n=1
t

X  
= n cos(nt) + n sin(nt) sin(nx)
n=1
t

X 
= nn sin(nt) + nn cos(nt) sin(nx)
n=1

As:

2u
X 

2
= nn sin(nt) + nn cos(nt) sin(nx) (9.5)
t t n=1

X 
= n2 2 n cos(nt) + n2 2 n sin(nt) sin(nx)
n=1

X  
= n2 2 n cos(nt) + n sin(nt) sin(nx)
n=1

Entonces, comparando la ultima ecuacion en (9.3) y en (9.5) y notando, ademas, que:


u(0, t) = u(1, t) = 0, t>0
pues sin(n) = 0, se tiene que u(x, t) dada por la ecuacion (9.2) satisface el problema
dado por (9.1). Finalmente, debemos calcular los coeficientes n y n de la solucion u(x, t)
sabiendo que dicha solucion cumple con las siguientes condiciones iniciales:
u(x, 0) = sin(7x), 0<x<1
u
(x, 0) = x(1 x), 0<x<1
t
En efecto, si t = 0, entonces de (9.2) obtenemos:

X
u(x, 0) = n sin(nx) (9.6)
n=1

355
Apuntes Mat023 (Segundo semestre 2014)

y de (9.4) obtenemos:

u X
(x, 0) = nn sin(nx) (9.7)
t n=1

Entonces, como u(x, 0) = sin(7x) es un elemento del sistema ortonormal , tenemos que:
D E
n = sin(7x), sin(nx)
Z 1
= sin(7x) sin(nx)dx
1
= 7 n
(
0 , n 6= 7
=
1 , n=7

Por otro lado, si en la ecuacion (9.7) escribimos Bn = nn , obtenemos:



X
x(1 x) = Bn sin(nx), 0<x<1
n=1

y entonces los coeficientes Bn corresponden a los coeficientes de Fourier en la extension


impar de x(1 x), 0 < x < 1 . Es decir:

356
Apuntes Mat023 (Segundo semestre 2014)

D E
Bn = x(1 x), sin(nx)
Z 1
=2 x(1 x) sin(nx) dx
0
Z 1 Z 1
2 0 2 0
= x cos(nx) dx + x2 cos(nx) dx
n 0 n 0
 Z 1
2 1 1
= x cos(nx) + cos(nx) dx + x2 cos(nx)

n 0 0 0
Z 1 
2 x cos(nx) dx
0
 Z 1 
2
= cos(n) + cos(n) 2 x cos(nx) dx
n 0
Z 1
4 0
= 2 2 x sin(nx) dx
n 0
4
 1 Z 1 
= 2 2 x sin(nx) sin(nx) dx

n 0 0

4 
= cos(n) 1
n3 3

0 , n es par
= 8
, n es impar
n3 3
Por lo tanto:
8
n = , n N
4 (2n 1)4
As, en resumen, los coeficientes n y n son:
8
n = n7 n =
4 (2n 1)4

para todo n 1. En particular, la solucion esta dada por:



8 X sin(nt) sin(nx)
u(x, t) = cos(7t) sin(7x) 4
n=1 (2n 1)4

357
Apuntes Mat023 (Segundo semestre 2014)

Ejercicios de la seccion
f f
1. Hallar x
y y
para las siguientes funciones

a) f (x, y) = xy 2
3 +y 2
b) f (x, y) = ex
c) f (x, y) = x ln (x2 + y 2 )

2. Si
x2 y 3

x2 +y 2
si (x, y) 6= (0, 0)
f (x, y) =

0 si (x, y) = (0, 0)

n o
determine el conjunto D = (x, y) R2 : fx
(x,y)
existe y estudiar si la funcion

G :D R2 R
f (x, y)
(x, y) G (x, y) =
x
es continua en todo D.
 2 2
y
3. Si u (x, y) = arctan x xy hallar ux , uy y verificar que

xux + yuy = 0

4. Si u (x, y, z) = x2 y + y 2 z + z 2 x verificar que


u u u
+ + = (x + y + z)2
x y z

5. Determine todas las funciones f : R3 R, (x, y, z) f (x, y, z) que cumplen


f
= ex+y + 3x2 y + 4yz + y 2 + 2
x
f
= ex+y + 2yz 3 + 2xy + 4xz + x3
y
f
= 3y 2 z 2 + 4xy
z
6. Si p
|x| + y 2 + |x|
si x+y >0


x+y

f (x, y) =

x3 si x + y 0 x2 + y 2 4
y2 + x si x + y 0 x2 + y 2 > 4

f f
determine si x
(0, 0) y y
(0, 0) existen.

358
Apuntes Mat023 (Segundo semestre 2014)

Interpretacion de la derivada parcial

En esta seccion veremos una interpretacion de las derivadas parciales y buscaremos la


ecuacion del plano tangente a la grafica de una funcion de dos variables.
Considere una funcion f : D R2 R, su grafica esta contenida en R3 . Suponga que
estamos interesados en estudiar el comportamiento de la funcion en un punto (x0 , y0 ) de
su dominio, como no contamos con tecnicas de varias variables, intentamos estudiar el
comportamiento descendiendo a una variable, para ello, vamos a intersectar la grafica con
dos planos x = x0 e y = y0 .
La interseccion de la grafica con el plano x = x0 es una curva, la cual puede ser
interpretada como el grafico de la funcion de una variable

z = f (x0 , y) = g (y)

en el plano x = x0 (note que si x = x0 y z = f (x, y) entonces z = f (x0 , y)). En


este plano podemos aplicar las tecnicas de una variable para conocer el crecimiento y
decrecimiento, concavidad y otros, sin embargo, este analisis solo es un comportamiento de
la curva, no necesariamente un comportamiento global, por ejemplo un maximo sobre esta
curva puede no ser un maximo de la funcion en su dominio. La recta tangente a la grafica
de esta funcion de una variable es

z g (y0 ) = g 0 (y0 ) (y y0 )

esto es
f
z f (x0 , y0 ) = (x0 , y0 ) (y y0 )
y
pues

g (y0 + h) g (y0 )
g 0 (y0 ) = lm
h0 h
f (x0 , y0 + h) f (x0 , y0 )
= lm
h0 h
f
= (x0 , y0 )
y
note que la recta esta en el plano x = x0 , se sigue que los puntos de ella cumplen

x = x0
y = y
f
z = f (x0 , y0 ) + (x0 , y0 ) (y y0 )
y

359
Apuntes Mat023 (Segundo semestre 2014)

o en ecuaciones parametricas

x x0 0
y = 0 +y 1


z f (x0 , y0 ) y0 f
y
(x0 , y0 ) f
y
(x0 , y0 )

donde y R, el vector director es en este caso



0
1


f
y
(x0 , y0 )

De manera similar, la interseccion de la grafica con el plano y = y0 es una curva, la


cual puede ser interpretada como el grafico de la funcion de una variable

z = f (x, y0 ) = p (x)

sobre el plano y = y0 (note que si y = y0 y z = f (x, y) entonces z = f (x, y0 )). La recta


tangente a la grafica de esta funcion de una variable es

z p (x0 ) = p0 (x0 ) (x x0 )

esto es
f
z f (x0 , y0 ) = (x0 , y0 ) (x x0 )
x
pues
p (x0 + h) p (x0 )
p0 (x0 ) = lm
h0 h
f (x0 + h, y0 ) f (x0 , y0 )
= lm
h0 h
f
= (x0 , y0 )
x
se sigue que los puntos de esta recta cumplen

x = x
y = y0
f
z = f (x0 , y0 ) + (x0 , y0 ) (x x0 )
x
o en ecuaciones parametricas

x 0 1
y = y0 + x 0
f f
z f (x0 , y0 ) x0 x (x0 , y0 ) x
(x0 , y0 )

360
Apuntes Mat023 (Segundo semestre 2014)

el vector director corresponde a



1
0
f
x
(x0 , y0 )
de estos calculos se sigue que en el punto (x0 , y0 , f (x0 , y0 )) de la grafica los vectores

1 0
0 y 1


f f
x
(x0 , y0 ) y
(x0 , y0 )

son tangentes, se sigue que el vector



f

1 0 x
(x 0 , y0 )
f
0 1 = y (x0 , y0 )


f f
x
(x0 , y0 ) y
(x0 , y0 ) 1

es perpendicular a la superficie, as
 
f f
(x0 , y0 ) , (x0 , y0 ) , 1 (x x0 , y y0 , z f (x0 , y0 )) = 0
x y

debera corresponder a un plano tangente a la superficie, el problema para definir esto


directamente como el plano tangente a la grafica es el de formalizar en que sentido queremos
decir que la funcion y el plano se parecen cerca del punto, el calculo realizado depende solo
de la existencia de las derivadas parciales pero por ejemplo en el caso de la funcion

1 si xy = 0
f (x, y) =
0 si xy 6= 0

los calculos nos llevaran a que el plano tangente es z = 1 sin embargo los valores que toma
la funcion en todo entorno abierto de (0, 0) no se parecen a uno (tan cerca como queramos
existen valores cero).
Veamos un ejemplo concreto, consideremos la funcion f : R2 R, (x, y) f (x, y) =
2 4
(x2 2xy) e2x y +2y y estudiemos la funcion en el punto (x0 , y0 ) = (1, 0). La grafica de f

361
Apuntes Mat023 (Segundo semestre 2014)

es

2
si cortamos con el plano y = 0 se obtiene la curva z = f (x, 0) = x2 e2x

la recta tangente en x = 1 es

d  2 2x2 
z f (1, 0) = xe (x 1)
dx
x=1

esto es

2 2x2 3 2x2
ze = 2xe 4x e (x 1)
x=1
z e2 = 2e2 4e2 (x 1)


362
Apuntes Mat023 (Segundo semestre 2014)

es decir
x = x
y = 0
z = e2 2e2 (x 1)

de manera similar, si cortamos con el plano x = 1 se obtiene la curva z = f (1, y) =


4
(1 2y) e2y +2y la recta tangente en y = 0 es
d  
2y 4 +2y
z f (1, 0) = (1 2y) e (y 0)
dy y=0

esto es
4 +2y2 
z e2 = 4yey 2y 3 + y 2 + 1 y
y=0
z e2 = 0
es decir
x = 1
y = y
z = e2

1 0
los directores de estas rectas son 0 y 1 se sigue que el normal corresponde
2e2 0
a 2
1 0 2e
0 1 = 0
2e2 0 1

363
Apuntes Mat023 (Segundo semestre 2014)

y as el plano tangente corresponde a

2e2 , 0, 1 x 1, y 0, z e2 = 0
 

es decir
2e2 (x 1) + z e2 = 0
o equivalentemente
z = e2 2e2 (x 1)

en la figura, en blanco estan las curvas y las rectas tangentes, la interseccion de estas
rectas y curvas corresponde al punto (1, 0, f (1, 0)) y el plano que contiene a estas rectas
tangentes es el plano tangente, como se puede apreciar la propiedad de tangencia es local,
no significa que toque a la grafica en un unico punto.

Ejercicios de la seccion

1. Considere la funcion f : R2 R, (x, y) f (x, y) = ex sin y + e2y cos x. Determine


las ecuaciones parametricas de las rectas tangentes a la grafica de f en el punto
(0, 0, 1) que son paralelas a los planos x = 0 e y = 0.

2. Determine una recta normal a la superficie


y2
x2 + + z2 = 1
4
364
Apuntes Mat023 (Segundo semestre 2014)

 
en el punto 1 , 1, 1 .
2 2

2 4
3. Si f : R2 R, (x, y) f (x, y) = (x2 2xy) e2x y +2y estudiar los extremos
(maximos y mnimos) locales y globales de la funcion g : R R, t g (t) = f (t, 0).

Diferenciabilidad

Definicion 9.3.1. Sean U Rn y f : U R una funcion. Diremos que f es diferenciable



en a U si existe una transformacion lineal T : Rn R tal que:
|f (a + h) f (a) T (h)|
lm =0
h0 khk
Observacion 9.3.1. Si n = 2, por ejemplo, el lmite en la definicion anterior toma la
forma:
|f (a + h, b + k) f (a, b) T (h, k)|
lm =0
(h,k)(0,0) h2 + k 2
Recordemos, ademas, que si T : Rn R es una transformacion lineal, entonces
existen constantes A1 , A2 , . . . , An R (respecto, por ejemplo, de las bases canonicas,
respectivamente) tales que:

[T ] = A1 A2 An M1n (R)

El siguiente teorema indica la condicion necesaria que deben cumplir las constantes
A1 , A2 , . . . , An en caso de que una funcion f sea diferenciable en a:

Teorema 9.3.1. Sean U Rn y f : U R una funcion diferenciable en a U , entonces:
f
Ai = (a)
xi
para cada i = 1, 2, . . . , n.

Demostracion. Como f : U R es diferenciable en a, existe una transformacion lineal


T : Rn R tal que:
|f (a + h) f (a) T h|
lm =0
h0 khk
Note que haciendo:
h = t ei
el lmite anterior implica el siguiente lmite:
|f (a + t ei ) f (a) T (t ei )|
lm =0 (9.8)
t0 kt ei k

365
Apuntes Mat023 (Segundo semestre 2014)

Por otro lado, suponga que la transformacion lineal T : Rn R tiene representacion


matricial:
[T ]1C = A1 A2 An


donde C es la base canonica de Rn y 1 representa la base canonica de R. As:

[T (t ei )]1 = [T ]1C [t ei ]C
= t Ai

As:
|f (a + t ei ) f (a) T (t ei )| |f (a + t ei ) f (a) t Ai |
lm = lm
t0 kt ei k t0 |t|

f (a + t ei ) f (a)
= lm
Ai
t0 t

f
= (a) Ai
xi

y como el valor del lmite en (9.8) es 0 (por la diferenciabilidad de f en a) se tiene que:



f

xi (a) Ai
=0

Esto es:
f
(a) = Ai
xi
para cada i = 1, 2, . . . , n.

Observacion 9.3.2. Se sigue entonces que si f es diferenciable en x = a las derivadas


parciales deben existir en el punto.

Podemos entonces reformular la definicion como:

Definicion 9.3.2. Sean U Rn y f : U R una funcion. Diremos que f es diferenciable



en a U si:
f
1. Para cada i = 1, 2, . . . , n, existen las derivadas parciales xi
(a).

2. Asumiendo, h = (h1 , h2 , . . . , hn ) Rn , se tiene que:


Pn f
f (a + h) f (a) i=1 x (a) h

i
i
lm =0
h0 khk

366
Apuntes Mat023 (Segundo semestre 2014)

Observacion 9.3.3. Con el cambio de variables h = x a el lmite


Pn f
f (a + h) f (a) i=1 x (a) h

i
i
lm =0
h0 khk
se puede escribir en la forma
Pn f
f (x) f (a) i=1 x (a) (x a )

i
i i
lm =0
xa kx ak
Observacion 9.3.4. Se debe hacer notar que la suma:
n
X f
(a) hi
i=1
x i

se obtiene va el isomorfismo canonico que existe entre M11 (R) y el espacio vectorial real
unidimensional. En efecto sabemos que:
 
1 f f f
[T ]C = x1 (a) x2 (a) xn (a)

y como h = (h1 , h2 , . . . , hn ), tenemos:

[T h]1 = [T ]1C [h]C



h1

f f f
 h2
= (a) (a) (a)

x1 x2 xn ..
.
hn
n
!
X f
= (a) hi
i=1
x i
11

Ejemplo 9.3.1. Considere la funcion f : R2 R definida por:


2 4

xy
si (x, y) 6= (0, 0)
x2 + y 2
0 si (x, y) = (0, 0)

estudiar la diferenciabilidad de f en (0, 0).


Solucion. Notemos que
f f (h, 0) f (0, 0)
(0, 0) = lm
x h0 h
h2 04
2 2 0
= lm h +0
h0 h
= lm 0
h0
= 0

367
Apuntes Mat023 (Segundo semestre 2014)

de manera similar
f f (0, h) f (0, 0)
(0, 0) = lm
y h0 h
02 h4
2 2 0
= lm 0 +h
h0 h
= lm 0
h0
= 0

luego las derivadas parciales existen, notemos que



f (x, y) f (0, 0) f (0, 0) (x 0) f
(0, 0) (y 0)

x y
lm p
(x,y)(0,0) x2 + y 2
2 4
xy
x2 +y2
= lm p
(x,y)(0,0) x2 + y 2
x2 y 4
= lm (por polares)
(x,y)(0,0) (x2 + y 2 )3/2

= 0

se sigue que f es diferenciable en (0, 0).

Ejemplo 9.3.2. Considere la funcion g : R3 R definida por:



sin (xyz) 6 (0, 0, 0)
, (x, y, z) =
g (x, y, z) = x2 + y 2 + |z|
0 , (x, y, z) = (0, 0, 0)

Demuestre que g es diferenciable en el punto (0, 0, 0).

Solucion. Calculamos, primeramente, las derivadas parciales de f en (0, 0, 0). Esto es:

f f (t, 0, 0) f (0, 0, 0)
(0, 0, 0) = lm
x t0 t
0
= lm
t0 t
= 0

368
Apuntes Mat023 (Segundo semestre 2014)

f f
Analogamente, se tiene que (0, 0, 0) = (0, 0, 0) = 0. Luego:
y z

f
f (h, k, l) f (0, 0, 0) (0, 0, 0) h + f (0, 0, 0) k + f
(0, 0, 0) l

x y z
lm =
(h,k,l)(0,0,0) h2 + k 2 + l2
|f (h, k, l)|
= lm
(h,k,l)(0,0,0) h2 + k 2 + l2
|sin (hkl)|
= lm
(h,k,l)(0,0,0) (h2 + k 2 + |l|) h2 + k 2 + l2
|hkl|
lm
(h,k,l)(0,0,0) |l| h2 + k 2 + l2
|hkl|
lm
(h,k,l)(0,0,0) |l| |k|

= lm |h|
(h,k,l)(0,0,0)

=0

Por tanto, f es diferenciable en (0, 0, 0).


Ejemplo 9.3.3. Consideremos f : R2 R la funcion definida por:
(
sin(xy)
x2 +|y|
, (x, y) 6= (0, 0)
f (x, y) =
0 , (x, y) = (0, 0)

Verifique que f no es diferenciable en (0, 0).


Solucion. Notamos, primeramente que:
f f (t, 0) f (0, 0)
(0, 0) = lm
x t0
 t 
1 0
= lm 2 =0
t0 t t +0
Analogamente se obtiene que:
f
(0, 0) = 0
y
Ahora bien, como:

f (h, k) f (0, 0) f (0, 0) h f
(0, 0) k

x y |f (h, k)|
lm = lm
(h,k)(0,0) h2 + k 2 (h,k)(0,0) h2 + k 2
1 |sin (hk)|
= lm
(h,k)(0,0) h2 + k 2 h2 + |k|

369
Apuntes Mat023 (Segundo semestre 2014)

Ahora bien, note que:


 
1 |sin (hk)|
lm lm = lm 0 = 0
h0 k0 h2 + k 2 h2 + |k| h0

y si consideramos la trayectoria : h = k, con h > 0 tenemos que:

1 |sin (h2 )| 1 |sin (h2 )|


lm+ = lm+
h0 h2 + h2 h2 + h h0 2 h3 + h2
1 |sin (h2 )| h2
= lm
2 h0 h2 h3 + h2
1
=
2
Por tanto, f no es diferenciable en (0, 0).

Ejemplo 9.3.4. Toda transformacion lineal es diferenciable, la transformacion lineal que


mas se parece es la misma funcion.

Observacion 9.3.5. Supongamos que f : U R2 R es una funcion diferenciable en



(a, b) U . Esto quiere decir que:

f (a + h, b + k) f (a, b) f (a, b) h f
(a, b) k

x y
lm =0 (9.9)
(h,k)(0,0) h2 + k 2
Ahora bien, haciendo el cambio de variables:

x=a+h
y =b+k

tenemos que:
(h, k) (0, 0) (x, y) (a, b)
Por tanto, el lmite en (9.9) queda:

f (x, y) f (a, b) f (a, b) (x a) f
(a, b) (y b)

x y
lm q =0
(x,y)(a,b)
(x a)2 + (y b)2

Intuitivamente, para (x, y) suficientemente cercano a (a, b), tenemos que:



f (x, y) f (a, b) f (a, b) (x a) f
(a, b) (y b)

x y
q '0
2 2
(x a) + (y b)

370
Apuntes Mat023 (Segundo semestre 2014)

As:
f f
f (x, y) f (a, b) (a, b) (x a) (a, b) (y b) ' 0
x y
y por tanto:
f f
f (x, y) ' (a, b) (x a) + (a, b) (y b) + f (a, b)
x y
Es decir, para (x, y) suficientemente cercano a (a, b), f (x, y) puede ser aproximada por el
plano:
f f
z = f (a, b) + (a, b) (x a) + (a, b) (y b)
x y
el cual es tangente a la superficie:

S : z = f (x, y)

en una vecindad de (a, b). As, tenemos la siguiente definicion:


Definicion 9.3.3. Sea f : U R2 R una funcion diferenciable en (a, b). Llamaremos
plano tangente a la superficie z = f (x, y) en (a, b) al plano de ecuacion:
f f
z f (a, b) = (x a) (a, b) + (y b) (a, b)
x y
Ejemplo 9.3.5. Encontrar la ecuacion del plano tangente a la superficie S dada por la
grafica de la funcion:
z = (x + y)2 2x
en el punto (1, 1, 0).
Solucion. Calculamos las derivadas parciales
z
(x + y)2 2x = 2x + 2y 2

=
x x

z
(1, 1) = 2
x
y
z
(x + y)2 2x = 2x + 2y

=
y y

z
(1, 1) = 4
y
el plano tangente es
z = 0 + 2 (x 1) + 4 (y 1)
es decir
z = 2x + 4y 6

371
Apuntes Mat023 (Segundo semestre 2014)

Ejemplo 9.3.6. Sean a, b, c numeros reales positivos. Verifique si el cono:


x2 y 2 z2
+ =
a2 b2 c2
y la esfera:
2
b2 + c 2 b2

2 2
= 2 b2 + c 2

x +y + z
c c
son tangentes entre s en los puntos (0, b, c).
Solucion. Notemos que los puntos entregados pertenecen a la interseccion
02 (b)2 c2
+ =
a2 b2 c2
y
2
b2 + c 2 b2

2 2
= 2 b2 + c 2

0 + (b) + c
c c
Notemos que
x2 y 2 z2
+ 2 =
a2 b c2
r
x2 y 2
z = c + 2
a2 b
(el punto (0, b, c) tiene tercera coordenada positiva, por eso consideramos solo la raz
positiva). El vector normal al plano tangente es
r ! r ! !
x2 y 2 x2 y 2
c + 2 , c + 2 ,1
x a2 b y a2 b


cx cy
= q , , 1

q
2 2 2 x2 2 2 2 x2
a2 a ya2+b b2 a ya2+b


b2 (x,y)=(0,b) b2 (x,y)=(0,b)
 c 
= 0, , 1
b
para la superficie
2
b2 + c 2 b2 2

2 2
= 2 b + c2

x +y + z
c c
se tiene r 2
2 2


z b + c = b (b2 + c2 ) (x2 + y 2 )
c c2
luego r
b2 + c 2 b2 2
z = (b + c2 ) (x2 + y 2 )
c c2
372
Apuntes Mat023 (Segundo semestre 2014)

evaluando en (0, b, c) se tiene


r
b2 + c 2 b2 2
c = (b + c2 ) b2
c c2

b2 + c 2 b2
c =
c c
se sigue que el signo correcto es . Luego
r
b2 + c 2 b2 2
z= (b + c2 ) (x2 + y 2 )
c c2
as el normal corresponde a
(zx , zy , 1)|(0,b)
donde
r !
b2 + c 2 b2 2
zx = (b + c2 ) (x2 + y 2 )
x c c2
x
= q
b4 +b2 c2 c2 x2 c2 y 2
c2

r !
b2 + c 2 b2 2
zy = (b + c2 ) (x2 + y 2 )
y c c2
y
= q
b4 +b2 c2 c2 x2 c2 y 2
c2

evaluando

zx = 0
b b c
zy = q = b2
=
b4 +b2 c2 c2 (b) 2
c
b
c2

as

(zx , zy , 1)|(0,b)
 c 
= 0, , 1
b
se sigue que tienen los mismos planos tangentes.

Teorema 9.3.2. Sean U Rn y f : U R una funcion diferenciable en a U , entonces
f es continua en a.

373
Apuntes Mat023 (Segundo semestre 2014)

Demostracion. Dado > 0, existe > 0 tal que:


n

X f
f (a + h) f (a) (a) hi khk

xi
i=1

para todo khk < . Entonces, por la desigualdad triangular, tenemos que:
n
X f
|f (a + h) f (a)| khk + (a) hi


i=1
xi
n
X f
khk + khk
xi (a)

i=1

( + n A) khk
n o
f 

donde A = max x (a) : i = 1, 2, . . . , n . Ahora bien, eligiendo = mn , se

i +n A
obtiene que:
khk < = |f (a + h) f (a)| <

Observacion 9.3.6. Como ya es sabido desde el calculo diferencial en una variable, el


teorema anterior dice que la continuidad es una condicion necesaria para la diferenciabilidad,
pero no es suficiente. En efecto, considere:

Ejemplo 9.3.7. Verifique que f : R2 R definida por:


3/2

x |y|
, si (x, y) 6= (0, 0)
f (x, y) = 2 2
x +y
0 , si (x, y) = (0, 0)

es continua, pero no diferenciable en (0, 0).

Solucion. La continuidad es inmediata. En efecto:

|x| |y|3/2 |xy| p


= |y|
x2 + y 2 x2 + y 2
p
1 x2 + y 2 p |y|
|y| =
2 x2 + y 2 2

donde la ultima expresion converge a 0, cuando (x, y) (0, 0). Ahora bien, por el Teorema
del Sandwich, se obtiene que:

x |y|3/2
lm = 0 = f (0, 0)
(x,y)(0,0) x2 + y 2

374
Apuntes Mat023 (Segundo semestre 2014)

Por tanto, f es continua en (0, 0). Por otro lado, es facil ver que:
f f
(0, 0) = (0, 0) = 0
x y
Luego:

f (h, k) f (0, 0) f (0, 0) h f
(0, 0) k

x y |f (h, k)|
lm = lm
(h,k)(0,0) h2 + k 2 (h,k)(0,0) h2 + k 2
|h | |k|3/2
= lm
(h,k)(0,0) (h2 + k 2 )3/2
Al tomar la recta de aproximacion h = k en el ultimo lmite se observa que no existe. Por
tanto, la funcion f no es diferenciable en el (0, 0).

Teorema 9.3.3 (Algebra de funciones diferenciables). Sean f, g : U Rn R funciones



diferenciables en a U y R una constante, entonces f, f + g, f g, f g son
diferenciables en a. Si, ademas, g (a) 6= 0, entonces f /g tambien es diferenciable en a.

f
Teorema 9.3.4. Sean f : U Rn R y a U . Si las derivadas parciales x i
existen en
una bola alrededor de a y son continuas en a entonces f es diferenciable en a.

Observacion 9.3.7. La condicion en el teorema anterior es solo suficiente, mas no necesaria


como puede verse con el siguiente ejemplo:

Ejemplo 9.3.8. Verificar que la funcion f : R2 R definida por:


!
(x2 + y 2 ) sin p
1
, si (x, y) 6= (0, 0)
f (x, y) = x 2 + y2

0 , si (x, y) = (0, 0)

es diferenciable en todo punto de R2 pero las funciones derivadas parciales no son continuas
en R2 .

Solucion. Del algebra de funciones diferenciables se obtiene que f es diferenciable en todo


punto distinto de (0, 0). Notemos que para (x, y) 6= (0, 0) se cumple
!
1
x2 + y 2 sin p

x x2 + y 2
! ! 
1 2 2
 1 1 2 2 3/2

= 2x sin p + x + y cos p x +y 2x
x2 + y 2 x2 + y 2 2
! !
1 x 1
= 2x sin p p cos p
x2 + y 2 x2 + y 2 x2 + y 2

375
Apuntes Mat023 (Segundo semestre 2014)

y por simetra
!
1
x2 + y 2 sin

p
y x2 + y 2
! !
1 y 1
= 2y sin p p cos p
x2 + y 2 x2 + y 2 x2 + y 2
en (0, 0) calculamos las derivadas parciales
f f (h, 0) f (0, 0)
(0, 0) = lm
x h0
h
1
h2 sin |h|
= lm
h0 h
 
1
= lm h sin
h0 |h|
= 0
f
y por simetra y
(0, 0) = 0. La funcion derivada parcial esta definida por
   
2x sin 2 2 2 2 cos 2 2
1 x 1

si (x, y) 6= (0, 0)
f x +y x +y x +y
(x, y) =
x

0 si (x, y) = (0, 0)

esta funcion no es continua en (0, 0), en efecto, considere los puntos


 
1
(xn , yn ) = ,0
2n + 4
entonces si n +, (xn , yn ) (0, 0) pero
f 2    
(xn , yn ) = sin 2n + cos 2n +
x 2n + 4
4 4
2 1 1
=
2n + 4 2 2
luego cuando n + se sigue (xn , yn ) (0, 0) pero f x
(xn , yn ) 12 , el lmite no es
cero (usando  otra sucesion
 se muestra que en realidad el lmite no existe por ejemplo con
1
xn , yen ) = 2n+ , 0 da otro lmite). Se sigue que f no es de clase C 1 sin embargo
(f
3
 
2
(x + y 2 ) sin 1
|f (x, y)| 2
x +y 2
lm p = lm p
(x,y)(0,0) x2 + y 2 (x,y)(0,0) x2 + y 2
!
p
2 2
1
= lm x + y sin p

2 2

(x,y)(0,0) x +y
= 0

376
Apuntes Mat023 (Segundo semestre 2014)

aqu hemos usado el acotamiento


!
p 1 p
0 x2 + y 2 sin x2 + y 2

p
x2 + y 2
luego f es diferenciable en (0, 0) y en todos los otros puntos por algebra de funciones
diferenciables pero f no tiene derivadas parciales continuas en el origen.
Ejemplo 9.3.9. Considere la funcion f : R2 R definida por
2 2

x + y si x < y
f (x, y) =
x + y 4 si x y 2

1. Determine el mayor subconjunto de R2 en el cual f es continua.


Por algebra de continuas y la definicion de la funcion esta es continua en todos los
puntos (x0 , y0 ) en los cuales x0 < y02 y x0 > y02 . Vamos a analizar en los puntos de la
forma (a2 , a) que son los puntos en el cual la definicion de la funcion cambia.

Notemos que
2 2
a +a por la region x < y 2
lm f (x, y) =
(x,y)(a2 ,a)
a2 + a4 por la region x y 2

377
Apuntes Mat023 (Segundo semestre 2014)

luego la funcion es continua si a2 + a2 = a2 + a4 esto es a = 1, 0, 1. Se sigue que el


mayor subconjunto de R2 en el cual f es continua es

(x, y) R2 : x 6= y 2 {(0, 0) , (1, 1) , (1, 1)}




2. Estudiar la diferenciabilidad de f en (0, 0).


Primero determinamos las derivadas parciales en el origen
f f (h, 0) f (0, 0)
(0, 0) = lm
x h0 h
f (h, 0)
= lm
h0 h
note que los puntos de la forma (h, 0) estan en las dos regiones de definicion de f
dependiendo del signo de h
f (h, 0) h + 04
lm+ = lm+ =1
h0 h h0 h
f (h, 0) h + 02
lm = lm+ =1
h0 h h0 h
se sigue que
f
(0, 0) = 1
x
de manera similar
f f (0, h) f (0, 0)
(0, 0) = lm
y h0 h
f (0, h)
= lm
h0 h
los puntos de la forma (0, h) con h 6= 0 siempre pertenecen a la region x < y 2 se sigue
f (0, h) h2
lm = lm =0
h0 h h0 h

f
as y
(0, 0) = 0. Ahora estudiamos la diferenciabilidad

f (x, y) f (0, 0) f (0, 0) (x 0) f
(0, 0) (y 0)

x y
lm p
(x,y)(0,0) x2 + y 2
|f (x, y) x|
= lm p
(x,y)(0,0) x2 + y 2
pero 2 2
y si x < y
|f (x, y) x| =
y 4 si x y 2

378
Apuntes Mat023 (Segundo semestre 2014)

se sigue que
2
y si x < y2
|f (x, y) x| x2 +y2

p =
x2 + y 2 y
4
si x y 2 , (x, y) 6= (0, 0)


x2 +y 2

de donde obtenemos
|f (x, y) x|
lm p =0
(x,y)(0,0) x2 + y 2
luego f es diferenciable en (0, 0).

3. Si existe, determine el plano tangente a la grafica de f en (2, 0, f (2, 0)).


En un entorno de (2, 0) la funcion esta definida como x + y 4 luego es diferenciable y
el plano tangente corresponde a
z f (2, 0) = fx (2, 0) (x 2) + fy (2, 0) (y 0)

z 2 = (x 2) + 0

z = x

Ejemplo 9.3.10. Determine (justificando) los valores de R+ para los cuales las funcion
|x| y 2

si (x, y) 6= (0, 0)
(x + y 2 )

2
f (x, y) =


0 si (x, y) = (0, 0)

1. Es continua en (0, 0).


La funcion es continua en (0, 0) si y solo si
lm f (x, y) = f (0, 0) = 0
(x,y)(0,0)

esto es
|x| y 2
lm = 0
(x,y)(0,0) (x2 + y 2 )

usando polares
r r2 |cos | sin2
lm = lm r+22 |cos | sin2
r0
libre
r2 r0
libre
= lm r2 |cos | sin2
r0
libre
= 0
si y solo si 2 > 0 esto es 0 < < 2.

379
Apuntes Mat023 (Segundo semestre 2014)

2. Es diferenciable en (0, 0).


Primero calculamos las derivadas parciales
f f (h, 0) f (0, 0)
(0, 0) = lm
x h0 h
f (h, 0)
= lm
h0 h
1 |h| 02
= lm = 0
h0 h (h2 + 02 )

de manera similar
f f (0, h) f (0, 0)
(0, 0) = lm
y h0 h
f (0, h)
= lm
h0 h
1 |0| h2
= lm
h0 h (02 + h2 )
= 0
luego, la funcion es diferenciable si y solo si
|f (x, y) f (0, 0) fx (0, 0) (x 0) fy (0, 0) (y 0)|
lm p =0
(x,y)(0,0) x2 + y 2
es decir
|f (x, y)| |x| y 2
lm = lm =0
x2 + y 2 (x,y)(0,0) (x2 + y 2 ) x2 + y 2
p p
(x,y)(0,0)

usando polares
r r2 |cos | sin2
lm 2 ) r
= lm r+221 |cos | sin2
r0
libre
(r r0
libre
= lm r1 |cos | sin2
r0
libre
= 0
si y solo si 1 > 0 esto es 0 < < 1.

Ejemplo 9.3.11. Sean g : R2 R dada por:


3

y
si (x, y) 6= (0, 0)
g(x, y) = x2 + y 2
0 si (x, y) = (0, 0)

y f : R2 R definida por:

1 si (x, y) = (0, 0)
f (x, y) = g(x,y)
e si (x, y) 6= (0, 0),

380
Apuntes Mat023 (Segundo semestre 2014)

1. Es f (x, y) continua en (0,0)?


f f
2. Determine (0, 0) y (0, 0).
x y
3. Es f (x, y) diferenciable en (0, 0)?

Solucion. 1. Puesto que la funcion e() es continua, estudiar la continuidad de f en


el origen equivale a estudiar la continuidad de g en dicho punto. Para probar la
continuidad de g en el origen utilizamos acotamiento. Se tiene:
|y 3 | y2
0 |g(x, y) 0| = = |y|
x2 + y 2 x2 + y 2
x2 + y 2
|y| 2
x + y2
= |y| 0

cuando (x, y) (0, 0). Por lo tanto


 
lm f (x, y) = exp lm g(x, y) = e0 = 1
(x,y)(0,0) (x,y)(0,0)

y la funcion f es continua en el origen.

2. Procedemos por definicion. Se tiene que:


2
f e0/h 1
(0, 0) = lm =0
x h0 h
Por otro lado:
f ek 1
(0, 0) = lm = lm ek = 1
y k0 k k0

3. Para verificar la diferenciabilidad de f debemos probar que el siguiente lmite:


|f (h, k) (f (0, 0) + fx (0, 0)h + fy (0, 0)k)|
lm =
(h,k)(0,0)) h2 + k 2
 3 
exp h2k+k2 1 k

= lm
(h,k)(0,0)) h2 + k 2
sea cero. Sin embargo, este lmite no es nulo. En efecto, basta elegir la trayectoria
h = k (el puntaje se otorga por elegir una trayectoria apropiada) y calcular:

|ek/2 1 k|
k/2
e 1 k 1 ek/2 1 1
lm = lm = lm 2

= 6= 0

k0 2|k| k0 2k k0 2 2 2

luego f no es diferenciable en el origen.

381
Apuntes Mat023 (Segundo semestre 2014)

Ejercicios de la seccion

1. Hallar la ecuacion del plano tangente a la superficie z = x2 + y 3 en el punto (3, 1, 10).

2. Por que deben llamarse tangentes en (0, 0) las graficas de f (x, y) = x2 + y 2 y


g (x, y) = x2 y 2 + xy 3 ?

3. Sea f : R2 R definida por



x2 y 4

x2 +y 2
si (x, y) 6= (0, 0)
f (x, y) =

0 si (x, y) = (0, 0)
Probar que f es de clase C 1 (R2 ).

4. Sea p 1. Estudiar la diferenciabilidad en el origen de la funcion


kkp : Rn R
n
!1/p
X p
x kxkp = |xi |
k=1

5. Para que valores de la funcion



x3 +xy 2

|x| +|y|
si (x, y) 6= (0, 0)
f (x, y) =

0 si (x, y) = (0, 0)
es diferenciable en (0, 0)?

6. Considere la superficie definida por


   
3 x
S = (x, y, z) R : x sin z =0
y
Si (a, b, c) S determine el plano tangente a la superficie en ese punto y mostrar que
pasa por el origen.

7. Utilizar la aproximacion mediante el plano tangente para estimar el valor de:


10
a) (0,99e0,1 )
q
b) (3,99)2 + (4,01)2 + (2,01)2

8. Sean > 1 y f : Rn R, x f (x) una funcion tal que, para todo x, y Rn


|f (x) f (y)| kx yk
Muestre que f es diferenciable.

382
Apuntes Mat023 (Segundo semestre 2014)

Derivadas de orden superior y funciones de clase C n

Observacion 9.4.1. Sean U Rn un conjunto abierto no vaco y f : U R una funcion


f
tal que las derivadas parciales x i
(x) existen para todo x U . Entonces, podemos definir
f f
la funcion derivada parcial de f como la funcion x i
: U Rn R dada por x 7 x i
(x).
Las derivadas parciales de orden superior son, entonces, derivadas parciales de la funcion
f
xi
: U Rn R. Mas precisamente:

Definicion 9.4.1. Sean U Rn un conjunto abierto no vaco y f : U Rm una funcion


f
tal que las derivadas parciales x i
(x) existen para todo x U . Se define la derivada parcial
de segundo orden para f como:
2f
 
f
(x) = (x)
xj xi xj xi
Mas precisamente:
2f
 
1 f f
(x) = lm (x + tej ) (x)
xj xi t0 t xi xi
2f
Ademas, si i 6= j la derivada xj xi
(x) recibe el nombre de derivada parcial mixta. Si i = j,
2f
anotamos x2 (x).
i
La derivada parcial de tercer orden para f se define como:
3f
 2 
f
(x) = (x)
xk xj xi xk xj xi
2
considerando xj x
f
i
: U Rn R, en donde U corresponde al conjunto de puntos de
U para los cuales la segunda derivada parcial esta definida. En particular, si i = j = k,
anotamos entonces:
3f
(x)
x3i
Finalmente, las derivadas parciales de orden superior se definen como las derivadas
f
parciales sucesivas de x i
: U Rn R.

Ejemplo 9.4.1. Hallar las derivadas parciales de segundo orden de la funcion:


x
f (x, y) = arctan
y
Solucion.
 
f x y
= arctan = 2
x x y x + y2
 
f x x
= arctan = 2
y y y x + y2

383
Apuntes Mat023 (Segundo semestre 2014)

y las de segundo orden

2f
 
y 2xy
= =
x2 x x2 + y 2 (x2 + y 2 )2
2f x2 y 2
 
y
= =
yx y x2 + y 2 (x2 + y 2 )2
2f x2 y 2
 
x
= 2 =
xy x x + y2 (x2 + y 2 )2
2f
 
x 2xy
= 2 =
y 2 y x +y 2
(x2 + y 2 )2
notar que, en este caso,
2f 2f
+ =0
x2 y 2
y
2f 2f
=
yx xy
3z
Ejemplo 9.4.2. Hallar , si:
xy 2
z = sin xy

Solucion.

zy = (sin xy) = x cos xy
y
luego

zyy = (x cos xy) = x2 sin xy
y
y finalmente

x2 sin xy = 2x sin xy x2 y cos xy

zyyx =
x
Definicion 9.4.2. Sean U Rn un conjunto abierto no vaco. Diremos que la funcion
f : U R es de clase C n sobre U, o bien que f C n (U ) si todas las derivadas parciales
hasta el orden nesimo son funciones continuas en U .

Observacion 9.4.2. A modo de resumen, tenemos que:

f C 1 (U ) = f es diferenciable en U = f es continua en U

Teorema 9.4.1 (Schwarz). Sean U Rn un conjunto abierto no vaco y f C 2 (U ),


entonces, para todo x U y para todo i 6= j

2f 2f
(x) = (x)
xj xi xi xj

384
Apuntes Mat023 (Segundo semestre 2014)

Observacion 9.4.3. Del teorema anterior y bajo la hipotesis de clase C n adecuada se


puede establecer igualdad de derivadas parciales mixtas de tercer orden y superior, note
por ejemplo que

3f 2
 
f
=
yx2 yx x
2
 
f
=
xy x
 2 
f
=
x yx
 2 
f
=
x xy
3f
=
x2 y

si f C 3 .

Observacion 9.4.4. La continuidad de las derivadas parciales de segundo orden es funda-


mental en el teorema anterior. Esto se puede ilustrar considerando el siguiente ejemplo:

Ejemplo 9.4.3. Sea f : R2 R la funcion definida por:


(  2 2
y
xy xx2 +y 2 si (x, y) 6= (0, 0)
f (x, y) =
0 si (x, y) = (0, 0)

Verifique que:
2f 2f
(0, 0) 6= (0, 0)
yx xy
Solucion. Note que
  2
x y2

f
(x, y) = xy
x x x2 + y 2
y (x4 + 4x2 y 2 y 4 )
=
(x2 + y 2 )2

para (x, y) 6= (0, 0) y


f f (h, 0) f (0, 0)
(0, 0) = lm =0
x h0 h
luego
y (x4 +4x2 y 2 y 4 )
(
f si (x, y) 6= (0, 0)
(x, y) = (x2 +y 2 )2
x 0 si (x, y) = (0, 0)

385
Apuntes Mat023 (Segundo semestre 2014)

se sigue que
2f
 
1 f f
(0, 0) = lm (0, h) (0, 0)
yx h0 h x x
1 h (0 h4 )
 
= lm
h0 h (02 + h2 )2
= 1
por otro lado
  2
x y2

f
(x, y) = xy
y y x2 + y 2
x (x + 4x2 y 2 + y 4 )
4
=
(x2 + y 2 )2
y
f
(0, 0) = 0
y
se sigue
2f
 
1 f f
(0, 0) = lm (h, 0) (0, 0)
xy h0 h y y
1 h (h4 )
 
= lm
h0 h (h2 + 02 )2
= 1
2f 2f
se sigue yx
(0, 0) 6= xy
(0, 0) en este caso la funcion no es de clase C 2 (R2 ).
Ejemplo 9.4.4. Si u = ln (x2 + y 2 ) verificar que
2u 2u
+ =0
x2 y 2
Solucion. Derivando
u 2x
ln x2 + y 2 = 2

=
x x x + y2
u 2y
ln x2 + y 2 = 2

=
y y x + y2
y de segundo orden
2u x2 y 2
 
2x
= = 2
x2 x x2 + y 2 (x2 + y 2 )2
2u x2 y 2
 
2y
= = 2
y 2 y x2 + y 2 (x2 + y 2 )2
as
2u 2u x2 y 2 x2 y 2
+ = 2 + 2 =0
x2 y 2 (x2 + y 2 )2 (x2 + y 2 )2

386
Apuntes Mat023 (Segundo semestre 2014)

Ejercicios de la seccion
 
3/2
p
1. Si u = arctan xy/ 1 + x + y probar que uxy = (1 + x2 + y 2 )
2 2 y

15xy
uxxyy =
(1 + x2 + y 2 )7/2

2. Sean a, b, c constantes positivas. Si z = z (x, y) es tal que a2 x2 + b2 y 2 = c2 z 2 entonces


muestre que
zxx zyy = (zxy )2

3. Si u = ln (x2 + y 2 + z 2 ) probar que


xuyz = yuzx = zuxy
xy
4. Si f (x, y) = x+y
demuestre que

2f 2f 2
2 f
x2 + 2xy + y =0
x2 xy y 2

Gradiente y matriz jacobiana

Definicion 9.5.1. Sean U Rn y f : U R una funcion tal que todas las derivadas

f
parciales x i
(a), i = 1, 2, . . . , n de f existen en a U . Se define el gradiente de f en a
como el vector en Rn dado por:
 
f f f
f (a) = (a) , (a) , . . . , (a)
x1 x2 xn
Ejemplo 9.5.1. Sea f (x, y, z) = 3x2 + sin y 2 + 2xz 2 . Calcule f (x, y, z).
Solucion. En este caso
 
f (x, y, z) f (x, y, z) f (x, y, z)
f (x, y, z) = , ,
x y z
es decir
f (x, y, z) = 2z 2 + 6x, 2y cos y 2 , 4xz


Ejemplo 9.5.2. Considere f (x, y, z) = 3x2 + 2xy 4z 3 . Sean S la superficie dada por:
S : f (x, y, z) = 1
y el plano tangente a S en (1, 1, 1). Si u, v linealmente independientes, entonces
pruebe que existe R tal que:
u v = f (1, 1, 1)

387
Apuntes Mat023 (Segundo semestre 2014)

Solucion. Si u, v son linealmente independientes entonces u v es normal al plano,


pero de las secciones anteriores sabemos que el normal a la grafica de una funcion z = h (x, y)
tiene la forma k (hx , hy , 1) donde k es una constante, la superficie f (x, y, z) = 1 la
podemos mirar como la grafica de
r
3 3 2 1 1
z= x + yx
4 2 4
se sigue

(zx , zy , 1)
q
3 3 2 1 1
23 3
x + 1
y 4
x + 2
yx 4
2 2
= 2 2 , 2x 2
, 1
3 2
(3x + 2yx 1) 3 9x + 6yx 3

evaluando en (1, 1)
q
3
3
+ 12 1
r 3 1
43 3 1 1 + 2 2 4 4
(zx , zy , 1) = + , 2 , 1
3 4 2 43+21 9+63
 
2 1
= , ,1
3 6
note que
f (x, y, x) = 6x + 2y, 2x, 12z 2


luego

f (1, 1, 1) = (6 + 2, 2, 12)
= (8, 2, 12)

se sigue que existe R tal que


 
2 1
u v =k , , 1 = f (1, 1, 1)
3 6
Observacion 9.5.1. El isomorfismo canonico : Rn ' M1n (R) vuelve a jugar un rol
importante en relacionar el vector gradiente f (a) y la matriz de la diferencial Df (a).
Sabemos que la diferencial de una funcion f : Rn R esta dada por la transformacion
lineal Df (a) : Rn R tal que:
|f (a + h) f (a) Df (a) h|
lm =0
h0 khk
pero 1 (Df (a)) = f (a), por lo tanto, podemos escribir:
|f (a + h) f (a) f (a) h|
lm =0
h0 khk

388
Apuntes Mat023 (Segundo semestre 2014)

Observacion 9.5.2. Nuestro proposito, ahora, es tratar la diferenciabilidad para funciones


vectoriales de la forma f : U Rn Rm .. Teniendo en cuenta que la topologa del espacio
euclidiano Rm se obtiene considerando la norma kkRm , tenemos que:

Definicion 9.5.2. Sean U Rn y f : U Rm una funcion vectorial. Diremos que f es



diferenciable en a U si existe una transformacion lineal T : Rn Rm tal que:
kf (a + h) f (a) T (h)k
lm =0
h0 khk
Definicion 9.5.3. Sean U Rn y f : U Rm una funcion tal que todas las derivadas
fi
parciales existen en a. Llamaremos matriz jacobiana de f en a a la matriz de derivadas
xj
parciales:
 
fi
J f (a) = (a)
xj mn

f1 f1 f1
x1 (a) x2 (a) xn (a)

f2 f 2 f 2

x (a) x (a) x (a)

1 2 n Mmn (R)
=
.
.. .
.. . .. .
..




fm fm fm
(a) (a) (a)
x1 x2 xn

Ejemplo 9.5.3. Sea f : R2 R2 definida por f (x, y) = (2x2 y + y, 2xy y 2 ). Luego, si


anotamos u = 2x2 y + y y v = 2xy y 2 , entonces la matriz jacobiana de f esta dada por:

u u
x y
Jf (x, y) = v v

x y
2x2
 
4xy
=
2y 2x 2y

Teorema 9.5.1. Sean f : U Rn Rm una funcion definida por f (x) = (f1 (x) , f2 (x) , . . . , fm (x))

y a U . f es diferenciable en a, si y solo si, fi : U R es diferenciable en a, para cada
i = 1, 2, . . . , m. Ademas:
 
fi
Df (a) = (a) = Jf (a)
xj mn

Ejemplo 9.5.4. Es diferenciable g (x, y, z) = x + sin x cos y 2 , x2 y + 1, z 2 + 1 en R3 ?

389
Apuntes Mat023 (Segundo semestre 2014)

Solucion. La respuesta es si, pues las funciones componentes son todas diferenciables en
R3 .

Observacion 9.5.3. Por definicion, tenemos que si f : U Rn Rm es diferenciable en


a, entonces existe una transformacion lineal T : Rn Rm tal que:
kf (a + h) f (a) T (h)k
lm =0
h0 khk
Ademas, note que para cada i = 1, 2, . . . , m, se tiene que:

|fi (a + h) fi (a) Ti (h)| kf (a + h) f (a) T (h)k

para todo h. As, si h = t ej , tenemos que:



fi (a + t ej ) fi (a)
0 Ti (ej )
t

fi (a + t ej ) fi (a) Ti (tej )
=
t
kf (a + t ej ) f (a) T (tej )k
0
ktej k
si acaso t 0. Lo anterior implica que:
fi
[Ti ]j = (a)
xj
en donde [Ti ]j representa el jesimo coeficiente de la matriz [Ti ], respecto de las bases
canonicas. Todo lo anterior implica que en caso de que f sea diferenciable en a, la matriz
jacobiana Jf (a) es la matriz de la diferencial Df (a). Desde ahora en adelante, anotamos:

Df (a) = Jf (a)

La regla de la cadena

Teorema 9.6.1. Sean f : U Rn Rm , con U abierto y g : V Rm Rp , con


V abierto, tales que f (U ) V . Suponga que f es diferenciable en a U y que g es
diferenciable en b = f (a) V , entonces:

g f : U Rn Rp

es diferenciable en a. Ademas:

D (g f ) (a) = Dg (f (a)) Df (a)


= Dg (b) Df (a)

390
Apuntes Mat023 (Segundo semestre 2014)

Observacion 9.6.1. Es importante destacar que Dg (b) Mpm (R) y que Df (a)
Mmn (R).
Observacion 9.6.2. Supongamos que z = g (f (x)) e y = f (x), con b = f (a), entonces
la regla de la cadena en terminos de las matrices jacobianas queda en la forma:

D (g f ) (a)

= Dg (b) Df (a)

z1 z1 z1 y1 y1 y1
y1 (b) y2 (b) (b) (a) (a) (a)
ym
x1 x2 xn
z2 z2 z2 y2 y2 y2
y1 (b) y2 (b)
(b) (a) (a) (a)
ym
x1 x2 x2
=
.. .. .. .. .. .. ... ..

. . . .
. . .


zp zp zp ym ym ym
(b) (b) (b) (a) (a) (a)
y1 y2 ym x1 x2 xn
m
!
X zi yk
= (b) (a) Mpn (R)
k=1
y k x j
ij

Por tanto, en notaciones clasicas, mirando las componentes del producto, la regla viene
dado por las formulas:
m
zi X zi yk
=
xj k=1
yk xj
zi y1 zi y2 zi ym
= + + +
y1 xj y2 xj ym xj
para i = 1, 2, . . . , p y j = 1, 2, . . . , n.

Ejemplo 9.6.1. Sea f : R2 R, (u, v) f (u, v) una funcion diferenciable. Se define la


funcion w : R2 R por
w (x, y) = f x2 y 2 , y 2 x2


calcular la Jacobiana de w y muestre que


w w
y +x =0
x y
Solucion. Por la regla de la cadena
 
 2x 2y

Jw = wx wy = fu fv
2x 2y

= 2xfu 2xfv 2yfu + 2yfv

391
Apuntes Mat023 (Segundo semestre 2014)

as
w w
y +x
x y
= y (2xfu 2xfv ) + x (2yfu + 2yfv )
= 0
Ejemplo 9.6.2. Sean g : R3 R y f : R3 R3 tales que:

f (x, y, z) = u (x, y, z) ; v (x, y, z) ; w (x, y, z)
Definamos F : R3 R mediante la ecuacion:
F (x, y, z) = (g f ) (x, y, z)
Por la regla de la cadena:
DF (x, y, z) = Dg (u, v, w) Df (x, y, z)
u u u

 x y z


g g g v v v
= x

u v w y z

w w w
x y z
As, por ejemplo:
F g u g v g w
= + +
x u x v x w x
3
Ejemplo 9.6.3. Sea g : R R y consideremos el cambio de variables a coordenadas
esfericas:
x = r cos sin
(r, , ) : y = r sin sin

z = r cos
Defina z (r, , ) = g x (r, , ) , y (r, , ) , z (r, , ) . Calcule z , z y
z

r
.
Solucion.
z g x g y g z
= + +
r x r y r z r
g g g
= (cos sin ) + (sin sin ) + (cos )
x y z

z g x g y g z
= + +
x y z
g g g
= (r sin sin ) + (r cos sin ) + (0)
x y z
g g
= (r sin sin ) + (r cos sin )
x y

392
Apuntes Mat023 (Segundo semestre 2014)

z g x g y g z
= + +
x y z
g g g
= (r cos cos ) + (r sin cos ) + (r sin )
x y z
Ejemplo 9.6.4. Considere:
w = x3 yez
Suponga que:
2 2

x=s t
y = s2 + t2

z =s+t
Pruebe que:
w w
(1, 0) + (1, 0) = 10e
s t
Solucion. Usando la regla de la cadena
w w x w y w y
= + +
s x s y s y s
w w x w y w y
= + +
t x t y t y t

recordar que las derivadas de w quedan evaluadas en x = s2 t2 , y = s2 + t2 , z = s + t as


w w 2  x
(s, t) = s t2 , s2 + t2 , s + t (s, t)
s x s
w 2  y
+ s t2 , s2 + t2 , s + t (s, t)
y s
w 2  y
+ s t2 , s2 + t2 , s + t (s, t)
y s
y
w w 2  x
(s, t) = s t2 , s2 + t2 , s + t (s, t)
t x t
w 2  y
+ s t2 , s2 + t2 , s + t (s, t)
y t
w 2  y
+ s t2 , s2 + t2 , s + t (s, t)
y t

evaluando en (1, 0) se obtiene

w w x w y w z
(1, 0) = (1, 1, 1) (1, 0) + (1, 1, 1) (1, 0) + (1, 1, 1) (1, 0)
s x s y s z s
w w x w y w z
(1, 0) = (1, 1, 1) (1, 0) + (1, 1, 1) (1, 0) + (1, 1, 1) (1, 0)
t x t y t z t

393
Apuntes Mat023 (Segundo semestre 2014)

note que
w
3x2 yez

(x, y, z) =
x
w
x3 e z

(x, y, z) =
y
w
x3 yez

(x, y, z) =
z
evaluando
w (1, 1, 1) = (3e, e, e)
por otro lado
x x
(s, t) = 2s y (s, t) = 2t
s t
y y
(s, t) = 2s y (s, t) = 2t
s t
z z
(s, t) = 1 y (s, t) = 1
s t
evaluando
x x
(1, 0) = 2 y (1, 0) = 0
s t
y y
(1, 0) = 2 y (1, 0) = 0
s t
z z
(1, 0) = 1 y (1, 0) = 1
s t
se sigue
w w x w y w z
(1, 0) = (1, 1, 1) (1, 0) + (1, 1, 1) (1, 0) + (1, 1, 1) (1, 0)
s x s y s z s
= (3e) (2) + (e) (2) + (e) (1)
= 9e

y
w w x w y w z
(1, 0) = (1, 1, 1) (1, 0) + (1, 1, 1) (1, 0) + (1, 1, 1) (1, 0)
t x t y t z t
= (3e) (0) + (e) (0) + (e) (1)
= e

se sigue
w w
(1, 0) + (1, 0) = 10e
s t

394
Apuntes Mat023 (Segundo semestre 2014)

p 
Ejemplo 9.6.5. Sea w = f (r) una funcion diferenciable, definamos h (x, y, z) = f 2 2 2
x +y +z .
Demuestre que:  2
dw
= khk2
dr
Solucion. La derivada de w respecto a r es
dw
= f 0 (r)
dr
luego  2
dw 2
= (f 0 (r))
dr
por otro lado, usando la regla de la cadena
x
hx = f 0 (r) rx = f 0 (r)
r
0 0 y
hy = f (r) ry = f (r)
r
0 0 z
hz = f (r) rz = f (r)
r
se sigue

2
 x 2  0
0 y 2  0 z 2
khk = f (r) + f (r) + f (r)
r r r
 0 2
f (r)
x2 + y 2 + z 2

=
r
 0 2
f (r)
r2

=
r
2
= (f 0 (r))

como se quera probar.

Ejemplo 9.6.6. Sean f : [0, +) R una funcion diferenciable y u : R R definida


por:
u = f k(x, y, z)k2


donde:
x = cos cos
y = cos sin

z = sin
u
Demuestre que = 0.

395
Apuntes Mat023 (Segundo semestre 2014)

Solucion. Usando la regla de la cadena, note que u (, , ) = f (g (h (, , ))) donde

p f (p)
(x, y, z) g (x, y, z) = x2 + y 2 + z 2
(, , ) h (, , ) = ( cos cos , cos sin , sin )

se sigue
u g x g y g z
= f0 + f0 + f0
x y z
0 2

= f (2 cos cos ) ( sin cos )
+f 0 2 (2 cos sin ) ( sin sin )


+f 0 2 (2 sin ) ( cos )


= 2f 0 2 cos sin cos2 cos sin sin2 + sin cos


 

= 2f 0 2 (0)


= 0

otra forma de verlo es


u = f k(x, y, z)k2


con
x = cos cos
y = cos sin

z = sin
queda
u = f 2


luego la derivada es 0.
Ejemplo 9.6.7. Cambiar las variables x, y, z en la ecuacion
u u u
x +y +z = nu
x y z
por , , de la forma
x y
= ,= y=z
z z
y probar que la ecuacion se transforma en
v
= nv

deducir de esto que u tiene la forma
x y 
n
u (x, y, z) = z F ,
z z
para una funcion F arbitraria.

396
Apuntes Mat023 (Segundo semestre 2014)

Solucion. Definimos
v ( (x, y, z) , (x, y, z) , (x, y, z)) = u (x, y, z)
con el cambio de variable propuesto
u v v v
= + +
x x x x
u v v v
= + +
y y y y
u v v v
= + +
z z z z
se sigue
 
u v 1 v v
= + (0) + (0)
x z
 
u v v 1 v
= (0) + + (0)
y z
u v  x  v  y  v
= 2 + 2 + (1)
z z z
se sigue que
u u u
x +y +z = nu
x y z
se transforma en
v  x  v  y  v  x  v  y  v
+ + + +z = nv
z z z z
es decir
v
= nv

esta ecuacion es de variables separadas, la resolvemos
ln |v| = n ln || + C
luego
v = Kn
note que la constante es respecto a la variable , luego puede depender de , , ponemos
v = K (, ) n
luego
u (x, y, z) = v ( (x, y, z) , (x, y, z) , (x, y, z))
= K ( (x, y, z) , (x, y, z)) (x, y, z)n
x y 
= K , zn
z z

397
Apuntes Mat023 (Segundo semestre 2014)

Observacion 9.6.3. Muchas veces abusando de la notacion, en lugar de poner

v ( (x, y, z) , (x, y, z) , (x, y, z)) = u (x, y, z)

simplemente se trabaja con u (, , ) y se escribe


u u u u
= + +
x x x x
Observacion 9.6.4. Otra manera de enfrentar el problema anterior, cuando es posible
despejar en forma explcita es la siguiente, note que
x y
= ,= y=z
z z
implica

x =
y =
z =

luego
P (, , ) = u (x (, , ) , y (, , ) , z (, , ))
luego
P u x u y u z
= + +
x y z
u u u
= () + () +
x y z
luego
P u u u
= () + () +
x y z
u u u
= x +y +z
x y z
= nu
= nP

de donde
P (, , ) = n K (, )
y as
u (x (, , ) , y (, , ) , z (, , )) = n K (, )
esto es  
n x z
u (x, y, z) = z K ,
y y

398
Apuntes Mat023 (Segundo semestre 2014)

Estudiaremos el comportamiento de la regla de la cadena en derivadas de orden superior

Ejemplo 9.6.8. Suponiendo z C 2 (R2 ), reescriba la ecuacion diferencial:

2z 2 2z
+ 2 3 =0
x2 xy y 2
en nuevas variables u y v, definidas por: u = 3x y y v = x + y.

Solucion. Definamos
P (u (x, y) , v (x, y)) = z (x, y)
entonces
zx = Pu ux + Pv vx
esto es
zx = 3Pu + Pv
derivando nuevamente respecto a x

zxx = 3 (Pu )x + (Pv )x


= 3 ((Pu )u ux + (Pu )v vx ) + ((Pv )u ux + (Pv )v vx )
= 3 ((Pu )u 3 + (Pu )v ) + ((Pv )u 3 + (Pv )v )
= 9Puu + 6Puv + Pvv

zxy = (3Pu + Pv )y
= 3 (Pu )y + (Pv )y
= 3 (Puu uy + Puv vy ) + (Pvu uy + Pvv vy )
= 3 (Puu (1) + Puv ) + (Pvu (1) + Pvv )
= 3Puu + 2Puv + Pvv

con respecto a la variable y

zy = Pu uy + Pv vy
= Pu + Pv

derivando nuevamente

zyy = (Puu uy + Puv vy ) + (Pvu uy + Pvv vy )


= (Puu + Puv ) + (Pvu + Pvv )
= Puu 2Puv + Pvv

399
Apuntes Mat023 (Segundo semestre 2014)

se sigue que
2z 2 2z
+2 3 2 =0
x2 xy y
queda reescrita como

(9Puu + 6Puv + Pvv ) + 2 (3Puu + 2Puv + Pvv ) 3 (Puu 2Puv + Pvv ) = 0

es decir
(9 6 3) Puu + (6 + 4 + 6) Puv + (1 + 2 3) Pvv = 0
as
Puv = 0
esta ecuacion implica
P (u, v) = F (u) + G (v)
donde F , G son funciones de clase C 2 de una variable se sigue

z (x, y) = P (u (x, y) , v (x, y))


= F (u (x, y)) + G (v (x, y))
= F (3x y) + G (x + y)
2z 2 2z
como ejercicio, verificar que si z (x, y) = F (3x y)+G (x + y) entonces x2
+2 xy 3 y 2 =
0.

Ejemplo 9.6.9. Las ecuaciones u = f (x, y) , x = X (s, t) e y = Y (s, t) definen u como


una funcion de s y t: u = F (s, t). Si las funciones son de clase C 2 pruebe que:
2 2
2F f 2 X 2 f X X Y 2 f f 2 Y 2 f Y
 
= + 2 +2 + + 2
s2 x s2 x s x s xy y s2 y s
Solucion. Por la regla de la cadena

F s = f x xs + f y y s
Fss = (fx )s xs + fx xss + (fy )s ys + fy ys
= (fxx xs + fxy ys ) xs + fx xss
+ (fyx xs + fyy ys ) ys + fy ys
= fxx (xs )2 + 2fxy ys xs + fyy (ys )2 + fx xss + fy ys

se sigue
2 2
2F 2f x y 2 f 2f f 2 x f 2 y
 
x y
= +2 + 2 + +
s2 x2 s s s xy y s x s2 y s2

400
Apuntes Mat023 (Segundo semestre 2014)

y

Ejemplo 9.6.10. Sea g : R R una funcion de clase C 1 (R) muestre que f (x, y) = g x
para x 6= 0 cumple
f f
x +y =0
x y
Solucion. La funcion es diferenciable y por la regla de la cadena se tiene
   
y y
fx = 2
g0
x x
   
1 y
fy = g0
x x
luego
 
y y y y
xfx + yfy = g0 + g0
x x x x
= 0

Ejemplo 9.6.11. Sea p : R2 R, una funcion diferenciable tal que


p p
x +y =0
x y
h
defina h (u, v) = p (v, uv) muestre que = 0.
v
Solucion. La compuesta de funciones diferenciables es diferenciables luego

hv = px xv + py yv
= p x + py u

as

vhv = vpx + py uv
= xpx + ypy
= 0

se sigue
hv = 0

Ejemplo 9.6.12. Sea u = u(x, t) una funcion de clase C 2 . Transformar la ecuacion de las
vibraciones de una cuerda:
2u 2
2 u
= c , c>0
t2 x2
a unas nuevas variables independientes y , en donde:

= x ct = x + ct

401
Apuntes Mat023 (Segundo semestre 2014)

Solucion. Sea u = u(x, t) C 2 . Considere, ademas, las nuevas variables independientes


y dadas por las ecuaciones = x ct y = x + ct. Entonces, por la regla de la cadena,
tenemos que:
u u u
= +
t t t
 u u 
=c

y tambien:
u u u
= +
x
As, por la regla de la cadena nuevamente, tenemos que:
2u
  
u u 
= c
t2 t
   
u  u 
=c
t t
 2
u 2 u
 2
2 u
 
u
=c + c +
t 2 t 2 t t
 2
2u 2u 2u
  
u
=c (c) + c c (c) + c
2 2
 2
2u 2u

2
=c 2 +
2 2
Analogamente:
2u
 
u u
= +
x2 x
 u   u 
= +
x x
2 u 2 u 2 u 2 u
= + + +
2 x x x 2 x
2u 2u 2u
= + 2 +
2 2
2 2
en donde la igualdad
u u
=
se obtiene a traves del Teorema de Schwarz, pues u C 2 .
Reemplazando las derivadas anteriores en la ecuacion:
2u 2
2 u
= c
t2 x2
tenemos:  2
2u 2u
 2
2u 2u
 
2 u 2 u
c 2 + =c 2 +
2 2 2 2

402
Apuntes Mat023 (Segundo semestre 2014)

Simplificando la ecuacion anterior se obtiene, finalmente, que:


2u
=0

Ejemplo 9.6.13. Sea g : R R una funcion derivable tal que g(0) = 1 y g 0 (0) = 2.
Considere la funcion f : R2 R definida por:
 
f (x, y) = xy g x1 y1

1. Determine la ecuacion del plano tangente a la superficie z = f (x, y) en el punto



1, 1, f (1, 1) .

2. Obtenga una funcion h(x, y) tal que:


z z
x2 + y2 = z h(x, y)
x y

Solucion. Definimos a u = u(x, y) = x1 y1 . Puesto que u es diferenciable en (1, 1) y g()


es derivable, entonces z es diferenciable en (1, 1). Se tiene entonces que:

z(1, 1) = 1 1 g(u(1, 1)) = g(0) = 1

Ademas, por la regla de la cadena:

zx = y g(u) + xy g 0 (u)ux
y
= yg(u) g 0 (u)
x
zx (1, 1) = 3

Y, de forma analoga:

zy = x g(u) + xy g 0 (u)uy
x
= xg(u) + g 0 (u)
y
zy (1, 1) = 1

Con esto, la ecuacion del plano tangente es:

3(x 1) + (y 1) (z + 1) = 0
3x + y z = 1

Hacemos:

x2 zx = x2 yg(u) xyg 0 (u)


y 2 zy = xy 2 g(u) + xyg 0 (u)

403
Apuntes Mat023 (Segundo semestre 2014)

Basta sumar y obtenemos:

x2 zx + y 2 zy = (x + y)xyg(u)

Pero z = xy g(u), de modo que:

x2 zx + y 2 zy = (x + y)z = f (x, y) = x + y

Ejemplo 9.6.14. Sea f : R2 R, (x, y) f (x, y) una funcion de clase C 2 .

1. Muestre que si g (r, ) = f (r cos , r sin ) entonces

2f 2f 2g 1 2 g 1 g
+ = + +
x2 y 2 r2 r2 2 r r
Usando la regla de la cadena

gr = f x x r + f y y r
= fx cos + fy sin

grr = (fxx xr + fxy yr ) cos + (fyx xr + fyy yr ) sin


= (fxx cos + fxy sin ) cos + (fyx cos + fyy sin ) sin
= fxx cos2 + 2 sin cos fxy + fyy sin2

g = fx x + fy y
= fx r sin + fy r cos

y as

g = r ((fxx x + fxy y ) sin + fx cos )


+r ((fyx x + fyy y ) cos fy sin )
= r ((fxx (r) sin + fxy r cos ) sin + fx cos )
+r ((fyx (r) sin + fyy r cos ) cos fy sin )
= r2 fxx sin2 2r2 fxy sin cos + r2 fyy cos2
rfx cos rfy sin
= r2 fxx sin2 2fxy sin cos + fyy cos2


r (fx cos + fy sin )

404
Apuntes Mat023 (Segundo semestre 2014)

entonces
2g 1 2 g 1 g
+ +
r2 r2 2 r r
= fxx cos2 + 2 sin cos fxy + fyy sin2


 (fx cos + fy sin )


+ fxx sin2 2fxy sin cos + fyy cos2
r
1
+ (fx cos + fy sin )
r
= fxx + fyy

como se quera demostrar.

2. Determine una funcion f : {(x, y) R2 : 1 x2 + y 2 9} R tal que


2f 2f
+ = 0 para 1 < x2 + y 2 < 9
x2 y 2
f (x, y) = 1 si x2 + y 2 = 1
f (x, y) = 3 si x2 + y 2 = 9
g
suponiendo que g (r, ) = f (r cos , r sin ) no depende del angulo, es decir,
= 0.
Pasando a polares
2f 2f
+ = 0 para 1 < x2 + y 2 < 9
x2 y 2

2g 1 2 g 1 g
+ + = 0 para 1 < r < 3
r2 r2 2 r r
f (x, y) = 1 si x2 + y 2 = 1

g (r, ) = 1 si r=1
y
f (x, y) = 3 si x2 + y 2 = 9

g (r, ) = 3 si r=3
si no depende del angulo tenemos
d2 g 1 dg
+ = 0 para 1 < r < 3
dr2 r dr
g (1) = 1
g (3) = 3

esto tiene solucion


g (r) = A + B ln r

405
Apuntes Mat023 (Segundo semestre 2014)

donde

1 = A
3 = A + B ln 3
2
que tiene solucion A = 1, B = ln 3
, as
2 p
f (x, y) = 1 + ln x2 + y 2
ln 3

Gradiente y planos tangentes

Observacion 9.7.1. Sea f : U Rn R una funcion diferenciable tal que f (a) =


6 0.
Consideremos el conjunto de nivel:

S : Lc (f ) = {x U : f (x) = c}

Suponga que : t 7 (t) una curva diferenciable de R Rn tal que Im() S. Note
que, en vista de las hipotesis para la curva , tenemos que:

f ( (t)) = c, t

entonces, por la regla de la cadena, tenemos que:


n
X f
f ( (t)) 0 (t) = ( (t)) 0i (t) = 0
i=1
x i

en particular, si existe t0 R tal que (t0 ) = a, entonces, de la ecuacion anterior conclumos


que:
hf (a) , 0 (t0 )i = 0
como es cualquier curva diferenciable tal que se tiene que:

f (a) S

puesto que 0 es tangente a S. Se tiene entonces:


Teorema 9.7.1. Sean U un conjunto abierto no vaco, f : U Rn R una funcion
diferenciable y S : Lc (f ) el conjunto de nivel de c para f . Suponga que que f (a) = c (es
decir, a S) y que f (a) 6= 0 entonces:

f (a) a S

donde a significa que la ortogonalidad de f (a) al conjunto de nivel S : Lc (f ) se da en


el punto a.

406
Apuntes Mat023 (Segundo semestre 2014)

Ejemplo 9.7.1. Hallar un vector normal unitario a la superficie definida por:

2xy 3 z + z ln x + y sin y = 0

en (1, 2, 0).

Solucion. El punto dado pertenece a la superficie 2xy 3 z + z ln x + y sin y = 0 la cual


corresponde al conjunto de nivel cero de la funcion f : D R3 R dada por

f (x, y, z) = 2xy 3 z + z ln x + y sin y

Sabemos que el gradiente es normal a la superficie,


 
1 3 2 3
f (x, y, z) = z + 2y z, sin y + y cos y + 6xy z, 2xy + ln x
x
evaluando

0, 2, 2 (2)3

f (1, 2, 0) =
0, 2, 16 3

=

es un vector normal a la superficie en el punto dado.

Observacion 9.7.2. Recordemos que si u0 Rn la ecuacion del plano que pasa por u0
y que es normal n es dada por:
n (x u0 ) = 0
Por lo tanto, el plano tangente a una superficie S : f (x) = c en a, esta dado por:

: f (a) (x a) = 0

pues f (a) a S y a S

Ejemplo 9.7.2. Encontrar el plano tangente a la superficie:

x2 + y 2 + z 2 = 1

en (1, 0, 0).

Solucion. La superficie es el conjunto de nivel 1 de la funcion h (x, y, z) = x2 + y 2 + z 2 y


el punto (1, 0, 0) pertenece a la superficie, se sigue que el vector normal al plano tangente
es dado por
h (x, y, z) = (2x, 2y, 2z)
as el plano tangente es

h (1, 0, 0) (x 1, y 0, z 0) = 0

407
Apuntes Mat023 (Segundo semestre 2014)

es decir
(2, 0, 0) (x 1, y 0, z 0) = 0
luego
x=1
es el plano tangente.

Ejemplo 9.7.3. Sea C la curva interseccion de la esfera x2 + y 2 + z 2 = R2 (donde R > 0)


y el plano x + y + z = 0. Demuestre que si (x0 , y0 , z0 ) C entonces la recta tangente a la
curva en (x0 , y0 , z0 ) donde x0 , y0 , z0 son reales distintos tiene ecuacion
x x0 y y0 z z0
= =
y0 z0 z0 x0 x0 y 0
Solucion. Definamos

(x, y, z) R3 : x2 + y 2 + z 2 = R2

E =
= (x, y, z) R3 : f (x, y, z) = 0


P = (x, y, z) R3 : x + y + z = 0


= (x, y, z) R3 : h (x, y, z) = 0


donde f (x, y, z) = x2 + y 2 + z 2 R2 y h (x, y, z) = x + y + z entonces C = E P . entonces

f (x0 , y0 , z0 ) C en (x0 , y0 , z0 )
h (x0 , y0 , z0 ) C en (x0 , y0 , z0 )

(el gradiente es perpendicular a los conjuntos de nivel) se sigue que el vector director de la
recta tangente a C en (x0 , y0 , z0 ) es paralelo a

f (x0 , y0 , z0 ) h (x0 , y0 , z0 )

pero

f (x0 , y0 , z0 ) = (2x0 , 2y0 , 2z0 )


h (x0 , y0 , z0 ) = (1, 1, 1)

as

f (x0 , y0 , z0 ) h (x0 , y0 , z0 )
= (2x0 , 2y0 , 2z0 ) (1, 1, 1)
= (2y0 2z0 , 2z0 2x0 , 2x0 2y0 )

as la recta es

(x, y, z) = (x0 , y0 , z0 ) + t (2y0 2z0 , 2z0 2x0 , 2x0 2y0 )

408
Apuntes Mat023 (Segundo semestre 2014)

esto es

x = x0 + 2t (y0 z0 )
y = y0 + 2t (z0 x0 )
z = z0 + 2t (x0 y0 )

despejando t
x x0
= 2t
y0 z0
y y0
= 2t
z0 x0
z z0
= 2t
x0 y 0
de donde obtenemos
x x0 y y0 z z0
= =
y0 z0 z0 x0 x0 y 0
como se queria demostrar.

Observacion 9.7.3. En secciones anteriores encontramos la ecuacion del plano tangente


a la grafica de una funcion diferenciable z = f (x, y) en un punto (x0 , y0 , f (x0 , y0 )) el cual
tiene ecuacion
f f
z = f (x0 , y0 ) + (x0 , y0 ) (x x0 ) + (x0 , y0 ) (y y0 )
x y
Note que la grafica corresponde al conjunto

S = (x, y, z) R3 : z f (x, y) = 0


el cual puede ser interpretado como el conjunto de nivel cero de F (x, y, z) = z f (x, y),
luego en el punto
(x0 , y0 , f (x0 , y0 )) = (x0 , y0 , z0 )
el plano tangente a S tiene ecuacion

F (x0 , y0 , z0 ) (x x0 , y y0 , z z0 ) = 0

pero
F f
(x0 , y0 , z0 ) = (x0 , y0 )
x x
F f
(x0 , y0 , z0 ) = (x0 , y0 )
y y
F
(x0 , y0 , z0 ) = 1
z
409
Apuntes Mat023 (Segundo semestre 2014)

as el plano es
 
f f
(x0 , y0 ) , (x0 , y0 ) , 1 (x x0 , y y0 , z z0 ) = 0
x y

es decir
f f
z = f (x0 , y0 ) + (x0 , y0 ) (x x0 ) + (x0 , y0 ) (y y0 )
x y
La propiedad del gradiente entonces nos permite obtener el resultado obtenido por otros
metodos.

Derivada direccional y direcciones de crecimiento maximo


Definicion 9.8.1. Sean a U , f : U Rn R una funcion cualquiera y u Rn un
vector unitario en Rn (es decir, un vector tal que kuk = 1). Se define la derivada direccional
de f en a en la direccion de u como el lmite:

f f (a + tu) f (a)
Du (a) = (a) = lm
u t0 t
si acaso existe.

Ejemplo 9.8.1. Calcular la derivada direccional def : R2  R, (x, y) f (x, y) =


x3 y + 2x2 y 3 en el punto a = (1, 2) en la direccion u = 12 , 12

Solucion. Por definicion


f f (a + tu) f (a)
(a) = lm
u t0
 t  
f (1, 2) + t 12 , 12 f (1, 2)
= lm
t0
 t
f 1 + t2 , 2 + t2 f (1, 2)
= lm
t0 t
 3    2  3
1 + t2 2 + t2 + 2 1 + t2 2+ t
2
(2 + 24 )
= lm
t0 t
63
= 2 (Usando la regla de L Hopital)
2
Observacion 9.8.1. Note que los vectores canonicos ei , con i = 1, 2, . . . , n, de Rn son
vectores unitarios, luego las derivadas parciales son casos particulares de la derivada
direccional. Mas precisamente, las derivadas parciales son derivadas direccionales en la
direccion de los vectores de la base canonica de Rn .

410
Apuntes Mat023 (Segundo semestre 2014)

f
Observacion 9.8.2. Se debe hacer notar, que u
(a) mide la razon de cambio de f en la
direccion de la recta vectorial:

x = a + tu, tR

as como las derivadas direccionales miden la razon de cambio de f es la direccion de los


ejes coordenados. La interpretacion de estas derivadas es la misma.

Ejemplo 9.8.2. Sea f : R2 R la funcion definida por:

p |x| y

, si x2 + y 2 =
6 0
f (x, y) = x2 + y 2
0 , si x2 + y 2 = 0

Verifique que f posee derivada direccional en (0, 0) en cualquier direccion y que no es


diferenciable en dicho punto.

Solucion. Sea u = (u, v) un vector unitario

f f (tu, tv) f (0, 0)


(0, 0) = lm
u t0 t
|tu| tv
= lm q
t0
t (tu)2 + (tv)2
|t| |u| tv
= lm
t0 t |t| u2 + v 2

= lm |u| v
t0
= |u| v

luego las derivadas direccionales existen en toda direccion. Note que f x


(0, 0) = f
y
(0, 0) = 0
para ello tomar las direcciones (u, v) = (1, 0) y (0, 1) respectivamente.
Para la diferenciabilidad

f (x, y) f (0, 0) f (0, 0) (x 0) f
(0, 0) (y 0)

x y
lm
(x,y)(0,0) k(x, y) (0, 0)k
|xy|
p
x2 + y 2
= lm p
(x,y)(0,0) x2 + y 2
|xy|
= lm
(x,y)(0,0) x + y 2
2

note que este lmite no existe, usando las trayectorias y = x e y = 0 obtenemos lmites
distintos. La funcion no es diferenciable.

411
Apuntes Mat023 (Segundo semestre 2014)

Observacion 9.8.3. Del ejemplo anterior podemos afirmar que la existencia de todas las
derivadas direccionales en un punto no implica diferenciabilidad en el punto.

Teorema 9.8.1. Sea f : U Rn R una funcion diferenciable en a U . Si u =
(u1 , u2 , . . . , un ) es un vector unitario en Rn , entonces:
f
(a) = hf (a) , ui
u
n
X f
= (a) ui
i=1
x i

Demostracion. Definamos g (t) = f (a + tu), esta funcion nes diferenciable en 0 por se


compuesta de diferenciables, ademas por la regla de la cadena

g 0 (t) = f (a + tu) u

evaluando en t = 0 se obtiene
g 0 (0) = f (a) u
por otro lado
g (t) g (0)
g 0 (0) = lm
t0 t0
f (a + tu) f (a)
= lm
t0 t
f
= (a)
u
as
f
(a) = f (a) u
u

y2
Ejemplo 9.8.3. Verifique que la derivada direccional de z = x
en cualquier punto de la
elipse:
2x2 + y 2 = c2
en la direccion de la normal a la curva es nula.

Solucion. Note que la curva es el conjunto de nivel c2 de la funcion g (x, y) = 2x2 + y 2


luego en el punto (x0 , y0 ) de la elipse, la direccion normal es

g (x0 , y0 ) 1
n = =p (4x0 , 2y0 )
kg (x0 , y0 )k 16x20 + 4y02
1
= p 2 (2x0 , y0 )
4x0 + y02

412
Apuntes Mat023 (Segundo semestre 2014)

luego la derivada direccional es


z
(x0 , y0 )
n
g (x0 , y0 )
= z (x0 , y0 )
kg (x , y0 )k
 2 0 
1 y 2y0
= p 2 02 , (2x0 , y0 )
4x0 + y0 2 x0 x0
2y02 2y02
 
1
= p 2 +
4x0 + y02 x0 x0
= 0
como se quera demostrar.
Observacion 9.8.4. Suponiendo las condiciones del teorema anterior, utilizando las
propiedades euclidianas de Rn , obtenemos:
f
(a) = hf (a) , ui
u
= kf (a)k kuk cos
= kf (a)k cos
en donde = (f (a) ; u) [0, ]. Note que:
f
kf (a)k (a) kf (a)k
u
As, si en particular (f (a) ; u) = 0 entonces
f
(a) = kf (a)k
u
Es decir, la maxima razon de cambio de f se obtiene en la direccion del gradiente f (a) y
de manera similar, si (f (a) ; u) = entonces
f
(a) = kf (a)k
u
maximo decrecimiento.
Teorema 9.8.2. Sean U Rn abierto y f una funcion diferenciable en a U , entonces
f (a)
la direccion de maximo crecimiento de f es kf (a)k
. Ademas, obtenemos que:
 
f
kf (a)k = max (a) : kuk = 1
u
De manera similar, la menor derivada direccional de f se obtiene en la direccion de
f (a)
kf (a)k
y
 
f
kf (a)k = mn (a) : kuk = 1
u

413
Apuntes Mat023 (Segundo semestre 2014)

Ejemplo 9.8.4. Un objeto que busca el calor esta localizado en el punto (2, 3) en un
plato de metal plano cuya temperatura en el punto (x, y) es T (x, y) = 30 8x2 2y 2 .
Determine el camino que sigue el objeto si este se mueve en forma continua en la direccion
de maximo incremento de la temperatura en cada punto. Ind. Describir el camino como
curva parametrica x = x (t), y = y (t).

Solucion. El objeto se mueve en la direccion del gradiente luego


 
dx dy
, = kT = k (x, 4y)
dt dt

se sigue
dx
= 16kx
dt
dy
= 4ky
dt
con (x (0) , y (0)) = (2, 3) de donde
dy
dy dt 4ky y
= dx = =
dx dt
16kx 4x
y (2) = 3

resolvemos el P.V.I.
dy y
=
dx 4x
y (2) = 3

la ecuacion tiene solucion y = C 4 x de donde obtenemos

4 3
3=C 2C=
4
2
y as r 
x
y=3 4
2

Ejemplo 9.8.5. Sea f (x, y, z) = 3x2 y + 2xy 2 + z 3 . Calcule la derivada direccional de f en


(1, 0, 1) en la direccion de un vector normal a la superficie:

sin(xy 2 ) + xy 2 z 2 = 2x yz 1

en el punto (0, 1, 1).

414
Apuntes Mat023 (Segundo semestre 2014)

Solucion. Sea g(x, y, z) = sin(xy 2 ) + xy 2 z 2 2x + yz + 1 y considere la superficie S :


g(x, y, z) = 0. En primer lugar, note que P (0, 1, 1) S, pues g(0, 1, 1) = 0. Ademas,
sabemos que:
g(0, 1, 1) P S
en donde P denota perpendicularidad en P (0, 1, 1). As:

gx = y 2 cos(xy 2 ) + y 2 z 2 2
gy = 2xy cos(xy 2 ) + 2xyz 2 + z
gz = 2xzy 2 + y

y por tanto:

g(0, 1, 1) = gx (0, 1, 1), gy (0, 1, 1), gz (0, 1, 1)
= (0, 1, 1)

Por otro lado, como f es diferenciable en (1, 0, 1), puesto que sus derivadas parciales:

fx = 6xy + 2y 2
fy = 3x2 + 4xy
fz = 3z 2

son funciones continuas en (1, 0, 1), tenemos que:

f

(1, 0, 1) = f (1, 0, 1), u
u
en donde f (1, 0, 1) = (0, 3, 3). As, finalmente, un vector unitario en la direccion del
vector normal g(0, 1, 1) se obtiene haciendo u = g(0, 1, 1)/kg(0, 1, 1)k y entonces:

f

(1, 0, 1) = f (1, 0, 1), u
u
D E
= (0, 3, 3), 0, 2/2, 2/2

=0

Ejercicios del captulo

1. Calcule las derivadas parciales de las siguientes funciones:


 
a) f (x, y) = ln sin x+a

y

415
Apuntes Mat023 (Segundo semestre 2014)

z
b) f (x, y, z) = xy
1
c) f (x, y) = y
arctan( x )
d ) f (x, y, z) = (x + z)x+y
e) f (x, y) = arcsin x
x2 +y 2

2. Suponga que g : R R es una funcion continua. Calcule las derivadas parciales de f


si:
R x+y
a) f (x, y) = a
g (t) dt
Z sin(x sin(y+z 2 ))
b) f (x, y, z) = z g (t) dt
xy

3. Calcular las derivadas parciales de las siguientes funciones:

a) f (x, y) = 31 x3 3x2 y + 3xy 2 + y 3


p
b) f (x, y) = ln (x + 2 x2 + 3y 2 )
x2 2xy
c) f (x, y) =
x+y
d ) f (x, y) = x4 x3 y + x2 y 2 xy 3 + y 4
x+y
e) f (x, y) =
xy
f ) f (x, y) = ln(x2 + y 2 )
g) f (x, y) = xy + y x
 
x
h) f (x, y) = x cos
y
 
2
i ) f (x, y) = arctan
x+y
j ) f (x, y, z) = xy+x y zy + z x+y
k ) f (x, y, z) = logx (x + y + z)

2x
4. Pruebe que si f (x, y) = entonces fx (3, 1) + fy (3, 1) = 1
xy
f f
5. Pruebe que si f (x, y) = Ax4 + 28x2 y 2 + Cy 4 , entonces x +y = 4f (x, y)
x y
m+n f (x, y)
6. Sea f (x, y) = ex+y . Pruebe que: = ex+y .
xm y n

416
Apuntes Mat023 (Segundo semestre 2014)

2z 2z
7. Sea z = f (x, y). Verifique que =
xy yx

a) z = x2 4xy + 3y 2
b) z = ln(x + y)
c) z = (x3 + y 3 )10
d ) z = (x + y) sec(xy)
e) z = x2 cos(y 2 )
4
f ) z = arctan(xy)

y 2f 2f
8. Verifique que f (x, y) = arctan satisface la ecuacion + 2 =0
x x2 y
9. Hallar
2z
(x, y),
xy
p
si z = 2xy + y 2 , siendo 2xy + y 2 > 0.

2 kt u 2u
10. Muestre que u(x, t) = en sin(nx) satisface la ecuacion =k 2
t x
11. Sea
x2 y

si (x, y =6= (0, 0)

f (x) = x4 + y 2
0 si (x, y) = (0, 0)

Pruebe que existen todas las derivadas direccionales de f en (0,0) pero f no es


continua en el origen.
xy 3



2 si (x, y) 6= (0, 0)
x + y2
12. Sea f (x, y) =


0 si (x, y) = (0, 0)

a) f es continua en (0,0)?
f f
b) Existen (0, 0), (0, 0)?
x y
4
x (y + 2)2

si (x, y) 6= (0, 2)
x2 + y 2

13. Sea f (x, y) =


0 si (x, y) = (0, 2)

a) Analice la continuidad de f en (0,-2).

417
Apuntes Mat023 (Segundo semestre 2014)

f
b) Calcule, si existe, (0, 2)
x
14. Sea
!
2 2 k 1
(x + (y 1) ) sin si (x, y) 6= (0, 1)

p
x2 + (y 1)2
f (x, y) =



0 si (x, y) = (0, 1)

f f
Determine todos los valores de k R de modo que (0, 1) y (0, 1) existan.
x y
15. Sea y  
x

2
x arctan

y 2 arctan 6 0
si xy =
x y
f (x, y) =


0 si xy = 0

f f
Calcule, si existen, (0, 1) y (1, 0)
x y
16. Verifique que la funcion f : R R2 definida por f (x) = (x2 , x3 ) es diferenciable en
R. Calcule Df (x).

17. Sea f : R3 R2 la funcion definida por:

f (x, y, z) = xy 2 , yz 2


Demuestre, por definicion, que f es diferenciable en R3 . Calcule Df (x, y, z)

18. Considere f : Rn Rm . Suponga que existe M > 0 tal que:

kf (x) k < M kxk2

para todo x Rn . Verifique que f es derivable en 0 y que ademas Df (0) = 0.


p
19. Sea f : R2 R la funcion f (x, y) = |xy|. Es f diferenciable en (0, 0)?

20. Sea:
x2 y 2

, si (x, y) 6= (0, 0)

f (x, y) = x2 + y 4
0 , si (x, y) = (0, 0)

f
a) Calcular x
en cada punto donde exista.
f
b) Decidir x
es continua en el punto (0, 0) y si acaso f es diferenciable en (0, 0).

418
Apuntes Mat023 (Segundo semestre 2014)

21. Demuestre que las siguientes funciones son derivables y calcule su derivada:

a) f (x, y) = ey , sin xy, x2 + 2y 3

b) g (x, y, z, w) = xyzw2 , esin w , x2

22. Calcule f (x, y), en terminos de derivadas de g (x) y h (x) , para las funciones f (x, y)
siguientes:

a) f (x, y) = g (x)
b) f (x, y) = g (y)
c) f (x, y) = g (x + y)2 + g (x)h(y) 3g (x2 3y 2 )

23. Verifique lo siguiente:

z z
a) Si z = xy + xey/x , entonces x x + y y = xy + z.
u u
b) Si u = (x y) (y z) (z x), entonces x
+ y
= u
z
.

z x
a) Se supone que y
= x2 +y 2
. Calcule z = z (x, y).
u x2 +y 2
b) Se supone que x
= x
y que u (1, y) = sin y. Calcule u = u (x, y).

z
24. Sea z = logy x. Calcule y .
 
x+y
25. Sea z = f xy , con f una funcion clase C 1 (R). Verifique que:

z z
x +y =0
x y
 
x+y
26. Sean f : R R y u (x, y) = xy f xy
. Hallar una funcion escalar g (x, y) tal que
se verifique la relacion:
u u
x2 y2 = g (x, y) u (x, y)
x y

27. Calcule:
d
(g f ) (t)

dt t=2
2 t2 3
si f (t) = (t, t 4, e ) y g : R R es tal que:

g g g
(2, 0, 1) = 4 (2, 0, 1) = 2 (2, 0, 1) = 2
x y z

419
Apuntes Mat023 (Segundo semestre 2014)

28. Calcule D h (x, y, z), si:

h (x, y, z) = f (u (x, y, z) , v (x, y) , w (y, z))


 
2u u 2u
29. Se define xy = x y
. Considere z = u (x, y) eax+by tal que xy
= 0. Hallar los
valores de las constantes a, b R de modo que:

2z z z
+z =0
xy x y

30. Sea f : Rp R una funcion. Se dice que f es homogenea de grado m si, para cada
x Rp y cualquier t R se tiene que:

f (t x) = tm f (x)

Demuestre que si f es homogenea de grado m y diferenciable, entonces:

hDf (x) , xi = m f (x)

z
31. Calcule x
, si:
z = f (u)
con u = xy + xy .

32. Hallar u0 (x), si:


u = f (x, y, z)
donde y = (x) y (x, y).
 
33. Sea f (x, y, z) = x sin xy z definida sobre:

A = (x, y, z) R3 : y 6= 0


Considere la superficie:

S = (x, y, z) R3 : f (x, y, z) = 0


Verifique que el plano tangente a S en cualquier punto de S pasa por el origen.

34. Encontrar la ecuacion del plano tangente a la superficie S dada por la grafica de la
funcion:

a) z = x2 + y 2 en el punto (0, 0, 0).


b) z = (x + y)2 2x en el punto (1, 1, 0).

420
Apuntes Mat023 (Segundo semestre 2014)

p
c) f (x, y) = x2 + y 2 + (x2 + y 2 ) en (1, 0, 2).
p 
d ) f (x, y, z) = x2 + 2xy y 2 + 1 en el punto 1, 1, 2, 3 .

35. Considere la superficie:


 
S : z 2 = 3x2 y + cos 2xy +
4

a) Se puede calcular el plano tangente a S en el punto 0, 0, 2 ? Explique.
 
b) Se puede calcular el plano tangente a S en el punto 1, 4 , 23 ? Explique.

36. Sea f (x, y) = x2 xy + 2y 2 . Hallar la derivada direccional de f en el punto (1, 2) y



en direccion que forma con el eje x positivo un angulo de 60 .

37. Sea f (x, y) = x3 2x2 y + xy 2 + 1. Hallar la derivada direccional de f en el punto


(1, 2) en la direccion que va desde este punto al punto (4, 6).

38. Determinar los valores de las constantes a, b, c R de modo que la derivada direccional
de:
f (x, y, z) = a xy 2 + b yz + c z 2 x3
en el punto (1, 2, 1) tenga un valor maximo de 64 en una direccion paralela al eje


X.

39. Sea f : R2 R la funcion definida por:


( 2
x y
x3 y 2
, si x3 y 2 =
6 0
f (x, y) = 3 2
0 , si x y = 0

Verifique que f tiene en (0, 0) derivada en cualquier direccion, pero que no es diferen-
ciable en dicho punto.

40. Calcule la derivada direccional de la funcion f en el punto P y en direccion del vector


v.

a) f (x, y) = 2x2 + 3xy + 4y 2 , P (2, 1), v = (1, 1).


 
x
b) f (x, y) = arctan , (3, 3), v = (3, 4).
y
c) f (x, y, z) = x2 + 3xy + 4y 2 , (1, 1, 2), v = (2, 3, 0).
 z
x
d ) f (x, y, z) = , P (1, 1, 1), v = (2, 1, 1)
y

41. Calcule la derivada de f (x, y) = x4 + x3 y 2 + y en (1, 1) y en la direccion de la tangente


a la curva y = x4 .

421
Apuntes Mat023 (Segundo semestre 2014)

42. Calcule la derivada direccional de f (x, y) = xy + x2 + x4 y, en (1, 1) y en la direccion


del vector que forma una angulo de 60 con el eje x.

43. Encuentre la maxima derivada direccional de f en P .

a) f (x, y) = 2x2 + 3xy + 4y 2 , en P (1, 1).


b) f (x, y, z) = e(x+y+z) , en P (5, 2, 3).

44. Dada la funcion f (x, y) se sabe que:

f 0 (x0 ; v) = 2 si x0 = (1, 2) y v = (2, 2)


0
f (x0 ; v) = 2 si x0 = (1, 2) y v = (1, 1)

Calcule f (1, 2) y f 0 (x0 ; v) cuando x0 = (1, 2) y v = (4, 6)

45. Suponga que la temperatura en el punto (x, y, z) esta dada por T (x, y, z) = 3x2 +
2y 2 4z. Determine el valor de la razon de cambio de T en P (1, 3, 2) en direccion
a Q(4, 1, 2). Cual es la taza de maxima variacion?.

46. Hallar los puntos (x, y) y las direcciones para las que la derivada direccional de
f (x, y) = 3x2 + y 2 tiene el valor maximo si (x, y) esta en el crculo x2 + y 2 = 1.

47. Calcule la derivada direccional de f (x, y, z) = x2 + y 2 z 2 en (3, 4, 5) a lo largo de la


curva interseccion de las superficies 2x2 + 2y 2 z 2 = 25 y x2 + y 2 = z 2 .

48. Calcular la ecuacion del plano tangente a la superficie dada por:

a) f (x, y) = 3x2 + 8xy en el punto (x0 , y0 ) = (1, 0).


p
b) f (x, y) = x2 + y 2 en el punto (x0 , y0 ) = (1, 2)
Z x2 +y2
c) f (x, y) = e t dt en el punto (x0 , y0 ) = (1, 1)
0
 
1 1
d ) f (x, y, z) = sen(xyz) en el punto 3, ,
2 6 3

49. Hallar una constante c tal que en todo punto de la interseccion de las dos esferas:

(x c)2 + y 2 + z 2 = 3 x2 + (y 1)2 + z 2 = 1

los planos tangentes correspondientes sean perpendiculares el uno al otro.

50. Demuestre que la superficie x2 2yz + y 3 = 4 es ortogonal a cualquiera de las


superficies de la familia x2 + 1 = (2 4a)y 2 + az 2 en el punto de interseccion
(1, 1, 2).

422
Apuntes Mat023 (Segundo semestre 2014)

51. Determinar en que punto de la superficie z = 3xy x3 y 3 el plano tangente es


horizontal (paralelo al plano z = 0)
u u u
52. Pruebe que si u = x2 y + y 2 z + z 2 x, entonces + + = (x + y + z)2
x y z
53. Si w = x2 + y 2 + z 2 encuentre el diferencial total de w

xy 2f 2f 2
2 f
54. Sea f (x, y) = x+y
Demuestre que : x2 + 2xy + y =0
x2 xy y 2
55. Probar que el volumen formado por el plano tangente a la superficie xyz = m3 , en
cualquier punto, y los planos coordenados es constante.

56. Hallar
2z
(x, y),
xy
p
si z = 2xy + y 2 , siendo 2xy + y 2 > 0.

57. Calcular la ecuacion del plano tangente a la superficie dada por

z = 3x2 + 8xy
en el punto (x, y) = (1, 0).

58. Calcular las derivadas de segundo orden:

a) f (x, y) = x3 + 3x2 y + 6xy 2 y 3


b) 2x4 3x2 y 2 + y 4 = z

59. Sea f (x, y) = (x2 + y 2 ), considere diferenciable, pruebe que:

f f
y x =0
x y

2u 2
2 u
60. Demuestre que la funcion u(x, t) satisface la ecuacion = a para:
t2 x2
a) u(x, t) = (x at) + (x + at) donde y son funciones de clase C 2 .
b) u(x, t) = sen(akt) sen(kx) con k Z.

61. Se dice que una funcion es armonica si verifica que

2f 2f
2 f = + =0
x2 y 2

423
Apuntes Mat023 (Segundo semestre 2014)

Sea f : R2 R una funcion de clase C 2 armonica y sean

x = eu cos v y = eu sen v

Consideremos la funcion g : R2 R definida como

g(u, v) = f (x(u, v) , y(u, v)).

a) Pruebe que "


 2  2 2  2 #
g g f f
+ = e2u +
u v x y

b) Pruebe que g es armonica. Es decir

2 2g 2g
g= + =0
u2 v 2
62. Si x2 = y 2 + f (x2 + z 2 ), Calcular:

y y z z
xy + yz zx
x x x x

f f
63. Sea f (x, y) = x2 g(x2 y). Pruebe que x 2y = 2f (x, y).
x y
2g
64. Sea f (x, y) = g(x, y) eax+by con = 0. Determine los valores de las constantes a
xy
y b para que
2f f f
+f =0
xy x y

65. Sea f : R2 R la funcion definida por:


( |x|y
, si x3 y 2 6= 0
f (x, y) = x2 +y 2
0 , si x3 y 2 = 0

Verifique que f es continua en (0, 0), que posee derivada direccional en (0, 0) en
cualquier direccion que no es diferenciable en dicho punto.

66. Sea f : R2 R la funcion definida por:


(
x
y
, si y 6= 0
f (x, y) =
0 , si y = 0

f f
a) Calcule x
(0, 0) y y
(0, 0).

424
Apuntes Mat023 (Segundo semestre 2014)

f
b) Sea u = (, ) tal que 6= 0. Existe u
(0, 0)?

67. Una mosca se encuentra volando, circularmente, en alguna cocina. La duena de casa,
ante la presencia del insecto, roca el aire con algun insecticida. La distribucion del
veneno en el aire se puede modelar mediante la superficie:

z = 10 + 6 cos x cos y + 3 cos 2x + 4 cos 3y

Ante el peligro, la mosca establece (rapidamente) un sistema de referencia y determina


que se encuentra en el punto 3 , 3 , 6 . En que direccion debera arrancar la mosca


si quiere sobrevivir?

a) Encontrar la ecuacion del plano tangente a la superficie x = e2yz en (1, 1, 2) y


un vector unitario normal a este plano.
b) Sea f (x, y, z) = z ex sin y y sea a = ln 3, 3

2
, 3 . Calcular el valor de c tal
que a S, donde:
(x, yz) S f (x, y, z) = c
Hallar, ademas, el plano tangente a esta superficie en a.
c) Sea f (x, y) = x2 2 . Hallar el plano tangente a la superficie f (x, y) = z en el
 x +y
punto 3, 4, 35 .

68. Una funcion u esta definida por una ecuacion de la forma:


 
x+y
u = xy f
xy
Demostrar que u satisface una ecuacion en derivadas parciales de la forma:
u u
x2 y2 = u G (x, y)
x y
Calcule, ademas, G (x, y).
1 x
69. Sean k una constante positiva y g (x, t) = 2 kt
. Considere:
Z g(x,t)
2
f (x, t) = eu du
0

Demuestre que:
2f f
k =
x2 t
70. Las ecuaciones:

u = f (x, y) x = X (s, t) y = Y (s, t)

definen u como una funcion de s y t, digamos: u = F (s, t).

425
Apuntes Mat023 (Segundo semestre 2014)

a) Emplee una forma adecuada de la regla de la cadena para expresar las derivadas
parciales F
s
y F
t
en funcion de f , f , X , X , Y y Y
x y s t s t
.
b) Asumiendo que f C 2 , demuestre que:
2 2
2F f 2 X 2 f X X Y 2 f f 2 Y 2 f Y
 
= + 2 +2 + + 2
s2 x s2 x s s s xy y s2 y s
71. Resuelva el ejercicio anterior para los casos:

a) X (s, t) = s + t e Y (s, t) = st.


b) X (s, t) = st e Y (s, t) = st .
c) X (s, t) = 21 (s t) e Y (s, t) = 12 (s + t).

72. La sustitucion x = es e y = et transforma f (x, y) en g (s, t) siendo g (s, t) = f (es , et ).


Se sabe, ademas, que f satisface la ecuacion en derivadas parciales:
2f 2
2 f f f
x2 2
+ y 2
+x +y =0
x y x t
Demuestre que g satisface otra ecuacion de la forma:
2g 2g g g
a2
+ b 2
+c +d =0
s t s t
para ciertas constantes a, b, c, d R. Calcule los valores de dichas constantes.

73. Escriba la ecuacion:


z z
+y x = x2 y
x y
en las variables u y v mediante el cambio de variables:
y
u=x v=
x
74. Represente la ecuacion:
2 2z 2z
+ 2 3 =0
x2 xy y 2
en las variables u y v definidas por el cambio de variables:
u = 3x y v =x+y

75. Escriba la ecuacion:


2z 2z 2z
y + (x + y) + x =0
x2 xy y 2
en terminos de u y v las cuales vienen dadas por el cambio de variables:
u = x2 y 2 v =yx

426
Captulo 10 : Maximos y mnimos

Extremos locales

Definicion 10.1.1. Sean U Rn un conjunto abierto y f : U Rn R una funcion.


Diremos que x0 U es un punto de:

1. Mnimo local, si existe un > 0 tal que x B (x0 , ) U, f (x0 ) f (x)

2. Maximo local, si existe un > 0 tal que x B (x0 , ) U, f (x0 ) f (x)

3. Extremo local si es maximo o mnimo local.

Cuando trabajamos con funciones diferenciables de una variable los extremos locales
ocurren en los puntos en los cuales f 0 (x) = 0, este resultado se extiende a funciones de
varias variables.

Suponga que x0 = (x01 , x02 , . . . , x0n ) es un punto de maximo local de f : U Rn R


entonces existe un > 0 tal que

x B (x0 , ) U, f (x0 ) f (x)

es decir, x U, kx x0 k < f (x0 ) f (x). Definamos la funcion

g : IRR
t g (t) = f t, x02 , x03 , . . . , x0n


427
Apuntes Mat023 (Segundo semestre 2014)

donde I es un intervalo abierto con x01 I entonces



t, x02 , x03 , . . . , x0n x01 , x02 , . . . , x0n < g x01 = f (x0 ) f t, x02 , x03 , . . . , x0n = g (t)
   

pero
t, x02 , x03 , . . . , x0n x01 , x02 , . . . , x0n < t x01 <
 

luego |t x01 | < g (x01 ) g (t) de esta forma g tiene un maximo local en t = x01 se
sigue que como g es diferenciable
g 0 x01 = 0


pero

0 g (x01 + h) g (x01 )
x01

g = lm
h0 h
f (x01 + h, x02 , x03 , . . . , x0n ) f (x01 , x02 , x03 , . . . , x0n )
= lm
h0 h
f (x0 + he1 ) f (x0 ) f (x0 )
= lm =
h0 h x1
se sigue
f (x0 )
=0
x1
el mismo procedimiento se puede realizar para todas las variables.

Teorema 10.1.1. Sea f : U Rn R una funcion diferenciable, donde U es un conjunto


abierto. Si x0 U es un extremo local de f entonces f (x0 ) = 0.

Definicion 10.1.2. Sea f : U Rn R una funcion diferenciable, donde U es un


conjunto abierto. Llamaremos puntos crticos de f a todos aquellos puntos x0 U que
cumplen f (x0 ) = 0. Si x0 es un punto crtico que no es un extremo local entonces x0 se
dice punto de silla.

428
Apuntes Mat023 (Segundo semestre 2014)

Ejemplo 10.1.1. Considere la funcion f :


R2 R, (x, y) f (x, y) = x2 + y 2 , esta
funcion es diferenciable en todo R2 , sus puntos
crticos son aquellos (x, y) tales que

f (x, y) = (0, 0)

pero
 
f f
(x, y) , (x, y) = (2x, 2y) = (0, 0)
x y

es decir, el unico punto crtico es (0, 0). Por


la forma de la funcion sabemos que (0, 0) es
un punto de mnimo ya que

f (0, 0) = 0 x2 + y 2 = f (x, y)

mas aun es un mnimo global (en el, la funcion


asume el menor valor que puede tomar en todo
su dominio)
Ejemplo 10.1.2. Considere la funcion f : R2
R, (x, y) f (x, y) = x2 y 2 , esta funcion es
diferenciable en todo R2 , sus puntos crticos son
aquellos (x, y) tales que

f (x, y) = (0, 0)

pero
 
f f
(x, y) , (x, y) = (2x, 2y) = (0, 0)
x y
es decir, el unico punto crtico es (0, 0). Por la forma de la funcion sabemos que (0, 0)
es un punto de maximo ya que

f (0, 0) = 0 x2 y 2 = f (x, y)

mas aun es un maximo global (en el, la funcion asume el mayor valor que puede tomar en
todo su dominio).

Ejemplo 10.1.3. Considere la funcion f : R2 R, (x, y) f (x, y) = x2 y 2 , esta


funcion es diferenciable en todo R2 , sus puntos crticos son aquellos (x, y) tales que

f (x, y) = (0, 0)

429
Apuntes Mat023 (Segundo semestre 2014)

pero  
f f
(x, y) , (x, y) = (2x, 2y) = (0, 0)
x y
es decir, el unico punto crtico es (0, 0). Note que, si x, y 6= 0 entonces

y 2 = f (0, y) < 0 = f (0, 0) < f (x, 0) = x2

de esta forma, arbitrariamente cerca de (0, 0) hay puntos en los cuales la funcion toma
valores mayores y menores. (0, 0) es un punto de silla.

Figura: La silla de montar

Claramente, no siempre es posible realizar este analisis para ver si los puntos son
maximos, mnimos o puntos de silla, necesitamos algun criterio que nos permita distinguir
entre estos tipos de puntos.
En calculo de una variable tenemos el siguiente criterio: Si x0 es un punto crtico de f ,
entonces f 00 (x0 ) > 0 implica que x0 es punto de mnimo local y si f 00 (x0 ) < 0 de maximo
local. Si f 00 (x0 ) = 0 no hay informacion. Este metodo requiere el signo de la segunda
derivada veamos como se puede generalizar a funciones de varias variables:

Proposicion 10.1.1. Sea n N y f : [a, b] R una funcion tal que f 0 , f 00 , . . . , f (n)


existen en [a, b] ademas f (n) es continua en [a, b] y diferenciable en ]a, b[ entonces existe
c ]a, b[ tal que
n
X f (k) (a) k f (n+1) (c)
f (b) = (b a) + (b a)n+1
k=0
k! (n + 1)!

430
Apuntes Mat023 (Segundo semestre 2014)

Demostracion. La demostracion se basa en el teorema del valor medio, Para x [a, b]


defina n
X f (k) (a)
P (x) = (x a)k
k=0
k!
y F : [a, b] R
 
f (b) P (b)
x F (x) = f (x) P (x) (x a)n+1
(b a)n+1
la funcion esta bien definida ademas
F (a) = F (b) = 0
por el teorema de Rolle existe un c1 ]a, b[ tal que F 0 (c1 ) = 0 pero note que F 0 (a) = 0
entonces existe c2 ]a, c1 [ tal que
F 00 (c2 ) = 0
pero F 00 (a) = 0... este proceso continua hasta la existencia de un c = cn+1 ]a, cn [ tal que
F (n+1) (c) = 0 como P (n+1) (x) 0 se sigue
 
(n+1) f (b) P (b)
f (c) = (n + 1)!
(b a)n+1
es decir
f (n+1) (c)
(b a)n+1 + P (b) = f (b)
(n + 1)!

Si f : [a, b] R es una funcion tal que f 0 , f 00 existen en [a, b] ademas f 00 es continua en


[a, b] y diferenciable en ]a, b[ entonces existe c ]a, b[ tal que
2
X f (k) (a) f (3) (c)
f (b) = (b a)k + (b a)3
k=0
k! (n + 1)!
es decir
f 00 (a) f (3) (c)
f (b) = f (a) + f 0 (a) (b a) + (b a)2 + (b a)3
2 3!
pongamos x0 y x0 + h en lugar de a, b entonces
(3) x0 +h

00 f c
f (x 0 ) x
f (x0 + h) = f (x0 ) + f 0 (x0 ) h + h2 + 0
h3
2 3!
si x0 es un punto crtico f 0 (x0 ) = 0 y as

f 00 (x0 ) 2 f (3) cxx00 +h 3
f (x0 + h) f (x0 ) = h + h
2 3!
(3) x0 +h
 !
00 f c
f (x 0 ) x0
= h2 + h
2 3!

431
Apuntes Mat023 (Segundo semestre 2014)

Suponga que f 00 (x0 ) 6= 0, como



f (3) cxx00 +h
lm h=0
h0 3!
se sigue que existe un > 0 tal que 0 < |h| <

f (3) cx0 +h  |f 00 (x )|
x0 0
<

3! 4

luego para 0 < |h| <



f 00 (x0 ) |f 00 (x0 )| f 00 (x0 ) f (3) cxx00 +h f 00 (x0 ) |f 00 (x0 )|
< + h< +
2 4 2 3! 2 4
 
x +h
f 00 (x0 ) f (3) cx00
de donde podemos concluir que para 0 < |h| < la cantidad 2
+ 3!
h tiene el
mismo signo que f 00 (x0 ). As, si f 00 (x0 ) > 0 entonces
 !
f 00 (x0 ) f (3) cxx00 +h
f (x0 + h) f (x0 ) = h2 + h
2 3!
> h2

luego, para 0 < |h| < se cumple f (x0 + h) > f (x0 ) es decir x0 es un mnimo. El mismo
argumento entrega que x0 es un maximo local cuando f 00 (x0 ) < 0 (notar que el signo de h
no influye por estar al cuadrado).

Teorema 10.1.2 (Formula de Taylor de orden 2 en varias variables). Suponga que U Rn


es un abierto y f : U R tiene derivadas parciales continuas hasta el tercer orden entonces
1
f (x0 + h) = f (x0 ) + f (x0 ) h + hT Hf (x0 ) h + R2 (h, x0 )
2
donde
R2 (h, x0 )
lm =0
khk0 khk2

h1
h2
h= .

..
hn

432
Apuntes Mat023 (Segundo semestre 2014)

2f 2f 2f 2f

(x0 ) (x0 ) (x0 ) (x0 )

x21 x1 x2 x1 x3 x1 xn


2f 2f 2f 2

f
x2 x1 (x0 )
(x0 ) (x0 ) (x0 )
x22 x2 x3 x2 xn


2

Hf (x0 ) = f (x )
2f 2f 2f
0 (x0 ) (x0 ) (x0 )
x x
3 1 x3 x2 x23 x3 xn



.. .. .. .. ..

. . . . .



2f 2f 2f 2f
(x0 ) (x0 ) (x0 ) (x 0 )
xn x1 xn x2 xn x3 x2n

La demostracion es el teorema de Taylor de una variable y la regla de la cadena, en


efecto, si ponemos
g (t) = f (x0 + ht)
entonces

g (1) = f (x0 + h)
g (0) = f (x0 )

y por la regla de la cadena g 0 (t) = Df (x0 + ht) D (x0 + ht) es decir

g 0 (t) = f (x0 + ht) h


n
X f
= (x0 + ht) hi
i=1
xi

as n
0
X f
g (0) = (x0 ) hi = f (x0 ) h
i=1
xi
ademas
n
!
d X f
g 00 (t) = (x0 + ht) hi
dt i=1
x i
n  
X d f
= hi (x0 + ht)
i=1
dt xi

433
Apuntes Mat023 (Segundo semestre 2014)

usando la regla de la cadena


n  
00
X f
g (t) = hi (x0 + ht) h
i=1
xi
n n
!
X X 2f
= hi (x0 + ht) hj
i=1 j=1
xj xi
n Xn
X 2f
= (x0 + ht) hj hi
i=1 j=1
xj xi
n 2
X f
= (x0 + ht) hj hi
i,j=1
x j x i

de manera similar n
000
X 2f
g (t) = (x0 + ht) hj hi hk
i,j,k=1
xk xj xi

se sigue
g 00 (0) 2 g 000 (c) 3
g (1) = g (0) + g 0 (0) 1 + 1 + 1
2! 3!
reemplazando

f (x0 + h)
n n n
X f 1 X 2f 1 X 2f
x0 + hc10 hj hi hk

= f (x0 ) + (x0 ) hi + (x0 ) hj hi +
i=1
xi 2! i,j=1 xj xi 3! i,j,k=1 xk xj xi

note que
n
1 X 2f 1
(x0 ) hj hi = hT Hf (x0 ) h
2! i,j=1 xj xi 2

donde
h1
h2
h=

..
.
hn

434
Apuntes Mat023 (Segundo semestre 2014)

y
2f 2f 2f 2f

(x0 ) (x0 ) (x0 ) (x0 )

x21 x1 x2 x1 x3 x1 xn


2f 2f 2f 2

f
x2 x1 (x0 )
(x0 ) (x0 ) (x0 )
x22 x2 x3 x2 xn


2

Hf (x0 ) = f (x )
2f 2f 2
f
0 (x0 ) (x0 ) (x0 )
x x
3 1 x3 x2 x23 x3 xn



.. .. .. ... ..

. . . .



2f 2f 2f 2
f
(x0 ) (x0 ) (x0 ) (x 0 )
xn x1 xn x2 xn x3 x2n

Observacion 10.1.1. Bajo las hipotesis de continuidad del teorema se tiene


2f 2f
=
xi xj xj xi
por lo tanto la matriz Hf (x0 ) llamada Matriz Hessiana es una matriz simetrica.

Ahora bien, si x0 es punto crtico entonces f (x0 ) = 0 se sigue


n
X f
(x0 ) hi = 0
i=1
xi

luego la expansion en Taylor quedara


1
f (x0 + h) = f (x0 ) + hT Hf (x0 ) h + R2 (h, x0 )
2
y podemos generalizar el criterio de la segunda derivada de funciones reales.

Definicion 10.1.3. Sea A una matriz simetrica. Diremos que una forma cuadratica
CA : Rn R, x CA (x) = xT Ax es definida positiva si para todo x Rn se cumple
CA (x) = xT Ax 0 y CA (x) = 0 solo para x = 0. Diremos que es definida negativa si
para todo x Rn se cumple CA (x) = xT Ax 0 y CA (x) = 0 solo para x = 0.

Recordemos que toda matriz simetrica A es diagonalizable ortogonalmente, se sigue


que existe una matriz Q invertible QQT = I tal que

A = QT DQ

435
Apuntes Mat023 (Segundo semestre 2014)

donde D = diag (1 , 2 , . . . , n ) es una matriz diagonal entonces

xT Ax = (Qx)T D (Qx)

si ponemos
y1
y2
y = Qx =

..
.
yn
entonces n
X
T
x Ax = i yi2
i=1

la forma xT Ax es definida positiva (negativa) si y solo si todos los valores propios de A


son positivos (negativos).

Proposicion 10.1.2. Si la forma xT Ax es definida positiva entonces existe c > 0 tal que

xT Ax c kxk2

En efecto, basta considerar c como el mnimo de los valores propios de la matriz A.


entonces, siguiendo el calculo anterior con

xT Ax = (Qx)T D (Qx)

y1
y2
y = Qx = .

..
yn
se tiene n n
X X
T
x Ax = i yi2 c yi2 = c kyk2
i=1 i=1
pero
kyk2 = kQxk2 = (Qx)T (Qx) = xT QT Qx = xT Ix = kxk2
dando el resultado deseado.

Teorema 10.1.3. Sean U Rn abierto, f : U Rn R una funcion de clase C 3 (U ) y


x0 es un punto crtico de f . Si f (h) = 12 hT Hf (x0 ) h es definida positiva entonces x0 es
un mnimo relativo, si es definida negativa es un maximo relativo.

436
Apuntes Mat023 (Segundo semestre 2014)

Demostracion. Supongamos que es definida positiva, por la proposicion anterior existe un


c > 0 tal que
1 T
h Hf (x0 ) h c khk2
2
y de la expansion de Taylor tenemos
hT Hf (x0 ) h
f (x0 + h) = f (x0 ) + f (x0 ) h + + R2 (h, x0 )
2
donde
R2 (h, x0 )
lm =0
khk0 khk2
como x0 es punto crtico f (x0 ) h = 0 y as
hT Hf (x0 ) h
f (x0 + h) = f (x0 ) + + R2 (h, x0 )
2
f (x0 ) + c khk2 + R2 (h, x0 )
como
R2 (h, x0 )
lm =0
khk0 khk2

existe un > 0 tal que 0 < khk < R2khk
(h,x0 ) c
2 < 2 se sigue para 0 < khk <
 
2 R2 (h, x0 )
f (x0 + h) f (x0 ) khk c +
khk2
2
 c  c khk2
khk c = >0
2 2
as 0 < khk < implica
f (x0 + h) f (x0 )
luego x0 es un punto de mnimo local ( es lo mismo quer decir, si x B (x0 ; ) entonces
f (x) f (x0 )). La demostracion de Maximo local es similar.

Teorema 10.1.4. Sean U Rn abierto, f : U Rn R una funcion de clase C 3 (U ) y


x0 es un punto crtico de f . Si Hf (x0 ) tiene todos sus valores propios no nulos y existen
dos con distinto signo entonces x0 es un punto silla.

Existen otros metodos para determinar si una matriz es definida positiva o negativa.
Sea
a11 a12 . . . a1k . . . a1n
a21 a22 . . . a2k . . . a2
n
.. .
.. . .
. . .. .
.. .
..
.
A= ak1 ak2 . . . akk . . . akn


.. .. .. ..
. . ... . ... .
an1 an2 . . . ank . . . ann

437
Apuntes Mat023 (Segundo semestre 2014)

Denotaremos por Ak a la submatriz



a11 a12 . . . a1k
a21 a22 . . . a2k
Ak =

.. .. .. .
. ..

. .
ak1 ak2 . . . akk

de esta forma

A1 = a11
 
a11 a12
A2 =
a21 a22

a11 a12 a13
A3 = a21 a22 a23
a31 a32 a32
..
.
An = A

si los determinantes de todas estas submatrices son positivos la matriz generara una
forma cuadratica definida positiva y el punto crtico es un mnimo. Si los determinantes
se alternan en signo comenzando con el determinante de A1 negativo, A2 positivo, etc,
entonces la forma cuadratica sera definida negativa y el punto crtico es un maximo local.
Si los determinantes son no nulos y no cumplen con los ordenes de signos anteriores, el
punto critico es punto silla.

Observacion 10.1.2 (Importante). Los criterios anteriores no entregan informacion si


existen valores propios nulos o los subdeterminates son cero.

Ejemplo 10.1.4. Analizar los extremos locales de la funcion f : R2 R, (x, y)


f (x, y) = ln (x2 + y 2 + 1)

Solucion. Los extremos locales estan en los puntos crticos. Los puntos crticos de f son
aquellos puntos (x, y) tales que
f (x, y) = (0, 0)
pero
f 2x
(x, y) = 2
x x + y2 + 1
f 2y
(x, y) = 2
y x + y2 + 1

438
Apuntes Mat023 (Segundo semestre 2014)

se sigue que el unico punto crtico es (x, y) = (0, 0). Veamos si es un maximo, mnimo local
o un punto de silla. Calculemos la Hessiana, para ello necesitamos las derivadas de segundo
orden:
2f 2 (x2 + y 2 + 1) 2x (2x) 2y 2 2x2 + 2
(x, y) = =
x2 (x2 + y 2 + 1)2 (x2 + y 2 + 1)2
2f 4xy
(x, y) =
xy (x + y 2 + 1)2
2

2f 2 (x2 + y 2 + 1) 2y (2y) 2x2 2y 2 + 2


(x, y) = =
y 2 (x2 + y 2 + 1)2 (x2 + y 2 + 1)2
2y 2 2x2 + 2 4xy

(x2 + y 2 + 1)2 (x2 + y 2 + 1)2

Hf (x, y) =



4xy 2x2 2y 2 + 2
(x2 + y 2 + 1)2 (x2 + y 2 + 1)2
entonces  
2 0
Hf (0, 0) =
0 2

2 0
aplicando el criterio de los subdeterminantes vemos que |A1 | = 2 > 0 y =4>0
0 2
luego el extremo local es un mnimo.
Ejemplo 10.1.5. Analizar los extremos de f (x, y) = x3 y + xy 5 + xy.
Solucion. Buscamos los puntos crticos

f (x, y) = (0, 0)

es decir
f
(x, y) = 3x2 y + y 5 + y = y 3x2 + y 4 + 1 = 0

x
f
(x, y) = x3 + 5xy 4 + x = x x3 + 5y 4 + 1 = 0

y
luego el unico punto crtico es (x, y) = (0, 0). La Hessiana en este caso es

3x2 + 5y 4 + 1

6xy
Hf (x, y) =
3x2 + 5y 4 + 1 20xy 3
luego

0 1
Hf (0, 0) =
1 0

439
Apuntes Mat023 (Segundo semestre 2014)

entonces no funciona el metodo de los subdeterminantes pues el primero da cero. Note que
los valores propios de esta matriz son 1 y 1 por lo tanto es un punto de silla. Esto se
puede ver de la funcion misma pues

f (x, y) = xy x2 + y 4 + 1


cerca del (0, 0) hay puntos donde la funcion es negativa y positiva luego es un punto de
silla.
Ejemplo 10.1.6. Clasificar los puntos crticos de f : R3 R, (x, y, z) f (x, y, z) =
x2 z + y 2 z + 23 z 3 4x 4y 10z
Solucion. La funcion esta definida en un abierto y es de clase C , buscamos los puntos
crticos
f (x, y, z) = 2xz 4, 2yz 4, x2 + y 2 + 2z 2 10 = (0, 0, 0)


esto es

2xz 4 = 0
2yz 4 = 0
x + y + 2z 2 10 = 0
2 2

de las dos primeras ecuaciones se sigue que x, y, z 6= 0 y


2 2
x = ,y =
z z
reemplazado en la tercera
8
+ 2z 2 10 = 0
z2
as
z = 1, 2
tenemos 4 puntos crticos

(2, 2, 1) , (2, 2, 1) , (1, 1, 2) , (2, 2, 1)

los clasificaremos usando el criterio de la hessiana y subdeterminantes



2z 0 2x
Hf (x, y, z) = 0 2z 2y
2x 2y 4z
entonces

1 = 2z
2 = (2z)2
3 = 8z x2 + y 2 2z 2


440
Apuntes Mat023 (Segundo semestre 2014)

y as
Punto/Determinante 1 2 3 Tipo de punto
P1 = (2, 2, 1) + + - Punto silla
P2 = (2, 2, 1) - + + Punto silla
P3 = (1, 1, 2) + + + Punto de mnimo local
P4 = (2, 2, 1) - + - Punto de maximo local
notemos que
2
f (0, 0, z) = z 3 10z
3
luego

lm f (0, 0, z) = +
z+
lm f (0, 0, z) =
z

luego la funcion no tiene maximo ni mnimo en todo R3 .

Ejemplo 10.1.7. Muestre que 2 , 2 , 2 es un punto de maximo de la funcion f (x, y, z) =




sin x + sin y + sin z sin (x + y + z)

Solucion. Notemos que 2 , 2 , 2 es un punto crtico pues




f (x, y, z)
= (sin x + sin y + sin z sin (x + y + z))
= (cos x cos (x + y + z) , cos y cos (x + y + z) , cos z cos (x + y + z))

as
 
f , ,
2 2 2
     
= cos cos + , cos cos + , cos cos +
2 2 2 2 2 2
= (0, 0, 0)

la Hessiana es

Hf (x, y, z)

sin (x + y + z) sin x sin (x + y + z) sin (x + y + z)


= sin (x + y + z) sin (x + y + z) sin y sin (x + y + z)




sin (x + y + z) sin (x + y + z) sin (x + y + z) sin z

441
Apuntes Mat023 (Segundo semestre 2014)

as
 
Hf , ,
2 2 2
sin + 2 sin
  
2
sin + 2
sin + 2


sin + 2
   
= sin + sin sin +

2 2 2


sin + 2
   
sin + 2
sin + 2
sin 2

2 1 1


= 1 2 1




1 1 2

los subdeterminantes son

1 = 2
2 = 3
3 = 4

luego el punto crtico es un punto de maximo local.

Ejemplo 10.1.8. Sea f : R3 R una funcion de clase C 2 tal que:

f (x, y, z) = 2x y, 3y 2 x, 2z 2


1 1

Verifique que el punto P , ,
12 6
1 es un punto crtico de f y clasifique el punto crtico P .
1 1

Solucion. El punto P 12 , 6 , 1 es un punto crtico de f si anula el gradiente f . En
efecto:
1 1 1
61 , 3 1 1
 
f , ,
12 6
1 = 2 12 36
12
, 22
= (0, 0, 0)

Ahora bien, como f (x, y, z) = 2x y, 3y 2 x, 2z 2 , tenemos que:

fx = 2x y
fy = 3y 2 x
fz = 2z 2

442
Apuntes Mat023 (Segundo semestre 2014)

Luego, fxx = 2, fxy = 1, fxz = 0, fyy = 6y, fyz = 0 y fzz = 2 . Ademas, como f es de
clase C 2 sobre R3 , la matriz hessiana esta bien definida y vale:
2 1 0

Hf (x, y, z) = 1 6y 0
0 0 2
As:
2 1 0

1 1

Hf 12 , 6 , 1 = 1 1 0
0 0 2
y como:

2 1 2 1
1 = 2 > 0, 2 = = 1 > 0, 3 = 2 =2>0
1 1 1 1
1 1

se concluye que Hf 12 , 6 , 1 es definida positiva, y por el criterio del hessiano, f tiene un
1 1

mnimo local en P 12 , 6, 1 .

Maximos y mnimos en compactos y/o con restricciones

El siguiente teorema generaliza el teorema de una variable que afirma que toda funcion
continua f : [a, b] R alcanza su maximo y mnimo absolutos, es decir, existen x0 , x1
[a, b] tales que f (x0 ) f (x) f (x1 ) para todo x [a, b].

Teorema 10.2.1 (De Weiertrass). Sean K Rn un conjunto cerrado y acotado y f :


K R una funcion continua entonces existen puntos x0 y x1 en K tales que
x K, f (x0 ) f (x) f (x1 )
esto es f alcanza su maximo y mnimos absolutos en K.
Para utilizar tal teorema es conveniente la siguiente proposicion.
Proposicion 10.2.1. La imagen inversa de un conjunto abierto por una funcion continua
es un conjunto abierto. La imagen inversa de un cerrado por una funcion continua es un
conjunto cerrado.
Por ejemplo, si consideramos la funcion continua f (x, y) = y x2 entonces
f 1 (]0, +[) = f 1 ({u R : u > 0})
= (x, y) R2 : f (x, y) ]0, +[


= (x, y) R2 : y x2 > 0


= (x, y) R2 : y > x2


443
Apuntes Mat023 (Segundo semestre 2014)

es un conjunto abierto.
Si consideramos h (x, y) = x2 + y 2 entonces h es continua y

h1 ([1, 2]) = (x, y) R2 : h (x, y) [1, 2]




= (x, y) R2 : 1 x2 + y 2 2


es la imagen inversa de un cerrado por una funcion continua, entonces es un conjunto


cerrado.

Ejemplo 10.2.1. Hallar los valores extremos de f (x, y) = x2 + y 2 x y + 1 en el disco


D = {(x, y) R2 : x2 + y 2 1}

Solucion. La funcion f (x, y) = x2 + y 2 x y + 1 es continua. Note que el disco lo


podemos ver como
D = h1 (], 1])
donde h (x, y) = x2 + y 2 entonces D es cerrado y esta contenido en la bola B ((0, 0) ; 2)
luego es acotado. Por el teorema de los extremos absolutos se tiene que f (x, y) alcanza su
maximo y mnimos absolutos en D, es decir, existen puntos (u0 , v0 ) y (u1 , v1 ) en D tales
que
(x, y) D, f (u0 , v0 ) f (x, y) f (u1 , v1 )
Note que tales extremos pueden estar en la bola abierta B = {(x, y) R2 : x2 + y 2 < 1} o
en la circunferencia unitaria

S 1 = (x, y) R2 : x2 + y 2 = 1


Primero busquemos los puntos crticos en B.


 
f f
f (x, y) = (x, y) , (x, y) = (0, 0)
x x
= (2x 1, 2y 1) = (0, 0)

el unico punto crtico en B es (x, y) = 12 , 12 . El conjunto S 1 lo podemos parametrizar por




c : [0, 2] S 1 tal que t c (t) = (cos t, sin t), de esta forma la funcion f en el conjunto
S 1 es
f (cos t, sin t) = 2 cos t sin t
para t [0, 2] como es una funcion de una variable buscamos sus valores extremos con el
calculo diferencial y el teorema de los extremos en R.

f 0 = 0 sin t cos t = 0

444
Apuntes Mat023 (Segundo semestre 2014)

tiene las soluciones t = 4 y t = 4 + en el intervalo ]0, 2[ luego se tienen los puntos


crticos
     
1 1 1 1 1 1
, , , , , y (1, 0) = (cos 0, sin 0) = (cos 2, sin 2)
2 2 2 2 2 2
evaluando en la funcion obtenemos
   2  2
1 1 1 1 1 1 1
f , = + +1=
2 2 2 2 2 2 2

 
1 1 1 1 1 1
f , = + +1=2 2
2 2 2 2 2 2

 
1 1 1 1 1 1
f , = + + + +1=2+ 2
2 2 2 2 2 2
f (1, 0) = 1 1 + 1 = 1
 
de donde 2 , 2 es el punto de maximo y 21 , 12 es el punto de mnimo en el disco.
1 1


2 2
Nota: La funcion es f (x, y) = x 12 + y 12 + 12 el valor mnimo se alcanza en
1 1
yel maximoen el disco sera en el punto del disco que encuentre mas lejos de 12 , 21
 
,
2 2
este es 12 , 12 .

Extremos restringidos Multiplicadores de Lagrange

Es comun en problemas querer maximizar o minimizar alguna funcion sujeta a res-


tricciones dadas. Por ejemplo; En economa suponga que se quiere vender dos tipos de
productos I y II. Sean x e y la cantidad de productos I y II respectivamente. Representemos
por f (x, y) la ganancia obtenida cuando se vende una cantidad x de I y y de II, nuestra
produccion esta limitada o controlada por nuestro capital, de esta forma tenemos una
restriccion del tipo g (x, y) = c. El problema podra plantearse como

max f (x, y)
(x,y)R2 ,g(x,y)=c

Supongamos que x0 es un punto extremo del problema

Optimizar f (x)
xRn ,g(x)=c

donde f : U Rn R y optimizar es maximo o mnimo. Sea c : R Rn donde

g (c (t)) = c

445
Apuntes Mat023 (Segundo semestre 2014)

y c (0) = x0 entonces por la regla de la cadena

g (c (t)) c0 (t) = 0

se sigue que
g (x0 ) c0 (0) = 0
como x0 es extremo de la f se sigue que f (c (t)) tiene un maximo o mnimo en t = 0 luego

f (c (t)) c0 (t) = D (f c) (t)


f (x0 ) c0 (0) = 0

de esta forma f (x0 ) y g (x0 ) deben ser paralelos es decir R tal que

f (x0 ) = g (x0 )

el numero se llama multiplicador de Lagrange y la funcion de n + 1 variables

L (x, ) = f (x) (g (x) c)

es llamado Lagrangiano del problema. Podemos entonces formular el siguiente teorema:

Teorema 10.3.1. Sean f, g : U Rn R funciones de clase C 1 . Sea x0 U un punto


tal que g (x0 ) = c y g (x0 ) 6= 0. Si

S = {x U : g (x) = c}

y f |S tiene un extremo local en x0 entonces existe 0 R tal que

f (x0 ) = 0 g (x0 )

Observacion 10.3.1. Es lo mismo que buscar los puntos crticos del Lagrangiano L (x, ) =
f (x) (g (x) c)

Ejemplo 10.3.1. Sea S R2 la recta que pasa por (1, 0) inclinada en un angulo de 45
y sea f : R2 R, (x, y) x2 + y 2 . Hallar los extremos de f |S .

Solucion. La pendiente es tan (45) = 1 entonces

y0=x1

se sigue que la restriccion es


g (x, y) = y x + 1 = 0

446
Apuntes Mat023 (Segundo semestre 2014)

buscamos los puntos crticos del Lagrangiano


L (x, y, ) = x2 + y 2 (y x + 1)
as
L
(x, y, ) = 2x + = 0
x
L
(x, y, ) = 2y = 0
y
L
(x, y, ) = y x + 1 = 0

Note que de la primera y segunda se obtiene

= 2y = 2x y = x =
2
reemplazando en la tercera se obtiene
1 1
y= x=
2 2
1
, 12

geometricamente podemos deducir que este punto es un mnimo (x0 , y0 ) = 2
. Note
que el punto es un extremo de f |S pero no de f .
Ejemplo 10.3.2. Maximizar la funcion f (x, y, z) = x + z sujeta a la restriccion x2 + y 2 +
z 2 = 1.
Solucion. El conjunto S = {(x, y, z) R3 : x2 + y 2 + z 2 = 1} es cerrado y acotado y f es
una funcion continua en todo R2 de donde obtenemos que f |S debe tener un maximo y un
mnimo absolutos. Los buscamos con el metodo de Lagrange. Pongamos el Lagrangiano
L (x, y, z, ) = x + z x2 + y 2 + z 2 1


y debemos buscar sus puntos crticos


L
(x, y, z, ) = 1 2x = 0
x
L
(x, y, z, ) = 2y = 0
y
L
(x, y, z, ) = 1 2z = 0
z
L
(x, y, z, ) = x2 + y 2 + z 2 1 = 0

note que = 0 o y = 0. = 0 no puede ser (sistema inconsistente) entonces y = 0 se sigue
de la primera y tercera ecuaciones
1
x =
2
1
z =
2
447
Apuntes Mat023 (Segundo semestre 2014)

as x = z y reemplazando en la ultima
1
2x2 = 1 x =
2
as obtenemos dos puntos crticos
   
1 1 1 1
, 0, y , 0,
2 2 2 2
reemplazando se tiene que el primero es un maximo y el segundo un mnimo.
Ejemplo 10.3.3. Hallar el mayor volumen que pueda tener una caja rectangular con tapa
sujeta a la restriccion de que el area de la superficie sea 10 m2 .
Solucion. Sean x, y, z los lados de la caja entonces el volumen es V (x, y, z) = xyz, vemos
que esta funcion esta restringida a [0, 10] [0, 10] [0, 10] luego debe alcanzar el maximo
y mnimo absolutos. El area sera
2 (xy + xz + zy) = 10
as el Lagrangiano sera
L (x, y, z, ) = xyz (xy + xz + zy 5)
buscamos los puntos crticos
L
(x, y, z, ) = yz (y + z) = 0
x
L
(x, y, z, ) = xz (x + z) = 0
y
L
(x, y, z, ) = xy (x + y) = 0
z
L
(x, y, z, ) = xy + xz + zy 5 = 0

note que x 6= 0 pues si fuese cero zy = 5 pero z = 0 se sigue = 0 y as de la primera
yz = 0 contradiccion, lo mismo para y, z.
como son cantidades positivas tambien las sumas seran distintas de cero
yz xz xy
= =
y+z x+z x+y
esto implica x = y = z sustituyendo en la ultima ecuacion nos da
3x2 = 5
de donde obtenemos el punto crtico
r r r !
5 5 5
, ,
3 3 3
q 3
5
el volumen maximo es V = 3
.

448
Apuntes Mat023 (Segundo semestre 2014)

Ejemplo 10.3.4. Encuentre los extremos absolutos de la funcion f (x, y) = 2x2 +y 2 2y +1


sujeto a la restriccion x2 + y 2 4.

Solucion. La funcion es continua y el conjunto compacto, por el teorema de Weierstrass


se alcanza el maximo y mnimo absolutos en la region. Buscamos los puntos crticos al
interior y en el borde usamos multiplicadores de Lagrange

f (x, y) = (4x, 2y 2) = (0, 0)



(x, y) = (0, 1)

en el borde los extremos deben cumplir

(4x, 2y 2) = (2x, 2y)


x2 + y 2 = 4

es decir

4x = 2x
2y 2 = 2y
x2 + y 2 = 4

de la primera
4x 2x = 2x (2 ) = 0 x = 0 = 2
si x = 0 entonces y = 2 tenemos los puntos crticos (0, 2). Si = 2 entonces

2y 2 = 4y

y = 1 de donde
x2 + 1 = 4

x = 3, 3 tenemos los puntos crticos 3, 1 evaluando la funcion en (0, 1) , (0, 2)

y 3, 1 se obtiene

f (0, 1) = 2 02 + 12 2 (1) + 1 = 0

   2
f 3, 1 = 2 3 + (1)2 2 (1) + 1 = 10
f (0, 2) = 2 (0)2 + (2 1)2 = 1
f (0, 2) = 2 (0)2 + (2 1)2 = 9

se sigue que (0, 1) es el punto de mnimo global y 3, 1 puntos de maximo global.

449
Apuntes Mat023 (Segundo semestre 2014)

Ejemplo 10.3.5. Una pieza larga de lamina galvanizada con ancho w se ha de doblar en
forma simetrica con tres lados rectos para hacer un canal de agua lluvia. En la figura se
muestra una seccion transversal. Determine las dimensiones que permiten el maximo flujo
de agua.

Solucion. Vamos a maximizar el area de la seccion transversal. Con los datos de la figura
 
1
A (x, ) = 2 (x cos ) (x sin ) + (w 2x) (x sin )
2
= x2 sin cos + (w 2x) (x sin )
x2
= sin (2) + wx sin 2x2 sin
2

450
Apuntes Mat023 (Segundo semestre 2014)

donde (x, ) 0, w2 0, 2 buscamos los puntos crticos al interior


   

A = x2 cos (2) + wx cos 2x2 cos


Ax = x sin (2) + w sin 4x sin

despejando x en las dos

x2 cos (2) + wx cos 2x2 cos = 0


 
cos
x = w
cos 2 2 cos

x sin (2) + w sin 4x sin = 0


 
sin
x = w
sin 2 4 sin
de donde obtenemos
   
cos sin
w = w
cos 2 2 cos sin 2 4 sin

(cos ) (sin 2 4 sin ) = (sin ) (cos 2 2 cos )

as

2 cos2 sin 4 cos sin = cos2 sin 2 cos sin sin3



2 cos2 4 cos = cos2 2 cos sin2

luego

2 cos2 4 cos cos2 2 cos sin2



= 0

1 2 cos = 0

se sigue

=
3
y as
!
sin 3

x = w
sin 2
 
3
4 sin 3
w
=
3
451
Apuntes Mat023 (Segundo semestre 2014)

miramos la Hessiana
 
sin (2) 4 sin 2x cos (2) + w cos 4x cos
HA (x, ) =
2x cos (2) + w cos 4x cos 2x2 sin (2) wx sin + 2x2 sin
w

evaluando en el punto ,
3 3
se tiene
 w   3 3 12 w

HA , = 2
3 3 12 w 16 3w2

as
3
1 = 3<0
2
1 2
2 = w >0
2
es un maximo local. Note que

(w/3)2
w   
2 w    w 2  
A , = sin +w sin 2 sin
3 3 2 3 3 3 3 3
1 2
= 3w
12
La funcion en los bordes de la caja es

A (0, )
= 0
w  (w/2)2 w 1 1
A , = sin (2) + w sin 2 (w/2)2 sin = w2 sin 2 w2
2 2 2 8 8
A (x, 0)
= 0
  1
A x, = wx 2x2 w2
2 8
2
se sigue que w3 , 3 es punto de maximo global y el maximo es 12 1

3w

x2
sin (2) + 3x sin 2x2 sin
2

452
Apuntes Mat023 (Segundo semestre 2014)

Este resultado tiene una extension a mas restricciones:

Si tenemos las restricciones una restriccion S dada por los puntos que satisfacen

g1 (x1 , x2 , . . . , xn ) = c1
g2 (x1 , x2 , . . . , xn ) = c2
..
.
gk (x1 , x2 , . . . , xn ) = ck

y f restringida a S tiene un extremo en x0 S entonces, si g1 (x0 ) , g2 (x0 ) , , gk (x0 )


son l.i. existiran 1 , 2 , . . . , k R tales que

f (x0 ) = 1 g1 (x0 ) + 2 g2 (x0 ) + + k gk (x0 )

Ejemplo 10.3.6. Determine los extremos absolutos de f (x, y, z) = x + 2y + z sujeto a


las restricciones x2 + y 2 = 1 y y + z = 1.

Solucion. f (x, y, z) = x + 2y + z es una funcion continua. El conjunto definido por las


restricciones

S = (x, y, z) R3 : x2 + y 2 = 1 y + z = 1


= (x, y, z) R3 : x2 + y 2 = 1 (x, y, z) R3 : y + z = 1
 

note que el primer conjunto restringe a x, y al conjunto [1, 1] y eso restringe a z en el


segundo conjunto. Es un conjunto acotado e interseccion de dos cerrados. Luego es cerrado
y acotado la funcion alcanza un maximo y mnimo absolutos en S.

453
Apuntes Mat023 (Segundo semestre 2014)

El Lagrangiano sera

L (x, y, z, , ) = (x + 2y + z) x2 + y 2 1 (y + z 1)


buscamos los puntos crticos


L
= 1 2x = 0
x
L
= 2 2y = 0
y
L
= 1=0
z
L
= x2 + y 2 1 = 0

L
= y+z1=0

resolvemos = 1 reemplazando en la segunda obtenemos

1 = 2y

de donde ni y pueden ser cero. De la primera obtenemos


1
x= =y
2
de donde obtenemos en la cuarta
1
x = y =
2
de donde podemos obtener z en la 5. As
   
1 1 1 1 1 1
, ,1 y , , 1 +
2 2 2 2 2 2
evaluando en la funcion f (x, y, z) = x + 2y + z se tiene
 
1 1 1 1 2 1 2
f , ,1 = + +1 = +1
2 2 2 2 2 2 2
 
1 1 1 1 2 1 2
f , , 1 + = + 1 + = 1
2 2 2 2 2 2 2
el primer es maximo y el segundo un mnimo.

454
Apuntes Mat023 (Segundo semestre 2014)

Ejemplo 10.3.7. Sean

A = (x, y, z, w) R4 : x + y z 2w = 1


y
B = (x, y, z, w) R4 : x y + z + w = 2


determine el punto de A B que se encuentra a menor distancia al cuadrado del origen.

Solucion. Vamos a resolver este problema de dos formas

mn (x2 + y 2 + z 2 + w2 )
x + y z 2w =1
xy+z+w =2

usamos Lagrange con dos restricciones

(2x, 2y, 2z, 2w) = (1, 1, 1, 2) + (1, 1, 1, 1)


x + y z 2w = 1
xy+z+w = 2

esto es

2x = +
2y =
2z = +
2w = 2
x + y z 2w = 1
xy+z+w = 2

sistema que tiene solucion x = 27


19
7
, y = 19 7
, z = 19 3
, w = 19 , = 20
19
, = 34
19
de donde
obtenemos que  
27 7 7 3
, , ,
19 19 19 19
es el punto de mnimo por la geometra del problema y por ser el unico punto crtico o ver
siguiente seccion. La distancia cuadrado mnima es
 2  2  2  2
27 7 7 3 44
+ + + =
19 19 19 19 19
Otra forma

x + y z 2w = 1
xy+z+w = 2

455
Apuntes Mat023 (Segundo semestre 2014)

implica
1 3
x = w+
2 2
3 1
y = w+z
2 2
se sigue que buscamos el mnimo de
 2  2
1 3 3 1
w+ + w+z + z 2 + w2
2 2 2 2

calculamos su gradiente para buscar puntos criticos


 2  2 !  2  2 !
1 3 3 1 1 3 3 1
w+ + w+z + z 2 + w2 , w+ + w+z + z 2 + w2
z 2 2 2 2 w 2 2 2 2
= (0, 0)

esto es

3w + 4z 1 = 0
7w + 3z = 0

se sigue
3
w =
19
7
z =
19
se sigue que es un punto de mnimo pues
 
4 3
H=
3 7

tiene determinante 19 y la primera entrada es 4. El valor mnimo es


   2      2  2  2
1 3 3 3 3 7 1 7 3 44
+ + + + + =
2 19 2 2 19 19 2 19 19 19

Ejemplo 10.3.8. Hallar y clasificar los extremos de la funcion f (x, y, z) = 2x2 +y 2 +z 2 xy


en la region R R3 definida por las ecuaciones:

x2 + y 2 + z 2 1 1/2 x 1/2 y 1/2

456
Apuntes Mat023 (Segundo semestre 2014)

Solucion. Analizamos los extremos de f (x, y, z) = 2x2 + y 2 + z 2 xy, primeramente, en


todo R3 a traves del criterio de la segunda derivada. Es decir, resolvemos el problema
sin restricciones y detectamos los puntos que pertenece al conjunto R. As, buscamos los
puntos (x, y, z) tales que:

f (x, y, z) = (4x y, 2y x, 2z) = (0, 0, 0)

Luego, el unico punto candidato a extremo es P 0 (0, 0, 0) R y para obtener su clasificacion


se procede mediante la matriz hessiana, que viene dada por:
4 1 0

Hf (x, y, z) = 1 2 0
0 0 2

Note que la matriz Hf (x, y, z) es definida positiva en el punto P 0 , pues:



4 1 4 1
1 = 4 > 0, 2 = = 7 > 0, 3 = 2 = 14 > 0
1 2 1 2

Por tanto, la funcion f tiene en P 0 mnimo local y ademas, f (P0 ) = 0. Por otro lado,
analizamos f restringida a la funcion g(x, y, z) = x2 + y 2 + z 2 1 mediante el metodo de
los multiplicadores de Lagrange. As, el lagrangeano del problema esta definido como:

L(x, y, z, ) = f (x, y, z) g(x, y, z)

y los posibles extremos cumplen que L(x, y, z, ) = (0, 0, 0, 0). Entonces, tenemos el
sistema:

4x y 2x = 0
2y x 2y = 0
2z 2z = 0
x + y2 + z2 = 1
2

Note que de la tercera ecuacion tenemos que las soluciones de 2z 2z = 0 son: z = 0 o


= 1. Luego, analizamos cada caso por separado:
Si = 1 se tiene el sistema:

4x y 2x = 0
2y x 2y = 0

el cual permite obtener como puntos crticos los puntos P1 = (0, 0, 1) y P2 = (0, 0, 1), de
 
donde se sigue que f P1 = 1 y f P2 = 1. Ahora bien, como la region R es compacta1 se
tiene que los puntos P1 y P2 son maximos.
1
i.e. R es cerrado y acotado en R3 .

457
Apuntes Mat023 (Segundo semestre 2014)

Para el caso z = 0, tenemos que x2 + y 2 = 1. Luego el sistema a resolver es:

4x y 2x = 0
2y x 2y = 0

el cual es un sistema lineal de la forma:


    
4 2 1 x 0
=
1 2 2 y 0

el cual tiene como solucion nula x = y = 0 si el determinante de la


matriz es no nulo,
esto
3+ 2 3 2
se tiene para los tal que 42 12 + 7 = 0; es decir, 1 = y 2 = (Este
2 2
caso ya fue analizado). Resta analizar el caso en que el determinante sea cero, es decir,
tenemos dos rectas paralelas las cuales son:

y = 1 2 x

x= 1 2 y
 
Luego, solo debemos analizar las rectas l1 : y = 1 + 2 x y l2 : y = 1 2 x y ocupar
la restriccion x2 + y 2 = 1. Para la recta l1 los puntos crticos son:
 
1
, 1+ 2
, 0
4+2 2 4+2 2
 
mientras que para la recta l2 los puntos crticos son: 1
, 1+ 2
, 0 . Luego,
42 2 42 2
usando las restricciones que definen R y las aproximaciones se descartan todos los puntos
a excepcion de:
 
P3 = 1 , 1+ 2 , 0 = (0,3827 , 0,9239 , 0)
4+2 2 4+2 2

y en el cual se tiene f P3 = 0,7929, el cual es un maximo local dada la compacidad de R.

Criterio de la segunda derivada para extremos condicionados

En esta seccion presentamos un teorema que nos permite clasificar el punto obtenido
con los multiplicadores de Lagrange:

Teorema 10.3.2 (Hessiana orlada). Sean f, g : U Rn R funciones de clase C 2 .


Sean v0 U, g (v0 ) = c, g (v0 ) =
6 0 y existe un 0 R tal que f (v0 ) = 0 g (v0 ). Si

458
Apuntes Mat023 (Segundo semestre 2014)

L (, x) = f (x) (g (x) c) y
g g g
0
x 1
x2
x n
g 2L 2L 2L
x1 x21 x1 x2
x1 xn

g 2L 2L 2L
H = HL (v0 ) = x2 x2 x

x1 x2 x22

n
.. .. .. ... ..
. . . .
g 2L 2L 2L

xn x1 xn x2 xn
x2

n

entonces:

1. Si

g g g
g g
0 x x x
0 x x 1 2 3

g 2L 2L 2L

x1
1 2
g 2L 2L x21 x1 x2 x1 x3
H3 = x1 < 0, H4 = g < 0, . . .

x21 x1 x2 2L 2L 2L
g
x 2L 2L
x2 x1 x2 x22 x2 x3
x22 2L 2L 2L
x1 x2
g
2
x3

x1 x3 x2 x3 x2

3

entonces v0 es mnimo local de f sujeto a g (x) = c.

2. Si los determinantes se alternan en la forma H3 > 0, H4 < 0, H5 > 0 etc, entonces


v0 es maximo local de f sujeto a g (x) = c

3. Si los determinantes anteriores son distintos de cero pero siguen los patrones anteriores
entonces el punto es de silla.

Ejemplo 10.3.9. Estudiar los extremos de la funcion f (x, y) = cos2 x + cos2 y sujeta a la
restriccion x y = 4 .

Solucion. la funcion de Lagrange es F (, x, y) = cos2 x + cos2 y x y 4 se sigue




F  
= xy
4
F
= 2 cos x sin x
x
F
= 2 cos y sin y +
y
luego

sin (2x) =
sin (2y) =

xy =
4
459
Apuntes Mat023 (Segundo semestre 2014)

se sigue
  
sin 2 y + = sin (2y)
4
cos 2y = sin (2y)

as

cos 2y + sin 2y = 0

 
sin 2y + = 0
4

k
y = con k Z
2 8
se sigue  
k k
+ , con k Z
2 8 2 8
son los puntos crticos. Calculamos la Hessiana orlada
2
F2 F F
0 1 1

x y

2F
F F

x x2 xy = 1 2 cos 2x
0

F F 2F
2 cos 2y


y xy y 2
1 0

pero

2 cos 2x = 2 (1)k

2 cos 2y = 2 (1)k

se sigue
2F

F F

2
0 1 1


F
x
2F
y
F
k+1

k
= 1 2 (1) 0 = 2 (1) 2

x x2 xy
F F 2F
2 (1)k+1

1 0
y xy y 2

2F

F F
2 x y
F 2F F
se sigue que si k es impar < 0 y el punto es mnimo y si k es par es

x x2 xy
F F 2F


y xy y 2

460
Apuntes Mat023 (Segundo semestre 2014)

un maximo. Note que

f (x, y) = cos2 x + cos2 y


     
k k 2 k 2 k
f + , = cos + + cos
2 8 2 8 2 8 2 8

1 + cos k 4
 
1 + cos k + 4
= +
2 2
1    
= 1+ cos k + + cos k
2 4 4
1  
= 1+ 2 (1)k
2
2 (1)k
= 1+
2
para k impar  
k k 2
f + , =1
2 8 2 8 2
para k par  
k k 2
f + , =1+
2 8 2 8 2

Ejemplo 10.3.10. Hallar las dimensiones del mayor paraleleppedo rectangular de aristas
paralelas a los ejes coordenados que puede ser inscrito en el elipsoide de ecuacion:
 x 2  y 2  z 2
+ + =1
3 4 5

461
Apuntes Mat023 (Segundo semestre 2014)

Solucion. El volumen del paraleleppedo es

V (x, y, z) = (2x) (2y) (2z) = 8xyz

pero (x, y, z) debe estar sobre el elipsoide


 x 2  y 2  z 2
+ + =1
3 4 5
y sus coordenadas son positivas o cero. Tenemos que resolver el problema

max V (x, y, z)
r(x,y,z)=0
x,y,z0

2 2 2
donde V (x, y, z) = 8xyz y r (x, y, z) = x3 + y4 + z5 1. Este problema tiene
solucion pues el elipsoide es un conjunto cerrado y acotado ademas la funcion es continua.
Determinaremos el valor maximo utilizando los multiplicadores de Lagrange

V (x, y, z) = r (x, y, z)
r (x, y, z) = 0

esto es
 
2x 2y 2z
(8yz, 8xz, 8xy) = , ,
32 42 52
 x 2  y 2  z 2
+ + = 1
3 4 5
el sistema a resolver es
2x
8yz =
32
2y
8xz =
42
2z
8xy =
52
 x 2  y 2  z 2
+ + = 1
3 4 5
multiplicando las primeras tres ecuaciones por x, y, z respectivamente se obtiene

x2
4xyz =
32
y 2
4xyz =
42
z 2
4xyz =
52
462
Apuntes Mat023 (Segundo semestre 2014)

sumando las tres ecuaciones


x2 y 2 z 2
12xyz = 2
+ 2 + 2
3 4 5
 x 2  y 2  z 2 
= + +
3 4 5
=

as
x2 y 2 z 2
= 4xyz = 2 = 2 = 2
3 3 4 5
si 6= 0 entonces

2 32 3
x = x=
3 3
42 4
y2 = y=
3 3
52 5
z2 = z=
3 3
si = 0 entonces xyz = 0 y

yz = 0
xz = 0
xy = 0
 x 2  y 2  z 2
+ + = 1
3 4 5
esto nos da los puntos (3, 0, 0) , (0, 0, 5) , (0, 4, 0). Estos 3 puntos son puntos de mnimo
V =0y
160
     
3 4 5 3 4 5
V , , =8 = 3
3 3 3 3 3 3 3
es el valor maximo.

Ejercicios del captulo

1. Determine los valores de a, b R para los cuales la funcion f (x, y) = ax2 + by 2 tiene
un maximo, mnimo o punto silla en (0, 0).

2. Sea C R. Muestre que si f : U Rn R tiene un maximo (mnimo) local en


x = x0 entonces la funcion F : U R definida por F (x) = f (x) + C tambien tiene
un maximo (mnimo) local en x = x0 .

463
Apuntes Mat023 (Segundo semestre 2014)

3. Determinar los valores de a, b R para los cuales la funcion f : R2 R definida por

f (x, y) = x3 + 3ax + y 3 + by + 1

no tiene extremos locales.

4. Para cada una de las siguientes funciones determinar los puntos crticos (o puntos
estacionarios), clasificarlos en maximos, mnimos y puntos silla:
a) z = x2 + (y 1)2 i) z = x2 xy + y 2 2x + y
b) z = x2 (y 2)2 j) z = sin (x) sin (y) sin (x + y)
z = (5x + 7y 25) e(x +xy+y )
2 2
c) z = 1 x2 y 2 k)
= (x2 + y 2 ) e(x +y )
2 2
d) z = (x y 1)2 l) z
e) z = 2x2 xy 3y 2 3x + 7y m) z = xy 2 (x + 2y 1)
f) z = x (x2 + y 2 a2 ) n) z = x3 x2 y x2 + y 2
g) z = x3 + y 3 3 (x + y) + 1 o) z = (x + y) (x2 + y 2 6)
h) z = xy + (x y)3 p) z = (x y)3 + x4 + y 4

5. Sea f (x, y) = 3x4 4x2 y + y 2 . Demostrar que sobre toda recta de la forma y = mx la
funcion tiene un mnimo en (0, 0) pero no es mnimo en ningun entorno bidimensional
del origen. (Estudiar los puntos donde f (x, y) > 0 y f (x, y) < 0).

6. Considere la funcion z = f (x, y) que en los alrededores del punto (1, 1, 1) esta definida
implcitamente por
z 3 + 3x2 y y 3 z + y 2 3x 1 = 0
obtener la expansion de Taylor para z en (1, 1).

7. Sea f : R4 {(0, 0, 0, 0)} R dada por


y z u 1
f (x, y, z, u) = x + + + +
x y z u
tiene un punto crtico en (1, 1, 1, 1) y clasificarlo.

8. Sea g : R R una funcion diferenciable que no se anula. Analizar los extremos de


Z (x1)2 Z (y1)2
f (x, y) = g (t) dt + g (t) dt
0 0

si g (0) > 0.

9. Determine los puntos crticos de la funcion

f (x, y) = 2x4 + y 4 4x2 2y 2

y clasificarlos (obs.: Son 9 puntos).

464
Apuntes Mat023 (Segundo semestre 2014)

11.* Considere la funcion z = f (x, y) definida implcitamente por la expresion

F (x, y, z) = x3 + y 3 + z 3 3x 3y + z + 4 = 0

obtener sus puntos crticos y clasificarlos.

10. Determine los valores de las constantes a, b R para los cuales la integral
Z 1
(a + bx f (x))2 dx
0

sea mnimo (suponer f es continua en [0, 1]), estudiar los casos f (x) = x2 y f (x) =
x3 + x.

11. Sea f (x, y) = Ax2 + 2Bxy + Cy 2 + 2Dx + 2Ey + F donde A > 0 y B 2 < AC.

a) Demostrar que f tiene un mnimo.


b) Demostrar que en el punto de mnimo (x1 , y1 ) se cumple

f (x1 , y1 ) = Dx1 + Ey1 + F

c) Demostrar que

A B D
1
f (x1 , y1 ) = B C E
AC B 2
D E F

12. Metodo de los mnimos cuadrados: Dados n numeros distintos x1 , x2 , . . . , xn y


otros n numeros (no necesariamente distintos) es en general imposible encontrar una
recta f (x) = ax + b que pase por todos los puntos (xi , yi ) esto es f (xi ) = yi i, no
obstante se puede encontrar los valores de a y b que hacen que el error cuadratico
total n
X
E (a, b) = (f (xi ) yi )2
i=1

sea mnimo. Encontrar tales valores!!!

13. Comprobar que f (x, y, z) = x4 + y 4 + z 4 4xyz tiene un punto crtico en (1, 1, 1)


y clasificarlo como maximo, mnimo o punto silla. Expandir esta funcion en Taylor
entorno al punto (1, 1, 1) para visualizar directamente la forma cuadratica asociada a
la Hessiana Hf (1, 1, 1).

14. Estudiar los extremos relativos de la funcion f (x, y, z) = xyz (1 x y z)

465
Apuntes Mat023 (Segundo semestre 2014)

15. Sea A Mnn (R), B M1n (R) y C R, se define F : Rn R

F (X) = X T AX + BX + C

donde X Mn1 (R) (el vector se mira como columna). Estudiar las condiciones
bajo las cuales esta funcion tiene maximos, mnimos o puntos silla.

16. Considere la funcion f : R3 R dada por f (x, y, z) = sin x + sin y + sin z


sin (x + y + z) determine la expansion de Taylor de f en 2 , 2 , 2 , muestre ademas


que el punto es un mnimo.

17. Considere la funcion z = f (x, y) que en los alrededores del punto (1, 1, 1) esta definida
implcitamente por
z 3 + 3x2 y y 3 z + y 2 3x 1 = 0
obtener la expansion de Taylor para z en (1, 1).

18. La Hessiana de cierta funcion f : D R3 R es

x2 y xz

Hf (x, y, z) = y y 2 yx
xz yx z 2

En que subconjunto de R3 debe estar el punto critico (x, y, z) para que sea maximo,
mnimo o punto silla? Cuando esta Hessiana no entrega informacion?.

19. Determine los extremos de la funcion f (x, y) = 2x y con la restriccion g (x, y) =


3x2 + 2y 2 33/2 = 0

20. Determine los extremos de la funcion f (x, y) = x2 + 8y si (x, y) se encuentra sobre


la elipse x2 + 4y 2 = 5.

21. Utilizar el metodo de los multiplicadores para determinar los semiejes de la elipse
5x2 6xy + 5y 2 32 = 0.

22. Usando multiplicadores de Lagrange, determinar la menor distancia entre las rectas
x+4 y4 z+1
= =
2 1 2
x+5 y5
= =z55
4 3

23. Demostrar que el paraleleppedo de mayor volumen que se puede inscribir en una
esfera es un cubo.

466
Apuntes Mat023 (Segundo semestre 2014)

24. Hallar los puntos de la curva x2 + 4y 2 4 = 0 que se encuentren mas cercanos y mas
lejanos a los puntos de la curva

x2 + y 2 + 4x + 2y 20 = 0

x3
25. Hallar los extremos absolutos de la funcion f (x, y) = 3
32 x2 + 2x + y 2 2y + 1 en
la region K = {(x, y) : x 0, y 0, x + y 1}.

26. Hallar los extremos absolutos de la funcion f (x, y, z) = (x 1)2 + (y 1)2 + (z 1)2
en la region
K = (x, y, z) R3 : x2 + y 2 + z 2 12


28.* Suponga que queremos determinar los extremos de la funcion f (x, y, z) sujeta a la
restriccion g (x, y, z) = 0 y utilizando el teorema de Lagrange obtenemos el punto
crtico p0 = (x0 , y0 .z0 ), utilizando el teorema de la funcion implcita demostrar que
el problema se puede reducir a estudiar los extremos de una funcion de 2 variables
(con el mismo punto crtico), calcular la Hessiana de esta funcion de 2 variables y
expresarla en terminos de f y g para obtener un criterio que permita determinar si
el punto crtico entregado por Lagrange es maximo, mnimo o silla (Esto es llamado
criterio de la hessiana reducida).

27. La fabrica de gelatinas BOB esta interesada en minimizar sus costos de produccion.
Los ingredientes utilizados en la elaboracion de su gelatina son tres: Grenetina, azucar
y frutas. La formula de produccion dice que el numero de gelatinas que se puede
producir es
g (x, y, z) = x y z
donde x es la cantidad de grenetina, y es la cantidad de azucar y z es la cantidad
de frutas (todas las cantidades en kilogramos) y las constantes , , son secretas.
Suponga que el precio por kilogramo de los ingredientes es: La grenetina $10,000,
azucar $250 y fruta $500. Si se quiere producir 10,000 gelatinas, Que combinacion
de ingredientes minimiza los costos c (x, y, z) = 10,000x + 250y + 500z? (la respuesta
puede quedar en terminos de , , ).

467
Captulo 11 : Funciones implcitas e inversas

El teorema de la funcion implcita

En los cursos anteriores, frecuentemente nos hemos encontrado con relaciones de la


forma
F (x, y) = 0
donde F : R2 R es una funcion de clase C 1 (), por ejemplo, una elipse
x2 y 2
+ 2 = 1 F (x, y) = 0
a2 b
donde
x2 y 2
F (x, y) = + 2 1
a2 b
o la ecuacion cuadratica general
Ax2 + Bxy + Cy 2 + Dx + Ey + F = 0
Estas ecuaciones generalmente definen una curva en el plano que localmente (esto es, en
intervalos alrededor de un punto dado) puede ser descrita como la grafica de una funcion
y = (x) o de una funcion x = (y).

Por ejemplo considere la funcion F : R2 R, (x, y) F (x, y) = x2 + y 2 1, esta


funcion es clase C (R2 ) y
F (x, y) = 0 x2 + y 2 = 1
Todos los puntos (x, y) con y > 0 que satisfacen
x2 + y 2 = 1
corresponden a puntos de la grafica de la funcion

(x) = 1 x2 para x [1, 1]


y= 1 x2
1

0,5

1 0,5 0,5 1

0,5

468
Apuntes Mat023 (Segundo semestre 2014)

esto es, si (x, y) Graf (), entonces F (x, y) = 0. En efecto

(x, y) Graf () (x, y) = (x, (x)) para algun x [1, 1]


 
(x, y) = x, 1 x2 para algun x [1, 1]

luego (x, y) Graf () implica


 
F (x, y) = F x, 1 x2
 2
= x2 + 1 x2 1
= 0

lo que demuestra que todos los puntos de la grafica de cumplen la relacion F (x, y) = 0
(estan sobre la circunferencia unitaria). De manera similar, los puntos (x, y) con y < 0 que
satisfacen F (x, y) = 0 corresponden a puntos de la grafica de la funcion

y = (x) = 1 x2 para x [1, 1]

sin embargo, cerca de los puntos (1, 0) y (1, 0) no podemos interpretar, los puntos de
la curva como puntos del grafico de una funcion de y = f (x), en este caso los puntos
corresponden a la grafica de
p p
x = (y) = 1 y 2 para y [1, 1] x = (y) = 1 y 2 para y [1, 1]

respectivamente. En resumen, en cada punto (x0 , y0 ) R2 que cumple F (x0 , y0 ) = 0


existe una funcion : ]x0 , x0 + [ R tal que (x0 ) = y0 y que cumple para cada
x ]x0 , x0 + [, F (x, (x)) = 0 o bien existe una funcion : ]y0 , y0 + [ R tal
que (y0 ) = x0 y
F ( (y) , y) = 0
para cada y ]y0 , y0 + [.

469
Apuntes Mat023 (Segundo semestre 2014)

y=(x)
y0

x0 x0 x0 +

Si tenemos la funcion G : R2 R, (x, y) G (x, y) = x7 + y 7 + xy 1 y la ecuacion

G (x, y) = 0
x7 + y 7 + xy = 1

el problema no es tan sencillo. Cerca de que puntos (x, y) podemos encontrar una funcion
y = (x) cuya grafica coincida con la curva dada? es decir, los puntos del grafico de
y = (x) esten sobre la curva

x7 + 7 (x) + x (x) = 1

supuesto que hemos resuelto el problema de la existencia de tal funcion, Que propiedades
de G hereda la funcion y = (x)? es diferenciable? si es as Cuanto vale su derivada?.
Supongamos que la funcion que encontramos y = (x) es derivable, entonces tenemos

G (x, (x)) = 0

por la regla de la cadena


G (x) G
(x, (x)) + (x, (x)) 0 (x) = 0
x x y

470
Apuntes Mat023 (Segundo semestre 2014)

de esto se tiene
G (x, (x))
0 (x) = x
G
y
(x, (x))
G
siempre que y
(x, (x)) 6= 0.

Definicion 11.1.1. Sea F : U Rn Rm Rm una funcion. Diremos que la funcion


: V Rn Rm , x (x) esta definida implcitamente por la ecuacion
F (x, y) = 0
si x V se cumple
F (x, (x)) = 0
Ejemplo 11.1.1. La funcion : R R definida por
sr
3 1 6 1 2x
(x) = x 8x3 + x3 + rq
4 2 1
3
4
x6 8x3 + 12 x3

esta definida implcitamente por la ecuacion x3 + 6xy = y 3 . En la figura se muestra la curva


x3 + 6xy = y 3 y en verde la parte de ella descrita por . En este caso, puede resultar mas
simple trabajar directamente con la ecuacion que define la funcion en lugar de trabajar
con la expresion explcita de la funcion.

471
Apuntes Mat023 (Segundo semestre 2014)

El teorema de la funcion implcita nos entrega condiciones sobre la funcion que define
la ecuacion, para garantizar todas estas propiedades

Teorema 11.1.1. Sean U abierto de Rn+1 y F : U R es una funcion de clase C p (U ).


Denotemos por (x, z) los puntos de Rn+1 donde x Rn y z R. Supongamos que (x0 , z0 )
satisface
F
F (x0 , z0 ) = 0 y (x0 , z0 ) 6= 0
z
entonces existe una bola abierta U que contiene a x0 Rn , una vecindad V de z0 R
y una unica funcion z = g (x) definida para x U y con recorrido en V que satisface
z0 = g (x0 ),
F (x, g (x)) = 0 para todo x U
ademas z = g (x) es de clase C p (U) y
Dx F (x, g (x))
Dg (x) = F
z
(x, g (x))

donde Dx F es la derivada respeto a x es decir


 
F F
Dx F = ,...,
x1 xn
en terminos de componentes
F
g x (x, g (x))
(x) = F i
xi z
(x, g (x))
para i = 1, 2, . . . , n.

Ejemplo 11.1.2. Sea G : R3 R una funcion de clase C 1 (R3 ). Considere la superficie

G (x, y, z) = 0

suponga ademas que (x0 , y0 , z0 ) es un punto sobre la superficie, es decir

G (x0 , y0 , z0 ) = 0

si
G
(x0 , y0 , z0 ) 6= 0
z
por el teorema de la funcion implcita existe un conjunto abierto U que contiene (x0 , y0 ) y
una funcion z = z (x, y) definida en U tal que z0 = z (x0 , y0 ) ademas

G (x, y, z (x, y)) = 0 para (x, y) U

472
Apuntes Mat023 (Segundo semestre 2014)

el plano tangente a la grafica de z = z (x, y) en el punto (x0 , y0 , z0 ) es

z z
z z0 = (x0 , y0 ) (x x0 ) + (x0 , y0 ) (y y0 )
x y
pero note que
z G (x0 .y0 , z0 )
(x0 , y0 ) = xG(x0 ,y0 ,z0 )
x
z
y
z G
y
(x0 .y0 , z0 )
(x0 , y0 ) = G(x0 ,y0 ,z0 )
y
z
reemplazando

G
x
(x0 .y0 , z0 ) G
y
(x0 .y0 , z0 )
z z0 = G(x0 ,y0 ,z0 )
(x x0 ) + G(x0 ,y0 ,z0 )
(y y0 )
z z

as
G G G (x0 , y0 , z0 )
(x0 .y0 , z0 ) (x x0 ) + (x0 .y0 , z0 ) (y y0 ) + (z z0 ) = 0
x y z
que es
G (x0 , y0 , z0 ) ((x, y, z) (x0 .y0 , z0 )) = 0
que es la ecuacion del plano tangente a una superficie que obtuvimos por otros medios (el
gradiente es perpendicular a los conjuntos de nivel)

Ejemplo
 11.1.3. Suponga que F : R3 R es una funcion de clase C 1 (R3 ) que cumple
F F F
 
x y z
6= 0 muestre que las funciones z = z (x, y), x = x (y, z) y y = y (x, z)
definidas implcitamente por
F (x, y, z) = 0
cumplen
z x y
= 1
x y z
Solucion. Como F x
6= 0 se sigue que la ecuacion define a x como funcion de y y z, es
decir, x = x (y, z) ademas
F
x y
= F
y x
 
de manera similar (usando F F F
 
x y z
6= 0) se tiene y = y (x, z) y z = z (x, y) donde

F F
z x y z
= F y = F
x z
z y

473
Apuntes Mat023 (Segundo semestre 2014)

entonces
! F
! !
F F
z x y x y z
= F
F
F
x y z z x y
= 1

Ejemplo 11.1.4. Cerca de cuales puntos es posible representar la superficie

x3 + 3y 2 + 8xz 2 3z 3 y = 1

como grafica de una funcion diferenciable z = k (x, y)?


Solucion. Son aquellos puntos para los cuales la derivada parcial respecto a la variable z
sea no nula, es decir

16xz 9z 2 y 6= 0
z (16x 9zy) 6= 0

Observacion 11.1.1. Tambien se puede intentar despejar las otras variables, formular el
teorema en tales casos.
Observacion 11.1.2. De la ecuacion

x3 + 3y 2 + 8xz 2 3z 3 y = 1

tambien se puede obtener una expresion para las derivadas parciales (derivando la ecuacion
respecto a x)
z z
3x2 + 8z 2 + 16xz 9z 2 y =0
x x
as
z 3x2 8z 2
=
x 16xz 9z 2 y

Ejemplo 11.1.5. Sea f : R2 R, (u, v) f (u, v) una funcion de clase C 1 (R2 ). Supon-
gamos que f
v
6= 0 en todo R2 . Muestre que la ecuacion

f x2 y 2 , y 2 z 2 = 0


define implcitamente una funcion z = z (x, y) para z 6= 0 y que la expresion


z z
E (x, y, z) = yz + zx
x y
no depende de f .

474
Apuntes Mat023 (Segundo semestre 2014)

Solucion. Definamos G (x, y, z) = f (x2 y 2 , y 2 z 2 ) entonces G es compuesta de funcio-


nes C 1 y por tanto es C 1 , ademas
G f u f v
= +
z u z v z
f
= (2z) 6= 0
v
por el teorema de la funcion implcita existe una unica funcion z = z (x, y) de clase C 1 tal
que
z G fu ux + fv vx
= Gx =
x z
fv (2z)
  
fu 2x + fv 0 fu x
= =
fv (2z) fv z
similarmente
z G
y fu uy + fv vy
= G =
y z
fv (2z)
fu (2y) + fv (2y) fu y + fv y
= =
fv (2z) fv z
se sigue
     
z z fu x fu y + fv y
yz + zx = yz + zx
x y fv z fv z
= xy

luego
z z
yz + zx = xy
x y
no depende de f .

Estudiaremos ahora el caso mas general en el cual F : RN Rm Rm . Supongamos


que tenemos un sistema de ecuaciones lineales

2u + 3v + x y = 0
u v + 2x + 3y = 0

sabemos que este sistema tiene infinitas soluciones 4 variables y dos ecuaciones (es cuestion
de rangos), las infinitas soluciones son porque podemos dejar variables en funcion de otras,
notemos lo siguiente

2u + 3v = b1 = y x
u v = b2 = 2x 3y

475
Apuntes Mat023 (Segundo semestre 2014)

para resolver el sistema

2u + 3v = b1
u v = b2

podemos utilizar Cramer y se obtiene



b1 3 yx 3

b2 1 2x 3y 1 7 8
u= = = x y
2 3 2 3 5 5

1 1 1 1

y
2
b1 2
yx

1 b2 1 2x 3y 3 7
v= = = x+ y
2 3 2 3 5 5

1 1 1 1

luego las ecuaciones

2u + 3v + x y = 0
u v + 2x + 3y = 0

permiten despejar a u y v como funciones de x e y,


7 8
u (x, y) = x y
5 5
3 7
v (x, y) = x+ y
5 5
Ahora suponga que tenemos ecuaciones no lineales

xu + yvu2 = 2
xu3 + y 2 v 4 = 2

en este caso se nos hace mucho mas difcil despejar u, v en funcion de x e y. Estas ecuaciones
las podemos reinterpretar como

F (x, y, u, v) = xu + yvu2 2, xu3 + y 2 v 4 2 = (F1 , F2 ) = (0, 0)




note que las ecuaciones se satisfacen en el punto (1, 1, 1, 1). Sabemos que una buena
aproximacion de F (x, y, u, v) cerca de (1, 1, 1, 1) es dada por

F (1, 1, 1, 1) + DF (1, 1, 1, 1) (x 1, y 1, u 1, v 1)T

476
Apuntes Mat023 (Segundo semestre 2014)

donde
!
F1 F1 F1 F1
x y u v
DF (x, y, u, v) = F2 F2 F2 F2
x y u v

u vu2 x + 2yvu yu2


 
=
u3 2yv 4 3xu2 4v 3 y 2
evaluando en el punto  
1 1 3 1
DF (1, 1, 1, 1) =
1 2 3 4
luego
 
1 1 3 1
F (x, y, u, v) (x 1, y 1, u 1, v 1)T
1 2 3 4
 
3u + v + x + y 6

3u + 4v + x + 2y 10
luego la ecuacion
F (x, y, u, v) = (0, 0)
debe ser similar a

3u + v + x + y = 6
3u + 4v + x + 2y = 10

en el cual podemos despejar u y v en funcion de x e y, para poder hacer esto necesitamos



3 1
= D(u,v) F (1, 1, 1, 1)
3 4
F1 F1


u
(1, 1, 1, 1) v
(1, 1, 1, 1)
= F2 F2

u
(1, 1, 1, 1) v (1, 1, 1, 1)
6= 0

(permite usar Cramer para despejar las variables) Despejando



6xy 1

10 x 2y 4 14 2 1
u= = y x
3 1 9 9 3

3 4
y
3 6xy

3 10 x 2y 4 1
v= = y
3 1 3 3

3 4

477
Apuntes Mat023 (Segundo semestre 2014)

de donde debera cumplirse


1 2
ux = , uy =
3 9
1
vx = 0, vy =
3
(note que deberamos poner
14 2 1
u (x, y) y x
9 9 3
4 1
v (x, y) y
3 3
(es una aproximacion de primer orden de estas funciones) cerca del punto (1, 1, 1, 1)). Estas
ideas se resumen en el siguiente:

Teorema 11.1.2 (De la funcion implcita). Sean abierto de RN , abierto de Rm ,


F : Rm una funcion de clase C p ( ). Pongamos (x, y) para los puntos en
RN Rm . Suponga que (x0 , y0 ) es un punto tal que

F (x0 , y0 ) = 0

y que Dy F (x0 , y0 ) es invertible, entonces existe un abierto U RN , una funcion y = g (x)


de U a Rm de clase C p (U ) tal que

F (x, g (x)) = 0 para todo x U

y0 = g (x0 ) ademas
Dg (x) = Dy (x, y)1 Dx (x, y)

Ejemplo 11.1.6. Mostrar que cerca del punto (x, y, u, v) = (1, 1, 1, 1) podemos resolver

xu + yvu2 = 2
xu3 + y 2 v 4 = 2
u

de manera unica para u y v como funciones de x y y. Calcular x
(1, 1)

Solucion. Definamos

F : R2 R2 R2
xu + yvu2 2, xu3 + y 2 v 4 2

((x, y) , (u, v))

entonces
F ((1, 1) , (1, 1)) = (0, 0)

478
Apuntes Mat023 (Segundo semestre 2014)

calculemos D(u,v) F (x, y, u, v)



(xu+yvu2 2) (xu+yvu2 2)
D(u,v) F (x, y, u, v) = u v
(xu3 +y 2 v 4 2) (xu3 +y 2 v 4 2)
u v

x + 2yuv yu2
 
=
3xu2 4y 2 v 3

en (1, 1, 1, 1) es
 
3 1
D(u,v) F (1, 1, 1, 1) =
3 4
el determinante es 12 3 = 9 6= 0 se sigue que es invertible y as por el teorema de la
funcion implcita podemos despejar de manera unica para u y v como funciones de x y y.
Ademas g (x, y) = (u (x, y) , v (x, y)) tiene por derivada
1
x + 2yuv yu2

Dg (x, y) = D(x,y) (x, y, u, v)
3xu2 4y 2 v 3

donde

(xu+yvu2 2) (xu+yvu2 2)
x y
D(x,y) (x, y, u, v) = (xu3 +y 2 v 4 2) (xu3 +y 2 v 4 2)

x y

u vu2
 
=
u3 2yv 4

entonces 1 
x + 2yuv yu2 u vu2
 
Dg (x, y) =
3xu2 4y 2 v 3 u3 2yv 4
en el punto en cuestion
1 
31 92
   
3 1 1 1
Dg (1, 1) = =
3 4 1 2 0 31

de donde obtenemos
!
u u
13 29
 
x
(1, 1) y
(1, 1)
v v =
x
(1, 1) y
(1, 1) 0 13

se sigue
u 1
(1, 1) =
x 3

479
Apuntes Mat023 (Segundo semestre 2014)

Ejemplo 11.1.7. Discutir sobre la resolubilidad del sistema


3x + 2y + z 2 + u + v 2 = 0
4x + 3y + z + u2 + v + w + 2 = 0
x + z + w + u2 + 2 = 0
para u, v, w en terminos de x, y, z en el punto x = y = z = 0, u = v = 0 y w = 2. Si es
posible, obtener la ecuacion del plano tangente a
w (x, y, z) = 0
en (0, 0, 0).
Solucion. Definamos F : R6 R3 dada por
F (x, y, z, u, v, w)
3x + 2y + z 2 + u + v 2 , 4x + 3y + z + u2 + v + w + 2, x + z + w + u2 + 2

=
el sistema pude verse como
F (x, y, z, u, v, w) = (0, 0, 0)
notamos que

1 2v 0
D(u,v,w) F (0, 0, 0) = 2u 1 1
2u 0 1 (0,0,0)

1 0 0
= 0 1 1
0 0 1
que tiene determinante 1, por el teorema de la funcion implcita se sigue que podemos
despejar (u, v, w) en terminos de (x, y, z) ademas
u u u 1
x y z

1 0 0 3 2 0
v v v
x y z = 0 1 1 4 3 1
w w w

x y z
0 0 1 1 0 1
(0,0,0)

3 2 0
= 3 3 0
1 0 1
de esto obtenemos
W = (1, 0, 1)
y as el plano es
(1, 0, 1) (x, y, z) = 0
x+z = 0

480
Apuntes Mat023 (Segundo semestre 2014)

Ejercicios de la seccion

1. Sea F : R2 R, (u, v) F (u, v) una funcion de clase C que satisface

F (0, 0) = 0
F F
(0, 0) + (0,0) 6= 0
u v

Muestre que la ecuacion


F x3 yz, y 3 xz = 0


define una funcion z = z (x, y) de clase C en un entorno del punto (x, y) = (1, 1)
que satisface z (1, 1) = 1 y la ecuacion
 z  z
xz + 3y 3 + yz + 3x3 = 9x2 y 2 z 2
x y

2. Probar que cerca del punto (x0 , y0 , z0 , u0 , v0 ) = 1, 1, 0, 2 , 0 se puede resolver el




sistema

x2 y cos (uv) + z 2 = 0
x2 + y 2 sin (uv) + 2z 2 2 = 0
xy sin u cos v + z = 0

de manera unica para x, y, z como funciones de u y v. Calcular x



v 2
,0
 
z z
3. Sea f : R R definida por f (x, y, z) = g x + y , y + x donde g : R2 R y
3

f
z
(x, y, z) 6= 0

a) Verificar que la ecuacion f (x, y, z) = 0 define implcitamente a z como una


funcion de clase C 1 de las variables x e y.
b) Comprobar que
z z
x +y = z xy
x y

4. Mostrar que el sistema

x2 y 2 + u2 + 2v 2 = 5
x2 + y 2 u2 v 2 = 4

define implcitamente a u = u (x, y), v = v (x, y) con u (0, 1) = 2 y v (0, 1) = 1.


2v
Encontrar las diferenciales Du (0,1) y Dv (0, 1) ademas de la derivada x2 (0, 1).

481
Apuntes Mat023 (Segundo semestre 2014)

5. Suponga que F, G : R3 R son dos funciones de clase C 1 (R3 ) y x0 = (x0 , y0 , z0 ) R3


es un punto en el cual F (x0 ) = 0, G (x0 ) = 0, esto es un punto de la interseccion de
las superficies

S1 : F (x, y, z) = 0
S2 : G (x, y, z) = 0

Determine una condicion que permita asegurar que es posible despejar x = x (y),
z = z (y) como funciones de clase C 1 en un entorno de y0 luego la interseccion se
puede parametrizar como

x = x (y)
y = y
z = z (y)

para y en un entorno de y0 (el problema nos dice cuando podemos parametrizar con
un parametro la interseccion de dos superficies)

482
Apuntes Mat023 (Segundo semestre 2014)

El teorema de la funcion inversa

Si f : R R es una funcion de clase C 1 (R) y x0 es un punto al que f 0 (x0 ) 6= 0 entonces,


por la continuidad de la derivada, podemos garantizar la existencia de un intervalo abierto
I tal que f 0 (x) para todo x I tiene el mismo signo de f 0 (x0 ), entonces la funcion f
es estrictamente creciente o estrictamente decreciente en I (dependiendo si f 0 (x0 ) > 0 o
f 0 (x0 ) < 0).

f f0

f 0 (x0 )

x0

En la grafica podemos observar en verde la grafica de una funcion y en azul la de su


funcion derivada, en el punto x0 la derivada es no nula (estrictamente positiva), existe un
intervalo (marcado en rojo) en el cual la funcion derivada es positiva, note que en todo ese
intervalo la funcion f es estrictamente creciente.
Si restringimos nuestra funcion f al intervalo I entonces f es inyectiva (toda funcion
estrictamente creciente o estrictamente decreciente es inyectiva pues enva numeros distintos
del dominio en imagenes distintas), si ademas restringimos el recorrido a f (I) entonces es
sobreyectiva y podemos definir una inversa (no de la funcion completa, se trata de una
inversa local.

483
Apuntes Mat023 (Segundo semestre 2014) f0

f 1

note que solo podemos garantizar la existencia, obtener una expresion para ella puede ser
muy complicado. Como podemos trabajar entonces con tal funcion?, Cuales propiedades
de f conserva su inversa?.
El teorema de la funcion inversa de una variable asegura que la inversa es una funcion
de la misma clase (en terminos de derivadas) ademas

f f 1 (x) = x


entonces
d
f f 1 (x) = 1
 
dx
por la regla de la cadena
df 1  df 1
f (x) (x) = 1
dx dx
entonces
df 1 1
(x) = 0 1
dx f (f (x))
si queremos calcular la derivada de segundo orden
d2 f 1 d 1
2
(x) = 0 1
dx dx f (f (x))
 
0 1
2 00 1  1
= f f (x) f f (x)
f 0 (f 1 (x))
f 00 (f 1 (x))
=
(f 0 (f 1 (x)))3
Si T : Rn Rn es una transformacion lineal, sabemos que es invertible si y solo si su
matriz asociada (por ejemplo respecto a las bases canonicas) es invertible, sabemos que la
matriz asociada respecto a las bases canonicas corresponde a la matriz Jacobiana de la T .

JT (x0 ) = [T ]CC

484
Apuntes Mat023 (Segundo semestre 2014)

luego, si det (JT (x0 )) 6= 0 la transformacion lineal es invertible (derivada no nula implica
invertible), es razonable esperar que si una funcion se parece a una transformacion lineal
localmente entonces podamos obtener informacion de ella a traves de la transformacion
lineal a la cual se parece, eso es justamente lo que afirma el teorema de la funcion inversa
de varias variables.

Observacion 11.2.1. Supongamos que U, V son abiertos y F : U Rn V Rm


(diferenciable) tiene inversa (diferenciable) entonces n = m. En efecto, si G : V U es su
inversa entonces

F G : V V
x (F G) (x) = x

GF : U U
x (G F ) (x) = x

luego

D (F G) = Im
D (G F ) = In

as

DF (G (x)) DG (x) = Im
DG (F (v)) DF (v) = In

si ponemos x =F (v) entonces G (x) = v entonces

DF (G (x)) DG (x) = Im
DG (x) DF (G (x)) = In

pongamos A = DF (G (x)) y B = DG (x) entonces

AB = In
BA = Im

definamos las transformaciones lineales

TA : Rn Rm
x TA (x) = Ax

485
Apuntes Mat023 (Segundo semestre 2014)

TB : Rm Rn
x TB (x) = Bx

entonces TA TB es IRm y TB TA es IRn de la primera se obtiene que TB es inyectiva

TB (x) = TB (y) TA (TB (x)) = TA (TB (y))


x=y

y de la segunda que TB es sobreyectiva (si y Rn entonces TA (y) = v Rm entonces

TB (v) = TB (TA (y)) = y

de donde Im(TB ) = Rn ) se sigue por el teorema de las dimensiones que

Dim (Rn ) = Dim (Rm )

as n = m.

Teorema 11.2.1 (De la funcion inversa). Sean U un abierto de Rn , F : U Rn Rn


una funcion de clase C p (U ) y x0 U un punto en el cual det (DF (x0 )) 6= 0 entonces
existe un abierto U1 U tal que x0 U1 y F : U1 V = F (U1 ) es invertible, su inversa
F 1 : V U1 es de clase C p (V ), ademas

DF 1 (F (x)) = (DF (x))1 para x U1

en particular, si F (x0 ) = y0 entonces

DF 1 (y0 ) = (DF (x0 ))1

Observacion 11.2.2. Si x = (x1 , x2 , . . . , xn ) y F : D Rn Rn , x F (x) =


(F1 (x) , F2 (x) , . . . , Fn (x)) entonces det (DF (x0 )) se representa por el smbolo

(F1 , F2 , . . . , Fn )
(x1 , x2 , . . . , xn ) x=x0

y es llamado el Jacobiano (es el determinate de la matriz Jacobiana).

486
Apuntes Mat023 (Segundo semestre 2014)

Ejemplo 11.2.1. Muestre que la funcion


F : R2 R2
(x, y) F (x, y) = (u (x, y) , v (x, y)) = (ex cos y, ex sin y)
no tiene una inversa global (definida en R2 ), sin embargo, cerca de todo punto es posible
definir una inversa local. Calcular
 
DF 1 2, 2

si F ln 2, 4 =

2, 2 y obtener una expresion para
DF 1 (u, v)
si (u, v) 6= 0.
Solucion. Notemos
F (0, 2) = F (0, 0) = (1, 0)
luego F no es inyectiva y por tanto no tiene inversa global.
Notemos que F C (R2 ) ademas

F1 F1 u u
x y x y
DF (x, y) = =


F2 F2 v v
x y x y

ex cos y ex sin y
 
=
ex sin y ex cos y
y
ex cos y ex sin y
 
(u, v)
= det = e2x 6= 0
(x, y) ex sin y ex cos y
as, por el teorema de la funcion inversa, F es invertible cerca de cada punto de R2 (inversa
local) ademas
DF 1 (F (x, y)) = (DF (x, y))1
 x 1
e cos y ex sin y
=
ex sin y ex cos y
de esto se obtiene
1
eln 2 cos eln 2 sin
   4 4
DF 1 F ln 2,

=
4
eln 2 sin eln 2 cos
4 4
1 1

4
2 4
2
=

14 2 1
4
2

487
Apuntes Mat023 (Segundo semestre 2014)

esto es
1 1
  4
2 4
2
DF 1 2, 2 =

41 2 1
4
2
si F (x, y) = (u, v) entonces F (u, v) = (x, y) donde x = x (u, v) e y = y (u, v) ademas
x  x
 1 1

u
2, 2 v
2, 2 4
2 4
2
=
y
 y
 1
1

u
2, 2 v
2, 2 4 2 4
2

as por ejemplo
x   1
2, 2 = 2
u 4
en general

DF 1 (u, v) = DF 1 (F (x, y))


 x 1
e cos y ex sin y
=
ex sin y ex cos y
 1
u v
=
v u
u v
u2 + v 2 u2 + v 2
=

v u
2
u + v2 u2 + v 2
Observacion 11.2.3. Note que el teorema de la funcion inversa afirma DF 1 (y0 ) =
(DF (x0 ))1 luego
(u, v) 1
= (x,y)
(x, y)
(u,v)

cuidando donde estan evaluadas las funciones.

Ejemplo 11.2.2. Sea f : R2 R2 la funcion definida por

f (u, v) = u2 + u2 v + 10v, u + v 3


1. Mostrar que tiene inversa cerca del punto (1, 1)

488
Apuntes Mat023 (Segundo semestre 2014)

Solucion. Notemos que la funcion es de clase C (R2 ) y


 F1 F1 
Df (1, 1) = u v
F2 F2
u 2 v
(u +u2 v+10v ) (u2 +u2 v+10v )
= u v
(u+v 3 ) (u+v 3 )
u v
2
 
2u (v + 1) u + 10
=
1 3v 2
(1,1)
 
4 11
=
1 3

luego  
4 11
det = 1 6= 0
1 3
y as, por el teorema de la funcion inversa, la funcion es localmente invertible, ademas

f (1, 1) = (12, 2)

2. Calcular la derivada de su inversa en el punto (12, 2).

Solucion. Como f (1, 1) = (12, 2) se sigue


 1
1 1 4 11
Df (12, 2) = Df (f (1, 1)) =
1 3
 
3 11
=
1 4

Ejemplo 11.2.3. Sea u : R2 R, (x, y) u (x, y) una funcion que satisface

u  u
2xy x2 y 2 =0
x y

1. Utilizar el teorema de la funcion inversa para determinar condiciones sobre la funcion


de modo que

F : D R2 R2
F (x, y) = ( (x, y) , (x, y))
 
x
= , (x, y)
x2 + y 2

sea invertible.

489
Apuntes Mat023 (Segundo semestre 2014)

Solucion. Por el teorema de la funcion inversa, si



x2 y2 2xy
x y (x2 +y2 )2 2 2
(x +y ) 2
=
x y x y

x2 y 2
   
2xy
= +
y (x2 + y 2 )2 x (x2 + y 2 )2
6= 0

2. Escribir la ecuacion
u  u
2xy x2 y 2 =0
x y
en las nuevas variables (, ) mediante regla de la cadena, resolver esa nueva ecuacion
y concluir que u debe tener la forma
 
x
u (x, y) = f
x2 + y 2

donde f C 1 (R) es una funcion arbitraria.

Solucion.
u u u
= +
x x x
u u u
= +
y y y
de donde
x2 y 2
 
u u u
= +
x (x2 + y 2 )2 x
 
u u 2xy u
= +
y (x2 + y 2 )2 y
se sigue
u  u
0 = 2xy x2 y 2
x y
x y2
2
   
u u
= 2xy +
(x2 + y 2 )2 x
   
2 2
 u 2xy u
x y +
(x2 + y 2 )2 y
u  u
= 2xy x2 y 2
x y
 
u 2 2

= 2xy x y
x y

490
Apuntes Mat023 (Segundo semestre 2014)

pero  

2xy x2 y 2 6= 0
x y
entonces
u
=0

se sigue
u (, ) = f ()
as  
x
u (x, y) = f
x + y2
2

donde f es una funcion arbitraria.

Ejercicios de la seccion

1. Sea f : R R una funcion de clase C 1 y sea

u = f (x)
v = y + xf (x)

si f 0 (x0 ) 6= 0 probar que la funcion T (x, y) = (u, v) es invertible cerca de (x0 , y0 ) y


que la inversa tiene la forma

x = f 1 (u)
y = v + uf 1 (u)

encontrar DT 1 .
 2

2. Considere la funcion F (x, y) = (x y)2 , xy para y 6= 0.

a) Probar que F admite inversa local en una vecindad de (1, 1)


b) Sea F 1 : V R2 R2 , (x, y) = (g (u, v) , h (u, v)) la inversa local de F ,
calcular la razon de cambio de h en (4, 1) en la direccion del vector (2, 1).

3. Sea F (x, y) = (f1 (x, y) , f2 (x, y)) = (x cos y, sin (x y)). Mostrar que F tiene inversa
local en una vecindad del punto 2 , 2 y obtener la Jacobiana de la inversa en (0, 0).


4. Definimos x : R2 R, (r, ) x (r, ) = r cos y y : R2 R, (r, ) y (r, ) =


r sin .

491
Apuntes Mat023 (Segundo semestre 2014)

a) Demostrar que
(x, y)
= r0
(r, ) (r0 ,0 )
b) cuando se puede formar una funcion inversa suave de F : R2 R2 , (r, )
(x (r, ) , y (r, ))? Comprobarlo directamente y con el teorema de la funcion
inversa.

5. Definamos F : R3 R3 , (, , ) F (, , ) = (x (, , ) , y (, , ) , z (, , ))
donde

x (, , ) = sin cos
y (, , ) = sin sin
z (, , ) = cos

a) Muestre que
(x, y, z)
= 2 sin
(, , )
b) Cuando se puede despejar (, , ) en terminos de (x, y, z)?

Ejercicios del captulo

1. Si u1 , u2 son las races de la ecuacion cuadratica en


x2 + y 2 z2
+ =1
2 1
y u3 es arctan (y/x) probar que
(u1 , u2 , u3 ) 4z
=
(x, y, z) u1 u2

2. La ecuacion x3 + y 3 = 6xy define cerca del punto 2 3 2, 2 3 4 a y como funcion de x.

Muestre que x = 2 3 2 es un punto crtico de tal funcion y clasificarlo en maximo,
mnimo o punto silla.

3. La ecuacion y z 
f , =0
x x
define a z como funcion de x, y, z = g (x, y). Muestre que
g g
x +y =g
x y

492
Apuntes Mat023 (Segundo semestre 2014)

4. Si , : D R3 R son funciones de clase C 1 tales que


(, )
6= 0
(y, z)
entonces las ecuaciones (x, y, z) = 0, (x, y, z) = 0 determinan y, z como funciones
de x y
(,) (,)
dy (z,x) dz (x.y)
dx
= (,) dx
= (,)
(y,z) (y,z)

5. Si z1 = er cos , z2 = er sin y x = r cos , y = r sin probar que


(z1 , z2 ) e2r
=
(x, y) r

6. Suponga que x, y, z, r, , estan relacionados por

f (x, y, z) = 0

y
x = r sin cos y = r sin sin z = r cos
probar que
  
z r r
sin sin + r cos + sin
x
 
r
sin cos cos r sin = 0

7. Sea a > 0. Si y 4 + 3a2 y 2 5a2 xy + 2a2 x2 = 0, probar que


dy dy 2
lm = 0 o lm =
x0 dx x0 dx 3
8. Si u, v son funciones de las variables x, y por medio de las ecuaciones f (x, y, u) = 0
y g (u, v, x) = 0 probar que
v fx gu fu gx
=
x f u gv
v fy gu
=
y fu gv

9. La ecuacion f
z
(x, z) = u define a z como funcion de x y u. Si ponemos z = g (x, u)
y se define (x, u) = f (x, g (x, u)) probar que
z
= u
u u
f z
= +u
x x x

493
Apuntes Mat023 (Segundo semestre 2014)

10. Suponiendo que u, v son funciones de x, y las cuales definen a u, v en terminos de


x, y y satisfacen las ecuaciones
u v u
+ +u = 0
x y y
v u v
+ +u = 0
x y y
Probar que
2x 2x x
2
2 =
u v v

11. Si u1 x1 = u2 x2 = u3 x3 = u4 x4 = (u1 u2 u3 u4 )1/2 entonces

(u1 , u2 , u3 , u4 )
= 1
(x1 , x2 , x3 , x4 )

12. Determinar el Jacobiano de la transformacion

x = a ( cos sin )
y = b ( sin sin )
z = c ( cos )

donde a, b, c, , . son constantes positivas y determinar cuando es posible invertir


tal transformacion. Ind.: Compuestas de funciones.

494
Parte III

Evaluaciones de anos anteriores

495
Captulo 12 : Controles

Control 1
q
xy
1. Dada la funcion f : D R2 R definida por f (x, y) = x+y

a) Determine y grafique el dominio de f .


b) El dominio de f es un conjunto abierto?
c) Determine la frontera del dominio de f y sus puntos de acumulacion.
1
d ) Determine las curvas de nivel f (x, y) = c y grafique para c = 0, c = 1, c = 2
y
c = 2.
e) Analice que sucede cuando c crece indefinidamente.

2. Sea T : R3 [x] R2 [x] definida por


Z 1
00
T [p (x)] = p (x) + p (x) dx
0

a) Pruebe que T es una transformacion lineal.


b) Sean

1, x 1, (x 1)2 , (x 1)3

B1 =
B2 = {1, x, x (x 1)}

bases de R3 [x] y R2 [x] respectivamente. Determine [T ]BB21 y use esta matriz para
obtener el nucleo de T .

Control 2
1. Sea T : R4 R3 una transformacion lineal definida por

T (x, y, z, w) = (x y + z + w, x + 2z w, x + y + 3z 3w)

Encuentre una base y la dimension de Ker(T ) e Im(T ).

2. Describa explcitamente una transformacion lineal T : R3 R2 tal que

Im (T ) = h{(1, 0, 1) , (1, 2, 2)}i

496
Apuntes Mat023 (Segundo semestre 2014)

Control 3
1. Considere la funcion T : R2 [x] R3 definida por T (p (x)) = (p (0) , p (1) , p (2)). Sean
B = {1, x, x2 } y C = {(1, 0, 0) , (0, 1, 0) , (0, 0, 1)} bases de R2 [x] y R3 respectivamente:

a) Pruebe que T es una transformacion lineal.


b) Determine Ker(T ) y [T ]CB .
c) T es un isomorfismo?.
p
2. Considere la funcion f : D R2 R, definida por f (x, y) = sin (x2 + y 2 ).

a) Encuentre el dominio de f y dibujarlo.


b) Determine si el dominio es abierto o cerrado.

Control 4
1. Sea
x + y si |x| + |y| 2
f (x, y) =
si |x| + |y| < 2

1
Existe lm f (x, y)? Justificar.
(x,y)(1,1)

2. Demuestre o de un contraejemplo de la propiedad: Para todo A, B Rn

(A B) A B

Control 5
1. Considere la funcion f : {(x, y) R2 : x 6= 0} R, definida por
y
f (x, y) = x2 + y 2 arctan

x
es posible definir f en (0, 1) de forma que f sea continua en tal punto?

2. Sea f : R2 R definida por



xy

x2 +y
si x2 6= y
f (x, y) =
si x2 = y

0

Hallar f
x
(x, y) y f
x
(x, y) en todos los puntos que estas existan y determinar si son
continuas en (0, 0).

497
Apuntes Mat023 (Segundo semestre 2014)

Control 6
1. Considere el sistema

x = u+v+w
y = u2 + v 2 + w 2
z = u3 + v 3 + w 3
v w
Calcule y (2, 6, 8) y x
(2, 6, 8) donde (x, y, z) = (2, 6, 8) es la imagen de (u, v, w) =
(1, 2, 1).

2. Considere el sistema

u3 + xv 2 + y = 0
v 3 + yv + u2 = 0

a) Pruebe que es posible despejar x = x (u, v), y = y (u, v) en vecindades de los


puntos (x, y) = (1, 1) y (u, v) = (0, 1).
u v
b) Hallar x
(1, 1) y x
(1, 1).

Control 7
1. Un servicio de entrega de paquetes requiere que las dimensiones de una caja rectan-
gular sean tales que la longitud mas el doble del ancho mas el doble de la altura
no rebase 108 cms. Cual es el volumen de la caja mas grande que podra enviar la
compana?

2. Considerar el sistema

xy 2 + xzu + yv 2 = 3
u3 yz + 2xv u2 v 2 = 2

Es posible despejar u = u (x, y, z) y v = v (x, y, z) en vecindades de U de (x, y, z) y


v
V de (u, v) = (1, 1). Calcular y (1, 1, 1).

3. Resuelva la ecuacion
dy 1 xy 2
=
dx 2x2 y
haciendo la sustitucion v = y/xn para algun n.

4. Muestre que la ecuacion


dy y m n
y
= +x y f
dx x x
498
Apuntes Mat023 (Segundo semestre 2014)

se transforma en una ecuacion de variables separables usando el cambio de variables


y = vx donde v = v (x). Use lo anterior para resolver la ecuacion
y

dy y sec2 x
= +
dx x y2

Control 8
1. Encuentre la solucion general de la ecuacion

d4 y d3 y d2 y dy
4
3
+ 2 3 6y = 0
dx dx dx dx

2. Un estanque contiene 50 litros de agua pura. Al estanque entra salmuera, que contiene
C gramos de sal por litro a razon de 1,5 litros por minuto. La mezcla bien revuelta,
sale a razon de 1 litro por minuto. Si despues de 30 minutos la concentracion de sal
en el estanque es de 30 gramos por litro. Hallar el valor de C.

Control 9
dz
1. Hallar dx
si se cumple x3 + y 3 + z 3 = 0 y x2 + y 2 + z 2 = 1

2. La matriz  
1 1
A=
1 2
representa el jacobiano de una funcion f de clase C 2 de R2 en R2 en el punto (1, 1).
Suponga que f (1, 1) = (1, 0) y que f (x, y) = (u, v). Pruebe que f es localmente
y
invertible en (1, 1) y encuentre u (1, 0).

Control 10
1. Muestre que la ecuacion diferencial

2x4 yy 0 + y 4 = 4x6

se reduce a una ecuacion homogenea mediante el cambio de variables y = z n para


cierto n R. Determine el valor de n y resuelva la ecuacion.

2. Resuelva la ecuacion x3 yy 0 + 2x2 y 2 1 = 0 usando el cambio de variables u = x2 y.

499
Apuntes Mat023 (Segundo semestre 2014)

Control 11
1. Usando la transformada de Laplace resuelva el problema de valor inicial

ty 00 ty 0 + y = 2 et 1


con y (0) = 0, y 0 (0) = 1.

2. Obtenga la transformada de Laplace inversa de


s2 s
s3 + s2 + 9s + 9

Control 12
1. Resuelva el problema de valores iniciales
dy
exy + ex = 0
dx
y (0) = 1

Ayuda. Use el cambio de variables u = ey .

2. Resuelva la ecuacion y 2 dx = (x3 xy) dy usando un factor integrante de la forma


xn y m .

Control 13
Sea R Z.

1. Encontrar el desarrollo en serie de Fourier de la funcion f : ], [ R

f (x) = cos (x)

2. Verificar que "


#
1 1 X 2
cot () =
n=1 n2 2

Control 14
1. La recta normal, en cada punto (x, y) de la curva dada, pasa por el punto (2, 0). Si
la curva pasa por (2, 3) encuentre su ecuacion. Justificar.

2. Resolver la ecuacion

(2x + 1)2 y 00 2 (2x + 1) y 0 12y = 6x

500
Apuntes Mat023 (Segundo semestre 2014)

Control 15
1. Obtenga la solucion de las ecuaciones

a) (6xy 2 3x2 ) dx = (6x2 y + 3y 2 7) dy


y
b) xy 0 y = ln yln x

2. En la schopera de Don Ramon se vende cerveza artesanal a 4 o C. Dado que se


esta en epoca de verano la temperatura interior de la schopera es de 30 o C. El senor
Ramon es muy exigente con la calidad de su producto y pide a los meseros no servir
Schops si es que estos estan a mas de 7 o C. Sabiendo que a los 60 segundos la cerveza
llega a los 5 o C y que ademas un mesero demora 2 minutos en transportar la cerveza
desde su fuente hasta la mesa podran los meseros cumplir con las exigencias de Don
Ramon? (justifique matematicamente)

Control 16
1. Calcular  
sin t
L (s)
t

2. Resolver
ty 00 + 2y 0 + ty = 0
con y (0) = 1 e y () = 0.

3. Encontrar el desarrollo en serie de Fourier para



0 < x < 0
f (x) =
x 0<x<

y usando su resultado calcular



X 1
n=1
(2n 1)2

Control 17
1. Si es una funcion de una variable y z = y (x2 y 2 ) probar que

1 z 1 z z
+ = 2
x x y y y

501
Apuntes Mat023 (Segundo semestre 2014)

Control 18
1. Sea T : R3 [x] R3 [x] definida por
T a0 + a1 x + a2 x2 + a3 x3 = a0 + a1 (x + 1) + a2 (x + 1)2 + a3 (x + 1)3


a) Demuestre que T es una transformacion lineal.


b) Si B = {1, x, x2 , x3 } encuentre [T ]BB
c) Determine dim (T ). Es T inyectiva? Justifique.

Control 19
1. Resolver la ecuacion
x2 y 00 3xy 0 + 3y = 2x4 ex

Control 20
1. Resuelva la ecuacion integro diferencial
Z t
0
y (t) + y (u) du = f (t)
0

donde 
1 0t<1
f (t) =
0 t>1
con la condicion inicial y (0) = 0.

Control 21
1. Para x > 0 resuelva el P.V.I.
x2 y 00 2xy 0 + 2y = x4
con y (1) = 0, y 0 (0) = 1.

Control 22
1. Considere las superficies en R3 dadas por las ecuaciones
y = f (x)
y
z 2 + 2xz + y = 0
Determine la funcion f si se sabe que ambas superficies tienen el mismo plano
tangente en todo punto donde intersectan.

502
Apuntes Mat023 (Segundo semestre 2014)

2. Dado el sistema

u = f (x)
v = g (x, y)
w = h (x, y, z)
(f,g,h)
para el caul se cumple (x,y,z)
6= 0:

a) Demuestre que localmente se pueden despejar las variables y, u, z como funciones


de (x, v, w) y que
u y
= =0
w w
b) Pruebe que  
z 1 (h, g)
=
x gy hz (x, y)

Control 23
1. Calcule, si existe, el siguiente lmite

y x sin (y 3 )
lm
(x,y)(0,0) x4 + y 4

2. Sea
xy

x3 y
si y x3 6= 0
f (x, y) =
si y x3 = 0

1
Encuentre el dominio de continuidad de f .

Control 24
2 /(4y)
1. Sea f (x, y) = y n ex . Hallar un valor de n tal que f cumpla la ecuacion
 
2 f 2 f
x = x
y x x

2. Demostrar que el tetraedro acotado por los planos coordenados y cada plano tangente
a la superficie xyz = a3 es de volumen constante.

503
Apuntes Mat023 (Segundo semestre 2014)

Control 25
1. Encuentre todas las soluciones de la ecuacion
dy 3
= x2 (y 1)3
dx 2
2. Resuelva el problema de valor inicial
5
y0 + y = 3x3 + x
9x
con y (1) = 4.

Control 26
1. Se esta celebrando una fiesta en una habitacion que contiene 1800 pies cubicos de
aire libre de monoxido de carbono. En el instante t = 0 varias personas comienzan
a fumar. El humo que contiene 6 por ciento de monoxido de carbono, se introduce
en la habitacion a razon de 0.15 pies cubicos por minuto, y la mezcla, removida por
ventilacion, sale a ese mismo ritmo por una ventana entreabierta. Cuando debera
abandonar una persona prudente esa fiesta, si el nivel de monoxido de carbono
comienza a ser peligroso a partir de una concentracion de 0.00018?

Control 27
1. Un edificio tiene dos pisos. El primer piso esta sujeto al suelo rgidamente y el segundo
es una masa m que pesa 16 toneladas. La estructura elastica del edificio se comporta
como un resorte que resiste a los desplazamientos horizontales del segundo piso;
requiere una fuerza horizontal de 5 toneladas para que el segundo piso se desplace
una distancia de 1 pie. Suponga que un temblor de tierra hace que el piso oscile
horizontalmente con una amplitud A0 y con una frecuencia , resultando una fuerza
externa F (t) = mA0 2 sin (t) sobre el segundo piso.

a) Cual es la frecuencia de las oscilaciones del segundo piso?


b) Si el suelo sufre una oscilacion cada 2.25 segundos con una amplitud de 3 pulgadas
cual es la amplitud de las oscilaciones forzadas resultantes del segundo piso?

Control 28
1. Resuelva el problema de valores iniciales

ty 00 ty y = 0

con y (0) = 0, y 0 (0) = 3.

504
Apuntes Mat023 (Segundo semestre 2014)

2. Resuelva el problema

00 0 t si 0 t < 3
y 4y + 4y =
t + 2 si t3

con y (0) = y 0 (0) = 0.

Control 29
1. Resolver las ecuaciones

a) xy 2 y 0 + y 3 = x cos x
dy 2
3x y+y 2
b) dx
= 2x 3 +3xy con y (1) = 2

Control 30
1. Sea T : R2 [x] R2 definida por T (p (x)) = (p (0) , p (1))

a) Pruebe que T es una transformacion lineal.


b) Determine una base para Ker(T ) e Im(T ). Cuales son sus dimensiones?
c) Sea B = {1 + x, 2 + x2 , 4 + x + x2 } base de R2 [x] y sea C la base canonica de
R2 . encuentre [T ]CB .

Control 31
1. Resolver la ecuacion
1 2
y0 =
y y + 4x (x + 4)
16x2
Sabiendo que una solucion particular es de la forma axb .

2. Determine la funcion M (x, y) mas general de modo que la ecuacion diferencial


 
xy 1
M (x, y) dx + xe + 2xy + dy = 0
x
sea exacta y resuelva la ecuacion para una de tales M .

Control 32
1. Determine los maximos y mnimos globales de

f (x, y, z) = 2x2 + y 2 + z 2 xy
x2 y2 z2
sujeto a 2
+ 4
+ 8
1.

505
Apuntes Mat023 (Segundo semestre 2014)

Control 33
1. Use la transformada de Laplace para resolver el sistema

x0 = 4x 2y
y 0 = 5x + 2y

con x (0) = 2 y y (0) = 2.

Control 34
1. Suponga que un tanque cilndrico recto con radio de la base 12 metro y altura 4 metros
tiene inicialmente 2 litros de agua pura. Una solucion de salmuera se bombea hacia
1

el tanque a una rapidez de 1 + 1+t litros por minuto, la concentracion de sal en el
1
flujo de entrada es de 2 kilogramo por litro. La solucion en el tanque es homogenea y
1
se extrae a 1+t litros por minuto. Determinar la cantidad de sal en el tanque cuando
este se llena.

2. Sean u = (1, 0, 1), v = (1, 0, 1) y w = u v vectores en R3 y C la base canonica de


R3 :

a) Si T : R3 R3 es una transformacion lineal definida por T (u) = w, T (v) = v


y T (w) = u, determine [T ]CC y [T T ]CC
b) Determine explcitamente T 1 o argumente que no esta definida.
c) Si B = {u, v, w} determine [T ]BB
d ) Encontrar una matriz A tal que A1 [T ]BB A = [T ]CC

Control 35
1. ean a, b R, a 6= 0. Considere la ecuacion diferencial autonoma
dx
= a ((x 1) (x 4) b)
dt
I) Determine los valores y/o condiciones sobre a y b de modo que la funcion
x (t) 5 sea una solucion de equilibrio y ademas un atractor.
II) Para los valores y/o condiciones obtenidos en la parte anterior, bosquejar el
diagrama de fases.

506
Apuntes Mat023 (Segundo semestre 2014)

III) Para los valores y/o condiciones obtenidos en la parte I), analizar el comporta-
miento de la solucion definida por el P.V.I.
dx
= a ((x 1) (x 4) b)
dt
x (0) = 1

esto es: Intervalos de crecimiento, decrecimiento y concavidad, limites a si


estos tienen sentido (examinar el intervalo maximal de definicion).
Obs.: No se entregaran puntos por resolver explcitamente y luego realizar el
analisis.

Control 36
1. Considere el sistema de ecuaciones
d
X = AX + B
dt
     t 
x (t) 2 4 e
donde X = ,A= yB=
y (t) 1 3 e2t

d
a) Resolver el sistema homogeneo dt
X=
AX, bosquejar el diagrama de fases y
   
x (0) 3
clasificar la solucion de equilibrio. Si = determine la ecuacion
y (0) 2
de la recta a la cual tiende la curva solucion cuando t +.
d
b) Resolver el sistema no homogeneo dt
X = AX + B

2. Resolver el problema
t
d2 y
Z
= t+2 e(tu) y (u) du
dt2 0
y (0) = 0
y 0 (0) = 0

Control 37
1. Considere la funcion
x y


si (x, y) 6= (0, 0)
x2 + y 2
f (x, y) =


0 si (x, y) = (0, 0)

507
Apuntes Mat023 (Segundo semestre 2014)

a) Determine los valores de R+ para los cuales f es diferenciable en (0, 0)


b) Determine (para todo R+ ) el plano tangente a la grafica en el punto 1, 1, 12 .


2. Sea f : D R2 R definida por


p
x2 + y 2 1
f (x, y) = p
4 x2 y 2

a) Determine el dominio D de f .
 
b) Demostrar que 32 , 0 D, 1, 3 D y (0, 0) 6 D.
c) Describir los conjuntos de nivel de f y graficar la funcion.

508
Captulo 13 : Certamenes

Certamen 1
1. Sea f : R2 R diferenciable tal que f (1, 1) = 1 y f (1, 1) = (2, 3). Sea
z = g (x, y) = f (x2 y, y 2 f (x, y)). Encuentre la ecuacion del plano tangente a la
superficie z = g (x, y) en el punto (1, 1, 1).

2. Considere la funcion
1
sin (x2 + x2 y 2 ) si x 6= 0

x
f (x, y) =

0 si x = 0

a) Pruebe que f es continua en R2


b) Encuentre fx (0, 0) y fy (0, 0)
c) Determine si f es diferenciable en (0, 0).

3. El sistema

x3 + uy 2 + v = 0
y 3 + yv + x2 = 0

define a u y v como funciones de x e y, en vecindades de los puntos (x, y) = (0, 1) y


(u, v) = (1, 1). Sea w = (1, 1), determine la derivada direccional de v = v (x, y) en
el punto (1, 0) en la direccion w.

4. El material para el fondo de una caja rectangular cuesta el triple por metro cuadrado
que el material para los lados y la tapa. Determine la maxima capacidad que tal caja
puede tener si la cantidad total de dinero disponible para el material es $12000 y el
material para el fondo cuesta $600 el metro cuadrado.

509
Apuntes Mat023 (Segundo semestre 2014)

Certamen 2
1. Considere la aplicacion lineal T : R3 R3 tal que T (1, 1, 0) = (2, 4, 2), T (1, 0, 1) =
(0, 2, 2) y T (0, 1, 1) = (2, 2, 2):

a) Es T invertible?. Justifique.
b) Pruebe que existe una base B de R3 tal que [T ]BB es una matriz diagonal.

2. Sea g : R R una funcion continua y defina f : R2 R por


Z y
f (x, y) = g (t) dt
x

a) Pruebe que f es diferenciable


b) Pruebe que el plano tangente a la superficie z = f (x, y) en el punto (a, b) pasa
por el origen si y solo si
Z b
bg (b) ag (a) = g (t) dt
a

3. Encuentre el maximo de ln x + ln y + 3 ln z sobre la porcion de esfera x2 + y 2 + z 2 = 5r2


donde x > 0, y > 0, z > 0. Use el resultado para probar que para numeros reales
positivos a, b, c tenemos  5
3 a+b+c
abc 27
5

4. Sea U = {(x, y, z) R3 : xy 6= 0}. Definamos f : U R3 R por


 
z z
f (x, y, z) = g x + , y +
y x

donde g : R2 R es una funcion de clase C 1 . Suponga que para cada


f
(x, y, z) U , (x, y, z) 6= 0
z
a) Considere un punto (x0 , y0 , z0 ) U tal que f (x0 , y0 , z0 ) = 0. Argumente que
f (x, y, z) = 0 define implcitamente a z como funcion de las variables x e y, es
decir, z = z (x, y) en una vecindad de (x0 , y0 ) tal que z0 = z (x0 , y0 ).
b) Demuestre que la funcion z de la parte anterior satisface la ecuacion

z z
x +y = z xy
x y

510
Apuntes Mat023 (Segundo semestre 2014)

Certamen 3
1. Resuelva la siguiente ecuacion

y 0 = e2x y 2 2y 9e2x

con y (0) = 4 sabiendo que tiene una solucion particular de la forma y (x) = aekx .

2. Un tanque contiene inicialmente 60 gal. de agua pura. Entra al tanque, a una tasa de
2 gal./min., salmuera que contiene 1 lb. de sal por galon, y la solucion (perfectamente
mezclada) sale de el a razon de 3 gal. por minuto; el tanque se vaca despues de una
hora exactamente.

a) Encuentre la cantidad de sal que hay en el tanque despues de t minutos.


b) Cual es la cantidad maxima de sal que llega a tener el tanque?.

3. Resuelva la ecuacion
2 2
x 1 x2 y 00 1 x2 y 0 + x3 y = 0

usando el cambio de variables t = 12 ln (1 x2 )

4. Encuentre la solucion general de la ecuacion


2
x2 2x y 00 + 2 (1 x) y 0 + 2y = 6 x2 2x


sabiendo que y = x 1 es una solucion de la ecuacion homogenea asociada.

511
Apuntes Mat023 (Segundo semestre 2014)

Certamen 4
1. Resuelva las ecuaciones
dy
a) x3 dx + x2 y = 2y 4/3
b) (x2 + y 2 + 1) dx (xy + y) dy = 0

2. Suponga que un individuo infectado se introduce en una poblacion de tamano N,


todos los individuos de la cual son susceptibles a la enfermedad . Si suponemos que
la tasa de infeccion es proporcional al producto del numero de infectados y el de
susceptibles presentes, cual sera el numero de infecciones en el tiempo t?.

3. Para x > 0, considere la ecuacion


2
xy 00 + x2 1 y 0 + x3 y = ex /4


a) Use el cambio de variables t = x2 /2 para encontrar la solucion general de la


homogenea.
b) Resuelva la no homogenea usando variacion de parametros.

512
Apuntes Mat023 (Segundo semestre 2014)

Certamen 5
1. a) Pruebe que el cambio de variables u = ax + by + c transforma la ecuacion
diferencial y 0 = f (ax + by + c) en una ecuacion diferencial autonoma, donde
a, b y c son constantes reales.
b) Use el resultado anterior para transformar la ecuacion
dy
= (y 2x) (y 2x)2 + 2 (ec-1)
dx
en una ecuacion autonoma.
c) Haga la linea de fase de la ecuacion autonoma obtenida en el item b).
d ) En el plano xy haga un esbozo de las soluciones de la ecuacion (ec-1).
e) Obtenga la solucion general de (ec-1).

2. Determine una solucion de la forma y (x) = xn para la ecuacion


x2 1 y 00 (x) 2xy 0 (x) + 2y (x) = 0, x > 1


donde n es algun entero positivo.


Usando el resultado anterior, hallar la solucion del problema de valores iniciales
(
2
(x2 1) y 00 (x) 2xy 0 (x) + 2y (x) = (x2 1) x > 1
y (2) = 2, y 0 (2) = 10

3. Considere el sistema lineal de ecuaciones diferenciales


dx
= y
dt
dy
= 4x 2ay
dt
con a R:

a) Para los diferentes valores del parametro real a, haga un retrato de fase del
sistema de ecuaciones indicando las direcciones en las cuales las curvas son
recorridas.
b) Obtenga la solucion general del sistema.

4. Supongamos que la tasa de cambio del precio x de un bien, crece en el tiempo a una
razon constante c como resultado de la inflacion constante, al mismo tiempo cae en
forma proporcional a la diferencia entre la oferta y en el tiempo t y alguna oferta de
equilibrio y0 , es decir, dx
dt
= c (y y0 ). Tambien asumimos que la tasa de cambio
de la oferta es proporcional a la diferencia entre el precio y algun precio de equilibrio
x0 , es decir, dy
dt
= (x x0 ). ( y )son constantes de proporcionalidad positivas.
Asumiendo que en t = 0,x = x0 y y = y0

513
Apuntes Mat023 (Segundo semestre 2014)

a) Muestre que el precio y la oferta oscilan alrededor de x0 y y0 + c/ respectiva-


mente.
b) Si en el tiempo t = T0 el precio es maximo en que tiempo la oferta es maxima?

514
Apuntes Mat023 (Segundo semestre 2014)

Certamen 6
1. Se ha determinado experimentalmente que la variacion de peso de un tipo de pez
sigue la ley
dp
= e 3 t p2/3 p
dt
donde p = p (t) representa el peso, y son constantes positivas. Si p (0) = p0 > 0
determine el peso maximo del pez.

a) Hallar una ecuacion diferencial lineal con solucion general

c1 et cos (2t) + c2 et sin (2t) + t5

b) Considere la ecuacion y 00 + q (x) y = 0 donde q es continua en todo R y dos


soluciones y1 , y2 las cuales satisfacen y1 (0) = 1, y10 (0) = 0 y y2 (0) = 3, y20 (0) = 1.
Si W (x) es el Wronskiano de y1 e y2 en x demuestre que W (x) = 1 para todo
x donde las soluciones esten definidas.
d y 2 dy
2. Muestre que (x) = ex es una solucion de la ecuacion x dx 2 (x + 2) dx + 2y = 0 y

determinar la solucion general de

d2 y dy
x 2
(x + 2) + 2y = x3
dx dx

3. Sea C la base canonica de R3 . Sean T, L : R3 R3 transformaciones lineales tales


que T (1, 1, 1) = (1, 3, 3), T (1, 1, 0) = (2, 3, 2), T (1, 0, 0) = (1, 1, 2) y

2 3 2

[L]CC = 1 1 1
4 6 5

a) Determine T explcitamente.
b) Determine [T L]CC
c) Que relacion existe entre T y L?
d ) Determinar Ker(T ) e Im(L).

515
Apuntes Mat023 (Segundo semestre 2014)

Certamen 7
1. a) Calcular la serie de Fourier de la funcion

1 si 0<x<
f (x) =
1 si < x < 0

1 1 1
y usando la serie muestre que = 1 + +
4 3 5 7
b) Usando la serie de Fourier anterior, determine la serie de Fourier de

a si 0<x<
g (x) =
b si < x < 0

para a, b R (No se entregaran puntos por calcular esta serie con las formulas
de los coeficientes de Fourier).

2. Resolver el P.V.I.
d2 y

t si t < 1
+ 9y =
dt2 0 si t > 1
con y (0) = y 0 (0) = 0.

3. Resolver el sistema
Z t Z t
tu
e x (u) du + (t u) y (u) du = t2
0
Z t Z 0t
x (u) du + (t u)2 y (u) du = t3
0 0

4. Sea R {1, 2}. Considere el sistema de ecuaciones


    
d x 2 x
=
dt y 1 y

a) Clasificar la solucion de equilibrio (silla, atractor, repulsor, etc.) para los distintos
valores de .
3
b) Para = 2
determine la solucion general del sistema y bosquejar el diagrama
de fases.

516
Apuntes Mat023 (Segundo semestre 2014)

Certamen 8
1. Hallar y clasificar los puntos crticos de f : R2 R definida por

f (x, y) = x3 + y 3 + 9x2 3y 2 + 15x 9y

Posee f extremos globales?.

2. Considere la funcion
x2 y


si (x, y) 6= 0
|x| + y 2

f (x, y) =


0 si (x, y) = 0

a) Determine los puntos del dominio de f en los cuales la funcion es continua.


b) Es f diferenciable en (0, 0)?
f
c) Si a es un vector unitario de R2 y (1, 1) es la derivada direccional de f en
a
(1, 1) en la direccion a, determine el valor de
f
max (1, 1)
aR ,2
kak=1 a

3. Determine las funciones f : R R de clase C 1 (R) tales que las superficies

S1 = (x, y, z) R3 : y = f (x)


S2 = (x, y, z) R3 : z 2 + 2xz + y = 0


son tangentes en los puntos de interseccion.

4. Suponga que la funcion de clase C 2 (R), f : R2 R, (x, y) f (x, y) satisface la


ecuacion en derivadas parciales
2f 2f 2f
2 3 =0
x2 xy y 2
y defina g : R2 R por

g (u, v) = f (u + v, u + v)

donde , , y son constantes. Determine valores enteros no nulos de , , y


para los cuales g cumple
2g
=0
uv

517
Apuntes Mat023 (Segundo semestre 2014)

Certamen 9
1. Resolver la ecuacion diferencial
d2 y dy
t + (1 2t) 2y = 0
dt2 dt
con las condiciones iniciales y (0) = 1, y 0 (0) = 2.

2. En el paraboloide 4x2 + y 2 + 4z 2 16x 6y 8z + 25 = 0 hallar el punto mas cercano


y mas alejado del plano 2x + 2y + z = 0.

3. Muestre que existen funciones u = u (x, y), v = v (x, y) definidas en un abierto U que
contiene a (1, 1) tales que u (1, 1) = 0, v (1, 1) = 1,

ux3 + xy 2 + u sin u = 1

y
v 3 2xv + y 3 = 0
ademas estudiar si la funcion F : U R2 R2 , (x, y) F (x, y) = (u (x, y) , v (x, y))
es localmente invertible cerca de (1, 1) y en caso afirmativo calcular DF 1 (0, 1).

4. Sea f : R2 R, (x, y) f (x, y) una funcion de clase C 2 .

a) Muestre que si g (r, ) = f (r cos , r sin ) entonces

2f 2f 2g 1 2 g 1 g
+ = + +
x2 y 2 r2 r2 2 r r

b) Determine una funcion f : {(x, y) R2 : 1 x2 + y 2 9} R tal que

2f 2f
2
+ 2
= 0 para 1 < x2 + y 2 < 9
x y
f (x, y) = 1 si x2 + y 2 = 1
f (x, y) = 3 si x2 + y 2 = 9

suponiendo que g (r, ) = f (r cos , r sin ) no depende del angulo, es decir,


g

= 0.

518
Apuntes Mat023 (Segundo semestre 2014)

Certamen 10
1. Determine todas la funciones de clase C 1 y positivas f : R R+ , que cumplan
f (0) = 1 y tales que en todo intervalo [a, b] el area bajo la grafica de la funcion y
sobre el eje x sea igual a la longitud de arco de la grafica.

2. Determine la solucion general de la ecuacion

d3 y d2 y dy
3
6 2
+ 11 6y = et + t
dt dt dt

3. Si se sabe que una solucion de

d2 y dy
2
+ f (x) + g (x) y = 0
dx dx
es (x) = x y f (x) = x1 (x + 2), determine g (x) y la solucion general de

d2 y dy
2
+ f (x) + g (x) y = xex
dx dx

4. Construir una transformacion lineal T : R4 R4 tal que

ker (T ) = {(x, y, z, w) : x + y + z = 0 x 2y + w = 0}

e
Im (T ) = {(x, y, z, w) : x y z = 0 x + 2w + y z = 0}
y determine [T ]BC donde

B = {(1, 0, 0, 0) ; (1, 1, 0, 0) ; (1, 1, 1, 0) ; (1, 1, 1, 1)}

y C es la base canonica de R4 .

519
Apuntes Mat023 (Segundo semestre 2014)

Certamen 11
1. Determine La serie de Fourier de : [2, 2] R

0 si x [2, 0]
x (x) =
x [0, 2]

x si

a que converge la serie de Fourier de al evaluarla en x = 6?

2. Resolver el P.V.I.
dy 2 0 si t < 4
y =
dt2
t2 si t > 4

donde y (0) = y (0) = 0.

3. Determine las funciones de orden exponencial , : R R que cumplen (0) = 0 y


Z t
(t u) (u) du = t2
0
Z t
etu 0 (u) du = t
0

a) Analizar para los distintos valores de en R la naturaleza de las soluciones de


equilibrio del sistema
    
d x 1 x
=
dt y 1 y

y bosquejar en cada caso el diagrama de fases.


b) Resolver el P.V.I.
    
d x 2 1 x
=
dt y 1 2 y
x (0) = y (0) = 1

520
Apuntes Mat023 (Segundo semestre 2014)

Certamen 12
1. Considere el siguiente modelo de dinamica poblacional:
dy  y
=r 1 y Ey donde 0 < E < r, k 6= 0
dt k
a) Hallar y clasificar las soluciones de equilibrio.
b) Si y (0) = k2 1 Er bosquejar la solucion y determinar lmt+ y (t).


c) Existen valores de los parametros E, k, r tal que dos puntos crticos consecutivos
sean atractores?

2. Resolver el problema de valores iniciales

xy 00 (x) + (2x 1) y 0 (x) 2y (x) = x2 e3x , x > 0

donde y (1/2) = y 0 (1/2) = 0, si se sabe que y (x) = ex es una solucion de la


homogenea asociada para un adecuado.

3. Considere el problema
00 2
 
u (t) + a u (t) = 2 sin t +
4
u (0) = u0 (0) = 0

donde a es una constante conocida.

a) Si a 6= resolver la ecuacion diferencial con las condiciones dadas.


b) Para que valores de ocurre el fenomeno de la resonancia? Justifique su
respuesta.

4. Considere el sistema de ecuaciones


dx
= 5x + 4y
dt
dy
= 4x + 5y
dt
a) Elaborar el retrato de fases del sistema y clasificar los puntos de equilibrio.
b) Bosquejar en el plano de fase la curva solucion con condiciones iniciales x (0) = 2,
y (0) = 3 Existe algun instante t en el cual x (t) = y (t)?
c) Bosquejar en el plano de fase la curva solucion con condiciones iniciales x (0) = 4,
y (0) = 2 Existe algun instante t en el cual x (t) = y (t)?

521
Apuntes Mat023 (Segundo semestre 2014)

Certamen 13
1. Sea f : R3 R diferenciable y tal que

f (0, 2, 1) = (1, 1, 2) y f (0, 2, 1) = 4

Considerar ademas la funcion g(u, v) = f (u v 2 , 3u v , 3u2 2v) . Encuentre la


ecuacion del plano tangente a la superficie z = g(u, v) en el punto (1, 1, 4) .

2. La ecuacion x2 + y 3 + z 4 + u5 = 2 define a u como funcion de x , y y z y la


ecuacion x + y 2 + z 3 = 1 define a z como funcion de x e y . Entonces, se puede
considerar u como funcion de x e y . Entonces:
du
Hallar (1 , 1) , donde
v esta en la direccion (1 , 1).
dv
3. Sean F : R3 R3 y G : R2 R3 dadas por F (x , y , z) = (u , v , w) determinado
por el sistema

u = xy
v = x2 y 2 + z
w = xz

y G(x , y) = (x 2y + 1 , 2x y , x2 y 2 )

a) Determinar los puntos de R3 en los que F es localmente invertible.


dy
b) Determinar (0, 1, 0)
dw
c) Calcular el diferencial de F 1 G en (1, 1)

4. Considerar la elipse que se obtiene al interceptar el cilindro x2 + y 2 = 1 y el plano


x + y + z = 0 . Encontrar la longitud del semieje mayor y del semieje menor.

522
Apuntes Mat023 (Segundo semestre 2014)

Certamen 14
1. Resolver Z t
00 0
y 2y + y 2 y(u)du = 5 ; t0
0

con y(0) = 0 ; y 0 (0) = 0 .

2. Resolver el sistema

x0 (t)

1 2 3 x(t)
y 0 (t) = 0 2 3 y(t)
z 0 (t) 0 0 2 z(t)

3. Considerar la funcion
f (x) = ex ; < x <

y f (x + 2) = f (x) . Pruebe o refute que:


1 X (1)n
= +2
sinh() n=1
2 + n2

523
Apuntes Mat023 (Segundo semestre 2014)

Certamen 15
1. Encuentre la solucion general de la ecuacion de Euler no homogenea:

x2 y 00 + 2xy 0 2y = 10 cos (ln x)

2. Una masa que pesa 8 lbs. estira 4 pies un resorte. Al principio esta masa parte
del reposo a 2 pies abajo de la posicion de equilibrio y el movimiento ocurre en un
medio que presenta una fuerza de amortiguamiento igual a la mitad de la velocidad
instantanea. Deducir la ecuacion del movimiento si se aplica una fuerza externa igual
a f (t) = t cos(2t)
1
3. Para x > 0 y haciendo el cambio x = encuentre la solucion general de la ecuacion
t
 
4 00 3 0 1
4x y + 8x y + y = tg
2x

4. Considere la ecuacion diferencial


2
y 00 + 4xy + 6 + 4x2 y = x2 ex .


2
a) Pruebe que si y(x) = u(x)ex es solucion, entonces u satisface una ecuacion
diferencial lineal no homogenea con coeficientes constantes.
b) Use el resultado de la parte anterior para determinar la solucion y(x) con

y(0) = 1 , y 0 (0) = 0

524
Apuntes Mat023 (Segundo semestre 2014)

certamen 16

1. La Ley de Malthus supone que la tasa de crecimiento de una poblacion p , es


directamente proporcional al tamano de la poblacion en cada instante.

a) Escribir la ecuacion diferencial que representa esta relacion y verificar que


p(t0 ) = p0 , entonces p(t) = pe(tt0 ) , para alguna constante > 0
b) Sabiendo que la poblacion de la tierra aumento, en promedio, el 2 % anual desde
1960 ( = 0,02) y que en 1965 se estimaba en 3.340 millones de personas.
Calcular mediante este modelo en cuanto tiempo la poblacion se duplico (respecto
de 1960).
c) Este modelo tiene una correccion propuesta por Verhulst en 1837, que asume
que al crecer mucho la poblacion y tener que competir por el alimento y espacio,
el crecimiento se ve afectado por la falta de recursos. Este modelo esta dado por
la ecuacion logstica
p0 = p p2
con , constantes reales positivas. Resuelva esta ecuacion.
d ) Calcular para ambos modelos el lmite cuando t tiene a infinito y explicar su
resultado.

2. Determine la distancia maxima y mnima del origen (0, 0, 0) a los puntos de la curva
definida por la interseccion de las superficies:

z = x2 + y 2 ; x + 2y + z = 10

3. Considere la transformacion F : R3 R3 dada por

F (x, y, z) = (u, v, w), donde u = xy 2 v = x + 3y w = z x.

Se verifica que F (4, 1, 2) = F (1, 2, 1) = (4, 7, 2).

a) Pruebe que en torno a los puntos (4, 1, 2) y (1, 2, 1) existen inversas locales

(x, y, z) = G(u, v, w) ; (x, y, z) = H(u, v, w)

que satisfacen G(4, 7, 2) = (4, 1, 2) y H(4, 7, 2) = (1, 2, 1) respectivamente.


x
b) Calcular en el punto (4, 7, 2) para G o H (ESCOGER SOLO UNA DE
v
ELLAS).

525
Apuntes Mat023 (Segundo semestre 2014)

Certamen 17
1. Resuelva la ecuacion diferencial

x3 y 000 + 4x2 y 00 + xy 0 y = x2 ln x

para x > 0.

2. Considerar la funcion 
0 si 0 t < 1
f (t) =
1 si t1
Resuelva el sistema

9x0 32y 0 32y = f (t)


Z t
2x0 + x (u) du + 8y 0 + 8y = 0
0

con x (0) = 32 e y (0) = 9.


z
3. Use el cambio y = 1+x2
para resolver la ecuacion

2 2
1 + x2 y 00 + 4xy 0 + 2 3 + 2x2 y =
 
+ + 2x
cos (2x) sin (2x)

526
Apuntes Mat023 (Segundo semestre 2014)

Certamen 18
1. Sea T : R3 R2 una transformacion lineal. Sean B = {(1, 1, 1) , (1, 1, 0) , (1, 0, 0)} y
U = {(1, 1) , (1, 1)} bases de R3 y R2 respectivamente.
Considere  
1 0 1
A= [T ]UB =
1 1 2
Determine:

a) [T (1, 1, 0)]U
b) T (3, 2, 1)
c) T es inyectiva?
 
a) Si w = f yx
xy
, zy
yz
. Probar que

w w w
x2 + y2 + z2 =0
x y z

b) Sea f : R2 R definida por

x2 si x2 y 2

f (x, y) =
y si x2 < y 2

determine el maximo dominio de continuidad de f .


 3
2 2 2 2 2 2 2 R2
2. Pruebe que el maximo valor de x y z bajo la condicion x + y + z = R es 3
.
Deduzca de esto que
p3 x2 + y 2 + z 2
x2 y 2 z 2
3
3. Un canaleta cuya seccion transversal tiene forma de trapecio, con angulos en la base
iguales, se va a construir doblando bandas iguales a lo largo de ambos lados de una
larga pieza de metal, de 12 pulgadas de ancho. Encuentre los angulos de la base y las
dimensiones de los lados que producen la maxima capacidad de acarreo.

527
Apuntes Mat023 (Segundo semestre 2014)

Certamen 19
1. Dada la funcion definida por
(
(y2)2 sin(xy)
x2 +y 2 4y+4
si (x, y) 6= (0, 2)
f (x, y) =
0 si (x, y) = (0, 2)

Es f una funcion continua en R2 ?

2. El volumen de un elipsoide de semiejes a, b, c es 4


3
abc. Hallar el elipsoide con centro
(0, 0, 0) de volumen mnimo que pasa por (2, 3, 5).

3. Considere las ecuaciones

uv 3x + 2y = 0
u4 v 4 = x2 y 2

Verifique que ellas definen funciones u = u (x, y), v = v (x, y) en torno al punto
(u, v, x, y) = (1, 1, 1, 1), ademas, determine la ecuacion del plano tangente a la
superficie u = u (x, y) en (1, 1, 1).

4. Sea f : R2 R de clase C 2

a) Pruebe que si f es homogenea de grado p, es decir, f (tx, ty) = tp f (x, y) entonces

2f 2f 2
2 f
x2 + 2xy + y = p (p 1) f
x2 xy y 2

Ind.: g (t) = f (tx, ty) derivar dos veces respecto a t.


b) Probar que si (x, y) f (x, y) = pf (x, y) para todo (x, y) entonces p es ho-
mogenea de grado p.
Ind.: Defina g (t) = f (tx, ty) tp f (x, y) y calcule la derivada.

528
Apuntes Mat023 (Segundo semestre 2014)

Certamen 20
1. Sea (
xy 3
x2 +y 2
si (x, y) 6= (0, 0)
f (x, y) =
0 si (x, y) = (0, 0)

a) Es f continua en (0, 0)?


b) Hallar fx (x, y) y fy (x, y).

2. (Plano tangente y regla de la cadena)

a) Dada la superficie S : x2 + 2y 2 + 3z 2 = 21 hallar la(s) ecuacion(es) del (de los)


plano(s) tangente(s) a S que es (son) paralelos(s) al plano x + 4y + 6z = 0.
b) Si z = f (x, y) es de clase C 1 y x = r cos e y = r sin , probar que

z cos z z
sin + =
r r y

3. (Maximos y mnimos)

a) Encuentre los maximos y/o mnimos de la funcion

f (x, y, z) = x2 + y 2 + z 2 + xy + yz + xz + x 2y

b) Determine los angulos , , de un triangulo de modo que el producto de sus


senos sea maximo.

529
Apuntes Mat023 (Segundo semestre 2014)

Certamen 21
1. Considere la funcion f definida como sigue:
(
2yx3
x 2 +y 2 si (x, y) 6= (0, 0)
f (x, y) =
0 si (x, y) = (0, 0)

a) Determine si la funcion f (x, y) es continua en todo R2 .


b) Determine si la funcion f (x, y) es diferenciable en todo R2 .
c) Determine el valor de fxy (0, 0) y fyx (0, 0).

2. Sean x la cantidad de sillas e y la cantidad de mesas producidas por un fabricante.


Si las funciones f (x, y) = 256 3x y y g (x, y) = 222 + x 5y corresponden a los
precios unitarios de venta de los productos respectivamente, hallar las cantidades
de sillas y mesas de modo de obtener maximas utilidades sabiendo que el costo de
produccion total es C (x, y) = x2 + xy + y 2 .

3. Dada la ecuacion
sin (yz) + sin (xz) + sin (xy) = 0

a) Encuentre las condiciones para que z este definida implcitamente como funcion
de las variables x e y cerca de (x, y, z) = (1, 0, ).
b) Encontrar el plano tangente a la grafica de z = g (x, y) en (x, y, z) = (1, 0, ).

4. Dada la funcion z = f (2x + 3g (y))

a) Encuentre las condiciones para que la funcion z = f (2x + 3g (y)) sea dos veces
diferenciable en R2 .
b) Bajo los supuestos encontrados en la parte anterior, determine el valor de k de
modo que
z 2 z z 2 z
 
k=
x xy y x2

530
Apuntes Mat023 (Segundo semestre 2014)

Certamen 22
1. Sea T : R3 R3 la transformacion lineal definida por

T (x, y, z) = (x + z, y + 3z, x + y + z)

con R:

a) Determine el valor de la constante para que dim Ker(T ) = 1 y en este caso


Calcule Ker (T ).
b) Para el valor anterior de calcule Im (T ).

2. Sean p (x) y q (x) dos funciones continuas. Verificar que la sustitucion y = ez con
z = z (x) transforma la ecuacion diferencial

y 0 + p (x) y = q (x) y ln y

en una ecuacion lineal de primer orden. Usando lo anterior, resolver la ecuacion

xy 0 = 2x2 y + (x + 1) y ln y

3. Hallar la solucion general de una ecuacion diferencial lineal a coeficientes constantes


homogenea, cuya ecuacion caracterstica es:

5 24 + 63 92 + 8 4 = 0
 
sabiendo que y = ex/2 cos 23 x es una solucion de dicha ecuacion.

4. Hallar la solucion general de la ecuacion diferencial

d2 y dy
2
+ tan (x) + cos2 (x) y = 0
dx dx
utilizando para ello el cambio de variables t = sin x.

531
Apuntes Mat023 (Segundo semestre 2014)

Certamen 23
1. (40 pts.) Sea f : R2 R la funcion definida por:


2y 2 , (x, y) A


f (x, y) = 2 2
0 , (x, y) = (0, 0)
x +y
sin (x + y) , (x, y) AC (x, y) 6= (0, 0)


2
x + |y|

en donde:
(x, y) R2 : y > 0 x y x

A=

f f
a) Calcule (0, 0) y (0, 0).
x y
b) Es diferenciable f en (0, 0)?

2. Sean f : R3 R una funcion tal que f (1, 1, 2) = (1, 1, 0) y h : R2 R otra


funcion definida por h(s, t) = 2s2 + st:

(a) Encuentre un vector unitario normal a la curva de nivel (s, t) : h(s, t) = 2 en
el punto (1, 0).
(b) Considere : R2 R definida por

(s, t) = f 3x + 2xy + z 2 , x + y 2 , x 2z 2


en donde: x(s, t) = 2st, y(s, t) = s2 2t y z(s, t) = s + t. Calcule


(1, 0).
(c) Encuentre la derivada direccional de en el punto (1, 0) en la direccion dada por
el vector calculado en la parte (a).

3. Sean f : R2 R una funcion definida por:

f (x, y) = x x2 y 2

y el conjunto U = {(x, y) : x2 + y 2 1}:

a) Mediante el criterio del hessiano, determine los extremos de f en U = {(x, y) : x2 + y 2 < 1},
en caso de existencia.
b) Mediante el metodo de los multiplicadores de Lagrange, calcule los extremos de f
en U = {(x, y) : x2 + y 2 = 1}, si acaso existen.

532
Apuntes Mat023 (Segundo semestre 2014)

Certamen 24
1. Sea x = x (t) una funcion que satisface el sistema de ecuaciones

x00 = y z
y 00 = x0 + z 0
z 00 = (1 + x + y)

donde x (0) = x0 (0) = 0, y (0) = 1, y 0 (0) = 1 y z (0) = 0, z 0 (0) = 1. Calcule, de ser


posible, el valor de x ().

2. Sea f : R R la funcion periodica definida por

f (x) = ex para < x <

y f (x + 2) = f (x). Calcule la serie de Fourier de f .

3. Sea f : R2 R una funcion de clase C 2 (R2 ). Considere el cambio de variables

x = u+v
y = uv 2

y la funcion g : R2 R definida por g (u, v) = f (x (u, v) , y (u, v)). Calcule el valor


de
2g 2g
(1, 1) + (1, 1)
u2 v 2
2f 2f 2f f
sabiendo que x2
(2, 1) = yx
(2, 1) = y 2
(2, 1) = 1 y y
(2, 1) = 2.

4. Se desea construir un tanque que consistir a de un cilindro circular recto de altura h


y radio r, una tapa superior semiesferica de radio r y finalmente una tapa inferior
plana del mismo radio. Suponga que los costos de construccion de la tapa semiesferica
son de $20 por [m2 ]; que los costos de construccion de la cara lateral cilndrica son
de $8 por [m2 ] y de $5 por [m2 ] para la base circular plana.

a) Hallar el valor de h y r de modo que el costo de construccion sea mnimo


asumiendo que el volumen debe ser de 200 [m3 ].
b) A cuanto es igual la relacion h : r?.

533
Apuntes Mat023 (Segundo semestre 2014)

Certamen 25
1. Resuelva usando la transformada de Laplace el siguiente problema de valores iniciales

ty 00 2y 0 + ty = 0 con y (0) = 1, y 0 (0) = 0

2. Sea f : [0, [ R la funcion definida por:


(
2t

si 0 t < 2
f (t) = 2(t)

si 2 t <

Desarrollar f (t) en una serie de Fourier en terminos del seno.

3. Considere la ecuacion diferencial de primer orden


dx
= (x 1) (x a) x x2 + x + 1
dt
en donde a es un parametro real. Determine condiciones sobre a de modo que la
solucion de equilibrio x = 0 sea un punto atractor.

534
Apuntes Mat023 (Segundo semestre 2014)

Certamen 26
1. Sea T : R2 [x] R2 la funcion definida por
 Z 1 
0
T (p (x)) = p (1) , 6 p (x) dx
0

a) Pruebe que T es una transformacion lineal.


b) Hallar el nucleo de T y una base para la imagen de T .
c) Sean B = {1, 1 + x, x + x2 } y D = {(1, 1) , (0, 1)} bases ordenadas de R2 [x] y
R2 respectivamente. Calcular [T ]D
B.

2. Hallar la solucion general de la ecuacion

x2 (1 x) y 00 + 2x (2 x) y 0 + 2 (1 + x) = x2

sabiendo que la homogenea asociada tiene una solucion de la forma y = x2 .

3. La ecuacion homogenea

y (4) 4y 000 + 11y 00 + 8y 0 26y = 0

tiene una solucion de la forma y = e2x cos 3x. Resolver la ecuacion

y (4) 4y 000 + 11y 00 + 8y 0 26y = x

4. Se suministra bacterias como alimento a una poblacion de protozoos a una tasa


 gr 
constante de 1 min . Se ha observado que las bacterias son consumidas a una tasa
de cuatro veces el cuadrado de su cantidad c (t). Hallar c (t) en funcion de c (0) = c0 .

535
Apuntes Mat023 (Segundo semestre 2014)

Certamen 27
1. Sea x > 1. Resuelva la ecuacion diferencial:

(1 x) y 00 + xy 0 y = (1 x)2 cosh x

sabiendo que una solucion de la ecuacion homogenea asociada es y = ex .

2. Laplace:

a) Calcule L t sen t .

b) Si L f (t) = X (s), calcule f (t), sabiendo para ello que X (s) satisface la
ecuacion:
2s (1 e2s )
s2 + 1 X (s) =

s2 + 1
3. Considere el siguiente sistema de ecuaciones:
Z t
x0 + 2x + 6 y (u) du = 2
0
x0 + y 0 + y = 0

en donde x (0) = 5 e y (0) = 6. Calcule el valor de x (ln 2).

4. Sea f : R2 R la funcion definida por:

xy 2

, si y > x2




x 2 + y2

f (x, y) = x2 y
2 + y2
, si y x2 (x, y) 6= (0, 0)



x
0 , si (x, y) = (0, 0)

Determine todos los puntos de R2 para los cuales la funcion f es continua.

536
Apuntes Mat023 (Segundo semestre 2014)

Certamen 28
1. Considere la funcion f : R2 R definida por:

xy sin x , (x, y) 6= (0, 0)
f (x, y) = x2 + y 2
0 , (x, y) = (0, 0)

a) Determine si f es continua en (0, 0).


b) Determine todas las derivadas direccionales de f en (0, 0).
c) Determine si f es diferenciable en (0, 0).

2. Sea z = z (x, y) una funcion de clase C 2 . Escriba la ecuacion:

2z 2z 2z
+ 2 3 =0
x2 xy y 2
en las variables u y v definidas por las ecuaciones:

u = 3x y v =x+y

3. Determine los valores de las constantes a, b y c en R de modo que la derivada


direccional de:
f (x, y, z) = acy 2 + byz + cz 2 x3
en el punto (1, 2, 1) tenga un valor maximo de 64 en la direccion del eje X positivo.

4. Hallar las dimensiones del paraleleppedo rectangular de mayor volumen con aristas
paralelas a los ejes coordenados que puede ser inscrito en el elipsoide de ecuacion:
 x 2  y 2  z 2
+ + =1
3 4 5

537
Apuntes Mat023 (Segundo semestre 2014)

Certamen 29
1. Resuelva la ecuacion diferencial:

(1 + x)2 y 00 3 (1 + x) y 0 + 4y = (1 + x)3

utilizando para ello el cambio de variables et = 1 + x.

2. Resuelva la ecuacion diferencial:

x00 + tx0 x = 0, x (0) = 0, x0 (0) = 1

3. Sea z C 2 . Simplificar al maximo la ecuacion:

2z 2z 2z
y + (x + y) + x =0
x2 xy y 2
considerando para ello el cambio de variables dado por:

u = y 2 x2 v =yx

4. Sean f : R3 R una funcion diferenciable tal que:

f (0, 2, 1) = (1, 1, 2) f (0, 2, 1) = 4

y g la funcion definida por g (u, v) = f (u v 2 , 3u v, 3u2 2v). Hallar la ecuacion


del plano tangente a la superficie z = g (u, v) en el punto (1, 1, 4) .

5. Determine el maximo y el mnimo absolutos de la funcion:

z = sin x + sin y + sin (x + y)

en la region:

0x< 0y
2 2

538
Apuntes Mat023 (Segundo semestre 2014)

Certamen 30
x
et
Z
1. Sea g (x) = dt. Hallar todos los valores de la constante a tales que la funcion
1 t
f definida por:
1 a g(x)
f (x) = e
x
satisfaga la ecuacion diferencial lineal:

x2 y 00 + 3x x2 y 0 + 1 x e2x y = 0
 

Utilizar la informacion anterior para determinar la solucion general de la ecuacion en


el intervalo (0, +).

2. Utilizando la Transformada de Laplace, resuelva la ecuacion diferencial:

xy 00 2y 0 + xy = 0

sabiendo que y (0) = 1 e y 0 (0) = 0.

3. Considere la funcion f : R2 R definida por:


(
x|y|3/2
x2 +y 2
, (x, y) 6= (0, 0)
f (x, y) =
0 , (x, y) = (0, 0)

a) Determine si f es continua en (x, y) = (0, 0)


b) Determine si f es diferenciable en (x, y) = (0, 0)
Z x
2
4. Sea f (x, y) = ln (x + 2xy + 1) + sin2 t dt
0

a) Calcule f (, 1)
f
b) Calcular

v R2 tal que k

v k = 1 y que (, 1) sea maxima.
v

5. Determine el maximo y el mnimo absoluto de la funcion:

z = x3 + y 3 3xy

en la region:
0x2 1 y 2

539
Apuntes Mat023 (Segundo semestre 2014)

Certamen 31
1. Hallar la solucion general de la ecuacion:

xy 00 2 (x + 1) y 0 + (x + 2) y = x3 e2x

para x > 0, bajo el supuesto que la ecuacion homogenea tiene una solucion de la
forma y = emx

2. Resuelva la ecuacion diferencial:

2 (1 + x)2 y 00 6 (1 + x) y 0 + 8y = (x + 1)3

3. Considere la funcion f : R2 R definida por:


(
x|y|3/2
x2 +y 2
, (x, y) 6= (0, 0)
f (x, y) =
0 , (x, y) = (0, 0)

a) Determine si f es continua en (x, y) = (0, 0)


b) Determine si f es diferenciable en (x, y) = (0, 0)

4. Sean f, g : R R funciones de clase C 2 . Considere z : U R2 R definida por:


y y
z (x, y) = x f +g
x x
2z 2z 2
2 z
Calcule el valor de: x2 + 2xy + y
x2 xy y 2
5. Determine el maximo y el mnimo absoluto de la funcion:

z = x3 + y 3 3xy

en la region:
0x2 1 y 2

540
Captulo : Bibliografa

[1] Kreyszig, E. Advanced Engineering Mathematics, 9th Ed., John Wiley & Sons Inc.,
Singapore, 2006.

[2] Piskunov, N. Calculo Diferencial e Integral, Editorial Limusa S.A. de C.V., Mexico,
2007.

[3] Osses, A. Ecuaciones Diferenciales Ordinarias, CMM, Departamento de Ingeniera


Matematica, U. de Chile, Santiago, 2010.

[4] Derrick, W & Grossman, S Ecuaciones diferenciales con aplicaciones, Fondo Educativo
Interamericano, Mexico, 1984.

[5] Apostol, T. Calculus: volumen 1, Editorial Reverte, Barcelona, 1967.

[6] Fernandez, C. & Rebolledo, R. Ecuaciones diferenciales ordinarias, Ediciones Univer-


sidad Catolica de Chile, Santiago, 1995.

[7] Hsu, H. Analisis de Fourier, Addison-Wesley Iberoamericana, EE.UU., 1987.

[8] Rocha, J.M. & Villa, G. Calculo infinitesimal de varias variables: vol. 1, I.P.N, Mexico,
2003.

[9] Taylor,H & Wade, T. Calculo diferencial e integral, LimusaWiley, Mexico, 1972.

[10] Gavilan, E. Dossier de problemas resueltos, Departamento de Matematicas, U. de


Concepcion, Concepcion.

[11] Martnez, C. Calculo real y vectorial en varias variables, Instituto de Matematicas, P.


U. Catolica de Valparaso, Valparaso, 2000.

[12] Jerrold E. Marsden & Anthony J. Tromba. Calculo vectorial, 5th Ed. Pearson. Estados
Unidos 2010.

541
Parte I

Ecuaciones diferenciales ordinarias

You might also like